Final/HESI

अब Quizwiz के साथ अपने होमवर्क और परीक्षाओं को एस करें!

What different pathophysiologic conditions can the health heart adapt to? (SATA) a. Menses b. Stress c. Gastroesophageal reflux disease d. Infection e. Hemorrhage

bde

the nurse assesses a client who had a coronary artery bypass graft yesterday. which assessment finding will the nurse report to the surgeon immediately? A. incisional pain B. BP of 136/76 C. decreased level of consciousness D. apical pulse of 88

C

Which statement by a client scheduled for a percutaneous transluminal coronary angioplasty (PTCA) indicates a need for further preoperative teaching? "I will be awake during this procedure." "I will have a balloon in my artery to widen it." "I must lie still after the procedure." "My angina will be gone for good."

"My angina will be gone for good." Reocclusion is possible after PTCA. The client is typically awake, but drowsy, during this procedure. PTCA uses a balloon to widen the artery, and the client will have to lie still after the procedure because of the large-bore venous access. Time is necessary to allow the hole to heal and prevent hemorrhage.

A client undergoing coronary artery bypass grafting asks why the surgeon has chosen to use the internal mammary artery for the surgery. Which response by the nurse is correct? "This way you will not need to have a leg incision." "The surgeon prefers this approach because it is easier." "These arteries remain open longer." "The surgeon has chosen this approach because of your age."

"These arteries remain open longer." Mammary arteries remain patent much longer than other grafts. Although no leg incision will be made with this approach, veins from the legs do not remain patent as long as the mammary artery graft does. Long-term patency, not ease of the procedure, is the primary concern. Age is not a determining factor in selection of these grafts.

A client with unstable angina has received education about acute coronary syndrome. Which statement indicates that the client has understood the teaching? "This is a big warning; I must modify my lifestyle or risk having a heart attack in the next year." "Angina is just a temporary interruption of blood flow to my heart." "I need to tell my wife I've had a heart attack." "Because this was temporary, I will not need to take any medications for my heart."

"This is a big warning; I must modify my lifestyle or risk having a heart attack in the next year." Among people who have unstable angina, 10% to 30% have a myocardial infarction within 1 year. Although anginal pain is temporary, it reflects underlying coronary artery disease (CAD), which requires attention, including lifestyle modifications. Unstable angina reflects tissue ischemia, but infarction represents tissue necrosis. Clients with underlying CAD may need medications such as aspirin, lipid-lowering agents, anti-anginals, or antihypertensives.

During discharge planning after admission for a myocardial infarction, the client says, "I won't be able to increase my activity level. I live in an apartment, and there is no place to walk." What is the nurse's best response? "You are right. Work on your diet then." "You must find someplace to walk." "Walk around the edge of your apartment complex." "Where might you be able to walk?"

"Where might you be able to walk?" Asking the client where he or she might be able to walk calls for cooperation and participation from the client; increased activity is imperative for this client. Telling the client to work on diet is an inappropriate response. Telling the client to find someplace to walk is too demanding to be therapeutic. Telling the client to walk around the apartment complex is domineering and will not likely achieve cooperation from the client.

(Chp. 31; elsevier resources)

(Chp. 31; elsevier resources)

81. The client with a continent urinary diversion is being discharged. Which discharge instructions should the nurse include in the teaching? 1. Have the client demonstrate catheterizing the stoma. 2. Instruct the client on how to pouch the stoma. 3. Explain the use of a bedside drainage bag at night. 4. Tell the client to call the HCP if the temperature is 99°F or less.

1. A continent urinary diversion is a surgical procedure in which a reservoir is created to hold urine until the client can selfcatheterize the stoma. The nurse should observe the client's technique before discharge. TEST-TAKING HINT: Options "2" and "3" are related to continuous drainage and could be eliminated on this basis. The word "continent" in the stem should key the test taker to the fact this diversion is a procedure in which there is no continuous drainage of urine.

73. The client is diagnosed with hypothyroidism. Which signs/symptoms should the nurse expect the client to exhibit? 1. Complaints of extreme fatigue and hair loss. 2. Exophthalmos and complaints of nervousness. 3. Complaints of profuse sweating and flushed skin. 4. Tetany and complaints of stiffness of the hands.

1. A decrease in thyroid hormone causes decreased metabolism, which leads to fatigue and hair loss. TEST-TAKING HINT: Often if the test taker does not know the specific signs/symptoms of the disease but knows the function of the system affected by the disease, some possible answers can be ruled out. Tetany and stiffness of the hands are related to calcium, the level of which is influenced by the parathyroid gland, not the thyroid gland; therefore, option "4" can be ruled out.

30. Which client teaching should the nurse implement for the client diagnosed with coronary artery disease? Select all that apply. 1. Encourage a low-fat, low-cholesterol diet. 2. Instruct the client to walk 30 minutes a day. 3. Decrease the salt intake to two (2) g a day. 4. Refer to a counselor for stress reduction techniques. 5. Teach the client to increase fiber in the diet.

1. A low-fat, low-cholesterol diet will help decrease the buildup of atherosclerosis in the arteries. 2. Walking will help increase collateral circulation. 4. Stress reduction is encouraged for clients with CAD because this helps prevent excess stress on the heart muscle. 5. Increasing fiber in the diet will help remove cholesterol via the gastrointestinal system. TEST-TAKING HINT: This is an alternate-type question where the test taker must select all interventions that are applicable to the situation. Coronary artery disease is a common disease, and the nurse must be knowledgeable about ways to modify risk factors.

20. The intensive care department nurse is assessing the client who is 12 hours post-myocardial infarction. The nurse assesses an S3 heart sound. Which intervention should the nurse implement? 1. Notify the health-care provider immediately. 2. Elevate the head of the client's bed. 3. Document this as a normal and expected finding. 4. Administer morphine intravenously.

1. An S3 indicates left ventricular failure and should be reported to the health-care provider. It is a potential life-threatening complication of a myocardial infarction. TEST-TAKING HINT: There are some situations in which the nurse must notify the healthcare provider, and the test taker should not automatically eliminate this as a possible correct answer. The test taker must decide if any of the other three options will help correct a life-threatening complication. Normal assessment concepts should help identify the correct option. The normal heart sounds are S1 and S2 ("lubb-dupp"); S3 is abnormal.

68. Which statement indicates the client diagnosed with calcium phosphate renal calculi understands the discharge teaching for ways to prevent future calculi formation? 1. "I should increase my fluid intake, especially in warm weather." 2. "I should eat foods containing cocoa and chocolate." 3. "I will walk about a mile every week and not exercise often." 4. "I should take one (1) vitamin a day with extra calcium."

1. An increased fluid intake ensuring 2 to 3 L of urine a day prevents the stoneforming salts from becoming concentrated enough to precipitate. TEST-TAKING HINT: This is a urinary problem and fluid is priority. Therefore, the test taker should select an option addressing fluid, and there is only one option addressing oral intake.

96. Which intervention should the nurse implement first for the client diagnosed with a hemothorax who has had a right-sided chest tube for three (3) days and has no fluctuation (tidaling) in the water compartment? 1. Assess the client's bilateral lung sounds. 2. Obtain an order for a STAT chest x-ray. 3. Notify the health-care provider as soon as possible. 4. Document the findings in the client's chart.

1. Assessment of the lung sounds could indicate the client's lung has reexpanded because it has been three (3) days since the chest tube has been inserted. TEST-TAKING HINT: When the stem asks the test taker to identify the first intervention, all four (4) answer options could be interventions appropriate for the situation, but only one (1) is the first intervention. Remember to apply the nursing process: the first step is assessment.

91. Which action should the nurse implement for the client with a hemothorax who has a rightsided chest tube with excessive bubbling in the water-seal compartment? 1. Check the amount of wall suction being applied. 2. Assess the tubing for any blood clots. 3. Milk the tubing proximal to distal. 4. Encourage the client to cough forcefully

1. Checking to see if someone has increased the suction rate is the simplest and a noninvasive action for the nurse to implement; if it is not on high, then the nurse must check to see if the problem is with the client or the system. TEST-TAKING HINT: The test taker should always think about assessing the client if there is a problem and the client is not in immediate danger. This would cause the test taker to eliminate options "3" and "4." If the test taker thinks about bubbling, he or she should know it has to do with suctioning.

67. The charge nurse is assigning clients for the shift. Which client should be assigned to the new graduate nurse? 1. The client diagnosed with cancer of the lung who has chest tubes. 2. The client diagnosed with laryngeal spasms who has stridor. 3. The client diagnosed with laryngeal cancer who has multiple fistulas. 4. The client who is two (2) hours post-partial laryngectomy.

1. Chest tubes are part of the nursing education curriculum. The new graduate should be capable of caring for this client or at least knowing when to get assistance. TEST-TAKING HINT: The question is asking for the least compromised or most stable client. The client in option "1" already has chest tubes in place and is presumably stable.

74. The nurse identifies the client problem "risk for imbalanced body temperature" for the client diagnosed with hypothyroidism. Which intervention should be included in the plan of care? 1. Discourage the use of an electric blanket. 2. Assess the client's temperature every two (2) hours. 3. Keep the room temperature cool. 4. Space activities to promote rest.

1. External heat sources (heating pads, electric or warming blankets) should be discouraged because they increase the risk of peripheral vasodilation and vascular collapse. TEST-TAKING HINT: The test taker must always know exactly what the question is asking. Option "4" can be ruled out because it does not address body temperature. If the test taker knows the normal function of the thyroid gland, this may help identify the answer; decreased metabolism will cause the client to be cold.

80. Which nursing interventions should the nurse implement for the client diagnosed with a pulmonary embolus who is undergoing thrombolytic therapy? Select all that apply. 1. Keep protamine sulfate readily available. 2. Avoid applying pressure to venipuncture sites. 3. Assess for overt and covert signs of bleeding. 4. Avoid invasive procedures and injections. 5. Administer stool softeners as ordered.

1. Heparin is administered during thrombolytic therapy, and the antidote is protamine sulfate and should be available to reverse the effects of the anticoagulant. 3. Obvious (overt) as well as hidden (covert) signs of bleeding should be assessed for. 4. Invasive procedures increase the risk of tissue trauma and bleeding. 5. Stool softeners help prevent constipation and straining, which may precipitate bleeding from hemorrhoids. TEST-TAKING HINT: Thrombolytic therapy is ordered to help dissolve the clot resulting in the PE. Therefore, all nursing interventions should address bleeding tendencies. The test taker must select all interventions applicable in these alternative questions.

82. The client diagnosed with a pulmonary embolus is being discharged. Which intervention should the nurse discuss with the client? 1. Increase fluid intake to two (2) to three (3) L/day. 2. Eat a low-cholesterol, low-fat diet. 3. Avoid being around large crowds. 4. Receive pneumonia and flu vaccines.

1. Increasing fluids will help increase fluid volume, which will, in turn, help prevent the development of deep vein thrombosis, the most common cause of PE. TEST-TAKING HINT: The test taker must know deep vein thrombosis is the most common cause of pulmonary embolus and preventing dehydration is an important intervention. The test taker can attempt to eliminate answers by trying to determine which disease process is appropriate for the intervention.

80. The 68-year-old client diagnosed with hyperthyroidism is being treated with radioactive iodine therapy. Which interventions should the nurse discuss with the client? 1. Explain it will take up to a month for symptoms of hyperthyroidism to subside. 2. Teach the iodine therapy will have to be tapered slowly over one (1) week. 3. Discuss the client will have to be hospitalized during the radioactive therapy. 4. Inform the client after therapy the client will not have to take any medication.

1. Radioactive iodine therapy is used to destroy the overactive thyroid cells. After treatment, the client is followed closely for three (3) to four (4) weeks until the euthyroid state is reached. TEST-TAKING HINT: Some questions require the test taker to be knowledgeable of the information, especially medical treatments, and there are no specific hints to help the test taker answer the question.

56. The client diagnosed with lung cancer is being discharged. Which statement made by the client indicates more teaching is required? 1. "It doesn't matter if I smoke now. I already have cancer." 2. "I should see the oncologist at my scheduled appointment." 3. "If I begin to run a fever, I should notify the HCP." 4. "I should plan for periods of rest throughout the day."

1. Research indicates smoking will still interfere with the client's response to treatment, so more teaching is needed. TEST-TAKING HINT: Two (2) options—"2" and "3"—are instructions given to all clients regardless of disease process.

50. The nurse writes a problem of "impaired gas exchange" for a client diagnosed with cancer of the lung. Which interventions should be included in the plan of care? Select all that apply. 1. Apply O2 via nasal cannula. 2. Have the dietitian plan for six (6) small meals per day. 3. Place the client in respiratory isolation. 4. Assess vital signs for fever. 5. Listen to lung sounds every shift.

1. Respiratory distress is a common finding in clients diagnosed with lung cancer. As the tumor grows and takes up more space or blocks air movement, the client may need to be taught positioning for lung expansion. The administration of oxygen will help the client to use the lung capacity that is available to get oxygen to the tissues. 2. Clients with lung cancer frequently become fatigued trying to eat. Providing six (6) small meals spaces the amount of food the client eats throughout the day. 4. Clients with cancer of the lung are at risk for developing an infection from lowered resistance as a result of treatments or from the tumor blocking secretions in the lung. Therefore, monitoring for the presence of fever, a possible indication of infection, is important. 5. Assessment of the lungs should be completed on a routine and prn basis. TEST-TAKING HINT: This alternative-type question is an all-or-nothing situation. The NCLEX-RN examination requires the test taker to answer each option correctly to receive credit for the question. Each option has the potential to be right or wrong.

34. The client with coronary artery disease is prescribed a Holter monitor. Which intervention should the nurse implement? 1. Instruct the client to keep a diary of activity, especially when having chest pain. 2. Discuss the need to remove the Holter monitor during a.m. care and showering. 3. Explain that all medications should be withheld while wearing a Holter monitor. 4. Teach the client the importance of decreasing activity while wearing the monitor.

1. The Holter monitor is a 24-hour electrocardiogram, and the client must keep an accurate record of activity so that the health-care provider can compare the ECG recordings with different levels of activity. TEST-TAKING HINT: In some instances, the test taker must be knowledgeable about diagnostic tests and there are no test-taking hints. The test taker might eliminate option "3" by realizing that, unless the client is NPO for a test or surgery, medications are usually taken.

87. Which intervention should the nurse implement for a male client who has had a left-sided chest tube for six (6) hours and who refuses to take deep breaths because of the pain? 1. Medicate the client and have the client take deep breaths. 2. Encourage the client to take shallow breaths to help with the pain. 3. Explain deep breaths do not have to be taken at this time. 4. Tell the client if he doesn't take deep breaths, he could die.

1. The client must take deep breaths to help push the air out of the pleural space into the water-seal drainage, and deep breaths will help prevent the client from developing pneumonia or atelectasis. TEST-TAKING HINT: If the test taker reads options "2" and "3" and notices that both reflect the same idea—namely, that deep breaths are not necessary—then both can either be eliminated as incorrect answers or kept as possible correct answers. Option "4" should be eliminated based on being a very rude and threatening comment.

76. The client diagnosed with cancer of the bladder is undergoing intravesical chemotherapy. Which instruction should the nurse provide the client about the pretherapy routine? 1. Instruct the client to remain NPO after midnight before the procedure. 2. Explain the use of chemotherapy in bladder cancer. 3. Teach the client to administer Neupogen, a biologic response modifier. 4. Have the client take Tylenol, an analgesic, before coming to the clinic.

1. The client will have medication instilled in the bladder, which must remain in the bladder for a prescribed length of time. For this reason, the client must remain NPO before the procedure. TEST-TAKING HINT: If the test taker is not aware of the term "intravesical," then dividing the word into its components may be useful. Intra- means "into" and vesical means "bladder." The test taker should choose an option that has a direct effect on urine production.

65. The client diagnosed with renal calculi is scheduled for a 24-hour urine specimen collection. Which interventions should the nurse implement? Select all that apply. 1. Check for the ordered diet and medication modifications. 2. Instruct the client to urinate, and discard this urine when starting collection. 3. Collect all urine during 24 hours and place in appropriate specimen container. 4. Insert an indwelling catheter in client after having the client empty the bladder. 5. Instruct the UAP to notify the nurse when the client urinates.

1. The health-care provider may order certain foods and medications when obtaining a 24-hour urine collection to evaluate for calcium oxalate or uric acid. 2. When the collection begins, the client should completely empty the bladder and discard this urine. The test is started after the bladder is empty. 3. All urine for 24 hours should be saved and put in a container with preservative, refrigerated, or placed on ice as indicated. Not following specific instructions will result in an inaccurate test result. TEST-TAKING HINT: This is an alternate-type question that has more than one correct answer. The test taker must be knowledgeable of specific laboratory tests.

23. The client diagnosed with a myocardial infarction asks the nurse, "Why do I have to rest and take it easy? My chest doesn't hurt anymore." Which statement would be the nurse's best response? 1. "Your heart is damaged and needs about four (4) to six (6) weeks to heal." 2. "There is necrotic myocardial tissue that puts you at risk for dysrhythmias." 3. "Your doctor has ordered bedrest. Therefore, you must stay in the bed." 4. "Just because your chest doesn't hurt anymore doesn't mean you are out of danger."

1. The heart tissue is dead, stress or activity may cause heart failure, and it does take about six (6) weeks for scar tissue to form. TEST-TAKING HINT: When attempting to answer a client's question, the nurse should provide factual information in simple, understandable terms. The test taker should select the answer option that provides this type of information.

66. The nurse and an unlicensed assistive personnel (UAP) are caring for a group of clients on a surgical floor. Which information provided by the UAP requires immediate intervention by the nurse? 1. There is a small, continuous amount of bright-red drainage coming out from under the dressing of the client who had a radical neck dissection. 2. The client who has had a right upper lobectomy is complaining that the patientcontrolled analgesia (PCA) pump is not providing any relief. 3. The client diagnosed with cancer of the lung is complaining of being tired and short of breath. 4. The client admitted with chronic obstructive pulmonary disease is making a whistling sound with every breath.

1. The most serious complication resulting from a radical neck dissection is rupture of the carotid artery. Continuous brightred drainage indicates bleeding, and this client should be assessed immediately. TEST-TAKING HINT: If the test taker is not sure of the answer and airway compromise is not one of the answer options, then an option dealing with bleeding is the best choice.

74. The client is suspected of having a pulmonary embolus. Which diagnostic test suggests the presence of a pulmonary embolus and requires further investigation? 1. Plasma D-dimer test. 2. Arterial blood gases. 3. Chest x-ray (CXR). 4. Magnetic resonance imaging (MRI).

1. The plasma D-dimer test is highly specific for the presence of a thrombus; an elevated D-dimer indicates a thrombus formation and lysis. This result would require a CT or V/Q scan to then confirm the diagnosis. TEST-TAKING HINT: The key to answering this question is "confirms the diagnosis." The test taker should eliminate options "2" and "3" based on the fact these are diagnostic tests used for many disease processes and conditions.

62. The client is diagnosed with cancer of the larynx and is to have radiation therapy to the area. Which prophylactic procedure will the nurse prepare the client for? 1. Removal of the client's teeth and fitting for dentures. 2. Take antiemetic medications every four (4) hours. 3. Wear sunscreen on the area at all times. 4. Placement of a nasogastric feeding tube.

1. The teeth will be in the area of radiation and the roots of teeth are highly sensitive to radiation, which results in root abscesses. The teeth are removed and the client is fitted for dentures prior to radiation. TEST-TAKING HINT: The test taker could eliminate option "4" as a form of nutritional treatment, not prophylaxis. The test taker must recognize which anatomical structures would lie within the radiation beam. The teeth of the lower jaw definitely are within the port, and the upper teeth possibly would be in range.

84. The female client diagnosed with bladder cancer who has a cutaneous urinary diversion states, "Will I be able to have children now?" Which statement is the nurse's best response? 1. "Cancer does not make you sterile, but sometimes the therapy can." 2. "Are you concerned you can't have children?" 3. "You will be able to have as many children as you want." 4. "Let me have the chaplain come to talk with you about this."

1. This client is asking for information and should be provided factual information. The surgery will not make the client sterile, but chemotherapy can induce menopause and radiation therapy to the pelvis can render a client sterile. TEST-TAKING HINT: When the stem has the client asking for specific information, then the nurse should provide the correct information. It is easy to confuse these questions with ones requiring therapeutic responses.

26. The client with coronary artery disease asks the nurse, "Why do I get chest pain?" Which statement would be the most appropriate response by the nurse? 1. "Chest pain is caused by decreased oxygen to the heart muscle." 2. "There is ischemia to the myocardium as a result of hypoxemia." 3. "The heart muscle is unable to pump effectively to perfuse the body." 4. "Chest pain occurs when the lungs cannot adequately oxygenate the blood."

1. This is a correct statement presented in layman's terms. When the coronary arteries cannot supply adequate oxygen to the heart muscle, there is chest pain. TEST-TAKING HINT: The nurse must select the option that best explains the facts in terms a client who does not have medical training can understand.

71. The client diagnosed with cancer of the larynx has had four (4) weeks of radiation therapy to the neck. The client is complaining of severe pain when swallowing. Which scientific rationale explains the pain? 1. The cancer has grown to obstruct the esophagus. 2. The treatments are working on the cancer and the throat is edematous. 3. Cancers are painful and this is expected. 4. The treatments are also affecting the esophagus, causing ulcerations.

1. This is an example of nonmalfeasance, where the nurse "does no harm." In attempting to discuss the client's refusal, the nurse is not trying to influence the client; the nurse is merely attempting to listen therapeutically. TEST-TAKING HINT: If the test taker were not aware of the terms of ethical principles, then dissecting the word "nonmalfeasance" into its portions might help. Non- means "nothing" or "none," and mal- means "bad," so "no bad action" could be inferred. This would eliminate option "2."

53. The client diagnosed with lung cancer has been told the cancer has metastasized to the brain. Which intervention should the nurse implement? 1. Discuss implementing an advance directive. 2. Explain the use of chemotherapy for brain involvement. 3. Teach the client to discontinue driving. 4. Have the significant other make decisions for the client.

1. This situation indicates a terminal process, and the client should make decisions for the end of life. TEST-TAKING HINT: The ethical principle of autonomy could help the test taker to discard option "4" as a correct answer.

77. The nurse is planning the care of a postoperative client with an ileal conduit. Which intervention should be included in the plan of care? 1. Provide meticulous skin care and pouching. 2. Apply sterile drainage bags daily. 3. Monitor the pH of the urine weekly. 4. Assess the stoma site every day.

1. Urine is acidic, and the abdominal wall tissue is not designed to tolerate acidic environments. The stoma is pouched so urine will not touch the skin. TEST-TAKING HINT: The test taker should look at time frames—daily and weekly. If the time frame is not sufficient, then the option can be eliminated as a possible correct answer.

82. Which information regarding the care of a cutaneous ileal conduit should the nurse discuss with the client? 1. Teach the client to instill a few drops of vinegar into the pouch. 2. Tell the client the stoma should be slightly dusky colored. 3. Inform the client large clumps of mucus are expected. 4. Tell the client it is normal for the urine to be pink or red in color.

1. Vinegar will act as a deodorizing agent in the pouch and help prevent a strong urine smell. TEST-TAKING HINT: A dusky color is never normal when discussing body functioning. There are very few procedures for which bloody urine is a normal expectation.

81. The nurse is teaching the client diagnosed with hyperthyroidism. Which information should be taught to the client? Select all that apply. 1. Notify the HCP if a three (3)-pound weight loss occurs in two (2) days. 2. Discuss ways to cope with the emotional lability. 3. Notify the HCP if taking over-the-counter medication. 4. Carry a medical identification card or bracelet. 5. Teach how to take thyroid medications correctly.

1. Weight loss indicates the medication may not be effective and will probably need to be increased. 2. The client needs to know emotional highs and lows are secondary to hyperthyroidism. With treatment, this emotional lability will subside. 3. Any over-the-counter medications (for example, alcohol-based medications) may negatively affect the client's hyperthyroidism or medications being used for treatment. 4. This will help any HCP immediately know of the client's condition, especially if the client is unable to tell the HCP. TEST-TAKING HINT: This alternate-type question instructs the test taker to select all the interventions that apply. The test taker must read and evaluate each option as to whether it applies or not.

28. The nurse is preparing to administer a beta blocker to the client diagnosed with coronary artery disease. Which assessment data would cause the nurse to question administering the medication? 1. The client has a BP of 110/70. 2. The client has an apical pulse of 56. 3. The client is complaining of a headache. 4. The client's potassium level is 4.5 mEq/L.

2. A beta blocker decreases sympathetic stimulation to the heart, thereby decreasing the heart rate. An apical rate less than 60 indicates a lower-than-normal heart rate and should make the nurse question administering this medication because it will further decrease the heart rate. TEST-TAKING HINT: If the test taker does not know when to question the use of a certain medication, the test taker should evaluate the options to determine if any options include abnormal data based on normal parameters. This would make the test taker select option "2" because the normal apical pulse in an adult is 60 to 100.

77. The client is admitted to the intensive care department diagnosed with myxedema coma. Which assessment data warrant immediate intervention by the nurse? 1. Serum blood glucose level of 74 mg/dL. 2. Pulse oximeter reading of 90%. 3. Telemetry reading showing sinus bradycardia. 4. The client is lethargic and sleeps all the time.

2. A pulse oximeter reading of less than 93% is significant. A 90% pulse oximeter reading indicates a Pao2 of approximately 60 on an arterial blood gas test; this is severe hypoxemia and requires immediate intervention. TEST-TAKING HINT: The words "warrant immediate intervention" means the test taker should select an option that is abnormal for the disease process or a life-threatening symptom.

75. The client diagnosed with cancer of the bladder is scheduled to have a cutaneous urinary diversion procedure. Which preoperative teaching intervention specific to the procedure should be included? 1. Demonstrate turn, cough, and deep breathing. 2. Explain a bag will drain the urine from now on. 3. Instruct the client on the use of a PCA pump. 4. Take the client to the ICD so the client can become familiar with it.

2. A urinary diversion procedure involves the removal of the bladder. In a cutaneous procedure, the ureters are implanted in some way to allow for stoma formation on the abdominal wall, and the urine drains into a pouch. There are numerous methods used for creating the stoma. TEST-TAKING HINT: The test taker must notice the phrase "specific to the procedure" to be able to correctly answer this question. All of the options are standard interventions for major surgeries, but only one is specific to the procedure.

16. The nurse is caring for a client diagnosed with a myocardial infarction who is experiencing chest pain. Which interventions should the nurse implement? Select all that apply. 1. Administer morphine intramuscularly. 2. Administer an aspirin orally. 3. Apply oxygen via a nasal cannula. 4. Place the client in a supine position. 5. Administer nitroglycerin subcutaneously.

2. Aspirin is an antiplatelet medication and should be administered orally. 3. Oxygen will help decrease myocardial ischemia, thereby decreasing pain. TEST-TAKING HINT: This is an alternate-type question that requires the test taker to select all options that are applicable. The test taker must identify all correct answer options to receive credit for a correct answer; no partial credit is given. Remember to read the question carefully—it is not meant to be tricky.

62. The client diagnosed with renal calculi is admitted to the medical unit. Which intervention should the nurse implement first? 1. Monitor the client's urinary output. 2. Assess the client's pain and rule out complications. 3. Increase the client's oral fluid intake. 4. Use a safety gait belt when ambulating the client.

2. Assessment is the first part of the nursing process and is priority. The renal colic pain can be so intense it can cause a vasovagal response, with resulting hypotension and syncope. TEST-TAKING HINT: Remember, if the question asks which intervention is first, all four (4) options may be appropriate for the client's diagnosis but only one has priority. Assessment is the first part of the nursing process and it is the first intervention a nurse should implement if the client is not in distress.

92. Which assessment data indicate to the nurse the chest tubes inserted three (3) days ago have been effective in treating the client with a hemothorax? 1. Gentle bubbling in the suction compartment. 2. No fluctuation (tidaling) in the water-seal compartment. 3. The drainage compartment has 250 mL of blood 4. The client is able to deep breathe without any pain.

2. At three (3) days postinsertion, no fluctuation (tidaling) indicates the lung has reexpanded, which indicates the treatment has been effective. TEST-TAKING HINT: The test taker must be knowledgeable about chest tubes to be able to answer this question. The test taker must know the normal time frame and what is expected for each compartment of the chest tube drainage system.

14. Along with persistent, crushing chest pain, which signs/symptoms would make the nurse suspect that the client is experiencing a myocardial infarction? 1. Midepigastric pain and pyrosis. 2. Diaphoresis and cool, clammy skin. 3. Intermittent claudication and pallor. 4. Jugular vein distention and dependent edema.

2. Diaphoresis (sweating) is a systemic reaction to the MI. The body vasoconstricts to shunt blood from the periphery to the trunk of the body; this, in turn, leads to cold, clammy skin. TEST-TAKING HINT: The stem already addresses chest pain; therefore, the test taker could eliminate option "1" as a possible answer. Intermittent claudication, option "3," is the classic sign of arterial occlusive disease, and JVD is very specific to congestive heart failure. The nurse must be able to identify at least two or three signs/symptoms of disease processes.

84. Which signs/symptoms should make the nurse suspect the client is experiencing a thyroid storm? 1. Obstipation and hypoactive bowel sounds. 2. Hyperpyrexia and extreme tachycardia. 3. Hypotension and bradycardia. 4. Decreased respirations and hypoxia.

2. Hyperpyrexia (high fever) and heart rate above 130 beats per minute are signs of thyroid storm, a severely exaggerated hyperthyroidism. TEST-TAKING HINT: If the test taker does not have the knowledge to answer the question, the test taker should look at the options closely. Options "1," "3," and "4" all have signs/symptoms of "decrease"—hypoactive, hypotension, and hypoxia. The test taker should select the option that does not match.

54. The client diagnosed with lung cancer is in an investigational program and receiving a vaccine to treat the cancer. Which information regarding investigational regimens should the nurse teach? 1. Investigational regimens provide a better chance of survival for the client. 2. Investigational treatments have not been proven to be helpful to clients. 3. Clients will be paid to participate in an investigational program. 4. Only clients who are dying qualify for investigational treatments.

2. Investigational treatments are just that— treatments being investigated to determine if they are effective in the care of clients diagnosed with cancer. There is no guarantee the treatments will help the client. TEST-TAKING HINT: The test taker should think about the word "investigational" and understand its meaning. Investigational means looking into something. This should lead the test taker to choose option "2."

29. Which intervention should the nurse implement when administering a loop diuretic to a client diagnosed with coronary artery disease? 1. Assess the client's radial pulse. 2. Assess the client's serum potassium level. 3. Assess the client's glucometer reading. 4. Assess the client's pulse oximeter reading.

2. Loop diuretics cause potassium to be lost in the urine output. Therefore, the nurse should assess the client's potassium level, and if the client is hypokalemic, the nurse should question administering this medication. TEST-TAKING HINT: Knowing that diuretics increase urine output would lead the test taker to eliminate glucose level and oxygenation (options "3" and "4"). In very few instances does the nurse assess the radial pulse; the apical pulse is assessed.

69. Which intervention is most important for the nurse to implement for the client diagnosed with rule-out renal calculi? 1. Assess the client's neurological status every two (2) hours. 2. Strain all urine and send any sediment to the laboratory. 3. Monitor the client's creatinine and BUN levels. 4. Take a 24-hour dietary recall during the client interview.

2. Passing a renal stone may negate the need for the client to have lithotripsy or a surgical procedure. Therefore, all urine must be strained, and a stone, if found, should be sent to the laboratory to determine what caused the stone. TEST-TAKING HINT: Remember, if the question asks for "most important," more than one of the options could be appropriate but only one is most important. Assessment is priority if the client is not in distress, but the test taker should make sure it is appropriate for the situation.

79. The male client diagnosed with metastatic cancer of the bladder is emaciated and refuses to eat. Which nursing action is an example of the ethical principle of paternalism? 1. The nurse allows the client to talk about not wanting to eat. 2. The nurse tells the client if he does not eat, a feeding tube will be placed. 3. The nurse consults the dietitian about the client's nutritional needs. 4. The nurse asks the family to bring favorite foods for the client to eat.

2. Paternalism is deciding for the client what is best, similar to a parent making decisions for a child. Feeding a client, as with a feeding tube, without the client wishing to eat is paternalism. TEST-TAKING HINT: The question asks for an ethical principle, and only two of the options could be considered to represent ethical principles. Option "1" is allowing the client a voice in the situation; the term "paternalism" eliminates this option.

70. The HCP has recommended a total laryngectomy for a male client diagnosed with cancer of the larynx but the client refuses. Which intervention by the nurse illustrates the ethical principle of nonmalfeasance? 1. The nurse listens to the client explain why he is refusing surgery. 2. The nurse and significant other insist that the client have the surgery. 3. The nurse refers the client to a counselor for help with the decision. 4. The nurse asks a cancer survivor to come and discuss the surgery with the client.

2. Placing a towel over the mirror indicates the client is having difficulty looking at his reflection, a body-image problem. TEST-TAKING HINT: The test taker must try to match the problem with the answer choices. This would eliminate options "1" and "4." Shaving himself is a positive action, and the question asks for an action indicating a disturbance in body image.

84. The client is getting out of bed and becomes very anxious and has a feeling of impending doom. The nurse thinks the client may be experiencing a pulmonary embolism. Which action should the nurse implement first? 1. Administer oxygen 10 L via nasal cannula. 2. Place the client in high Fowler's position. 3. Obtain a STAT pulse oximeter reading. 4. Auscultate the client's lung sounds.

2. Placing the client in this position facilitates maximal lung expansion and reduces venous return to the right side of the heart, thus lowering pressures in the pulmonary vascular system. TEST-TAKING HINT: The test taker must select the option that will directly help the client breathe easier. Therefore, assessment is not the first intervention and option "4" can be eliminated as the correct answer. When the client is in distress, do not assess.

33. The nurse is discussing angina with a client who is diagnosed with coronary artery disease. Which action should the client take first when experiencing angina? 1. Put a nitroglycerin tablet under the tongue. 2. Stop the activity immediately and rest. 3. Document when and what activity caused angina. 4. Notify the health-care provider immediately.

2. Stopping the activity decreases the heart's need for oxygen and may help decrease the angina (chest pain). TEST-TAKING HINT: The question is asking which action the client should take first. This implies that more than one of the answer options could be appropriate for the chest pain, but that only one is done first. The test taker should select the answer that will help the client directly and quickly—and that is stopping the activity.

35. Which statement by the client diagnosed with coronary artery disease indicates that the client understands the discharge teaching concerning diet? 1. "I will not eat more than six (6) eggs a week." 2. "I should bake or grill any meats I eat." 3. "I will drink eight (8) ounces of whole milk a day." 4. "I should not eat any type of pork products."

2. The American Heart Association recommends a low-fat, low-cholesterol diet for a client with coronary artery disease. The client should avoid any fried foods, especially meats, and bake, broil, or grill any meat. TEST-TAKING HINT: The test taker must be knowledgeable of prescribed diets for specific disease processes. This is mainly memorizing facts. There is no test-taking hint to help eliminate any of the options.

88. The unlicensed assistive personnel (UAP) assists the client with a chest tube to ambulate to the bathroom. Which situation warrants immediate intervention from the nurse? 1. The UAP keeps the chest tube below chest level. 2. The UAP has the chest tube attached to suction. 3. The UAP allowed the client out of the bed. 4. The UAP uses a bedside commode for the client.

2. The chest tube system can function as a result of gravity and does not have to be attached to suction. Keeping it attached to suction could cause the client to trip and fall. Therefore, this is a safety issue and the nurse should intervene and explain this to the UAP. TEST-TAKING HINT: "Warrants immediate intervention" means the test taker must identify the situation in which the nurse should correct the action, demonstrate a skill, or somehow intervene with the UAP's behavior.

63. The client is three (3) days post-partial laryngectomy. Which type of nutrition should the nurse offer the client? 1. Total parenteral nutrition. 2. Soft, regular diet. 3. Partial parenteral nutrition. 4. Clear liquid diet.

2. The client should be eating normal foods by this time. The consistency should be soft to allow for less chewing of the food and easier swallowing because a portion of the throat musculature has been removed. The client should be taught to turn the head toward the affected side when swallowing to help prevent aspiration. TEST-TAKING HINT: The keys to this question are "three (3) days" and "partial." Clients are progressed rapidly after surgery to as normal a life as possible.

95. The alert and oriented client is diagnosed with a spontaneous pneumothorax, and the healthcare provider is preparing to insert a left-sided chest tube. Which intervention should the nurse implement first? 1. Gather the needed supplies for the procedure. 2. Obtain a signed informed consent form. 3. Assist the client into a side-lying position. 4. Discuss the procedure with the client.

2. The insertion of a chest tube is an invasive procedure and requires informed consent. Without a consent form, this procedure should not be done on an alert and oriented client. TEST-TAKING HINT: The test taker must know invasive procedures require informed consent, and legally it must be obtained first before anyone can touch the client.

75. Which nursing assessment data support that the client has experienced a pulmonary embolism? 1. Calf pain with dorsiflexion of the foot. 2. Sudden onset of chest pain and dyspnea. 3. Left-sided chest pain and diaphoresis. 4. Bilateral crackles and low-grade fever.

2. The most common signs of a pulmonary embolism are sudden onset of chest pain when taking a deep breath and shortness of breath. TEST-TAKING HINT: The key to selecting option "2" as the correct answer is sudden onset. The test taker would need to note "left-sided" in option "3" to eliminate this as a possible correct answer, and option "4" is nonspecific for a pulmonary embolism.

18. The client is one (1) day postoperative coronary artery bypass surgery. The client complains of chest pain. Which intervention should the nurse implement first? 1. Medicate the client with intravenous morphine. 2. Assess the client's chest dressing and vital signs. 3. Encourage the client to turn from side to side. 4. Check the client's telemetry monitor.

2. The nurse must always assess the client to determine if the chest pain that is occurring is expected postoperatively or if it is a complication of the surgery. TEST-TAKING HINT: The stem asks the nurse to identify the first intervention that should be implemented. Therefore, the test taker should apply the nursing process and select an assessment intervention. Both options "2" and "4" involve assessment, but the nurse— not a machine or diagnostic test—should always assess the client.

27. The client is scheduled for a right femoral cardiac catheterization. Which nursing intervention should the nurse implement after the procedure? 1. Perform passive range-of-motion exercises. 2. Assess the client's neurovascular status. 3. Keep the client in high Fowler's position. 4. Assess the gag reflex prior to feeding the client.

2. The nurse must make sure that blood is circulating to the right leg, so the client should be assessed for pulses, paresthesia, paralysis, coldness, and pallor. TEST-TAKING HINT: The nurse should apply the nursing process when determining the correct answer. Therefore, either option "2" or option "4" could possibly be the correct answer. The test taker then should apply anatomy concepts—where is the left femoral artery? Neurovascular assessment is performed on extremities.

64. Which clinical manifestations should the nurse expect to assess for the client diagnosed with a ureteral renal stone? 1. Dull, aching flank pain and microscopic hematuria. 2. Nausea; vomiting; pallor; and cool, clammy skin. 3. Gross hematuria and dull suprapubic pain with voiding. 4. The client will be asymptomatic.

2. The severe flank pain associated with a stone in the ureter often causes a sympathetic response with associated nausea; vomiting; pallor; and cool, clammy skin. TEST-TAKING HINT: Options "1" and "3" both have assessment data indicating bleeding. The test taker can usually eliminate these as possible answers or eliminate the other two options not addressing blood. Renal stones are painful; therefore, option "4" could be eliminated as a possible answer.

93. The nurse is caring for a client with a right-sided chest tube secondary to a pneumothorax. Which interventions should the nurse implement when caring for this client? Select all that apply. 1. Place the client in the low Fowler's position. 2. Assess chest tube drainage system frequently. 3. Maintain strict bedrest for the client. 4. Secure a loop of drainage tubing to the sheet. 5. Observe the site for subcutaneous emphysema.

2. The system must be patent and intact to function properly. 4. Looping the tubing prevents direct pressure on the chest tube itself and keeps tubing off the floor, addressing both a safety and a potential clogging of the tube. 5. Subcutaneous emphysema is air under the skin, which is a common occurrence at the chest tube insertion site. TEST-TAKING HINT: The test taker should be careful with adjectives. In option "1," the word "low" makes it incorrect; in option "3," the word "strict" makes this option incorrect.

79. The nurse identified the client problem "decreased cardiac output" for the client diagnosed with a pulmonary embolus. Which intervention should be included in the plan of care? 1. Monitor the client's arterial blood gases 2. Assess skin color and temperature. 3. Check the client for signs of bleeding. 4. Keep the client in the Trendelenburg position.

2. These assessment data monitor tissue perfusion, which evaluates for decreased cardiac output. TEST-TAKING HINT: The test taker must think about which answer option addresses the problem of the heart's inability to pump blood. Decreased blood to the extremities results in cyanosis and cold extremities.

58. The client is four (4) hours post-lobectomy for cancer of the lung. Which assessment data warrant immediate intervention by the nurse? 1. The client has an intake of 1,500 mL IV and an output of 1,000 mL. 2. The client has 450 mL of bright-red drainage in the chest tube. 3. The client is complaining of pain at a "10" on a 1-to-10 scale. 4. The client has absent lung sounds on the side of the surgery.

2. This is about a pint of blood loss and could indicate the client is hemorrhaging TEST-TAKING HINT: Blood is always a priority.

85. The client is admitted to the emergency department with chest trauma. Which signs/ symptoms indicate to the nurse the diagnosis of pneumothorax? 1. Bronchovesicular lung sounds and bradypnea. 2. Unequal lung expansion and dyspnea. 3. Frothy, bloody sputum and consolidation. 4. Barrel chest and polycythemia.

2. Unequal lung expansion and dyspnea indicate a pneumothorax. TEST-TAKING HINT: The test taker can use "chest trauma" or "pneumothorax" to help select the correct answer. Both of these terms should cause the test taker to select option "2" because unequal chest expansion would result from trauma.

61. The nurse is admitting a client with a diagnosis of rule-out cancer of the larynx. Which information should the nurse teach? 1. Demonstrate the proper method of gargling with normal saline. 2. Perform voice exercises for 30 minutes three (3) times a day. 3. Explain that a lighted instrument will be placed in the throat to biopsy the area. 4. Teach the client to self-examine the larynx monthly.

3. A laryngoscopy will be performed to allow for visualization of the vocal cords and to obtain a biopsy for pathological diagnosis. TEST-TAKING HINT: The test taker must understand that, if the question states the client is admitted to "rule out" a disease, then diagnostic tests and procedures will be done to determine if, in fact, the client has the diagnosis that the HCP suspects.

36. The charge nurse is making assignments for clients on a cardiac unit. Which client should the charge nurse assign to a new graduate nurse? 1. The 44-year-old client diagnosed with a myocardial infarction. 2. The 65-year-old client admitted with unstable angina. 3. The 75-year-old client scheduled for a cardiac catheterization. 4. The 50-year-old client complaining of chest pain.

3. A new graduate should be able to complete a preprocedure checklist and get this client to the catheterization laboratory. TEST-TAKING HINT: "New graduate" is the key to answering this question correctly. What type of client should be assigned to an inexperienced nurse? The test taker should not assign the new graduate a client who is unstable or at risk

83. The nurse is preparing to administer medications to the following clients. Which medication should the nurse question administering? 1. The oral coagulant warfarin (Coumadin) to the client with an INR of 1.9. 2. Regular insulin to a client with a blood glucose level of 218 mg/dL. 3. Hang the heparin bag on a client with a PT/PTT of 12.9/98. 4. A calcium channel blocker to the client with a BP of 112/82.

3. A normal PTT is 39 seconds, and for heparin to be therapeutic, it should be 1.5 to 2 times the normal value, or 58 to 78. A PTT of 98 indicates the client is not clotting and the medication should be held. TEST-TAKING HINT: This question is asking the test taker to select a distracter with assessment data that are unsafe for administering the medication. The test taker must know normal laboratory values to administer medication safely.

78. The nurse is preparing to administer the oral anticoagulant warfarin (Coumadin) to a client who has a PT/PTT of 22/39 and an INR of 2.8. What action should the nurse implement? 1. Assess the client for abnormal bleeding. 2. Prepare to administer vitamin K (AquaMephyton). 3. Administer the medication as ordered. 4. Notify the HCP to obtain an order to increase the dose.

3. A therapeutic INR is 2 to 3; therefore, the nurse should administer the medication. TEST-TAKING HINT: The test taker must know normal laboratory values; this is the only way the test taker will be able to answer this question. The test taker should make a list of laboratory values that must be memorized for successful test taking.

70. The client with a history of renal calculi calls the clinic and reports having burning on urination, chills, and an elevated temperature. Which instruction should the nurse discuss with the client? 1. Increase water intake for the next 24 hours. 2. Take two (2) Tylenol to help decrease the temperature. 3. Come to the clinic and provide a urinalysis specimen. 4. Use a sterile 4 × 4 gauze to strain the client's urine.

3. A urinalysis can assess for hematuria, the presence of white blood cells, crystal fragments, or all three, which can determine if the client has a urinary tract infection or possibly a renal stone, with accompanying signs/symptoms of UTI. TEST-TAKING HINT: Fever, chills, and burning on urination require some type of assessment. Therefore, the test taker should select an option that helps determine what is wrong with the client and "3" is the only such option.

19. The client diagnosed with a myocardial infarction is six (6) hours post-right femoral percutaneous transluminal coronary angioplasty (PTCA), also known as balloon surgery. Which assessment data would require immediate intervention by the nurse? 1. The client is keeping the affected extremity straight. 2. The pressure dressing to the right femoral area is intact. 3. The client is complaining of numbness in the right foot. 4. The client's right pedal pulse is 3+ and bounding.

3. Any neurovascular assessment data that are abnormal require intervention by the nurse; numbness may indicate decreased blood supply to the right foot. TEST-TAKING HINT: This question requires the test taker to identify abnormal, unexpected, or life-threatening data. The nurse must know that a PTCA is performed by placing a catheter in the femoral artery and that internal or external bleeding is the most common complication.

52. The nurse and an unlicensed assistive personnel (UAP) are caring for a group of clients on a medical unit. Which information provided by the UAP warrants immediate intervention by the nurse? 1. The client diagnosed with cancer of the lung has a small amount of blood in the sputum collection cup. 2. The client diagnosed with chronic emphysema is sitting on the side of the bed and leaning over the bedside table. 3. The client receiving Procrit, a biologic response modifier, has a T 99.2˚F, P 68, R 24, and BP of 198/102. 4. The client receiving prednisone, a steroid, is complaining of an upset stomach after eating breakfast.

3. Biologic response modifiers stimulate the bone marrow and can increase the client's blood pressure to dangerous levels. This BP is high and warrants immediate attention. TEST-TAKING HINT: Even if the test taker did not know the side effects of Procrit, a BP of 198/102 warrants immediate attention.

67. The client diagnosed with renal calculi is scheduled for lithotripsy. Which postprocedure nursing task is the most appropriate to delegate to the UAP? 1. Monitor the amount, color, and consistency of urine output. 2. Teach the client about care of the indwelling Foley catheter. 3. Assist the client to the car when being discharged home. 4. Take the client's postprocedural vital signs.

3. The UAP could assist the client to the car once the discharge has been completed. TEST-TAKING HINT: There are some basic rules about delegation: the nurse cannot delegate assessment, teaching, evaluation, or any task requiring judgment.

17. The client who has had a myocardial infarction is admitted to the telemetry unit from intensive care. Which referral would be most appropriate for the client? 1. Social worker. 2. Physical therapy. 3. Cardiac rehabilitation. 4. Occupational therapy.

3. Cardiac rehabilitation is the most appropriate referral. The client can start rehabilitation in the hospital and then attend an outpatient cardiac rehabilitation clinic, which includes progressive exercise, diet teaching, and classes on modifying risk factors. TEST-TAKING HINT: The test taker must be familiar with the responsibilities of the other members of the health-care team. If the test taker had no idea which would be the most appropriate referral, the word "cardiac," which means "heart," should help the test taker in deciding that this is the most sensible option because the client had a myocardial infarction, a "heart attack."

74. Which modifiable risk factor should the nurse identify for the development of cancer of the bladder in a client? 1. Previous exposure to chemicals. 2. Pelvic radiation therapy. 3. Cigarette smoking. 4. Parasitic infections of the bladder.

3. Cigarette smoke contains more than 400 chemicals, 17 of which are known to cause cancer. The risk is directly proportional to the amount of smoking. TEST-TAKING HINT: The question asks for a modifiable risk factor. Modifiable factors involve lifestyle changes, weight loss, tobacco use, and eating habits.

61. The laboratory data reveal a calcium phosphate renal stone for a client diagnosed with renal calculi. Which discharge teaching intervention should the nurse implement? 1. Encourage the client to eat a low-purine diet and limit foods such as organ meats. 2. Explain the importance of not drinking water two (2) hours before bedtime. 3. Discuss the importance of limiting vitamin D-enriched foods. 4. Prepare the client for extracorporeal shock wave lithotripsy (ESWL).

3. Dietary changes for preventing renal stones include reducing the intake of the primary substance forming the calculi. In this case, limiting vitamin D will inhibit the absorption of calcium from the gastrointestinal tract. TEST-TAKING HINT: The test taker should remember to read the question carefully. The question asks for a "discharge teaching" intervention. This rules out option "4," which is a treatment, as a potential answer.

24. The client has just returned from a cardiac catheterization. Which assessment data would warrant immediate intervention from the nurse? 1. The client's BP is 110/70 and pulse is 90. 2. The client's groin dressing is dry and intact. 3. The client refuses to keep the leg straight. 4. The client denies any numbness and tingling.

3. If the client bends the leg, it could cause the insertion site to bleed. This is arterial blood and the client could bleed to death very quickly, so this requires immediate intervention. TEST-TAKING HINT: "Warrants immediate intervention" means the nurse should probably notify the health-care provider or do something independently because a complication may occur. Therefore, the test taker must select an answer option that is abnormal or unsafe. In the data listed, there are three normal findings and one abnormal finding.

68. The nurse is writing a care plan for a client newly diagnosed with cancer of the larynx. Which problem is the highest priority? 1. Wound infection. 2. Hemorrhage. 3. Respiratory distress. 4. Knowledge deficit.

3. Respiratory distress is the highest priority. Hemorrhaging and infection are serious problems, but airway is priority. TEST-TAKING HINT: In cardiopulmonary resuscitation the steps are airway, breathing, and then circulation. This concurs with Maslow's hierarchy of needs, which places oxygenation at the top of the hierarchy.

31. The elderly client has coronary artery disease. Which question should the nurse ask the client during the client teaching? 1. "Do you have a daily bowel movement?" 2. "Do you get yearly chest x-rays (CXRs)?" 3. "Are you sexually active?" 4. "Have you had any weight change?"

3. Sexual activity is a risk factor for angina resulting from coronary artery disease. The client's being elderly should not affect the nurse's assessment of the client's concerns about sexual activity. TEST-TAKING HINT: Remember, if the client is described with an adjective such as "elderly," this may be the key to selecting the correct answer. The nurse must not be judgmental about the elderly, especially about issues concerning sexual activity.

55. The nursing staff on an oncology unit is interviewing applicants for the unit manager position. Which type of organizational structure does this represent? 1. Centralized decision making. 2. Decentralized decision making. 3. Shared governance. 4. Pyramid with filtered-down decisions.

3. Shared governance is a system where the staff is empowered to make decisions such as scheduling and hiring of certain staff. Staff members are encouraged to participate in developing policies and procedures to reach set goals. TEST-TAKING HINT: Answer options "1" and "4" basically say the same thing and could be eliminated for this reason.

15. The client diagnosed with rule-out myocardial infarction is experiencing chest pain while walking to the bathroom. Which action should the nurse implement first? 1. Administer sublingual nitroglycerin. 2. Obtain a STAT electrocardiogram (ECG). 3. Have the client sit down immediately. 4. Assess the client's vital signs.

3. Stopping all activity will decrease the need of the myocardium for oxygen and may help decrease the chest pain. TEST-TAKING HINT: Whenever the test taker wants to select an assessment intervention, be sure to think about whether that intervention will help the client, especially if the client is experiencing pain. Do not automatically select the answer option that is assessment.

59. The client is admitted to the outpatient surgery center for a bronchoscopy to rule out cancer of the lung. Which information should the nurse teach? 1. The test will confirm the results of the MRI. 2. The client can eat and drink immediately after the test. 3. The HCP can do a biopsy of the tumor through the scope. 4. There is no discomfort associated with this procedure.

3. The HCP can take biopsies and perform a washing of the lung tissue for pathological diagnosis during the procedure. TEST-TAKING HINT: The test taker must read each stem and answer option carefully. When in medicine does one (1) test confirm another? Tests are done to confirm diagnoses. Option "4" has a false promise.

65. The client has had a total laryngectomy. Which referral is specific for this surgery? 1. CanSurmount. 2. Dialogue. 3. Lost Chord Club. 4. SmokEnders.

3. The Lost Chord Club is an American Cancer Society-sponsored group of survivors of larynx cancer. These clients are able to discuss the feelings and needs of clients who have had laryngectomies because they have all had this particular surgery. TEST-TAKING HINT: The larynx is commonly referred to as the "vocal chords." If the test taker were not aware of the various support groups, option "3" has "lost" and "chords" in it and is the best choice.

75. The client diagnosed with hypothyroidism is prescribed the thyroid hormone levothyroxine (Synthroid). Which assessment data indicate the medication has been effective? 1. The client has a three (3)-pound weight gain. 2. The client has a decreased pulse rate. 3. The client's temperature is WNL. 4. The client denies any diaphoresis.

3. The client with hypothyroidism frequently has a subnormal temperature, so a temperature WNL indicates the medication is effective. TEST-TAKING HINT: One way of determining the effectiveness of medication is to determine if the signs/symptoms of the disease are no longer noticeable.

86. The client had a right-sided chest tube inserted two (2) hours ago for a pneumothorax. Which action should the nurse implement if there is no fluctuation (tidaling) in the water-seal compartment? 1. Obtain an order for a STAT chest x-ray. 2. Increase the amount of wall suction. 3. Check the tubing for kinks or clots. 4. Monitor the client's pulse oximeter reading.

3. The key to the answer is "2 hours." The air from the pleural space is not able to get to the water-seal compartment, and the nurse should try to determine why. Usually the client is lying on the tube, it is kinked, or there is a dependent loop. TEST-TAKING HINT: The test taker should apply the nursing process to answer the question correctly. The first step in the nursing process is assessment, and "check" (option "3") is a word that can be used synonymously for "assess." Monitoring (option "4") is also assessing, but the test taker should not check a diagnostic test result before caring for the client.

76. The client diagnosed with a pulmonary embolus is in the intensive care department. Which assessment data warrant immediate intervention from the nurse? 1. The client's ABGs are pH 7.36, Pao2 95, Paco2 38, Hco3 24. 2. The client's telemetry exhibits occasional premature ventricular contractions (PVCs). 3. The client's pulse oximeter reading is 90%. 4. The client's urinary output for the 12-hour shift is 800 mL.

3. The normal pulse oximeter reading is 93% to 100%. A reading of 90% indicates the client has an arterial oxygen level of around 60. TEST-TAKING HINT: This question is asking the test taker to select abnormal, unexpected, or life-threatening assessment data in relationship to the client's disease process. A pulse oximeter reading of less than 93% indicates severe hypoxia and requires immediate intervention.

22. The client diagnosed with a myocardial infarction is on bedrest. The unlicensed assistive personnel (UAP) is encouraging the client to move the legs. Which action should the nurse implement? 1. Instruct the UAP to stop encouraging the leg movements. 2. Report this behavior to the charge nurse as soon as possible. 3. Praise the UAP for encouraging the client to move the legs. 4. Take no action concerning the UAP's behavior

3. The nurse should praise and encourage UAPs to participate in the client's care. Clients on bedrest are at risk for deep vein thrombosis, and moving the legs will help prevent this from occurring. TEST-TAKING HINT: This is a management question. The test taker must know the chain of command and when to report behavior. The test taker could eliminate options "1" and "2" with the knowledge that moving the legs is a safe activity for the client. When having to choose between options "3" and "4," the test taker should select doing something positive, instead of taking no action. This is a management concept.

90. The nurse is presenting a class on chest tubes. Which statement best describes a tension pneumothorax? 1. A tension pneumothorax develops when an airfilled bleb on the surface of the lung ruptures. 2. When a tension pneumothorax occurs, the air moves freely between the pleural space and the atmosphere. 3. The injury allows air into the pleural space but prevents it from escaping from the pleural space. 4. A tension pneumothorax results from a puncture of the pleura during a central line placement.

3. This describes a tension pneumothorax. It is a medical emergency requiring immediate intervention to preserve life. TEST-TAKING HINT: The test taker must always be clear about what the question is asking before answering the question. If the test taker can eliminate options "1" and "2" and can't decide between options "3" and "4," the test taker must go back to the stem and clarify what the question is asking.

80. The client diagnosed with cancer of the bladder states, "I have young children. I am too young to die." Which statement is the nurse's best response? 1. "This cancer is treatable and you should not give up." 2. "Cancer occurs at any age. It is just one of those things." 3. "You are afraid of dying and what will happen to your children." 4. "Have you talked to your children about your dying?"

3. This is an example of restating, a therapeutic technique used to clarify the client's feelings and encourage a discussion of those feelings. TEST-TAKING HINT: When the question requires a therapeutic response, the test taker should select an option addressing the client's feelings.

83. The nurse is preparing to administer the following medications. Which medication should the nurse question administering? 1. The thyroid hormone to the client who does not have a T3, T4 level. 2. The regular insulin to the client with a blood glucose level of 210 mg/dL. 3. The loop diuretic to the client with a potassium level of 3.3 mEq/L. 4. The cardiac glycoside to the client who has a digoxin level of 1.4 mg/dL.

3. This potassium level is below normal, which is 3.5 to 5.5 mEq/L. Therefore, the nurse should question administering this medication because loop diuretics cause potassium loss in the urine. TEST-TAKING HINT: When administering medication, the nurse must know when to question the medication, how to know it is effective, and what must be taught to keep the client safe while taking the medication. The test taker may want to turn the question around and say, "I should give this medication."

13. Which cardiac enzyme would the nurse expect to elevate first in a client diagnosed with a myocardial infarction (MI)? 1. Creatine kinase (CK-MB). 2. Lactate dehydrogenase (LDH). 3. Troponin. 4. White blood cells (WBCs).

3. Troponin is the enzyme that elevates within 1 to 2 hours. TEST-TAKING HINT: The test taker should be aware of the words "cardiac enzyme," which would eliminate option "4" as a possible answer. The word in the stem is "first." This questionrequires the test taker to have knowledge of laboratory values.

78. Which medication order should the nurse question in the client diagnosed with untreated hypothyroidism? 1. Thyroid hormones. 2. Oxygen. 3. Sedatives. 4. Laxatives.

3. Untreated hypothyroidism is characterized by an increased susceptibility to the effects of most hypnotic and sedative agents; therefore, the nurse should question this medication. TEST-TAKING HINT: When a question asks which order the nurse should question, three of the options are medications the nurse expects to administer to the client. Sometimes saying, "The nurse administers this medication," may help the test taker select the correct answer.

72. The client is diagnosed with a uric acid stone. Which foods should the client eliminate from the diet to help prevent reoccurrence? 1. Beer and colas. 2. Asparagus and cabbage. 3. Venison and sardines. 4. Cheese and eggs.

3. Venison, sardines, goose, organ meats, and herring are high-purine foods, which should be eliminated from the diet to help prevent uric acid stones. TEST-TAKING HINT: The nurse has to be knowledgeable of foods included in specific diets. This is memorizing, but the test taker must have this knowledge to answer questions evaluating types of diets for specific diseases and disorders.

32. The nurse is discussing the importance of exercise with the client diagnosed with coronary artery disease. Which intervention should the nurse implement? 1. Perform isometric exercises daily. 2. Walk for 15 minutes three (3) times a week. 3. Do not walk outside if it is less than 40˚F. 4. Wear open-toed shoes when ambulating.

3. When it is cold outside, vasoconstriction occurs, and this will decrease oxygen to the heart muscle. Therefore, the client should not exercise when it is cold outside. TEST-TAKING HINT: The test taker should be aware of adjectives such as "isometric," which makes option "1" incorrect, and "open-toed," which makes option "4" incorrect.

COMPLETION 1. A nurse prepares a client with acute renal insufficiency for a cardiac catheterization. The provider prescribes 0.9% normal saline to infuse at 125 mL/hr for renal protection. The nurse obtains gravity tubing with a drip rate of 15 drops/mL. At what rate (drops/min) should the nurse infuse the fluids? (Record your answer using a whole number, and rounding to the nearest drop.) drops/min

31 drops/min

33. The nurse and the idetitian are planning dietary intake for a patient with AKI who is currently not on dialysis therapy. The dietitian informs the nurse that 0.6 g/kg of body weight of protein are needed. The patient weighs 130 pounds. How many grams of protein should the patient receive? (Round grams to the nearest whole number.)

35

94. The charge nurse is making client assignments on a medical floor. Which client should the charge nurse assign to the licensed practical nurse (LPN)? 1. The client with pneumonia who has a pulse oximeter reading of 91%. 2. The client with a hemothorax who has Hb of 9 g/dL and Hct of 20%. 3. The client with chest tubes who has jugular vein distention and BP of 96/60. 4. The client who is two (2) hours post- bronchoscopy procedure.

4. A client two (2) hours post-bronchoscopy procedure could safely be assigned to an LPN. TEST-TAKING HINT: The test taker must understand that the LPN should be assigned the least critical client or the client who is stable and not exhibiting any complications secondary to the admitting disease or condition.

82. The nurse is providing an in-service on thyroid disorders. One of the attendees asks the nurse, "Why don't the people in the United States get goiters as often?" Which statement by the nurse is the best response? 1. "It is because of the screening techniques used in the United States." 2. "It is a genetic predisposition rare in North Americans." 3. "The medications available in the United States decrease goiters." 4. "Iodized salt helps prevent the development of goiters in the United States."

4. Almost all of the iodine entering the body is retained in the thyroid gland. A deficiency in iodine will cause the thyroid gland to work hard and enlarge, which is called a goiter. Goiters are commonly seen in geographical regions having an iodine deficiency. Most table salt in the United States has iodine added. TEST-TAKING HINT: The nurse must know about disease processes. There is no testtaking hint to help with knowledge

73. The nurse is caring for clients on a renal surgery unit. After the afternoon report, which client should the nurse assess first? 1. The male client who just returned from a CT scan who states he left his glasses in the x-ray department. 2. The client who is one (1) day postoperative and has a moderate amount of serous drainage on the dressing. 3. The client who is scheduled for surgery in the morning and wants an explanation of the operative procedure before signing the permit. 4. The client who had ileal conduit surgery this morning and has not had any drainage in the drainage bag.

4. An ileal conduit is a procedure diverting urine from the bladder and provides an alternate cutaneous pathway for urine to exit the body. Urinary output should always be at least 30 mL/hr. This client should be assessed to make sure the stents placed in the ureters have not become dislodged or blocked. TEST-TAKING HINT: Basic care of any postoperative client is to ensure urinary output. Two of the options involve tasks that can be delegated or are not in the realm of the nurse.

a client weighing 174 lbs had thrombolytic therapy followed by a one-time dose of IV lovenox 30 mg. the physician prescribes lovenox 1 mg/kg subcutaneously after the IV administration. the nurse will give ____ mg of lovenox to the client.

79

77. The client is admitted to the medical unit diagnosed with a pulmonary embolus. Which intervention should the nurse implement? 1. Administer oral anticoagulants. 2. Assess the client's bowel sounds. 3. Prepare the client for a thoracentesis. 4. Institute and maintain bedrest.

4. Bedrest reduces the risk of another clot becoming an embolus leading to a pulmonary embolus. Bedrest reduces metabolic demands and tissue needs for oxygen in the lungs. TEST-TAKING HINT: The test taker must be aware of adjectives such as "oral" in option "1," which makes this option incorrect. The test taker should apply the body system of the disease process to eliminate option "2" as a correct answer.

57. The clinic nurse is interviewing clients. Which information provided by a client warrants further investigation? 1. The client uses Vicks VapoRub every night before bed. 2. The client has had an appendectomy. 3. The client takes a multiple vitamin pill every day. 4. The client has been coughing up blood in the mornings.

4. Coughing up blood is not normal and is cause for investigation. It could indicate lung cancer. TEST-TAKING HINT: The test taker should read all distracters carefully. "Further investigation" means something abnormal is occurring. Coughing up blood is always abnormal.

78. The nurse and a licensed practical nurse (LPN) are caring for a group of clients. Which intervention should be assigned to the LPN? 1. Assessment of the client who has had a Kock pouch procedure. 2. Monitoring of the postop client with a WBC of 22,000/mm3. 3. Administration of the prescribed antineoplastic medications. 4. Care for the client going for an MRI of the kidneys.

4. It is in the scope of practice for the LPN to care for this client. TEST-TAKING HINT: The client who is the least ill or the client having the least invasive procedure is the client who should be assigned to the LPN.

63. The client with possible renal calculi is scheduled for a renal ultrasound. Which intervention should the nurse implement for this procedure? 1. Ask if the client is allergic to shellfish or iodine. 2. Keep the client NPO eight (8) hours prior to the ultrasound. 3. Ensure the client has a signed informed consent form. 4. Explain the test is noninvasive and there is no discomfort.

4. No special preparation is needed for this noninvasive, nonpainful test. A conductive gel is applied to the back or flank and then a transducer is applied, which produces sound waves, resulting in a picture. TEST-TAKING HINT: The nurse must be aware of preprocedure and postprocedure teaching and care. The test taker must know the invasive and noninvasive diagnostic tests in general. Ultrasound, computed tomography (CT), and magnetic resonance imaging (MRI) are a few of the noninvasive diagnostic tests.

66. The client is diagnosed with an acute episode of ureteral calculi. Which client problem is priority when caring for this client? 1. Fluid volume loss. 2. Knowledge deficit. 3. Impaired urinary elimination. 4. Alteration in comfort.

4. Pain is priority. The pain can be so severe a sympathetic response may occur, causing nausea; vomiting; pallor; and cool, clammy skin. TEST-TAKING HINT: Remember Maslow's hierarchy of needs: airway and pain are priority. No option mentions possible airway problems, so pain is priority.

49. The nurse is taking the social history from a client diagnosed with small cell carcinoma of the lung. Which information is significant for this disease? 1. The client worked with asbestos for a short time many years ago. 2. The client has no family history for this type of lung cancer. 3. The client has numerous tattoos covering both upper and lower arms. 4. The client has smoked two (2) packs of cigarettes a day for 20 years.

4. Smoking is the number-one risk factor for developing cancer of the lung. More than 85% of lung cancers are attributable to inhalation of chemicals. There are more than 400 chemicals in each puff of cigarette smoke, 17 of which are known to cause cancer. TEST-TAKING HINT: If the test taker did not know this information, option "3" has no anatomical connection to the lungs and could be eliminated. This information has been widely disseminated in the media for more than 40 years since the Surgeon General's office first warned about the dangers of smoking in the early 1960s.

89. The nurse is caring for a client with a rightsided chest tube that is accidentally pulled out of the pleural space. Which action should the nurse implement first? 1. Notify the health-care provider to have chest tubes reinserted STAT. 2. Instruct the client to take slow shallow breaths until the tube is reinserted. 3. Take no action and assess the client's respiratory status every 15 minutes. 4. Tape a petroleum jelly occlusive dressing on three (3) sides to the insertion site.

4. Taping on three sides prevents the development of a tension pneumothorax by inhibiting air from entering the wound during inhalation but allowing it to escape during exhalation. TEST-TAKING HINT: The words "implement first" in the stem of the question indicate to the test taker that possibly more than one (1) intervention could be warranted in the situation but only one (1) is implemented first. Remember, do not select assessment first without reading the question. If the client is in any type of crisis, then the nurse should first do something to help the client's situation.

83. The client is two (2) days postureterosigmoidostomy for cancer of the bladder. Which assessment data warrant notification of the HCP by the nurse? 1. The client complains of pain at a "3," 30 minutes after being medicated. 2. The client complains it hurts to cough and deep breathe. 3. The client ambulates to the end of the hall and back before lunch. 4. The client is lying in a fetal position and has a rigid abdomen.

4. The client is drawn up in a position that relieves pressure off the abdomen; a rigid abdomen is an indicator of peritonitis, a medical emergency. TEST-TAKING HINT: When the test taker is deciding on a priority question, the test taker should decide if the situation is expected or if it is life threatening.

64. The nurse is preparing the client diagnosed with laryngeal cancer for a laryngectomy in the morning. Which intervention is the nurse's priority? 1. Take the client to the intensive care unit for a visit. 2. Explain that the client will need to ask for pain medication. 3. Demonstrate the use of an antiembolism hose. 4. Find out if the client can read and write.

4. The client is having the vocal cords removed and will be unable to speak. Communication is a high priority for this client. If the client is able to read and write, a Magic Slate or pad of paper should be provided. If the client is illiterate, the nurse and the client should develop a method of communication using pictures. TEST-TAKING HINT: Questions addressing highest priority include all of the options being possible interventions, but only one is priority. Use Maslow's hierarchy of needs to answer the question—safety is priority.

25. The male client is diagnosed with coronary artery disease (CAD) and is prescribed sublingual nitroglycerin. Which statement indicates the client needs more teaching? 1. "I should keep the tablets in the dark-colored bottle they came in." 2. "If the tablets do not burn under my tongue, they are not effective." 3. "I should keep the bottle with me in my pocket at all times." 4. "If my chest pain is not

4. The client should take one tablet every five (5) minutes and, if no relief occurs after the third tablet, have someone drive him to the emergency department or call 911. TEST-TAKING HINT: This question is an "except" question, requiring the test taker to identify which statement indicates the client doesn't understand the teaching. Sometimes the test taker could restate the question and think which statement indicates the client understands the teaching.

81. Which statement by the client diagnosed with a pulmonary embolus indicates the discharge teaching is effective? 1. "I am going to use a regular-bristle toothbrush." 2. "I will take antibiotics prior to having my teeth cleaned." 3. "I can take enteric-coated aspirin for my headache." 4. "I will wear a Medic Alert band at all times."

4. The client should wear a Medic Alert band at all times so that, if any accident or situation occurs, the health-care providers will know the client is receiving anticoagulant therapy. The client understands the teaching. TEST-TAKING HINT: This is a higher level question in which the test taker must know clients with a pulmonary embolus are prescribed anticoagulant therapy on discharge from the hospital. If the test taker had no idea of the answer, the option stating "wear a Medic Alert band" is a good choice because many disease processes require the client to take long-term medication and a health-care provider should be aware of this.

76. Which nursing intervention should be included in the plan of care for the client diagnosed with hyperthyroidism? 1. Increase the amount of fiber in the diet. 2. Encourage a low-calorie, low-protein diet. 3. Decrease the client's fluid intake to 1,000 mL/day. 4. Provide six (6) small, well-balanced meals a day.

4. The client with hyperthyroidism has an increased appetite; therefore, well-balanced meals served several times throughout the day will help with the client's constant hunger. TEST-TAKING HINT: If the test taker knows the metabolism is increased with hyperthyroidism, then increasing the food intake is the most appropriate choice.

21. The nurse is administering a calcium channel blocker to the client diagnosed with a myocardial infarction. Which assessment data would cause the nurse to question administering this medication? 1. The client's apical pulse is 64. 2. The client's calcium level is elevated. 3. The client's telemetry shows occasional PVCs. 4. The client's blood pressure is 90/58.

4. The client's blood pressure is low, and a calcium channel blocker could cause the blood pressure to bottom out. TEST-TAKING HINT: The test taker must know when to question administering medications. The test taker is trying to select an option that, if the medication is administered, would cause serious harm to the client.

72. The client who has undergone a radical neck dissection and tracheostomy for cancer of the larynx is being discharged. Which discharge instructions should the nurse teach? Select all that apply. 1. The client will be able to speak again after the surgery area has healed. 2. The client should wear a protective covering over the stoma when showering. 3. The client should clean the stoma and then apply a petroleum-based ointment. 4. The client should use a humidifier in the room. 5. The client can get a special telephone for communication.

4. The esophagus is extremely radiosensitive, and esophageal ulcerations are common. The pain can become so severe the client cannot swallow saliva. This is a situation in which the client will be admitted to the hospital for IV narcotic pain medication and possibly total parenteral nutrition. TEST-TAKING HINT: The test taker must remember not to jump to conclusions and to realize what a word is actually saying. Swallowing is an action involving the esophagus, so the best choice would be either option "1" or option "4," both of which contain the word "esophagus."

60. The client diagnosed with oat cell carcinoma of the lung tells the nurse, "I am so tired of all this. I might as well just end it all." Which statement should be the nurse's first response? 1. Say, "This must be hard for you. Would you like to talk?" 2. Tell the HCP of the client's statement. 3. Refer the client to a social worker or spiritual advisor. 4. Find out if the client has a plan to carry out suicide.

4. The priority action anytime a client makes a statement regarding taking his or her own life is to determine if the client has thought it through enough to have a plan. A plan indicates an emergency situation. TEST-TAKING HINT: In a question that asks for a first response, all answer options may be actions the nurse would take. Ranking the options in order of action—"4," "1," "2," "3"—may help the test taker to make a decision. Applying Maslow's hierarchy of needs, safety comes first.

79. Which statement made by the client makes the nurse suspect the client is experiencing hyperthyroidism? 1. "I just don't seem to have any appetite anymore." 2. "I have a bowel movement about every three (3) to four (4) days." 3. "My skin is really becoming dry and coarse." 4. "I have noticed all my collars are getting tighter."

4. The thyroid gland (in the neck) enlarges as a result of the increased need for thyroid hormone production; an enlarged gland is called a goiter. TEST-TAKING HINT: If the test taker does not know the answer, sometimes thinking about the location of the gland or organ causing the problem may help the test taker select or rule out specific options.

71. The client had surgery to remove a kidney stone. Which laboratory assessment data warrant immediate intervention by the nurse? 1. A serum potassium level of 3.8 mEq/L. 2. A urinalysis shows microscopic hematuria. 3. A creatinine level of 0.8 mg/100 mL. 4. A white blood cell count of 14,000/mm3.

4. The white blood cell count is elevated; normal is 5,000 to 10,000/mm3. TEST-TAKING HINT: The nurse must know normal laboratory data and be able to apply the normal and abnormal results to specific diseases and disorders.

73. The client is diagnosed with a pulmonary embolus (PE) and is receiving a heparin drip. The bag hanging is 20,000 units/500 mL of D5W infusing at 22 mL/hr. How many units of heparin is the client receiving each hour? ________

880 units. If there are 20,000 units of heparin in 500 mL of D5W, there are 40 units in each mL: 20,000 ÷ 500 = 40 units If 22 mL are infused per hour, then 880 units of heparin are infused each hour: 40 × 22 = 880 TEST-TAKING HINT: The test taker must know how to calculate heparin drips from two aspects: the question may give the mL/hr and the test taker has to determine units/hr, or the question may give units/hr and the test taker has to determine mL/hr. Remember to learn how to use the drop-down calculator on the computer. During the NCLEX-RN, the test taker can request an erase slate.

13. A nurse cares for a client admitted from a nursing home after several recent falls. What prescription should the nurse complete first? a. Obtain urine sample for culture and sensitivity. b. Administer intravenous antibiotics. c. Encourage protein intake and additional fluids. d. Consult physical therapy for gait training.

A Although all interventions are or might be important, obtaining a urine sample for urinalysis takes priority. Often urinary tract infection (UTI) symptoms in older adults are atypical, and a UTI may present with new onset of confusion or falling. The urine sample should be obtained before starting antibiotics. Dietary requirements and gait training should be implemented after obtaining the urine sample.

10. After teaching a client with a history of renal calculi, the nurse assesses the clients understanding. Which statement made by the client indicates a correct understanding of the teaching? a. I should drink at least 3 liters of fluid every day. b. I will eliminate all dairy or sources of calcium from my diet. c. Aspirin and aspirin-containing products can lead to stones. d. The doctor can give me antibiotics at the first sign of a stone.

A Dehydration contributes to the precipitation of minerals to form a stone. Although increased intake of calcium causes hypercalcemia and leads to excessive calcium filtered into the urine, if the client is well hydrated the calcium will be excreted without issues. Dehydration increases the risk for supersaturation of calcium in the urine, which contributes to stone formation. The nurse should encourage the client to drink more fluids, not decrease calcium intake. Ingestion of aspirin or aspirin-containing products does not cause a stone. Antibiotics neither prevent nor treat a stone.

5. An emergency room nurse obtains the health history of a client. Which statement by the client should alert the nurse to the occurrence of heart failure? a. I get short of breath when I climb stairs. b. I see halos floating around my head. c. I have trouble remembering things. d. I have lost weight over the past month.

A Dyspnea on exertion is an early manifestation of heart failure and is associated with an activity such as stair climbing. The other findings are not specific to early occurrence of heart failure.

6. A nurse cares for adult clients who experience urge incontinence. For which client should the nurse plan a habit training program? a. A 78-year-old female who is confused b. A 65-year-old male with diabetes mellitus c. A 52-year-old female with kidney failure d. A 47-year-old male with arthritis

A For a bladder training program to succeed in a client with urge incontinence, the client must be alert, aware, and able to resist the urge to urinate. Habit training will work best for a confused client. This includes going to the bathroom (or being assisted to the bathroom) at set times. The other clients may benefit from another type of bladder training.

5. After teaching a client who has stress incontinence, the nurse assesses the clients understanding. Which statement made by the client indicates a need for additional teaching? a. I will limit my total intake of fluids. b. I must avoid drinking alcoholic beverages. c. I must avoid drinking caffeinated beverages. d. I shall try to lose about 10% of my body weight.

A Limiting fluids concentrates urine and can irritate tissues, leading to increased incontinence. Many people try to manage incontinence by limiting fluids. Alcoholic and caffeinated beverages are bladder stimulants. Obesity increases intra-abdominal pressure, causing incontinence.

12. A nurse assesses a client who is recovering from a myocardial infarction. The clients pulmonary artery pressure reading is 25/12 mm Hg. Which action should the nurse take first? a. Compare the results with previous pulmonary artery pressure readings. b. Increase the intravenous fluid rate because these readings are low. c. Immediately notify the health care provider of the elevated pressures. d. Document the finding in the clients chart as the only action.

A Normal pulmonary artery pressures range from 15 to 26 mm Hg for systolic and from 5 to 15 mm Hg for diastolic. Although this clients readings are within normal limits, the nurse needs to assess any trends that may indicate a need for medical treatment to prevent complications. There is no need to increase intravenous fluids or notify the provider.

16. A nurse obtains the health history of a client with a suspected diagnosis of bladder cancer. Which question should the nurse ask when determining this clients risk factors? a. Do you smoke cigarettes? b. Do you use any alcohol? c. Do you use recreational drugs? d. Do you take any prescription drugs?

A Smoking is known to be a factor that greatly increases the risk of bladder cancer. Alcohol use, recreational drug use, and prescription drug use (except medications that contain phenacetin) are not known to increase the risk of developing bladder cancer.

21. A nurse assesses a client who has aortic regurgitation. In which location in the illustration shown below should the nurse auscultate to best hear a cardiac murmur related to aortic regurgitation? a. Location A b. Location B c. Location C d. Location D

A The aortic valve is auscultated in the second intercostal space just to the right of the sternum.

18. A nurse teaches a young female client who is prescribed amoxicillin (Amoxil) for a urinary tract infection. Which statement should the nurse include in this clients teaching? a. Use a second form of birth control while on this medication. b. You will experience increased menstrual bleeding while on this drug. c. You may experience an irregular heartbeat while on this drug. d. Watch for blood in your urine while taking this medication.

A The client should use a second form of birth control because penicillin seems to reduce the effectiveness of estrogen-containing contraceptives. She should not experience increased menstrual bleeding, an irregular heartbeat, or blood in her urine while taking the medication.

23. A nurse assesses a client who presents with renal calculi. Which question should the nurse ask? a. Do any of your family members have this problem? b. Do you drink any cranberry juice? c. Do you urinate after sexual intercourse? d. Do you experience burning with urination?

A There is a strong association between family history and stone formation and recurrence. Nephrolithiasis is associated with many genetic variations; therefore, the nurse should ask whether other family members have also had renal stones. The other questions do not refer to renal calculi but instead are questions that should be asked of a client with a urinary tract infection.

1. A nurse assesses a client who had a myocardial infarction and is hypotensive. Which additional assessment finding should the nurse expect? a. Heart rate of 120 beats/min b. Cool, clammy skin c. Oxygen saturation of 90% d. Respiratory rate of 8 breaths/min

A When a client experiences hypotension, baroreceptors in the aortic arch sense a pressure decrease in the vessels. The parasympathetic system responds by lessening the inhibitory effect on the sinoatrial node. This results in an increase in heart rate and respiratory rate. This tachycardia is an early response and is seen even when blood pressure is not critically low. An increased heart rate and respiratory rate will compensate for the low blood pressure and maintain oxygen saturations and perfusion. The client may not be able to compensate for long, and decreased oxygenation and cool, clammy skin will occur later.

An emergency nurse cares for a client who is experiencing an acute adrenal crisis. Which action should the nurse take first? a. Obtain intravenous access. b. Administer hydrocortisone succinate (Solu-Cortef). c. Assess blood glucose. d. Administer insulin and dextrose.

A (All actions are appropriate for the client with adrenal crisis. However, therapy is given intravenously, so the priority is to establish IV access. Solu-Cortef is the drug of choice. Blood glucose is monitored hourly and treatment is provided as needed. Insulin and dextrose are used to treat any hyperkalemia.)

A nurse cares for a client with chronic hypercortisolism. Which action should the nurse take? a. Wash hands when entering the room. b. Keep the client in airborne isolation. c. Observe the client for signs of infection. d. Assess the clients daily chest x-ray.

A (Excess cortisol reduces the number of circulating lymphocytes, inhibits maturation of macrophages, reduces antibody synthesis, and inhibits production of cytokines and inflammatory chemicals. As a result, these clients are at greater risk of infection and may not have the expected inflammatory manifestations when an infection is present. The nurse needs to take precautions to decrease the clients risk. It is not necessary to keep the client in isolation. The client does not need a daily chest x-ray.)

A client with hyperaldosteronism is being treated with spironolactone (Aldactone) before surgery. Which precautions does the nurse teach this client? a. Read the label before using salt substitutes. b. Do not add salt to your food when you eat. c. Avoid exposure to sunlight. d. Take Tylenol instead of aspirin for pain.

A (Spironolactone is a potassium-sparing diuretic used to control potassium levels. Its use can lead to hyperkalemia. Although the goal is to increase the clients potassium, unknowingly adding potassium can cause complications. Some salt substitutes are composed of potassium chloride and should be avoided by clients on spironolactone therapy. Depending on the client, he or she may benefit from a low-sodium diet before surgery, but this may not be necessary. Avoiding sunlight and Tylenol is not necessary.)

An LPN/LVN is scheduled to work on the inpatient "stepdown" cardiac unit. Which client does the charge nurse assign to the LPN/LVN? A 60-year-old who was admitted today for pacemaker insertion because of third-degree heart block and who is now reporting chest pain A 62-year-old who underwent open heart surgery 4 days ago for mitral valve replacement and who has a temperature of 38.2° C A 66-year-old who has a prescription for a nitroglycerin (Nitro-Dur) patch and is scheduled for discharge to a group home later today A 69-year-old who had a stent placed 2 hours ago in the left anterior descending artery and who has bursts of ventricular tachycardia

A 66-year-old who has a prescription for a nitroglycerin (Nitro-Dur) patch and is scheduled for discharge to a group home later today The LPN/LVN scope of practice includes administration of medications to stable clients. Third-degree heart block is characterized by a very low heart rate and usually by required pacemaker insertion; the skills of the RN are needed to care for this client. Fever after surgery requires collaboration with the health care provider, which is more consistent with the role of the RN. The client with a recent stent placement and having bursts of ventricular tachycardia is unstable and is showing ventricular irritability; he will need medications and monitoring beyond the scope of practice of the LPN/LVN.

p. 818, Prioritization, Delegation, and Supervision The patient is a 52-year-old woman who has undergone an autologous stem cell transplantation for non-Hodgkin's lymphoma. She is recovering, and her white blood cell count is improving but still very low. She remains on neutropenic precautions. The LPN reports that the patient's heart rate, respiratory rate, temperature, and blood pressure are all elevated. 4. The student asks you whether a book still wrapped in shrink wrap just now brought in by a friend of the patient can be taken to the patient's room. How will you help the student know what to do in this situation?

A book in shrink wrap could have potential organisms on it. However, the shrink wrap is waterproof. Tell the student to cleanse the shrink wrap with an alcohol-based hand rub and then remove the shrink wrap before giving the book to the patient. Although the book and its jacket are not sterile, they most likely have been handled only by machines in the printing process and are unlikely to convey pathogenic microorganisms (or any other organisms) to the patient.

A nurse plans care for an older adult who is admitted to the hospital for pneumonia. The client has no known drug allergies and no significant health history. Which action should the nurse include in this client's plan of care? a. Initiate Airborne Precautions. b. Offer fluids every hour or two. c. Place an indwelling urinary catheter. d. Palpate the client's thyroid gland.

A normal age-related endocrine change is decreased antidiuretic hormone (ADH) production. This results in a more diluted urine output, which can lead to dehydration. If no contraindications are known, the nurse should offer (or delegate) the client something to drink at least every 2 hours. A client with simple pneumonia would not require Airborne Precautions. Indwelling urinary catheterization is not necessary for this client and would increase the client's risk for infection. The nurse should plan a toileting schedule and assist the client to the bathroom if needed. Palpating the client's thyroid gland is a part of a comprehensive examination but is not specifically related to this client.

13. Patients who have central nervous system lesions from stroke, multiple sclerosis, or parasacral spinal cord lesions may have which type for urinary incontinence? a. Detrusor hyperreflexia b. Mixed c. Stress d. Functional

a

The nurse is providing health teaching for a patient at risk for heart disease. Which factor is the most modifiable, controllable risk factor? a. Obesity b. Diabetes mellitus c. Ethnic background d. Family history of cardiovascular disease

a

28. An older adult male patient reports an acute problem with urine retention. The nurse advises the patient to seek medical attention because permanent kidney damage can occur in what time frame? a. In less than 6 hours b. In less than 48 hours c. within several weeks d. Within several years

b

A 70-year-old client has a complicated medical history, including chronic obstructive pulmonary disease. Which client statement indicates the need for further teaching about the disease? A) "I am here to receive the yearly pneumonia shot again." B) "I am here to get my yearly flu shot again." C) "I should avoid large gatherings during cold and flu season." D) "I should cough into my upper sleeve instead of my hand." (Chp. 31; elsevier resources)

A) "I am here to receive the yearly pneumonia shot again." (Chp. 31; elsevier resources)

The community health nurse is planning tuberculosis treatment for a client who is homeless and heroin-addicted. Which action will be most effective in ensuring that the client completes treatment? A) Arrange for a health care worker to watch the client take the medication. B) Give the client written instructions about how to take prescribed medications. C) Have the client repeat medication names and side effects. D) Instruct the client about the possible consequences of nonadherence. (Chp. 31; elsevier resources)

A) Arrange for a health care worker to watch the client take the medication. (Chp. 31; elsevier resources)

An older adult resident in a long-term-care facility becomes confused and agitated, telling the nurse, "Get out of here! You're going to kill me!" Which action will the nurse take first? A) Check the resident's oxygen saturation. B) Do a complete neurologic assessment. C) Give the prescribed PRN lorazepam (Ativan). D) Notify the resident's primary care provider. (Chp. 31; elsevier resources)

A) Check the resident's oxygen saturation. (Chp. 31; elsevier resources)

A client with tuberculosis (TB) who is homeless and has been living in shelters for the past 6 months asks the nurse why he must take so many medications. What information will the nurse provide in answering this question? Select all that apply. A) Combination drug therapy is effective in preventing transmission. B) Combination drug therapy is the most effective method of treating TB. C) Combination drug therapy will decrease the length of required treatment to 2 months. D) Multiple drug regimens destroy organisms as quickly as possible. E) The use of multiple drugs reduces the emergence of drug-resistant organisms. (Chp. 31; elsevier resources)

A) Combination drug therapy is effective in preventing transmission. B) Combination drug therapy is the most effective method of treating TB. D) Multiple drug regimens destroy organisms as quickly as possible. E) The use of multiple drugs reduces the emergence of drug-resistant organisms. (Chp. 31; elsevier resources)

A client is taking isoniazid, rifampin, pyrazinamide, and ethambutol for tuberculosis. The client calls to report visual changes, including blurred vision and reduced visual fields. Which medication may be causing these changes? A) Ethambutol B) Isoniazid C) Pyrazinamide D) Rifampin (Chp. 31; elsevier resources)

A) Ethambutol (Chp. 31; elsevier resources)

A client who has been homeless and has spent the past 6 months living in shelters has been diagnosed with confirmed tuberculosis (TB). Which medications does the nurse expect to be ordered for the client? A) Isoniazid (INH), rifampin (Rifadin), pyrazinamide (Zinamide), ethambutol (Myambutol) B) Metronidazole (Flagyl), acyclovir (Zovirax), flunisolide (AeroBid), rifampin (Rifadin) C) Prednisone (Prednisone), guaifenesin (Organidin), ketorolac (Toradol), pyrazinamide (Zinamide) D) Salmeterol (Serevent), cromolyn sodium (Intal), dexamethasone (Decadron), isoniazid (INH) (Chp. 31; elsevier resources)

A) Isoniazid (INH), rifampin (Rifadin), pyrazinamide (Zinamide), ethambutol (Myambutol) (Chp. 31; elsevier resources)

A client has recently been released from prison and has just tested positive for tuberculosis (TB). What teaching points does the community health nurse want to stress for this client regarding medications? Select all that apply. A) Not taking the medication could lead to an infection that is difficult to treat or to total drug resistance. B) The medications may cause nausea. The client should take them at bedtime. C) The client is generally not contagious after 2 to 3 consecutive weeks of treatment. D) These medications must be taken for 2 years. E) These medications may cause kidney failure. (Chp. 31; elsevier resources)

A) Not taking the medication could lead to an infection that is difficult to treat or to total drug resistance. B) The medications may cause nausea. The client should take them at bedtime. (Chp. 31; elsevier resources)

A client has been admitted to the hospital with suspected TB. What drugs should the nurse plan to teach the client about before discharge? Select all that apply. A) Rifampin (Rifadin); contact lenses can become stained orange B) Isoniazid (INH); report yellowing of the skin or darkened urine C) Pyrazinamide (PZA); maintain a fluid restriction of 1200 mL/day D) Ethambutol (Myambutol); report any changes in vision E) Amoxicillin (Amoxil); take this drug with food or milk (Chp. 31; p. 598)

A) Rifampin (Rifadin); contact lenses can become stained orange B) Isoniazid (INH); report yellowing of the skin or darkened urine D) Ethambutol (Myambutol); report any changes in vision (Chp. 31; p. 598)

A nurse is caring for an 89-year-old client admitted with pneumonia. He has an IV of normal saline running at 100 mL/hr and antibiotics that were initiated in the emergency department 3 hours ago. He has oxygen at 2 liters/nasal cannula. What assessment finding by the nurse indicates that goals for a priority diagnosis have been met for this client? A) The client is alert and oriented to person, place, and time. B) Blood pressure is within normal limits and client's baseline. C) Skin behind the ears demonstrates no redness or irritation. D) Urine output has been >30 mL/hr per Foley catheter. (Chp. 31; p. 593)

A) The client is alert and oriented to person, place, and time. (Chp. 31; p. 593)

A client is being admitted for pneumonia. The sputum culture is positive for streptococcus, and the client asks about the length of the treatment. On what does the nurse base the answer? A) The client will be treated for 5 to 7 days. B) The client will require IV antibiotics for 7 to 10 days. C) The client will complete 6 days of therapy. D) The client must be afebrile for 24 hours. (Chp. 31; elsevier resources)

A) The client will be treated for 5 to 7 days. (Chp. 31; elsevier resources)

1. A nurse is caring for a client with a history of renal insufficiency who is scheduled for a cardiac catheterization. Which actions should the nurse take prior to the catheterization? (Select all that apply.) a. Assess for allergies to iodine. b. Administer intravenous fluids. c. Assess blood urea nitrogen (BUN) and creatinine results. d. Insert a Foley catheter. e. Administer a prophylactic antibiotic. f. Insert a central venous catheter.

A, B, C If the client has kidney disease (as indicated by BUN and creatinine results), fluids and Mucomyst may be given 12 to 24 hours before the procedure for renal protection. The client should be assessed for allergies to iodine, including shellfish; the contrast medium used during the catheterization contains iodine. A Foley catheter and central venous catheter are not required for the procedure and would only increase the clients risk for infection. Prophylactic antibiotics are not administered prior to a cardiac catheterization.

7. A nurse teaches a client about self-care after experiencing a urinary calculus treated by lithotripsy. Which statements should the nurse include in this clients discharge teaching? (Select all that apply.) a. Finish the prescribed antibiotic even if you are feeling better. b. Drink at least 3 liters of fluid each day. c. The bruising on your back may take several weeks to resolve. d. Report any blood present in your urine. e. It is normal to experience pain and difficulty urinating.

A, B, C The client should be taught to finish the prescribed antibiotic to ensure that he or she does not get a urinary tract infection. The client should drink at least 3 liters of fluid daily to dilute potential stone-forming crystals, prevent dehydration, and promote urine flow. After lithotripsy, the client should expect bruising that may take several weeks to resolve. The client should also experience blood in the urine for several days. The client should report any pain, fever, chills, or difficulty with urination to the provider as these may signal the beginning of an infection or the formation of another stone.

1. A nurse assesses a client who has had two episodes of bacterial cystitis in the last 6 months. Which questions should the nurse ask? (Select all that apply.) a. How much water do you drink every day? b. Do you take estrogen replacement therapy? c. Does anyone in your family have a history of cystitis? d. Are you on steroids or other immune-suppressing drugs? e. Do you drink grapefruit juice or orange juice daily?

A, B, D Fluid intake, estrogen levels, and immune suppression all can increase the chance of recurrent cystitis. Family history is usually insignificant, and cranberry juice, not grapefruit or orange juice, has been found to increase the acidic pH and reduce the risk for bacterial cystitis.

5. A nurse cares for clients with urinary incontinence. Which types of incontinence are correctly paired with their clinical manifestation? (Select all that apply.) a. Stress incontinence Urine loss with physical exertion b. Urge incontinence Large amount of urine with each occurrence c. Functional incontinence Urine loss results from abnormal detrusor contractions d. Overflow incontinence Constant dribbling of urine e. Reflex incontinence Leakage of urine without lower urinary tract disorder

A, B, D Stress incontinence is a loss of urine with physical exertion, coughing, sneezing, or exercising. Urge incontinence presents with an abrupt and strong urge to void and usually has a large amount of urine released with each occurrence. Overflow incontinence occurs with bladder distention and results in a constant dribbling of urine. Functional incontinence is the leakage of urine caused by factors other than a disorder of the lower urinary tract. Reflex incontinence results from abnormal detrusor contractions from a neurologic abnormality.

4. A nurse assesses a client with hypothyroidism who is admitted with acute appendicitis. The nurse notes that the client's level of consciousness has decreased. Which actions should the nurse take? (Select all that apply.) a. Infuse intravenous fluids. b. Cover the client with warm blankets. c. Monitor blood pressure every 4 hours. d. Maintain a patent airway. e. Administer oral glucose as prescribed.

A, B, D. A client with hypothyroidism and an acute illness is at risk for myxedema coma. A decrease in level of consciousness is a symptom of myxedema. The nurse should infuse IV fluids, cover the client with warm blankets, monitor blood pressure every hour, maintain a patent airway, and administer glucose intravenously as prescribed.

6. A nurse teaches a client with a history of calcium phosphate urinary stones. Which statements should the nurse include in this clients dietary teaching? (Select all that apply.) a. Limit your intake of food high in animal protein. b. Read food labels to help minimize your sodium intake. c. Avoid spinach, black tea, and rhubarb. d. Drink white wine or beer instead of red wine. e. Reduce your intake of milk and other dairy products.

A, B, E Clients with calcium phosphate urinary stones should be taught to limit the intake of foods high in animal protein, sodium, and calcium. Clients with calcium oxalate stones should avoid spinach, black tea, and rhubarb. Clients with uric acid stones should avoid red wine.

8. A nurse teaches a female client who has stress incontinence. Which statements should the nurse include about pelvic muscle exercises? (Select all that apply.) a. When you start and stop your urine stream, you are using your pelvic muscles. b. Tighten your pelvic muscles for a slow count of 10 and then relax for a slow count of 10. c. Pelvic muscle exercises should only be performed sitting upright with your feet on the floor. d. After you have been doing these exercises for a couple days, your control of urine will improve. e. Like any other muscle in your body, you can make your pelvic muscles stronger by contracting them.

A, B, E The client should be taught that the muscles used to start and stop urination are pelvic muscles, and that pelvic muscles can be strengthened by contracting and relaxing them. The client should tighten pelvic muscles for a slow count of 10 and then relax the muscles for a slow count of 10, and perform this exercise 15 times while in lying-down, sitting-up, and standing positions. The client should begin to notice improvement in control of urine after several weeks of exercising the pelvic muscles.

A nurse assesses clients with potential endocrine disorders. Which clients are at high risk for hypopituitarism? (Select all that apply.) a. A 20-year-old female with benign pituitary tumors b. A 32-year-old male with diplopia c. A 41-year-old female with anorexia nervosa d. A 55-year-old male with hypertension e. A 60-year-old female who is experiencing shock f. A 68-year-old male who has gained weight recently

A, C, D, E (Pituitary tumors, anorexia nervosa, hypertension, and shock are all conditions that can cause hypopituitarism. Diplopia is a manifestation of hypopituitarism, and weight gain is a manifestation of Cushings disease and syndrome of inappropriate antidiuretic hormone. They are not risk factors for hypopituitarism.)

A nurse assesses a client with anterior pituitary hyperfunction. Which clinical manifestations should the nurse expect? (Select all that apply.) a. Protrusion of the lower jaw b. High-pitched voice c. Enlarged hands and feet d. Kyphosis e. Barrel-shaped chest f. Excessive sweating

A, C, D, E, F (Anterior pituitary hyperfunction typically will cause protrusion of the lower jaw, deepening of the voice, enlarged hands and feet, kyphosis, barrel-shaped chest, and excessive sweating.)

4. A nurse reviews a clients laboratory results. Which findings should alert the nurse to the possibility of atherosclerosis? (Select all that apply.) a. Total cholesterol: 280 mg/dL b. High-density lipoprotein cholesterol: 50 mg/dL c. Triglycerides: 200 mg/dL d. Serum albumin: 4 g/dL e. Low-density lipoprotein cholesterol: 160 mg/dL

A, C, E A lipid panel is often used to screen for cardiovascular risk. Total cholesterol, triglycerides, and low-density lipoprotein cholesterol levels are all high, indicating higher risk for cardiovascular disease. High-density lipoprotein cholesterol is within the normal range for both males and females. Serum albumin is not assessed for atherosclerosis.

6. A nurse cares for a client who is recovering from a right-sided heart catheterization. For which complications of this procedure should the nurse assess? (Select all that apply.) a. Thrombophlebitis b. Stroke c. Pulmonary embolism d. Myocardial infarction e. Cardiac tamponade

A, C, E Complications from a right-sided heart catheterization include thrombophlebitis, pulmonary embolism, and vagal response. Cardiac tamponade is a risk of both right- and left-sided heart catheterizations. Stroke and myocardial infarction are complications of left-sided heart catheterizations.

3. A nurse teaches a client with hyperthyroidism. Which dietary modifications should the nurse include in this client's teaching? (Select all that apply.) a. Increased carbohydrates b. Decreased fats c. Increased calorie intake d. Supplemental vitamins e. Increased proteins

A, C, E. The client is hypermetabolic and has an increased need for carbohydrates, calories, and proteins. Proteins are especially important because the client is at risk for a negative nitrogen balance. There is no need to decrease fat intake or take supplemental vitamins.

2. A nurse teaches a client about self-catheterization in the home setting. Which statements should the nurse include in this clients teaching? (Select all that apply.) a. Wash your hands before and after self-catheterization. b. Use a large-lumen catheter for each catheterization. c. Use lubricant on the tip of the catheter before insertion. d. Self-catheterize at least twice a day or every 12 hours. e. Use sterile gloves and sterile technique for the procedure. f. Maintain a specific schedule for catheterization.

A, C, F The key points in self-catheterization include washing hands, using lubricants, and maintaining a regular schedule to avoid distention and retention of urine that leads to bacterial growth. A smaller rather than a larger lumen catheter is preferred. The client needs to catheterize more often than every 12 hours. Self-catheterization in the home is a clean procedure.

4. Which hormone is released from the posterior pituitary and makes the distal convoluted tubule and the collecting duct permeable to water to maximize reabsorption and produce concentrated urine? a. Aldosterone b. Vasopressin c. ZBradykinins d. Natriuretic

b

A nurse assesses a client who potentially has hyperaldosteronism. Which serum laboratory values should the nurse associate with this disorder? (Select all that apply.) a. Sodium: 150 mEq/L b. Sodium: 130 mEq/L c. Potassium: 2.5 mEq/L d. Potassium: 5.0 mEq/L e. pH: 7.28 f. pH: 7.50

A, C, F (Aldosterone increases reabsorption of sodium and excretion of potassium. Hyperaldosteronism causes hypernatremia, hypokalemia, and metabolic alkalosis. Hyponatremia, hyperkalemia, and acidosis are manifestations of adrenal insufficiency.)

5. A nurse teaches a client who is prescribed an unsealed radioactive isotope. Which statements should the nurse include in this client's education? (Select all that apply.) a. "Do not share utensils, plates, and cups with anyone else." b. "You can play with your grandchildren for 1 hour each day." c. "Eat foods high in vitamins such as apples, pears, and oranges." d. "Wash your clothing separate from others in the household." e. "Take a laxative 2 days after therapy to excrete the radiation."

A, D, E. A client who is prescribed an unsealed radioactive isotope should be taught to not share utensils, plates, and cups with anyone else; to avoid contact with pregnant women and children; to avoid eating foods with cores or bones, which will leave contaminated remnants; to wash clothing separate from others in the household and run an empty cycle before washing other people's clothing; and to take a laxative on days 2 and 3 after receiving treatment to help excrete the contaminated stool faster.

After receiving change-of-shift report about these four clients, which client does the nurse attend to first? A. Client with acute adrenal insufficiency who has a blood glucose of 36 mg/dL B. Client with diabetes insipidus who has a dose of desmopressin (DDAVP) due C. Client with hyperaldosteronism who has a serum potassium of 3.4 mEq/L D. Client with pituitary adenoma who is reporting a severe headache

A. A glucose level of 36 mg/dL is considered an emergency; this client must be assessed and treated immediately. Although it is important to maintain medications on schedule, the client requiring a dose of desmopressin is not the first client who needs to be seen. A serum potassium of 3.4 mEq/L in the client with hyperaldosteronism may be considered normal (or slightly hypokalemic), based on specific hospital levels. The client reporting a severe headache needs to be evaluated as soon as possible after the client with acute adrenal insufficiency. As an initial measure, the RN could delegate obtaining vital signs to unlicensed assistive personnel.

A client with iatrogenic Cushing's syndrome is a resident in a long-term care facility. Which nursing action included in the client's care would be best to delegate to unlicensed assistive personnel (UAP)? A. Assist with personal hygiene and skin care. B. Develop a plan of care to minimize risk for infection. C. Instruct the client on the reasons to avoid overeating. D. Monitor for signs and symptoms of fluid retention.

A. Assisting a client with bathing and skin care is included in UAP scope of practice. It is not within their scope of practice to develop a plan of care, although they will play a very important role in following the plan of care. Client teaching requires a broad education and should not be delegated to UAP. Monitoring for signs and symptoms of fluid retention is part of client assessment, which requires a higher level of education and clinical judgment.

A client has been admitted to the medical intensive care unit with a diagnosis of diabetes insipidus (DI) secondary to lithium overdose. Which medication is used to treat the DI? A. Desmopressin (DDAVP) B. Dopamine hydrochloride (Intropin) C. Prednisone D. Tolvaptan (Samsca)

A. Desmopressin is the drug of choice for treatment of severe DI. It may be administered orally, nasally, or by intramuscular or intravenous routes. Dopamine hydrochloride is a naturally occurring catecholamine and inotropic vasopressor; it would not be used to treat DI. Prednisone would not be used to treat DI. Tolvaptan is a selective competitive arginine vasopressin receptor 2 antagonist and is not used with DI.

A client with diabetes insipidus (DI) has dry lips and mucous membranes and poor skin turgor. Which intervention does the nurse provide first? A. Force fluids B. Offer lip balm C. Perform a 24-hour urine test D. Withhold desmopressin acetate (DDAVP)

A. Dry lips and mucous membranes and poor skin turgor are indications of dehydration, which can occur with DI. This is a serious condition that must be treated rapidly. Encouraging fluids is the initial step, provided the client is able to tolerate oral intake. Lip balm may make the client more comfortable, but does not address the problem of dehydration. A 24-hour urine test will identify loss of electrolytes and adrenal androgen metabolites, but will not correct the dehydration that this client is experiencing. Desmopressin acetate is a synthetic form of antidiuretic hormone that is given to reduce urine production; it improves DI and should not be withheld.

A client with pheochromocytoma is admitted for surgery. What does the nurse do for the admitting assessment? A. Avoids palpating the abdomen B. Monitors for pulmonary edema with a chest x-ray C. Obtains a 24-hour urine specimen on admission D. Places the client in a room with a roommate for distraction

A. The abdomen must not be palpated in a client with pheochromocytoma because this action could cause a sudden release of catecholamines and severe hypertension. The tumor on the adrenal gland causes sympathetic hyperactivity, increasing blood pressure and heart rate, not pulmonary edema. A 24-hour urine collection will already have been completed to determine the diagnosis of pheochromocytoma. A client diagnosed with a pheochromocytoma may feel anxious as part of the disease process; providing a roommate for distraction will not reduce the client's anxiety.

The nurse is teaching a client about how to monitor therapy effectiveness for syndrome of inappropriate antidiuretic hormone. What does the nurse tell the client to look for? A. Daily weight gain of less than 2 pounds B. Dry mucous membranes C. Increasing heart rate D. Muscle spasms

A. The client must monitor daily weights because this assesses the degree of fluid restriction needed. A weight gain of 2 pounds or more daily or a gradual increase over several days is cause for concern. Dry mucous membranes are a sign of dehydration and an indication that therapy is not effective. An increased heart rate indicates increased fluid retention or dehydration and hypovolemia, and either condition is an indication that therapy is not effective. Muscle spasms are associated with hyponatremia and are an indication of a change in the client's neurologic status. Untreated hyponatremia can lead to seizures and coma.

A client is being discharged with propylthiouracil (PTU). Which statement by the client indicates a need for further teaching by the nurse? A. "I can return to my job at the nursing home." B. "I must call if my urine is dark." C. "I must faithfully take the drug every 8 hours." D. "I need to report weight gain."

A. The client should avoid large crowds and people who are ill because PTU reduces blood cell counts and the immune response, which increases the risk for infection. The client does not, however, need to remain completely at home. Dark urine may indicate liver toxicity or failure, and the client must notify the provider immediately. Taking PTU regularly at the same time each day provides better drug levels and ensures better drug action. The client must notify the provider of weight gain because this may indicate hypothyroidism; a lower drug dose may be required.

A client with syndrome of inappropriate antidiuretic hormone is admitted with a serum sodium level of 105 mEq/L. Which request by the health care provider does the nurse address first? A. Administer infusion of 150 mL of 3% NaCl over 3 hours. B. Draw blood for hemoglobin and hematocrit. C. Insert retention catheter and monitor urine output. D. Weigh the client on admission and daily thereafter.

A. The client with a sodium level of 105 mEq/L is at high risk for seizures and coma. The priority intervention is to increase the sodium level to a more normal range. Ideally, 3% NaCl should be infused through a central line or with a small needle through a large vein to prevent irritation. Monitoring laboratory values for fluid balance and monitoring urine output are important, but are not the top priority. Monitoring client weight will help in the assessment of fluid balance; however, this is also not the top priority.

The nurse manager for the medical-surgical unit is making staff assignments. Which client will be most appropriate to assign to a newly graduated RN who has completed a 6-week unit orientation? A. Client with chronic hypothyroidism and dementia who takes levothyroxine (Synthroid) daily B. Client with follicular thyroid cancer who has vocal hoarseness and difficulty swallowing C. Client with Graves' disease who is experiencing increasing anxiety and diaphoresis D. Client with hyperparathyroidism who has just arrived on the unit after a parathyroidectomy

A. The client with chronic hypothyroidism and dementia is the most stable of the clients described and would be most appropriate to assign to an inexperienced RN. A client with vocal hoarseness and difficulty swallowing is at higher risk for complications and requires close observation by a more experienced nurse. Increasing anxiety and diaphoresis in a client with Graves' disease can be an indication of impending thyroid storm, which is an emergency; this is not a situation to be managed by a newly graduated RN. A client who has just arrived on the unit after a parathyroidectomy requires close observation for bleeding and airway compromise and requires assessment by an experienced nurse.

A client presents to the emergency department with a history of adrenal insufficiency. The following laboratory values are obtained: Na+ 130 mEq/L, K+ 5.6 mEq/L, and glucose 72 mg/dL. Which is the first request that the nurse anticipates? A. Administer insulin and dextrose in normal saline to shift potassium into cells. B. Give spironolactone (Aldactone) 100 mg orally. C. Initiate histamine2 (H2) blocker therapy with ranitidine for ulcer prophylaxis. D. Obtain arterial blood gases to assess for peaked T waves.

A. This client is hyperkalemic. The nurse should anticipate a request to administer 20 to 50 units of insulin with 20 to 50 mg of dextrose in normal saline as an IV infusion to shift potassium into the cells. Spironolactone is a potassium-sparing diuretic that helps the body keep potassium, which the client does not need. Although H2 blocker therapy would be appropriate for this client, it is not the first priority. Arterial blood gases are not used to assess for peaked T waves associated with hyperkalemia; an electrocardiogram needs to be obtained instead.

13. While assessing a client with Graves' disease, the nurse notes that the client's temperature has risen 1° F. Which action should the nurse take first? a. Turn the lights down and shut the client's door. b. Call for an immediate electrocardiogram (ECG). c. Calculate the client's apical-radial pulse deficit. d. Administer a dose of acetaminophen (Tylenol).

A. A temperature increase of 1° F may indicate the development of thyroid storm, and the provider needs to be notified. But before notifying the provider, the nurse should take measures to reduce environmental stimuli that increase the risk of cardiac complications. The nurse can then call for an ECG. The apical-radial pulse deficit would not be necessary, and Tylenol is not needed because the temperature increase is due to thyroid activity.

The nurse is preparing the room for the client returning from a thyroidectomy. Which items are important for the nurse to have available for this client? (Select all that apply.) A. Calcium gluconate B. Emergency tracheotomy kit C. Furosemide (Lasix) D. Hypertonic saline E. Oxygen F. Suction

A. B. E. F. Calcium gluconate should be available at the bedside to treat hypocalcemia and tetany that might occur if the parathyroid glands have been injured during the surgery. Equipment for an emergency tracheotomy must be kept at the bedside in the event that hemorrhage or edema should occlude the airway. Respiratory distress can result from swelling or damage to the laryngeal nerve leading to spasm, so it is important that the nurse work with respiratory therapy to have oxygen ready at the bedside for the client on admission. Because of the potential for increased secretions, it is important that a working suction device is present at the bedside for admission of the client from the operating room. Furosemide might be useful in the postoperative client to assist with urine output; however, this is not of added importance for this client. Hypertonic saline would not be of benefit to this client as the client is not hyponatremic.

1. An emergency department nurse cares for a client who is severely dehydrated and is prescribed 3 L of intravenous fluid over 6 hours. At what rate (mL/hr) should the nurse set the intravenous pump to infuse the fluids? (Record your answer using a whole number.) ____ mL/hr

ANS: 500 mL/hr Because IV pumps deliver in units of milliliters per hour, the pump would have to be set at 500 mL/hr to deliver 3 L (3000 mL) over 6 hours. 6x = 3000 x = 500

1. A client in the intensive care unit with acute kidney injury (AKI) must maintain a mean arterial pressure (MAP) of 65 mm Hg to promote kidney perfusion. What is the client's MAP if the blood pressure is 98/50 mm Hg? (Record your answer using a whole number.) _____ mm Hg

ANS: 66 mm Hg

1. A 242-pound client is being mechanically ventilated. To prevent lung injury, what setting should the nurse anticipate for tidal volume? (Record your answer using a whole number.) ___ mL

ANS: 660 mL A low tidal volume of 6 mL/kg is used to prevent lung injury. 242 pounds = 110 kg. 110 kg × 6 mL/kg = 660 mL.

A nurse prepares to administer prescribed regular and NPH insulin. Place the nurse's actions in the correct order to administer these medications. 1. Inspect bottles for expiration dates. 2. Gently roll the bottle of NPH between the hands. 3. Wash your hands. 4. Inject air into the regular insulin. 5. Withdraw the NPH insulin. 6. Withdraw the regular insulin. 7. Inject air into the NPH bottle. 8. Clean rubber stoppers with an alcohol swab.

ANS: 3, 1, 2, 8, 7, 4, 6, 5 After washing hands, it is important to inspect the bottles and then to roll the NPH to mix the insulin. Rubber stoppers should be cleaned with alcohol after rolling the NPH and before sticking a needle into either bottle. It is important to inject air into the NPH bottle before placing the needle in a regular insulin bottle to avoid mixing of regular and NPH insulin. The shorter-acting insulin is always drawn up first.

A preoperative nurse assesses a client who has type 1 diabetes mellitus prior to a surgical procedure. The client's blood glucose level is 160 mg/dL. Which action should the nurse take? a. Document the finding in the client's chart. b. Administer a bolus of regular insulin IV. c. Call the surgeon to cancel the procedure. d. Draw blood gases to assess the metabolic state.

ANS: A Clients who have type 1 diabetes and are having surgery have been found to have fewer complications, lower rates of infection, and better wound healing if blood glucose levels are maintained at between 140 and 180 mg/dL throughout the perioperative period. The nurse should document the finding and proceed with other operative care. The need for a bolus of insulin, canceling the procedure, or drawing arterial blood gases is not required.

21. A client is on a dopamine infusion via a peripheral line. What action by the nurse takes priority for safety? a. Assess the IV site hourly. b. Monitor the pedal pulses. c. Monitor the client's vital signs. d. Obtain consent for a central line.

ANS: A Dopamine should be infused through a central line to prevent extravasation and necrosis of tissue. If it needs to be run peripherally, the nurse assesses the site hourly for problems. When the client is getting the central line, ensuring informed consent is on the chart is a priority. But at this point, the client has only a peripheral line, so caution must be taken to preserve the integrity of the client's integumentary system. Monitoring pedal pulses and vital signs give indications as to how well the drug is working. DIF: Applying/Application REF: 773 KEY: Inotropic agents| adverse effects| medication safety MSC: IntegratedProcess:NursingProcess:Assessment NOT: Client Needs Category: Physiological Integrity: Pharmacological and Parenteral Therapies

A nurse teaches a client who is diagnosed with diabetes mellitus. Which statement should the nurse include in this client's plan of care to delay the onset of microvascular and macrovascular complications? a. "Maintain tight glycemic control and prevent hyperglycemia." b. "Restrict your fluid intake to no more than 2 liters a day." c. "Prevent hypoglycemia by eating a bedtime snack." d. "Limit your intake of protein to prevent ketoacidosis."

ANS: A Hyperglycemia is a critical factor in the pathogenesis of long-term diabetic complications. Maintaining tight glycemic control will help delay the onset of complications. Restricting fluid intake is not part of the treatment plan for clients with diabetes. Preventing hypoglycemia and ketosis, although important, are not as important as maintaining daily glycemic control.

After teaching a client who is recovering from pancreas transplantation, the nurse assesses the client's understanding. Which statement made by the client indicates a need for additional education? a. "If I develop an infection, I should stop taking my corticosteroid." b. "If I have pain over the transplant site, I will call the surgeon immediately." c. "I should avoid people who are ill or who have an infection." d. "I should take my cyclosporine exactly the way I was taught."

ANS: A Immunosuppressive agents should not be stopped without the consultation of the transplantation physician, even if an infection is present. Stopping immunosuppressive therapy endangers the transplanted organ. The other statements are correct. Pain over the graft site may indicate rejection. Anti-rejection drugs cause immunosuppression, and the client should avoid crowds and people who are ill. Changing the routine of anti-rejection medications may cause them to not work optimally.

A nurse prepares to administer insulin to a client at 1800. The client's medication administration record contains the following information: • Insulin glargine: 12 units daily at 1800 • Regular insulin: 6 units QID at 0600, 1200, 1800, 2400 Based on the client's medication administration record, which action should the nurse take? a. Draw up and inject the insulin glargine first, and then draw up and inject the regular insulin. b. Draw up and inject the insulin glargine first, wait 20 minutes, and then draw up and inject the regular insulin. c. First draw up the dose of regular insulin, then draw up the dose of insulin glargine in the same syringe, mix, and inject the two insulins together. d. First draw up the dose of insulin glargine, then draw up the dose of regular insulin in the same syringe, mix, and inject the two insulins together.

ANS: A Insulin glargine must not be diluted or mixed with any other insulin or solution. Mixing results in an unpredictable alteration in the onset of action and time to peak action. The correct instruction is to draw up and inject first the glargine and then the regular insulin right afterward.

A nurse cares for a client with diabetes mellitus who is visually impaired. The client asks, "Can I ask my niece to prefill my syringes and then store them for later use when I need them?" How should the nurse respond? a. "Yes. Prefilled syringes can be stored for 3 weeks in the refrigerator in a vertical position with the needle pointing up." b. "Yes. Syringes can be filled with insulin and stored for a month in a location that is protected from light." c. "Insulin reacts with plastic, so prefilled syringes are okay, but you will need to use glass syringes." d. "No. Insulin syringes cannot be prefilled and stored for any length of time outside of the container."

ANS: A Insulin is relatively stable when stored in a cool, dry place away from light. When refrigerated, prefilled plastic syringes are stable for up to 3 weeks. They should be stored in the refrigerator in the vertical position with the needle pointing up to prevent suspended insulin particles from clogging the needle.

An emergency department nurse assesses a client with ketoacidosis. Which clinical manifestation should the nurse correlate with this condition? a. Increased rate and depth of respiration b. Extremity tremors followed by seizure activity c. Oral temperature of 102° F (38.9° C) d. Severe orthostatic hypotension

ANS: A Ketoacidosis decreases the pH of the blood, stimulating the respiratory control areas of the brain to buffer the effects of increasing acidosis. The rate and depth of respiration are increased (Kussmaul respirations) in an attempt to excrete more acids by exhalation. Tremors, elevated temperature, and orthostatic hypotension are not associated with ketoacidosis.

A nurse cares for a client who has a family history of diabetes mellitus. The client states, "My father has type 1 diabetes mellitus. Will I develop this disease as well?" How should the nurse respond? a. "Your risk of diabetes is higher than the general population, but it may not occur." b. "No genetic risk is associated with the development of type 1 diabetes mellitus." c. "The risk for becoming a diabetic is 50% because of how it is inherited." d. "Female children do not inherit diabetes mellitus, but male children will."

ANS: A Risk for type 1 diabetes is determined by inheritance of genes coding for HLA-DR and HLA-DQ tissue types. Clients who have one parent with type 1 diabetes are at increased risk for its development. Diabetes (type 1) seems to require interaction between inherited risk and environmental factors, so not everyone with these genes develops diabetes. The other statements are not accurate.

15. A nurse cares for a client with a 40-year smoking history who is experiencing distended neck veins and dependent edema. Which physiologic process should the nurse correlate with this client's history and clinical manifestations? a. Increased pulmonary pressure creating a higher workload on the right side of the heart b. Exposure to irritants resulting in increased inflammation of the bronchi and bronchioles c. Increased number and size of mucus glands producing large amounts of thick mucus d. Left ventricular hypertrophy creating a decrease in cardiac output

ANS: A Smoking increases pulmonary hypertension, resulting in cor pulmonale, or right-sided heart failure. Increased pressures in the lungs make it more difficult for blood to flow through the lungs. Blood backs up into the right side of the heart and then into the peripheral venous system, creating distended neck veins and dependent edema. Inflammation in bronchi and bronchioles creates an airway obstruction which manifests as wheezes. Thick mucus in the lungs has no impact on distended neck veins and edema. Left ventricular hypertrophy is associated with left heart failure and is not caused by a 40-year smoking history. DIF: Remembering/Knowledge REF: 58 KEY: Heart failure| cor pulmonale MSC: Integrated Process: Nursing Process: Implementation NOT: Client Needs Category: Physiological Integrity: Physiological Adaptation

After teaching a client with diabetes mellitus to inject insulin, the nurse assesses the client's understanding. Which statement made by the client indicates a need for additional teaching? a. "The lower abdomen is the best location because it is closest to the pancreas." b. "I can reach my thigh the best, so I will use the different areas of my thighs." c. "By rotating the sites in one area, my chance of having a reaction is decreased." d. "Changing injection sites from the thigh to the arm will change absorption rates."

ANS: A The abdominal site has the fastest rate of absorption because of blood vessels in the area, not because of its proximity to the pancreas. The other statements are accurate assessments of insulin administration.

A nurse assesses a client with diabetes mellitus and notes the client only responds to a sternal rub by moaning, has capillary blood glucose of 33 g/dL, and has an intravenous line that is infiltrated with 0.45% normal saline. Which action should the nurse take first? a. Administer 1 mg of intramuscular glucagon. b. Encourage the client to drink orange juice. c. Insert a new intravenous access line. d. Administer 25 mL dextrose 50% (D50) IV push.

ANS: A The client's blood glucose level is dangerously low. The nurse needs to administer glucagon IM immediately to increase the client's blood glucose level. The nurse should insert a new IV after administering the glucagon and can use the new IV site for future doses of D50 if the client's blood glucose level does not rise. Once the client is awake, orange juice may be administered orally along with a form of protein such as a peanut butter.

A nurse assesses a client who has diabetes mellitus and notes the client is awake and alert, but shaky, diaphoretic, and weak. Five minutes after administering a half-cup of orange juice, the client's clinical manifestations have not changed. Which action should the nurse take next? a. Administer another half-cup of orange juice. b. Administer a half-ampule of dextrose 50% intravenously. c. Administer 10 units of regular insulin subcutaneously. d. Administer 1 mg of glucagon intramuscularly.

ANS: A This client is experiencing mild hypoglycemia. For mild hypoglycemic manifestations, the nurse should administer oral glucose in the form of orange juice. If the symptoms do not resolve immediately, the treatment should be repeated. The client does not need intravenous dextrose, insulin, or glucagon.

24. A nurse auscultates a client's lung fields. Which pathophysiologic process should the nurse associate with this breath sound? (Click the media button to hear the audio clip.) a. Inflammation of the pleura b. Constriction of the bronchioles c. Upper airway obstruction d. Pulmonary vascular edema

ANS: A A pleural friction rub can be heard when the pleura is inflamed and rubbing against the lung wall. The other pathophysiologic processes would not cause a pleural friction rub. Constriction of the bronchioles may be heard as a wheeze, upper airway obstruction may be heard as stridor, and pulmonary vascular edema may be heard as crackles. DIF: Applying/Application REF: 575 KEY: Assessment/diagnostic examination MSC: Integrated Process: Nursing Process: Implementation NOT: Client Needs Category: Physiological Integrity: Physiological Adaptation

20. A client has been diagnosed with a very large pulmonary embolism (PE) and has a dropping blood pressure. What medication should the nurse anticipate the client will need as the priority? a. Alteplase (Activase) b. Enoxaparin (Lovenox) c. Unfractionated heparin d. Warfarin sodium (Coumadin)

ANS: A Activase is a "clot-busting" agent indicated in large PEs in the setting of hemodynamic instability. The nurse knows this drug is the priority, although heparin may be started initially. Enoxaparin and warfarin are not indicated in this setting.

A nurse cares for a client with a deficiency of aldosterone. Which assessment finding should the nurse correlate with this deficiency? a. Increased urine output b. Vasoconstriction c. Blood glucose of 98 mg/dL d. Serum sodium of 144 mEq/L

ANS: A Aldosterone, the major mineralocorticoid, maintains extracellular fluid volume. It promotes sodium and water reabsorption and potassium excretion in the kidney tubules. A client with an aldosterone deficiency will have increased urine output. Vasoconstriction is not related. These sodium and glucose levels are normal; in aldosterone deficiency, the client would have hyponatremia and hyperkalemia.

21. A nurse admits a client from the emergency department. Client data are listed below: History Physical Assessment Laboratory Values: 70 years of age History of diabetes On insulin twice a day Reports new-onset dyspnea and productive cough Crackles and rhonchi heard throughout the lungs Dullness to percussion LLL Afebrile Oriented to person only WBC: 5,200/mm3 PaO2 on room air 65mm Hg What action by the nurse is the priority? a. Administer oxygen at 4 liters per nasal cannula. b. Begin broad-spectrum antibiotics. c. Collect a sputum sample for culture. d. Start an IV of normal saline at 50 mL/hr.

ANS: A All actions are appropriate for this client who has manifestations of pneumonia. However, airway and breathing come first, so begin oxygen administration and titrate it to maintain saturations greater than 95%. Start the IV and collect a sputum culture, and then begin antibiotics.

21. A nurse admits a client from the emergency department. Client data are listed below: History Physical Assessment Laboratory Values: 70 years of age History of diabetes On insulin twice a day Reports new-onset dyspnea and productive cough Crackles and rhonchi heard throughout the lungs Dullness to percussion LLL Afebrile Oriented to person only WBC: 5,200/mm3 PaO2 on room air 65mm Hg What action by the nurse is the priority? a. Administer oxygen at 4 liters per nasal cannula. b. Begin broad-spectrum antibiotics. c. Collect a sputum sample for culture. d. Start an IV of normal saline at 50 mL/hr.

ANS: A All actions are appropriate for this client who has manifestations of pneumonia. However, airway and breathing come first, so begin oxygen administration and titrate it to maintain saturations greater than 95%. Start the IV and collect a sputum culture, and then begin antibiotics.

19. A client in the emergency department has several broken ribs. What care measure will best promote comfort? a. Allowing the client to choose the position in bed b. Humidifying the supplemental oxygen c. Offering frequent, small drinks of water d. Providing warmed blankets

ANS: A Allow the client with respiratory problems to assume a position of comfort if it does not interfere with care. Often the client will choose a more upright position, which also improves oxygenation. The other options are less effective comfort measures.

18. A nurse is caring for a client who is intubated and has an intra-aortic balloon pump. The client is restless and agitated. What action should the nurse perform first for comfort? a. Allow family members to remain at the bedside. b. Ask the family if the client would like a fan in the room. c. Keep the television tuned to the client's favorite channel. d. Speak loudly to the client in case of hearing problems.

ANS: A Allowing the family to remain at the bedside can help calm the client with familiar voices (and faces if the client wakes up). A fan might be helpful but may also spread germs through air movement. The TV should not be kept on all the time to allow for rest. Speaking loudly may agitate the client more. DIF: Applying/Application REF: 780 KEY: Intra-aortic balloon pump| nonpharmacologic comfort measures MSC: Integrated Process: Nursing Process: Implementation NOT: Client Needs Category: Physiological Integrity: Basic Care and Comfort

13. A nurse cares for a client admitted from a nursing home after several recent falls. What prescription should the nurse complete first? a. Obtain urine sample for culture and sensitivity. b. Administer intravenous antibiotics. c. Encourage protein intake and additional fluids. d. Consult physical therapy for gait training.

ANS: A Although all interventions are or might be important, obtaining a urine sample for urinalysis takes priority. Often urinary tract infection (UTI) symptoms in older adults are atypical, and a UTI may present with new onset of confusion or falling. The urine sample should be obtained before starting antibiotics. Dietary requirements and gait training should be implemented after obtaining the urine sample.

1. A nurse assesses clients on a cardiac unit. Which client should the nurse identify as being at greatest risk for the development of left-sided heart failure? a. A 36-year-old woman with aortic stenosis b. A 42-year-old man with pulmonary hypertension c. A 59-year-old woman who smokes cigarettes daily d. A 70-year-old man who had a cerebral vascular accident

ANS: A Although most people with heart failure will have failure that progresses from left to right, it is possible to have left-sided failure alone for a short period. It is also possible to have heart failure that progresses from right to left. Causes of left ventricular failure include mitral or aortic valve disease, coronary artery disease, and hypertension. Pulmonary hypertension and chronic cigarette smoking are risk factors for right ventricular failure. A cerebral vascular accident does not increase the risk of heart failure. DIF: Applying/Application REF: 679 KEY: Heart failure| health screening MSC: IntegratedProcess:NursingProcess:Assessment NOT: Client Needs Category: Safe and Effective Care Environment: Management of Care

6. A nurse cares for a client with an increased blood urea nitrogen (BUN)/creatinine ratio. Which action should the nurse take first? a. Assess the client's dietary habits. b. Inquire about the use of nonsteroidal anti-inflammatory drugs (NSAIDs). c. Hold the client's metformin (Glucophage). d. Contact the health care provider immediately.

ANS: A An elevated BUN/creatinine ratio is often indicative of dehydration, urinary obstruction, catabolism, or a high-protein diet. The nurse should inquire about the client's dietary habits. Kidney damage related to NSAID use most likely would manifest with elevations in both BUN and creatinine, but no change in the ratio. The nurse should obtain more assessment data before holding any medications or contacting the provider.

6. A nurse is teaching a client with heart failure who has been prescribed enalapril (Vasotec). Which statement should the nurse include in this client's teaching? a. "Avoid using salt substitutes." b. "Take your medication with food." c. "Avoid using aspirin-containing products." d. "Check your pulse daily."

ANS: A Angiotensin-converting enzyme (ACE) inhibitors such as enalapril inhibit the excretion of potassium. Hyperkalemia can be a life-threatening side effect, and clients should be taught to limit potassium intake. Salt substitutes are composed of potassium chloride. ACE inhibitors do not need to be taken with food and have no impact on the client's pulse rate. Aspirin is often prescribed in conjunction with ACE inhibitors and is not contraindicated. DIF: Applying/Application REF: 685 KEY: Heart failure| angiotensin-converting enzyme (ACE) inhibitor| medication| patient education MSC: IntegratedProcess:Teaching/Learning NOT: Client Needs Category: Physiological Integrity: Pharmacological and Parenteral Therapies

A nurse assesses a client who is prescribed a medication that stimulates beta1 receptors. Which assessment finding should alert the nurse to urgently contact the health care provider? a. Heart rate of 50 beats/min b. Respiratory rate of 18 breaths/min c. Oxygenation saturation of 92% d. Blood pressure of 144/69 mm Hg

ANS: A Stimulation of beta1 receptor sites in the heart has positive chronotropic and inotropic actions. The nurse expects an increase in heart rate and increased cardiac output. The client with a heart rate of 50 beats/min would be cause for concern because this would indicate that the client was not responding to the medication. The other vital signs are within normal limits and do not indicate a negative response to the medication.

11. A nurse admits a client who is experiencing an exacerbation of heart failure. Which action should the nurse take first? a. Assess the client's respiratory status. b. Draw blood to assess the client's serum electrolytes. c. Administer intravenous furosemide (Lasix). d. Ask the client about current medications.

ANS: A Assessment of respiratory and oxygenation status is the priority nursing intervention for the prevention of complications. Monitoring electrolytes, administering diuretics, and asking about current medications are important but do not take priority over assessing respiratory status. DIF: Applying/Application REF: 687 KEY: Heart failure| respiratory distress/failure| assessment/diagnostic examination MSC: Integrated Process: Nursing Process: Implementation NOT: Client Needs Category: Safe and Effective Care Environment: Management of Care

14. A nurse assesses a client with tachycardia. Which clinical manifestation requires immediate intervention by the nurse? a. Mid-sternal chest pain b. Increased urine output c. Mild orthostatic hypotension d. P wave touching the T wave

ANS: A Chest pain, possibly angina, indicates that tachycardia may be increasing the client's myocardial workload and oxygen demand to such an extent that normal oxygen delivery cannot keep pace. This results in myocardial hypoxia and pain. Increased urinary output and mild orthostatic hypotension are not life-threatening conditions and therefore do not require immediate intervention. The P wave touching the T wave indicates significant tachycardia and should be assessed to determine the underlying rhythm and cause; this is an important assessment but is not as critical as chest pain, which indicates cardiac cell death. DIF: Applying/Application REF: 663 KEY: Cardiac electrical conduction MSC: IntegratedProcess:NursingProcess:Assessment NOT: Client Needs Category: Safe and Effective Care Environment: Management of Care

4. A nursing student is caring for a client who had a myocardial infarction. The student is confused because the client states nothing is wrong and yet listens attentively while the student provides education on lifestyle changes and healthy menu choices. What response by the faculty member is best? a. "Continue to educate the client on possible healthy changes." b. "Emphasize complications that can occur with noncompliance." c. "Tell the client that denial is normal and will soon go away." d. "You need to make sure the client understands this illness."

ANS: A Clients are often in denial after a coronary event. The client who seems to be in denial but is compliant with treatment may be using a healthy form of coping that allows time to process the event and start to use problem-focused coping. The student should not discourage this type of denial and coping, but rather continue providing education in a positive manner. Emphasizing complications may make the client defensive and more anxious. Telling the client that denial is normal is placing too much attention on the process. Forcing the client to verbalize understanding of the illness is also potentially threatening to the client. DIF: Understanding/Comprehension REF: 769 KEY: Coronary artery disease| psychosocial response| coping| therapeutic communication MSC: Integrated Process: Communication and Documentation NOT: Client Needs Category: Psychosocial Integrity

26. A client had an inferior wall myocardial infarction (MI). The nurse notes the client's cardiac rhythm as shown below: What action by the nurse is most important? a. Assess the client's blood pressure and level of consciousness. b. Call the health care provider or the Rapid Response Team. c. Obtain a permit for an emergency temporary pacemaker insertion. d. Prepare to administer antidysrhythmic medication.

ANS: A Clients with an inferior wall MI often have bradycardia and blocks that lead to decreased perfusion, as seen in this ECG strip showing sinus bradycardia. The nurse should first assess the client's hemodynamic status, including vital signs and level of consciousness. The client may or may not need the Rapid Response Team, a temporary pacemaker, or medication; there is no indication of this in the question. DIF: Analyzing/Analysis REF: 769 KEY: Coronary artery disease| dysrhythmias| nursing assessment| hemodynamic status MSC: Integrated Process: Nursing Process: Implementation NOT: Client Needs Category: Physiological Integrity: Physiological Adaptation

12. A nurse reviews the allergy list of a client who is scheduled for an intravenous urography. Which client allergy should alert the nurse to urgently contact the health care provider? a. Seafood b. Penicillin c. Bee stings d. Red food dye

ANS: A Clients with seafood allergies often have severe allergic reactions to the standard dyes used during intravenous urography. The other allergies have no impact on the client's safety during an intravenous urography.

9. A client is in the clinic a month after having a myocardial infarction. The client reports sleeping well since moving into the guest bedroom. What response by the nurse is best? a. "Do you have any concerns about sexuality?" b. "I'm glad to hear you are sleeping well now." c. "Sleep near your spouse in case of emergency." d. "Why would you move into the guest room?"

ANS: A Concerns about resuming sexual activity are common after cardiac events. The nurse should gently inquire if this is the issue. While it is good that the client is sleeping well, the nurse should investigate the reason for the move. The other two responses are likely to cause the client to be defensive. DIF: Applying/Application REF: 781 KEY: Coronary artery disease| sexuality| anxiety| therapeutic communication MSC: IntegratedProcess:Caring NOT: Client Needs Category: Psychosocial Integrity

64. As a patient with ESKD experiences isosthenuria, what must the nurse be alert for? a. The diuretic stage b. Fluid volume overload c. Dehydration d. Alkalosis

b

5. A nurse cares for a client with right-sided heart failure. The client asks, "Why do I need to weigh myself every day?" How should the nurse respond? a. "Weight is the best indication that you are gaining or losing fluid." b. "Daily weights will help us make sure that you're eating properly." c. "The hospital requires that all inpatients be weighed daily." d. "You need to lose weight to decrease the incidence of heart failure."

ANS: A Daily weights are needed to document fluid retention or fluid loss. One liter of fluid equals 2.2 pounds. The other responses do not address the importance of monitoring fluid retention or loss. DIF: Remembering/Knowledge REF: 683 KEY: Heart failure| patient education MSC: Integrated Process: Teaching/Learning NOT: Client Needs Category: Physiological Integrity: Physiological Adaptation

10. After teaching a client with a history of renal calculi, the nurse assesses the client's understanding. Which statement made by the client indicates a correct understanding of the teaching? a. "I should drink at least 3 liters of fluid every day." b. "I will eliminate all dairy or sources of calcium from my diet." c. "Aspirin and aspirin-containing products can lead to stones." d. "The doctor can give me antibiotics at the first sign of a stone."

ANS: A Dehydration contributes to the precipitation of minerals to form a stone. Although increased intake of calcium causes hypercalcemia and leads to excessive calcium filtered into the urine, if the client is well hydrated the calcium will be excreted without issues. Dehydration increases the risk for supersaturation of calcium in the urine, which contributes to stone formation. The nurse should encourage the client to drink more fluids, not decrease calcium intake. Ingestion of aspirin or aspirin-containing products does not cause a stone. Antibiotics neither prevent nor treat a stone.

21. A nurse cares for an older adult client with heart failure. The client states, "I don't know what to do. I don't want to be a burden to my daughter, but I can't do it alone. Maybe I should die." How should the nurse respond? a. "Would you like to talk more about this?" b. "You are lucky to have such a devoted daughter." c. "It is normal to feel as though you are a burden." d. "Would you like to meet with the chaplain?"

ANS: A Depression can occur in clients with heart failure, especially older adults. Having the client talk about his or her feelings will help the nurse focus on the actual problem. Open-ended statements allow the client to respond safely and honestly. The other options minimize the client's concerns and do not allow the nurse to obtain more information to provide client-centered care. DIF: Applying/Application REF: 683 KEY: Heart failure| support| psychosocial response MSC: IntegratedProcess:Caring NOT: Client Needs Category: Psychosocial Integrity

11. While assessing a client who has facial trauma, the nurse auscultates stridor. The client is anxious and restless. Which action should the nurse take first? a. Contact the provider and prepare for intubation. b. Administer prescribed albuterol nebulizer therapy. c. Place the client in high-Fowler's position. d. Ask the client to perform deep-breathing exercises.

ANS: A Facial and neck tissue edema can occur in clients with facial trauma. Airway patency is the highest priority. Clients who experience stridor and hypoxia, manifested by anxiety and restlessness, should be immediately intubated to ensure airway patency. Albuterol decreases bronchi and bronchiole inflammation, not facial and neck edema. Although putting the client in high-Fowler's position and asking the client to perform breathing exercises may temporarily improve the client's comfort, these actions will not decrease the underlying problem or improve airway patency. DIF: Applying/Application REF: 534 KEY: Trauma MSC: Integrated Process: Nursing Process: Implementation NOT: Client Needs Category: Physiological Integrity: Physiological Adaptation

16. The nurse is teaching a client with chronic kidney disease (CKD) about the sodium restriction needed in the diet to prevent edema and hypertension. Which statement by the client indicates more teaching is needed? a. "I am thrilled that I can continue to eat fast food." b. "I will cut out bacon with my eggs every morning." c. "My cooking style will change by not adding salt." d. "I will probably lose weight by cutting out potato chips."

ANS: A Fast food restaurants usually serve food that is high in sodium. This statement indicates that more teaching needs to occur. The other statements show a correct understanding of the teaching.

5. A nurse evaluates a client with acute glomerulonephritis (GN). Which manifestation should the nurse recognize as a positive response to the prescribed treatment? a. The client has lost 11 pounds in the past 10 days. b. The client's urine specific gravity is 1.048. c. No blood is observed in the client's urine. d. The client's blood pressure is 152/88 mm Hg.

ANS: A Fluid retention is a major feature of acute GN. This weight loss represents fluid loss, indicating that the glomeruli are performing the function of filtration. A urine specific gravity of 1.048 is high. Blood is not usually seen in GN, so this finding would be expected. A blood pressure of 152/88 mm Hg is too high; this may indicate kidney damage or fluid overload.

6. A nurse cares for adult clients who experience urge incontinence. For which client should the nurse plan a habit training program? a. A 78-year-old female who is confused b. A 65-year-old male with diabetes mellitus c. A 52-year-old female with kidney failure d. A 47-year-old male with arthritis

ANS: A For a bladder training program to succeed in a client with urge incontinence, the client must be alert, aware, and able to resist the urge to urinate. Habit training will work best for a confused client. This includes going to the bathroom (or being assisted to the bathroom) at set times. The other clients may benefit from another type of bladder training.

A nurse cares for a client who is prescribed a 24-hour urine collection. The unlicensed assistive personnel (UAP) reports that, while pouring urine into the collection container, some urine splashed his hand. Which action should the nurse take next? a. Ask the UAP if he washed his hands afterward. b. Have the UAP fill out an incident report. c. Ask the laboratory if the container has preservative in it. d. Send the UAP to Employee Health right away.

ANS: A For safety, the nurse should find out if the UAP washed his or her hands. The UAP should do this for two reasons. First, it is part of Standard Precautions to wash hands after client care. Second, if the container did have preservative in it, this would wash it away. The preservative may be caustic to the skin. The nurse can call the laboratory while the UAP is washing hands, if needed. The UAP would then need to fill out an incident or exposure report and may or may not need to go to Employee Health. The UAP also needs further education on Standard Precautions, which include wearing gloves.

8. Which drug disrupts platelet action? a. Vitamin K b. Ibuprofen c. Methyldopa d. Azathioprine

b

12. A client is taking furosemide (Lasix) 40 mg/day for management of chronic kidney disease (CKD). To detect the positive effect of the medication, what action of the nurse is best? a. Obtain daily weights of the client. b. Auscultate heart and breath sounds. c. Palpate the client's abdomen. d. Assess the client's diet history.

ANS: A Furosemide (Lasix) is a loop diuretic that helps reduce fluid overload and hypertension in clients with early stages of CKD. One kilogram of weight equals about 1 liter of fluid retained in the client, so daily weights are necessary to monitor the response of the client to the medication. Heart and breath sounds should be assessed if there is fluid retention, as in heart failure. Palpation of the client's abdomen is not necessary, but the nurse should check for edema. The diet history of the client would be helpful to assess electrolyte replacement since potassium is lost with this diuretic, but this does not assess the effect of the medication.

11. The emergency department (ED) manager is reviewing client charts to determine how well the staff performs when treating clients with community-acquired pneumonia. What outcome demonstrates that goals for this client type have been met? a. Antibiotics started before admission b. Blood cultures obtained within 20 minutes c. Chest x-ray obtained within 30 minutes d. Pulse oximetry obtained on all clients

ANS: A Goals for treatment of community-acquired pneumonia include initiating antibiotics prior to inpatient admission or within 6 hours of presentation to the ED. Timely collection of blood cultures, chest x-ray, and pulse oximetry are important as well but do not coincide with established goals.

11. The emergency department (ED) manager is reviewing client charts to determine how well the staff performs when treating clients with community-acquired pneumonia. What outcome demonstrates that goals for this client type have been met? a. Antibiotics started before admission b. Blood cultures obtained within 20 minutes c. Chest x-ray obtained within 30 minutes d. Pulse oximetry obtained on all clients

ANS: A Goals for treatment of community-acquired pneumonia include initiating antibiotics prior to inpatient admission or within 6 hours of presentation to the ED. Timely collection of blood cultures, chest x-ray, and pulse oximetry are important as well but do not coincide with established goals.

8. A client in the intensive care unit is started on continuous venovenous hemofiltration (CVVH). Which finding is the cause of immediate action by the nurse? a. Blood pressure of 76/58 mm Hg b. Sodium level of 138 mEq/L c. Potassium level of 5.5 mEq/L d. Pulse rate of 90 beats/min

ANS: A Hypotension can be a problem with CVVH if replacement fluid does not provide enough volume to maintain blood pressure. The specially trained nurse needs to monitor for ongoing fluid and electrolyte replacement. The sodium level is normal and the potassium level is slightly elevated, which could be normal findings for someone with acute kidney injury. A pulse rate of 90 beats/min is normal.

5. After teaching a client who has stress incontinence, the nurse assesses the client's understanding. Which statement made by the client indicates a need for additional teaching? a. "I will limit my total intake of fluids." b. "I must avoid drinking alcoholic beverages." c. "I must avoid drinking caffeinated beverages." d. "I shall try to lose about 10% of my body weight."

ANS: A Limiting fluids concentrates urine and can irritate tissues, leading to increased incontinence. Many people try to manage incontinence by limiting fluids. Alcoholic and caffeinated beverages are bladder stimulants. Obesity increases intra-abdominal pressure, causing incontinence.

13. A nurse cares for a client with diabetes mellitus who is prescribed metformin (Glucophage) and is scheduled for an intravenous urography. Which action should the nurse take first? a. Contact the provider and recommend discontinuing the metformin. b. Keep the client NPO for at least 6 hours prior to the examination. c. Check the client's capillary artery blood glucose and administer prescribed insulin. d. Administer intravenous fluids to dilute and increase the excretion of dye.

ANS: A Metformin can cause lactic acidosis and renal impairment as the result of an interaction with the dye. This drug must be discontinued for 48 hours before the procedure and not started again after the procedure until urine output is well established. The client's health care provider needs to provide alternative therapy for the client until the metformin can be resumed. Keeping the client NPO, checking the client's blood glucose, and administering intravenous fluids should be part of the client's plan of care, but are not the priority, as the examination should not occur while the client is still taking metformin.

8. A nurse cares for a client after radiation therapy for lung cancer. The client reports a sore throat. Which action should the nurse take first? a. Ask the client to gargle with mouthwash containing lidocaine. b. Administer prescribed intravenous pain medications. c. Explain that soreness is normal and will improve in a couple days. d. Assess the client's neck for redness and swelling.

ANS: A Mouthwashes and throat sprays containing a local anesthetic agent such as lidocaine or diphenhydramine can provide relief from a sore throat after radiation therapy. Intravenous pain medications may be used if local anesthetics are unsuccessful. The nurse should explain to the client that this is normal and assess the client's neck, but these options do not decrease the client's discomfort. DIF: Remembering/Knowledge REF: 539 KEY: Cancer| pain| medication MSC: Integrated Process: Nursing Process: Implementation NOT: Client Needs Category: Physiological Integrity: Pharmacological and Parenteral Therapies

25. A nurse auscultates a client's lung fields. Which action should the nurse take based on the lung sounds? (Click the media button to hear the audio clip.) a. Assess for airway obstruction. b. Initiate oxygen therapy. c. Assess vital signs. d. Elevate the client's head.

ANS: A Stridor is the sound heard, and it indicates severe airway constriction. The nurse must administer a bronchodilator to get air into the lungs. Administering oxygen, assessing vital signs, and elevating the client's head will not help until the client's airways are open. DIF: Applying/Application REF: 575 KEY: Assessment/diagnostic examination MSC: Integrated Process: Nursing Process: Implementation NOT: Client Needs Category: Safe and Effective Care Environment: Management of Care

5. A nurse contacts the health care provider after reviewing a client's laboratory results and noting a blood urea nitrogen (BUN) of 35 mg/dL and a creatinine of 1.0 mg/dL. For which action should the nurse recommend a prescription? a. Intravenous fluids b. Hemodialysis c. Fluid restriction d. Urine culture and sensitivity

ANS: A Normal BUN is 10 to 20 mg/dL. Normal creatinine is 0.6 to 1.2 mg/dL (males) or 0.5 to 1.1 mg/dL (females). Creatinine is more specific for kidney function than BUN, because BUN can be affected by several factors (dehydration, high-protein diet, and catabolism). This client's creatinine is normal, which suggests a non-renal cause for the elevated BUN. A common cause of increased BUN is dehydration, so the nurse should anticipate giving the client more fluids, not placing the client on fluid restrictions. Hemodialysis is not an appropriate treatment for dehydration. The lab results do not indicate an infection; therefore, a urine culture and sensitivity is not appropriate.

11. An older adult is on cardiac monitoring after a myocardial infarction. The client shows frequent dysrhythmias. What action by the nurse is most appropriate? a. Assess for any hemodynamic effects of the rhythm. b. Prepare to administer antidysrhythmic medication. c. Notify the provider or call the Rapid Response Team. d. Turn the alarms off on the cardiac monitor.

ANS: A Older clients may have dysrhythmias due to age-related changes in the cardiac conduction system. They may have no significant hemodynamic effects from these changes. The nurse should first assess for the effects of the dysrhythmia before proceeding further. The alarms on a cardiac monitor should never be shut off. The other two actions may or may not be needed. DIF: Applying/Application REF: 769 KEY: Coronary artery disease| older adult| pathophysiology| nursing assessment MSC: IntegratedProcess:NursingProcess:Assessment NOT: Client Needs Category: Health Promotion and Maintenance

15. A nurse teaches a client who experiences occasional premature atrial contractions (PACs) accompanied by palpitations that resolve spontaneously without treatment. Which statement should the nurse include in this client's teaching? a. "Minimize or abstain from caffeine." b. "Lie on your side until the attack subsides." c. "Use your oxygen when you experience PACs." d. "Take amiodarone (Cordarone) daily to prevent PACs."

ANS: A PACs usually have no hemodynamic consequences. For a client experiencing infrequent PACs, the nurse should explore possible lifestyle causes, such as excessive caffeine intake and stress. Lying on the side will not prevent or resolve PACs. Oxygen is not necessary. Although medications may be needed to control symptomatic dysrhythmias, for infrequent PACs, the client first should try lifestyle changes to control them. DIF: Applying/Application REF: 663 KEY: Patient education| cardiac electrical conduction MSC: IntegratedProcess:Teaching/Learning NOT: Client Needs Category: Health Promotion and Maintenance

15. A 70-kg adult with chronic renal failure is on a 40-g protein diet. The client has a reduced glomerular filtration rate and is not undergoing dialysis. Which result would give the nurse the most concern? a. Albumin level of 2.5 g/dL b. Phosphorus level of 5 mg/dL c. Sodium level of 135 mmol/L d. Potassium level of 5.5 mmol/L

ANS: A Protein restriction is necessary with chronic renal failure due to the buildup of waste products from protein breakdown. The nurse would be concerned with the low albumin level since this indicates that the protein in the diet is not enough for the client's metabolic needs. The electrolyte values are not related to the protein-restricted diet.

7. A client appears dyspneic, but the oxygen saturation is 97%. What action by the nurse is best? a. Assess for other manifestations of hypoxia. b. Change the sensor on the pulse oximeter. c. Obtain a new oximeter from central supply. d. Tell the client to take slow, deep breaths.

ANS: A Pulse oximetry is not always the most accurate assessment tool for hypoxia as many factors can interfere, producing normal or near-normal readings in the setting of hypoxia. The nurse should conduct a more thorough assessment. The other actions are not appropriate for a hypoxic client.

8. A nurse reviews laboratory results for a client who was admitted for a myocardial infarction and cardiogenic shock 2 days ago. Which laboratory test result should the nurse expect to find? a. Blood urea nitrogen (BUN) of 52 mg/dL b. Creatinine of 2.3 mg/dL c. BUN of 10 mg/dL d. BUN/creatinine ratio of 8:1

ANS: A Shock leads to decreased renal perfusion. An elevated BUN accompanies this condition. The creatinine should be normal because no kidney damage occurred. A low BUN signifies overhydration, malnutrition, or liver damage. A low BUN/creatinine ratio indicates fluid volume excess or acute renal tubular acidosis.

16. A nurse obtains the health history of a client with a suspected diagnosis of bladder cancer. Which question should the nurse ask when determining this client's risk factors? a. "Do you smoke cigarettes?" b. "Do you use any alcohol?" c. "Do you use recreational drugs?" d. "Do you take any prescription drugs?"

ANS: A Smoking is known to be a factor that greatly increases the risk of bladder cancer. Alcohol use, recreational drug use, and prescription drug use (except medications that contain phenacetin) are not known to increase the risk of developing bladder cancer.

A nurse teaches a client who has been prescribed a 24-hour urine collection to measure excreted hormones. The client asks, "Why do I need to collect urine for 24 hours instead of providing a random specimen?" How should the nurse respond? a. "This test will assess for a hormone secreted on a circadian rhythm." b. "The hormone is diluted in urine; therefore, we need a large volume." c. "We are assessing when the hormone is secreted in large amounts." d. "To collect the correct hormone, you need to urinate multiple times."

ANS: A Some hormones are secreted in a pulsatile, or circadian, cycle. When testing for these substances, a collection that occurs over 24 hours will most accurately reflect hormone secretion. Dilution of hormones in urine, secretion of hormone amounts, and ability to collect the correct hormone are not reasons to complete a 24-hour urine test.

70. What change in diabetic therapy may be needed for a patient who has diabetic nephropathy? a. Fluid restriction b. Decreased activity level c. Decreased insulin dosages e. Increased caloric intake

c

6. A client admitted for pneumonia has been tachypneic for several days. When the nurse starts an IV to give fluids, the client questions this action, saying "I have been drinking tons of water. How am I dehydrated?" What response by the nurse is best? a. "Breathing so quickly can be dehydrating." b. "Everyone with pneumonia is dehydrated." c. "This is really just to administer your antibiotics." d. "Why do you think you are so dehydrated?"

ANS: A Tachypnea and mouth breathing, both seen in pneumonia, increase insensible water loss and can lead to a degree of dehydration. The other options do not give the client useful information.

18. After delegating to an unlicensed assistive personnel (UAP) the task of completing a bladder scan examination for a client, the nurse evaluates the UAP's performance. Which action by the UAP indicates the nurse must provide additional instructions when delegating this task? a. Selecting the female icon for all female clients and male icon for all male clients b. Telling the client, "This test measures the amount of urine in your bladder." c. Applying ultrasound gel to the scanning head and removing it when finished d. Taking at least two readings using the aiming icon to place the scanning head

ANS: A The UAP should use the female icon for women who have not had a hysterectomy. This allows the scanner to subtract the volume of the uterus from readings. If a woman has had a hysterectomy, the UAP should choose the male icon. The UAP should explain the procedure to the client, apply gel to the scanning head and clean it after use, and take at least two readings.

18. A nurse teaches a young female client who is prescribed amoxicillin (Amoxil) for a urinary tract infection. Which statement should the nurse include in this client's teaching? a. "Use a second form of birth control while on this medication." b. "You will experience increased menstrual bleeding while on this drug." c. "You may experience an irregular heartbeat while on this drug." d. "Watch for blood in your urine while taking this medication."

ANS: A The client should use a second form of birth control because penicillin seems to reduce the effectiveness of estrogen-containing contraceptives. She should not experience increased menstrual bleeding, an irregular heartbeat, or blood in her urine while taking the medication.

15. A nurse cares for a client with infective endocarditis. Which infection control precautions should the nurse use? a. Standard Precautions b. Bleeding precautions c. Reverse isolation d. Contact isolation

ANS: A The client with infective endocarditis does not pose any specific threat of transmitting the causative organism. Standard Precautions should be used. Bleeding precautions or reverse or contact isolation is not necessary. DIF: Applying/Application REF: 697 KEY: Infection| Standard Precautions MSC: Integrated Process: Nursing Process: Implementation NOT: Client Needs Category: Safe and Effective Care Environment: Safety and Infection Control

2. A nurse assesses a client who has a nasal fracture. The client reports constant nasal drainage, a headache, and difficulty with vision. Which action should the nurse take next? a. Collect the nasal drainage on a piece of filter paper. b. Encourage the client to blow his or her nose. c. Perform a test focused on a neurologic examination. d. Palpate the nose, face, and neck.

ANS: A The client with nasal drainage after facial trauma could have a skull fracture that has resulted in leakage of cerebrospinal fluid (CSF). CSF can be differentiated from regular drainage by the fact that it forms a halo when dripped on filter paper. The other actions would be appropriate but are not as high a priority as assessing for CSF. A CSF leak would increase the client's risk for infection. DIF: Applying/Application REF: 532 KEY: Trauma| medical emergencies MSC: Integrated Process: Nursing Process: Implementation NOT: Client Needs Category: Physiological Integrity: Reduction of Risk Potential

A nurse collaborates with an unlicensed assistive personnel (UAP) to provide care for a client who is prescribed a 24-hour urine specimen collection. Which statement should the nurse include when delegating this activity to the UAP? a. "Note the time of the client's first void and collect urine for 24 hours." b. "Add the preservative to the container at the end of the test." c. "Start the collection by saving the first urine of the morning." d. "It is okay if one urine sample during the 24 hours is not collected."

ANS: A The collection of a 24-hour urine specimen is often delegated to a UAP. The nurse must ensure that the UAP understands the proper process for collecting the urine. The 24-hour urine collection specimen is started after the client's first urination. The first urine specimen is discarded because there is no way to know how long it has been in the bladder, but the time of the client's first void is noted. The client adds all urine voided after that first discarded specimen during the next 24 hours. When the 24-hour mark is reached, the client voids one last time and adds this specimen to the collection. The preservative, if used, must be added to the container at the beginning of the collection. All urine samples need to be collected for the test results to be accurate.

11. A nurse assesses a client who is prescribed fluticasone (Flovent) and notes oral lesions. Which action should the nurse take? a. Encourage oral rinsing after fluticasone administration. b. Obtain an oral specimen for culture and sensitivity. c. Start the client on a broad-spectrum antibiotic. d. Document the finding as a known side effect.

ANS: A The drug reduces local immunity and increases the risk for local infection, especially Candida albicans. Rinsing the mouth after using the inhaler will decrease the risk for developing this infection. Use of mouthwash and broad-spectrum antibiotics is not warranted in this situation. The nurse should document the finding, but the best action to take is to have the client start rinsing his or her mouth after using fluticasone. An oral specimen for culture and sensitivity will not provide information necessary to care for this client. DIF: Applying/Application REF: 554 KEY: Medication| fungal infection MSC: Integrated Process: Nursing Process: Implementation NOT: Client Needs Category: Physiological Integrity: Pharmacological and Parenteral Therapies 12. A nurse cares for a client who is infected with Burkholderia cepacia. Which action should the nurse take first when admitting this client to a pulmonary care unit? a. Instruct the client to wash his or her hands after contact with other people. b. Implement Droplet Precautions and don a surgical mask. c. Keep the client isolated from other clients with cystic fibrosis. d. Obtain blood, sputum, and urine culture specimens. ANS: C Burkholderia cepacia infection is spread through casual contact between cystic fibrosis clients, thus the need for these clients to be separated from one another. Strict isolation measures will not be necessary. Although the client should wash his or her hands frequently, the most important measure that can be implemented on the unit is isolation of the client from other clients with cystic fibrosis. There is no need to implement Droplet Precautions or don a surgical mask when caring for this client. Obtaining blood, sputum, and urine culture specimens will not provide information necessary to care for a client with Burkholderia cepacia infection. DIF: Applying/Application REF: 568 KEY: Pulmonary infection| infection control MSC: Integrated Process: Nursing Process: Implementation NOT: Client Needs Category: Safe and Effective Care Environment: Safety and Infection Control

13. A nurse prepares to discharge a client with cardiac dysrhythmia who is prescribed home health care services. Which priority information should be communicated to the home health nurse upon discharge? a. Medication reconciliation b. Immunization history c. Religious beliefs d. Nutrition preferences

ANS: A The home health nurse needs to know current medications the client is taking to ensure assessment, evaluation, and further education related to these medications. The other information will not assist the nurse to develop a plan of care for the client. DIF: Applying/Application REF: 673 KEY: Hand-off communication MSC: Integrated Process: Communication and Documentation NOT: Client Needs Category: Safe and Effective Care Environment: Management of Care

11. A male client with chronic kidney disease (CKD) is refusing to take his medication and has missed two hemodialysis appointments. What is the best initial action for the nurse? a. Discuss what the treatment regimen means to him. b. Refer the client to a mental health nurse practitioner. c. Reschedule the appointments to another date and time. d. Discuss the option of peritoneal dialysis.

ANS: A The initial action for the nurse is to assess anxiety, coping styles, and the client's acceptance of the required treatment for CKD. The client may be in denial of the diagnosis. While rescheduling hemodialysis appointments may help, and referral to a mental health practitioner and the possibility of peritoneal dialysis are all viable options, assessment of the client's acceptance of the treatment should come first.

A nurse assesses a client diagnosed with adrenal hypofunction. Which client statement should the nurse correlate with this diagnosis? a. "I have a terrible craving for potato chips." b. "I cannot seem to drink enough water." c. "I no longer have an appetite for anything." d. "I get hungry even after eating a meal."

ANS: A The nurse correlates a client's salt craving with adrenal hypofunction. Excessive thirst is related to diabetes insipidus or diabetes mellitus. Clients who have hypothyroidism often have a decrease in appetite. Excessive hunger is associated with diabetes mellitus.

12. A nurse is caring for a client on mechanical ventilation and finds the client agitated and thrashing about. What action by the nurse is most appropriate? a. Assess the cause of the agitation. b. Reassure the client that he or she is safe. c. Restrain the client's hands. d. Sedate the client immediately.

ANS: A The nurse needs to determine the cause of the agitation. The inability to communicate often makes clients anxious, even to the point of panic. Pain and confusion can also cause agitation. Once the nurse determines the cause of the agitation, he or she can implement measures to relieve the underlying cause. Reassurance is also important but may not address the etiology of the agitation. Restraints and more sedation may be necessary, but not as a first step.

15. A client has been hospitalized with tuberculosis (TB). The client's spouse is fearful of entering the room where the client is in isolation and refuses to visit. What action by the nurse is best? a. Ask the spouse to explain the fear of visiting in further detail. b. Inform the spouse the precautions are meant to keep other clients safe. c. Show the spouse how to follow the isolation precautions to avoid illness. d. Tell the spouse that he or she has already been exposed, so it's safe to visit.

ANS: A The nurse needs to obtain further information about the spouse's specific fears so they can be addressed. This will decrease stress and permit visitation, which will be beneficial for both client and spouse. Precautions for TB prevent transmission to all who come into contact with the client. Explaining isolation precautions and what to do when entering the room will be helpful, but this is too narrow in scope to be the best answer. Telling the spouse it's safe to visit is demeaning of the spouse's feelings.

15. A client has been hospitalized with tuberculosis (TB). The client's spouse is fearful of entering the room where the client is in isolation and refuses to visit. What action by the nurse is best? a. Ask the spouse to explain the fear of visiting in further detail. b. Inform the spouse the precautions are meant to keep other clients safe. c. Show the spouse how to follow the isolation precautions to avoid illness. d. Tell the spouse that he or she has already been exposed, so it's safe to visit.

ANS: A The nurse needs to obtain further information about the spouse's specific fears so they can be addressed. This will decrease stress and permit visitation, which will be beneficial for both client and spouse. Precautions for TB prevent transmission to all who come into contact with the client. Explaining isolation precautions and what to do when entering the room will be helpful, but this is too narrow in scope to be the best answer. Telling the spouse it's safe to visit is demeaning of the spouse's feelings.

7. A nurse cares for a client who is experiencing epistaxis. Which action should the nurse take first? a. Initiate Standard Precautions. b. Apply direct pressure. c. Sit the client upright. d. Loosely pack the nares with gauze.

ANS: A The nurse should implement Standard Precautions and don gloves prior to completing the other actions. DIF: Applying/Application REF: 532 KEY: Trauma MSC: Integrated Process: Nursing Process: Implementation NOT: Client Needs Category: Safe and Effective Care Environment: Safety and Infection Control

4. While assessing a client on a cardiac unit, a nurse identifies the presence of an S3 gallop. Which action should the nurse take next? a. Assess for symptoms of left-sided heart failure. b. Document this as a normal finding. c. Call the health care provider immediately. d. Transfer the client to the intensive care unit.

ANS: A The presence of an S3 gallop is an early diastolic filling sound indicative of increasing left ventricular pressure and left ventricular failure. The other actions are not warranted. DIF: Remembering/Knowledge REF: 683 KEY: Heart failure| assessment/diagnostic examination MSC: IntegratedProcess:NursingProcess:Assessment NOT: Client Needs Category: Physiological Integrity: Reduction of Risk Potential

6. A client has a serum potassium level of 6.5 mmol/L, a serum creatinine level of 2 mg/dL, and a urine output of 350 mL/day. What is the best action by the nurse? a. Place the client on a cardiac monitor immediately. b. Teach the client to limit high-potassium foods. c. Continue to monitor the client's intake and output. d. Ask to have the laboratory redraw the blood specimen.

ANS: A The priority action by the nurse should be to check the cardiac status with a monitor. High potassium levels can lead to dysrhythmias. The other choices are logical nursing interventions for acute kidney injury but not the best immediate action.

26. The nurse instructs a client on the steps needed to obtain a peak expiratory flow rate. In which order should these steps occur? 1. "Take as deep a breath as possible." 2. "Stand up (unless you have a physical disability)." 3. "Place the meter in your mouth, and close your lips around the mouthpiece." 4. "Make sure the device reads zero or is at base level." 5. "Blow out as hard and as fast as possible for 1 to 2 seconds." 6. "Write down the value obtained." 7. "Repeat the process two additional times, and record the highest number in your chart." a. 4, 2, 1, 3, 5, 6, 7 b. 3, 4, 1, 2, 5, 7, 6 c. 2, 1, 3, 4, 5, 6, 7 d. 1, 3, 2, 5, 6, 7, 4

ANS: A The proper order for obtaining a peak expiratory flow rate is as follows. Make sure the device reads zero or is at base level. The client should stand up (unless he or she has a physical disability). The client should take as deep a breath as possible, place the meter in the mouth, and close the lips around the mouthpiece. The client should blow out as hard and as fast as possible for 1 to 2 seconds. The value obtained should be written down. The process should be repeated two more times, and the highest of the three numbers should be recorded in the client's chart. DIF: Applying/Application REF: 552 KEY: Assessment/diagnostic examination MSC: IntegratedProcess:NursingProcess:Assessment NOT: Client Needs Category: Physiological Integrity: Reduction of Risk Potential

8. A client has been diagnosed with tuberculosis (TB). What action by the nurse takes highest priority? a. Educating the client on adherence to the treatment regimen b. Encouraging the client to eat a well-balanced diet c. Informing the client about follow-up sputum cultures d. Teaching the client ways to balance rest with activity

ANS: A The treatment regimen for TB ranges from 6 to 12 months, making adherence problematic for many people. The nurse should stress the absolute importance of following the treatment plan for the entire duration of prescribed therapy. The other options are appropriate topics to educate this client on but do not take priority.

8. A client has been diagnosed with tuberculosis (TB). What action by the nurse takes highest priority? a. Educating the client on adherence to the treatment regimen b. Encouraging the client to eat a well-balanced diet c. Informing the client about follow-up sputum cultures d. Teaching the client ways to balance rest with activity

ANS: A The treatment regimen for TB ranges from 6 to 12 months, making adherence problematic for many people. The nurse should stress the absolute importance of following the treatment plan for the entire duration of prescribed therapy. The other options are appropriate topics to educate this client on but do not take priority.

3. A male client comes into the emergency department with a serum creatinine of 2.2 mg/dL and a blood urea nitrogen (BUN) of 24 mL/dL. What question should the nurse ask first when taking this client's history? a. "Have you been taking any aspirin, ibuprofen, or naproxen recently?" b. "Do you have anyone in your family with renal failure?" c. "Have you had a diet that is low in protein recently?" d. "Has a relative had a kidney transplant lately?"

ANS: A There are some medications that are nephrotoxic, such as the nonsteroidal anti-inflammatory drugs ibuprofen, aspirin, and naproxen. This would be a good question to initially ask the client since both the serum creatinine and BUN are elevated, indicating some renal problems. A family history of renal failure and kidney transplantation would not be part of the questioning and could cause anxiety in the client. A diet high in protein could be a factor in an increased BUN.

23. A nurse assesses a client who presents with renal calculi. Which question should the nurse ask? a. "Do any of your family members have this problem?" b. "Do you drink any cranberry juice?" c. "Do you urinate after sexual intercourse?" d. "Do you experience burning with urination?"

ANS: A There is a strong association between family history and stone formation and recurrence. Nephrolithiasis is associated with many genetic variations; therefore, the nurse should ask whether other family members have also had renal stones. The other questions do not refer to renal calculi but instead are questions that should be asked of a client with a urinary tract infection.

18. A client with chronic kidney disease (CKD) is experiencing nausea, vomiting, visual changes, and anorexia. Which action by the nurse is best? a. Check the client's digoxin (Lanoxin) level. b. Administer an anti-nausea medication. c. Ask if the client is able to eat crackers. d. Get a referral to a gastrointestinal provider.

ANS: A These signs and symptoms are indications of digoxin (Lanoxin) toxicity. The nurse should check the level of this medication. Administering antiemetics, asking if the client can eat, and obtaining a referral to a specialist all address the client's symptoms but do not lead to the cause of the symptoms.

2. A marathon runner comes into the clinic and states "I have not urinated very much in the last few days." The nurse notes a heart rate of 110 beats/min and a blood pressure of 86/58 mm Hg. Which action by the nurse is the priority? a. Give the client a bottle of water immediately. b. Start an intravenous line for fluids. c. Teach the client to drink 2 to 3 liters of water daily. d. Perform an electrocardiogram.

ANS: A This athlete is mildly dehydrated as evidenced by the higher heart rate and lower blood pressure. The nurse can start hydrating the client with a bottle of water first, followed by teaching the client to drink 2 to 3 liters of water each day. An intravenous line may be ordered later, after the client's degree of dehydration is assessed. An electrocardiogram is not necessary at this time.

17. A nurse supervises an unlicensed assistive personnel (UAP) applying electrocardiographic monitoring. Which statement should the nurse provide to the UAP related to this procedure? a. "Clean the skin and clip hairs if needed." b. "Add gel to the electrodes prior to applying them." c. "Place the electrodes on the posterior chest." d. "Turn off oxygen prior to monitoring the client."

ANS: A To ensure the best signal transmission, the skin should be clean and hairs clipped. Electrodes should be placed on the anterior chest, and no additional gel is needed. Oxygen has no impact on electrocardiographic monitoring. DIF: Remembering/Knowledge REF: 652 KEY: Assessment/diagnostic examination| interdisciplinary team| unlicensed assistive personnel (UAP) MSC: Integrated Process: Communication and Documentation NOT: Client Needs Category: Safe and Effective Care Environment: Management of Care

19. A nurse cares for a client who is on a cardiac monitor. The monitor displayed the rhythm shown below: Which action should the nurse take first? a. Assess airway, breathing, and level of consciousness. b. Administer an amiodarone bolus followed by a drip. c. Cardiovert the client with a biphasic defibrillator. d. Begin cardiopulmonary resuscitation (CPR).

ANS: A Ventricular tachycardia occurs with repetitive firing of an irritable ventricular ectopic focus, usually at a rate of 140 to 180 beats/min or more. Ventricular tachycardia is a lethal dysrhythmia. The nurse should first assess if the client is alert and breathing. Then the nurse should call a Code Blue and begin CPR. If this client is pulseless, the treatment of choice is defibrillation. Amiodarone is the antidysrhythmic of choice, but it is not the first action. DIF: Applying/Application REF: 670 KEY: Cardiac electrical conduction| medical emergency MSC: Integrated Process: Nursing Process: Implementation NOT: Client Needs Category: Physiological Integrity: Physiological Adaptation

A nurse assesses clients who have endocrine disorders. Which assessment findings are paired correctly with the endocrine disorder? (Select all that apply.) a. Excessive thyroid-stimulating hormone - Increased bone formation b. Excessive melanocyte-stimulating hormone - Darkening of the skin c. Excessive parathyroid hormone - Synthesis and release of corticosteroids d. Excessive antidiuretic hormone - Increased urinary output e. Excessive adrenocorticotropic hormone - Increased bone resorption

ANS: A, B Thyroid-stimulating hormone targets thyroid tissue and stimulates the formation of bone. Melanocyte-stimulating hormone stimulates melanocytes and promotes pigmentation or the darkening of the skin. Parathyroid hormone stimulates bone resorption. Antidiuretic hormone targets the kidney and promotes water reabsorption, causing a decrease in urinary output. Adrenocorticotropic hormone targets the adrenal cortex and stimulates the synthesis and release of corticosteroids.

A nurse provides diabetic education at a public health fair. Which disorders should the nurse include as complications of diabetes mellitus? (Select all that apply.) a. Stroke b. Kidney failure c. Blindness d. Respiratory failure e. Cirrhosis

ANS: A, B, C Complications of diabetes mellitus are caused by macrovascular and microvascular changes. Macrovascular complications include coronary artery disease, cerebrovascular disease, and peripheral vascular disease. Microvascular complications include nephropathy, retinopathy, and neuropathy. Respiratory failure and cirrhosis are not complications of diabetes mellitus.

2. A hospital nurse is participating in a drill during which many clients with inhalation anthrax are being admitted. What drugs should the nurse anticipate administering? (Select all that apply.) a. Amoxicillin (Amoxil) b. Ciprofloxacin (Cipro) c. Doxycycline (Vibramycin) d. Ethambutol (Myambutol) e. Sulfamethoxazole-trimethoprim (SMX-TMP) (Septra)

ANS: A, B, C Amoxicillin, ciprofloxacin, and doxycycline are all possible treatments for inhalation anthrax. Ethambutol is used for tuberculosis. SMX-TMP is commonly used for urinary tract infections and other common infections.

8. A nurse assesses a client who is recovering from a heart transplant. Which assessment findings should alert the nurse to the possibility of heart transplant rejection? (Select all that apply.) a. Shortness of breath b. Abdominal bloating c. New-onset bradycardia d. Increased ejection fraction e. Hypertension

ANS: A, B, C Clinical manifestations of heart transplant rejection include shortness of breath, fatigue, fluid gain, abdominal bloating, new-onset bradycardia, hypotension, atrial fibrillation or flutter, decreased activity tolerance, and decreased ejection fraction. DIF: Remembering/Knowledge REF: 703 KEY: Transplant| heart failure MSC: IntegratedProcess:NursingProcess:Assessment NOT: Client Needs Category: Physiological Integrity: Reduction of Risk Potential

3. A nurse teaches a client who has chronic obstructive pulmonary disease. Which statements related to nutrition should the nurse include in this client's teaching? (Select all that apply.) a. "Avoid drinking fluids just before and during meals." b. "Rest before meals if you have dyspnea." c. "Have about six small meals a day." d. "Eat high-fiber foods to promote gastric emptying." e. "Increase carbohydrate intake for energy."

ANS: A, B, C Fluids can make a client feel bloated and should be avoided with meals. Resting before the meal will help a client with dyspnea. Six small meals a day also will help to decrease bloating. Fibrous foods can produce gas, which can cause abdominal bloating and can increase shortness of breath. The client should increase calorie and protein intake to prevent malnourishment. The client should not increase carbohydrate intake as this will increase carbon dioxide production and increase the client's risk of for acidosis. DIF: Applying/Application REF: 565 KEY: Nutrition| patient education MSC: Integrated Process: Teaching/Learning NOT: Client Needs Category: Health Promotion and Maintenance

A nurse cares for clients with hormone disorders. Which are common key features of hormones? (Select all that apply.) a. Hormones may travel long distances to get to their target tissues. b. Continued hormone activity requires continued production and secretion. c. Control of hormone activity is caused by negative feedback mechanisms. d. Most hormones are stored in the target tissues for use later. e. Most hormones cause target tissues to change activities by changing gene activity.

ANS: A, B, C Hormones are secreted by endocrine glands and travel through the body to reach their target tissues. Hormone activity can increase or decrease according to the body's needs, and continued hormone activity requires continued production and secretion. Control is maintained via negative feedback. Hormones are not stored for later use, and they do not alter genetic activity

7. A nurse teaches a client about self-care after experiencing a urinary calculus treated by lithotripsy. Which statements should the nurse include in this client's discharge teaching? (Select all that apply.) a. "Finish the prescribed antibiotic even if you are feeling better." b. "Drink at least 3 liters of fluid each day." c. "The bruising on your back may take several weeks to resolve." d. "Report any blood present in your urine." e. "It is normal to experience pain and difficulty urinating."

ANS: A, B, C The client should be taught to finish the prescribed antibiotic to ensure that he or she does not get a urinary tract infection. The client should drink at least 3 liters of fluid daily to dilute potential stone-forming crystals, prevent dehydration, and promote urine flow. After lithotripsy, the client should expect bruising that may take several weeks to resolve. The client should also experience blood in the urine for several days. The client should report any pain, fever, chills, or difficulty with urination to the provider as these may signal the beginning of an infection or the formation of another stone.

1. The nurse is caring for five clients on the medical-surgical unit. Which clients would the nurse consider to be at risk for post-renal acute kidney injury (AKI)? (Select all that apply.) a. Man with prostate cancer b. Woman with blood clots in the urinary tract c. Client with ureterolithiasis d. Firefighter with severe burns e. Young woman with lupus

ANS: A, B, C Urine flow obstruction, such as prostate cancer, blood clots in the urinary tract, and kidney stones (ureterolithiasis), causes post-renal AKI. Severe burns would be a pre-renal cause. Lupus would be an intrarenal cause for AKI.

5. A nurse teaches a client who is being discharged after a fixed centric occlusion for a mandibular fracture. Which statements should the nurse include in this client's teaching? (Select all that apply.) a. "You will need to cut the wires if you start vomiting." b. "Eat six soft or liquid meals each day while recovering." c. "Irrigate your mouth every 2 hours to prevent infection." d. "Sleep in a semi-Fowler's position after the surgery." e. "Gargle with mouthwash that contains Benadryl once a day."

ANS: A, B, C, D The client needs to know how to cut the wires in case of emergency. If the client vomits, he or she may aspirate. The client should also be taught to eat soft or liquid meals multiple times a day, irrigate the mouth with a Waterpik to prevent infection, and sleep in a semi-Fowler's position to assist in avoiding aspiration. Mouthwash with Benadryl is used for clients who have mouth pain after radiation treatment; it is not used to treat pain in a client with a mandibular fracture. DIF: Applying/Application REF: 534 KEY: Surgical care| aspiration precautions MSC: IntegratedProcess:Teaching/Learning NOT: Client Needs Category: Physiological Integrity: Reduction of Risk Potential

. The nurse caring for mechanically ventilated clients uses best practices to prevent ventilator-associated pneumonia. What actions are included in this practice? (Select all that apply.) a. Adherence to proper hand hygiene b. Administering anti-ulcer medication c. Elevating the head of the bed d. Providing oral care per protocol e. Suctioning the client on a regular schedule

ANS: A, B, C, D The "ventilator bundle" is a group of care measures to prevent ventilator-associated pneumonia. Actions in the bundle include using proper hand hygiene, giving anti-ulcer medications, elevating the head of the bed, providing frequent oral care per policy, preventing aspiration, and providing pulmonary hygiene measures. Suctioning is done as needed.

4. A client has been diagnosed with an empyema. What interventions should the nurse anticipate providing to this client? (Select all that apply.) a. Assisting with chest tube insertion b. Facilitating pleural fluid sampling c. Performing frequent respiratory assessment d. Providing antipyretics as needed e. Suctioning deeply every 4 hours

ANS: A, B, C, D The client with an empyema is often treated with chest tube insertion, which facilitates obtaining samples of the pleural fluid for analysis and re-expands the lungs. The nurse should perform frequent respiratory system assessments. Antipyretic medications are also used. Suction is only used when needed and is not done deeply to prevent tissue injury.

1. A nurse assesses a client who is 6 hours post-surgery for a nasal fracture and has nasal packing in place. Which actions should the nurse take? (Select all that apply.) a. Observe for clear drainage. b. Assess for signs of bleeding. c. Watch the client for frequent swallowing. d. Ask the client to open his or her mouth. e. Administer a nasal steroid to decrease edema. f. Change the nasal packing.

ANS: A, B, C, D The nurse should observe for clear drainage because of the risk for cerebrospinal fluid leakage. The nurse should assess for signs of bleeding by asking the client to open his or her mouth and observing the back of the throat for bleeding. The nurse should also note whether the client is swallowing frequently because this could indicate postnasal bleeding. A nasal steroid would increase the risk for infection. It is too soon to change the packing, which should be changed by the surgeon the first time. DIF: Applying/Application REF: 531 KEY: Surgical care MSC: Integrated Process: Nursing Process: Implementation NOT: Client Needs Category: Physiological Integrity: Reduction of Risk Potential

5. A nursing student planning to teach clients about risk factors for coronary artery disease (CAD) would include which topics? (Select all that apply.) a. Advanced age b. Diabetes c. Ethnic background d. Medication use e. Smoking

ANS: A, B, C, E Age, diabetes, ethnic background, and smoking are all risk factors for developing CAD; medication use is not. DIF: Remembering/Knowledge REF: 760 KEY: Coronary artery disease| pathophysiology| patient education MSC: IntegratedProcess:Teaching/Learning NOT: Client Needs Category: Physiological Integrity: Physiological Adaptation

3. A client with a new pulmonary embolism (PE) is anxious. What nursing actions are most appropriate? (Select all that apply.) a. Acknowledge the frightening nature of the illness. b. Delegate a back rub to the unlicensed assistive personnel (UAP). c. Give simple explanations of what is happening. d. Request a prescription for antianxiety medication. e. Stay with the client and speak in a quiet, calm voice.

ANS: A, B, C, E Clients with PEs are often anxious. The nurse can acknowledge the client's fears, delegate comfort measures, give simple explanations the client will understand, and stay with the client. Using a calm, quiet voice is also reassuring. Sedatives and antianxiety medications are not used routinely because they can contribute to hypoxia. If the client's anxiety is interfering with diagnostic testing or treatment, they can be used, but there is no evidence that this is the case.

A nurse collaborates with the interdisciplinary team to develop a plan of care for a client who is newly diagnosed with diabetes mellitus. Which team members should the nurse include in this interdisciplinary team meeting? (Select all that apply.) a. Registered dietitian b. Clinical pharmacist c. Occupational therapist d. Health care provider e. Speech-language pathologist

ANS: A, B, D When planning care for a client newly diagnosed with diabetes mellitus, the nurse should collaborate with a registered dietitian, clinical pharmacist, and health care provider. The focus of treatment for a newly diagnosed client would be nutrition, medication therapy, and education. The nurse could also consult with a diabetic educator. There is no need for occupational therapy or speech therapy at this time.

5. A nurse teaches a client with polycystic kidney disease (PKD). Which statements should the nurse include in this client's discharge teaching? (Select all that apply.) a. "Take your blood pressure every morning." b. "Weigh yourself at the same time each day." c. "Adjust your diet to prevent diarrhea." d. "Contact your provider if you have visual disturbances." e. "Assess your urine for renal stones."

ANS: A, B, D A client who has PKD should measure and record his or her blood pressure and weight daily, limit salt intake, and adjust dietary selections to prevent constipation. The client should notify the provider if urine smells foul or has blood in it, as these are signs of a urinary tract infection or glomerular injury. The client should also notify the provider if visual disturbances are experienced, as this is a sign of a possible berry aneurysm, which is a complication of PKD. Diarrhea and renal stones are not manifestations or complications of PKD; therefore, teaching related to these concepts would be inappropriate.

3. A nurse is teaching a client with premature ectopic beats. Which education should the nurse include in this client's teaching? (Select all that apply.) a. Smoking cessation b. Stress reduction and management c. Avoiding vagal stimulation d. Adverse effects of medications e. Foods high in potassium

ANS: A, B, D A client who has premature beats or ectopic rhythms should be taught to stop smoking, manage stress, take medications as prescribed, and report adverse effects of medications. Clients with premature beats are not at risk for vasovagal attacks or potassium imbalances. DIF: Remembering/Knowledge REF: 673 KEY: Patient education MSC: IntegratedProcess:NursingProcess:Implementation NOT: Client Needs Category: Health Promotion and Maintenance

3. A nurse assesses clients on a cardiac unit. Which clients should the nurse identify as at greatest risk for the development of acute pericarditis? (Select all that apply.) a. A 36-year-old woman with systemic lupus erythematosus (SLE) b. A 42-year-old man recovering from coronary artery bypass graft surgery c. A 59-year-old woman recovering from a hysterectomy d. An 80-year-old man with a bacterial infection of the respiratory tract e. An 88-year-old woman with a stage III sacral ulcer

ANS: A, B, D Acute pericarditis is most commonly associated acute exacerbations of systemic connective tissue disease, including SLE; with Dressler's syndrome, or inflammation of the cardiac sac after cardiac surgery or a myocardial infarction; and with infective organisms, including bacterial, viral, and fungal infections. Abdominal and reproductive surgeries and pressure ulcers do not increase clients' risk for acute pericarditis. DIF: Applying/Application REF: 699 KEY: Inflammatory response| health screening MSC: IntegratedProcess:NursingProcess:Assessment NOT: Client Needs Category: Safe and Effective Care Environment: Management of Care

6. The nurse caring for mechanically ventilated clients knows that older adults are at higher risk for weaning failure. What age-related changes contribute to this? (Select all that apply.) a. Chest wall stiffness b. Decreased muscle strength c. Inability to cooperate d. Less lung elasticity e. Poor vision and hearing

ANS: A, B, D Age-related changes that increase the difficulty of weaning older adults from mechanical ventilation include increased stiffness of the chest wall, decreased muscle strength, and less elasticity of lung tissue. Not all older adults have an inability to cooperate or poor sensory acuity.

1. A nurse assesses a client who has had two episodes of bacterial cystitis in the last 6 months. Which questions should the nurse ask? (Select all that apply.) a. "How much water do you drink every day?" b. "Do you take estrogen replacement therapy?" c. "Does anyone in your family have a history of cystitis?" d. "Are you on steroids or other immune-suppressing drugs?" e. "Do you drink grapefruit juice or orange juice daily?"

ANS: A, B, D Fluid intake, estrogen levels, and immune suppression all can increase the chance of recurrent cystitis. Family history is usually insignificant, and cranberry juice, not grapefruit or orange juice, has been found to increase the acidic pH and reduce the risk for bacterial cystitis.

6. A client is undergoing hemodialysis. The client's blood pressure at the beginning of the procedure was 136/88 mm Hg, and now it is 110/54 mm Hg. What actions should the nurse perform to maintain blood pressure? (Select all that apply.) a. Adjust the rate of extracorporeal blood flow. b. Place the client in the Trendelenburg position. c. Stop the hemodialysis treatment. d. Administer a 250-mL bolus of normal saline. e. Contact the health care provider for orders.

ANS: A, B, D Hypotension occurs often during hemodialysis treatments as a result of vasodilation from the warmed dialysate. Modest decreases in blood pressure, as is the case with this client, can be maintained with rate adjustment, Trendelenburg positioning, and a fluid bolus. If the blood pressure drops considerably after two boluses and cooling dialysate, the hemodialysis can be stopped and the health care provider contacted.

6. A nurse plans care for a client who has chronic obstructive pulmonary disease and thick, tenacious secretions. Which interventions should the nurse include in this client's plan of care? (Select all that apply.) a. Ask the client to drink 2 liters of fluids daily. b. Add humidity to the prescribed oxygen. c. Suction the client every 2 to 3 hours. d. Use a vibrating positive expiratory pressure device. e. Encourage diaphragmatic breathing.

ANS: A, B, D Interventions to decrease thick tenacious secretions include maintaining adequate hydration and providing humidified oxygen. These actions will help to thin secretions, making them easier to remove by coughing. The use of a vibrating positive expiratory pressure device can also help clients remove thick secretions. Although suctioning may assist with the removal of secretions, frequent suctioning can cause airway trauma and does not support the client's ability to successfully remove secretions through normal coughing. Diaphragmatic breathing is not used to improve the removal of thick secretions. DIF: Applying/Application REF: 564 KEY: Respiratory distress/failure MSC: IntegratedProcess:NursingProcess:Implementation NOT: Client Needs Category: Physiological Integrity: Reduction of Risk Potential

2. A nurse assesses a client with nephrotic syndrome. For which clinical manifestations should the nurse assess? (Select all that apply.) a. Proteinuria b. Hypoalbuminemia c. Dehydration d. Lipiduria e. Dysuria f. Costovertebral angle (CV A) tenderness

ANS: A, B, D Nephrotic syndrome is caused by glomerular damage and is characterized by proteinuria (protein level higher than 3.5 g/24 hr), hypoalbuminemia, edema, and lipiduria. Fluid overload leading to edema and hypertension is common with nephrotic syndrome; dehydration does not occur. Dysuria is present with cystitis. CVA tenderness is present with inflammatory changes in the kidney.

7. A client is unsure of the decision to undergo peritoneal dialysis (PD) and wishes to discuss the advantages of this treatment with the nurse. Which statements by the nurse are accurate regarding PD? (Select all that apply.) a. "You will not need vascular access to perform PD." b. "There is less restriction of protein and fluids." c. "You will have no risk for infection with PD." d. "You have flexible scheduling for the exchanges." e. "It takes less time than hemodialysis treatments."

ANS: A, B, D PD is based on exchanges of waste, fluid, and electrolytes in the peritoneal cavity. There is no need for vascular access. Protein is lost in the exchange, which allows for more protein and fluid in the diet. There is flexibility in the time for exchanges, but the treatment takes a longer period of time compared to hemodialysis. There still is risk for infection with PD, especially peritonitis.

5. A nurse cares for clients with urinary incontinence. Which types of incontinence are correctly paired with their clinical manifestation? (Select all that apply.) a. Stress incontinence - Urine loss with physical exertion b. Urge incontinence - Large amount of urine with each occurrence c. Functional incontinence - Urine loss results from abnormal detrusor contractions d. Overflow incontinence - Constant dribbling of urine e. Reflex incontinence - Leakage of urine without lower urinary tract disorder

ANS: A, B, D Stress incontinence is a loss of urine with physical exertion, coughing, sneezing, or exercising. Urge incontinence presents with an abrupt and strong urge to void and usually has a large amount of urine released with each occurrence. Overflow incontinence occurs with bladder distention and results in a constant dribbling of urine. Functional incontinence is the leakage of urine caused by factors other than a disorder of the lower urinary tract. Reflex incontinence results from abnormal detrusor contractions from a neurologic abnormality.

6. A nurse prepares to discharge a client who has heart failure. Based on the Heart Failure Core Measure Set, which actions should the nurse complete prior to discharging this client? (Select all that apply.) a. Teach the client about dietary restrictions. b. Ensure the client is prescribed an angiotensin-converting enzyme (ACE) inhibitor. c. Encourage the client to take a baby aspirin each day. d. Confirm that an echocardiogram has been completed. e. Consult a social worker for additional resources.

ANS: A, B, D The Heart Failure Core Measure Set includes discharge instructions on diet, activity, medications, weight monitoring and plan for worsening symptoms, evaluation of left ventricular systolic function (usually with an echocardiogram), and prescribing an ACE inhibitor or angiotensin receptor blocker. Aspirin is not part of the Heart Failure Core Measure Set and is usually prescribed for clients who experience a myocardial infarction. Although the nurse may consult the social worker or case manager for additional resources, this is not part of the Core Measures. DIF: Understanding/Comprehension REF: 689 KEY: Heart failure| discharge| Core Measures| The Joint Commission MSC: Integrated Process: Nursing Process: Analysis NOT: Client Needs Category: Safe and Effective Care Environment: Management of Care

1. A nurse reviews a client's laboratory results. Which results from the client's urinalysis should the nurse identify as normal? (Select all that apply.) a. pH:6 b. Specific gravity: 1.015 c. Protein: 1.2 mg/dL d. Glucose: negative e. Nitrate: small f. Leukocyte esterase: positive

ANS: A, B, D The pH, specific gravity, and glucose are all within normal ranges. The other values are abnormal.

7. A nurse prepares to discharge a client who has heart failure. Which questions should the nurse ask to ensure this client's safety prior to discharging home? (Select all that apply.) a. "Are your bedroom and bathroom on the first floor?" b. "What social support do you have at home?" c. "Will you be able to afford your oxygen therapy?" d. "What spiritual beliefs may impact your recovery?" e. "Are you able to accurately weigh yourself at home?"

ANS: A, B, D To ensure safety upon discharge, the nurse should assess for structural barriers to functional ability, such as stairs. The nurse should also assess the client's available social support, which may include family, friends, and home health services. The client's ability to adhere to medication and treatments, including daily weights, should also be reviewed. The other questions do not address the client's safety upon discharge. DIF: Applying/Application REF: 689 KEY: Heart failure| discharge| safety MSC: Integrated Process: Nursing Process: Analysis NOT: Client Needs Category: Safe and Effective Care Environment: Safety and Infection Control

3. A nursing student studying acute coronary syndromes learns that the pain of a myocardial infarction (MI) differs from stable angina in what ways? (Select all that apply.) a. Accompanied by shortness of breath b. Feelings of fear or anxiety c. Lasts less than 15 minutes d. No relief from taking nitroglycerin e. Pain occurs without known cause

ANS: A, B, D, E The pain from an MI is often accompanied by shortness of breath and fear or anxiety. It lasts longer than 15 minutes and is not relieved by nitroglycerin. It occurs without a known cause such as exertion. DIF: Remembering/Knowledge REF: 762 KEY: Coronary artery disease| pathophysiology MSC: IntegratedProcess:Teaching/Learning NOT: Client Needs Category: Physiological Integrity: Physiological Adaptation

5. A nurse is caring for a client who is on mechanical ventilation. What actions will promote comfort in this client? (Select all that apply.) a. Allow visitors at the client's bedside. b. Ensure the client can communicate if awake. c. Keep the television tuned to a favorite channel. d. Provide back and hand massages when turning. e. Turn the client every 2 hours or more.

ANS: A, B, D, E There are many basic care measures that can be employed for the client who is on a ventilator. Allowing visitation, providing a means of communication, massaging the client's skin, and routinely turning and repositioning the client are some of them. Keeping the TV on will interfere with sleep and rest.

6. A nurse teaches a client with a history of calcium phosphate urinary stones. Which statements should the nurse include in this client's dietary teaching? (Select all that apply.) a. "Limit your intake of food high in animal protein." b. "Read food labels to help minimize your sodium intake." c. "Avoid spinach, black tea, and rhubarb." d. "Drink white wine or beer instead of red wine." e. "Reduce your intake of milk and other dairy products."

ANS: A, B, E Clients with calcium phosphate urinary stones should be taught to limit the intake of foods high in animal protein, sodium, and calcium. Clients with calcium oxalate stones should avoid spinach, black tea, and rhubarb. Clients with uric acid stones should avoid red wine.

MULTIPLE RESPONSE 1. A nurse is assessing a client with left-sided heart failure. For which clinical manifestations should the nurse assess? (Select all that apply.) a. Pulmonary crackles b. Confusion, restlessness c. Pulmonary hypertension d. Dependent edema e. Cough that worsens at night

ANS: A, B, E Left-sided heart failure occurs with a decrease in contractility of the heart or an increase in afterload. Most of the signs will be noted in the respiratory system. Right-sided heart failure occurs with problems from the pulmonary vasculature onward including pulmonary hypertension. Signs will be noted before the right atrium or ventricle including dependent edema. DIF: Remembering/Knowledge REF: 682 KEY: Heart failure| assessment/diagnostic examination MSC: IntegratedProcess:NursingProcess:Assessment NOT: Client Needs Category: Physiological Integrity: Physiological Adaptation

4. A nurse prepares a client for a percutaneous kidney biopsy. Which actions should the nurse take prior to this procedure? (Select all that apply.) a. Keep the client NPO for 4 to 6 hours. b. Obtain coagulation study results. c. Maintain strict bedrest in a supine position. d. Assess for blood in the client's urine. e. Administer antihypertensive medications.

ANS: A, B, E Prior to a percutaneous kidney biopsy, the client should be NPO for 4 to 6 hours. Coagulation studies should be completed to prevent bleeding after the biopsy. Blood pressure medications should be administered to prevent hypertension before and after the procedure. There is no need to keep the client on bedrest or assess for blood in the client's urine prior to the procedure; these interventions should be implemented after a percutaneous kidney biopsy.

8. A nurse teaches a female client who has stress incontinence. Which statements should the nurse include about pelvic muscle exercises? (Select all that apply.) a. "When you start and stop your urine stream, you are using your pelvic muscles." b. "Tighten your pelvic muscles for a slow count of 10 and then relax for a slow count of 10." c. "Pelvic muscle exercises should only be performed sitting upright with your feet on the floor." d. "After you have been doing these exercises for a couple days, your control of urine will improve." e. "Like any other muscle in your body, you can make your pelvic muscles stronger by contracting them."

ANS: A, B, E The client should be taught that the muscles used to start and stop urination are pelvic muscles, and that pelvic muscles can be strengthened by contracting and relaxing them. The client should tighten pelvic muscles for a slow count of 10 and then relax the muscles for a slow count of 10, and perform this exercise 15 times while in lying-down, sitting-up, and standing positions. The client should begin to notice improvement in control of urine after several weeks of exercising the pelvic muscles.

2. A nurse teaches a client with a new permanent pacemaker. Which instructions should the nurse include in this client's teaching? (Select all that apply.) a. "Until your incision is healed, do not submerge your pacemaker. Only take showers." b. "Report any pulse rates lower than your pacemaker settings." c. "If you feel weak, apply pressure over your generator." d. "Have your pacemaker turned off before having magnetic resonance imaging (MRI)." e. "Do not lift your left arm above the level of your shoulder for 8 weeks."

ANS: A, B, E The client should not submerge in water until the site has healed; after the incision is healed, the client may take showers or baths without concern for the pacemaker. The client should be instructed to report changes in heart rate or rhythm, such as rates lower than the pacemaker setting or greater than 100 beats/min. The client should be advised of restrictions on physical activity for 8 weeks to allow the pacemaker to settle in place. The client should never apply pressure over the generator and should avoid tight clothing. The client should never have MRI because, whether turned on or off, the pacemaker contains metal. The client should be advised to inform all health care providers that he or she has a pacemaker. DIF: Applying/Application REF: 675 KEY: Cardiac electrical conduction| patient education MSC: IntegratedProcess:Teaching/Learning NOT: Client Needs Category: Health Promotion and Maintenance

2. A nurse evaluates laboratory results for a client with heart failure. Which results should the nurse expect? (Select all that apply.) a. Hematocrit: 32.8% b. Serum sodium: 130 mEq/L c. Serum potassium: 4.0 mEq/L d. Serum creatinine: 1.0 mg/dL e. Proteinuria f. Microalbuminuria

ANS: A, B, E, F A hematocrit of 32.8% is low (should be 42.6%), indicating a dilutional ratio of red blood cells to fluid. A serum sodium of 130 mEq/L is low because of hemodilution. Microalbuminuria and proteinuria are present, indicating a decrease in renal filtration. These are early warning signs of decreased compliance of the heart. The potassium level is on the high side of normal and the serum creatinine level is normal. DIF: Applying/Application REF: 683 KEY: Heart failure| assessment/diagnostic examination MSC: Integrated Process: Nursing Process: Evaluation NOT: Client Needs Category: Physiological Integrity: Reduction of Risk Potential

5. A nurse plans care for an older adult client. Which interventions should the nurse include in this client's plan of care to promote kidney health? (Select all that apply.) a. Ensure adequate fluid intake. b. Leave the bathroom light on at night. c. Encourage use of the toilet every 6 hours. d. Delegate bladder training instructions to the unlicensed assistive personnel (UAP). e. Provide thorough perineal care after each voiding. f. Assess for urinary retention and urinary tract infection.

ANS: A, B, E, F The nurse should ensure that the client receives adequate fluid intake and has adequate lighting to ambulate safely to the bathroom at night, encourage the client to use the toilet every 2 hours, provide thorough perineal care after each voiding, and assess for urinary retention and urinary tract infections. The nurse should not delegate any teaching to the UAP, including bladder training instructions. The UAP may participate in bladder training activities, including encouraging and assisting the client to the bathroom at specific times.

A nurse teaches a client with diabetes mellitus about foot care. Which statements should the nurse include in this client's teaching? (Select all that apply.) a. "Do not walk around barefoot." b. "Soak your feet in a tub each evening." c. "Trim toenails straight across with a nail clipper." d. "Treat any blisters or sores with Epsom salts." e. "Wash your feet every other day."

ANS: A, C Clients who have diabetes mellitus are at high risk for wounds on the feet secondary to peripheral neuropathy and poor arterial circulation. The client should be instructed to not walk around barefoot or wear sandals with open toes. These actions place the client at higher risk for skin breakdown of the feet. The client should be instructed to trim toenails straight across with a nail clipper. Feet should be washed daily with lukewarm water and soap, but feet should not be soaked in the tub. The client should contact the provider immediately if blisters or sores appear and should not use home remedies to treat these wounds.

5. A nurse collaborates with an unlicensed assistive personnel (UAP) to provide care for a client with congestive heart failure. Which instructions should the nurse provide to the UAP when delegating care for this client? (Select all that apply.) a. "Reposition the client every 2 hours." b. "Teach the client to perform deep-breathing exercises." c. "Accurately record intake and output." d. "Use the same scale to weigh the client each morning." e. "Place the client on oxygen if the client becomes short of breath."

ANS: A, C, D The UAP should reposition the client every 2 hours to improve oxygenation and prevent atelectasis. The UAP can also accurately record intake and output, and use the same scale to weigh the client each morning before breakfast. UAPs are not qualified to teach clients or assess the need for and provide oxygen therapy. DIF: Applying/Application REF: 684 KEY: Heart failure| delegation| interdisciplinary team| unlicensed assistive personnel (UAP) MSC: Integrated Process: Communication and Documentation NOT: Client Needs Category: Safe and Effective Care Environment: Management of Care

1. A nurse is providing pneumonia vaccinations in a community setting. Due to limited finances, the event organizers must limit giving the vaccination to priority groups. What clients would be considered a priority when administering the pneumonia vaccination? (Select all that apply.) a. 22-year-old client with asthma b. Client who had a cholecystectomy last year c. Client with well-controlled diabetes d. Healthy 72-year-old client e. Client who is taking medication for hypertension

ANS: A, C, D, E Clients over 65 years of age and any client (no matter what age) with a chronic health condition would be considered a priority for a pneumonia vaccination. Having a cholecystectomy a year ago does not qualify as a chronic health condition.

6. A nurse is assessing clients on a rehabilitation unit. Which clients are at greatest risk for asphyxiation related to inspissated oral and nasopharyngeal secretions? (Select all that apply.) a. A 24-year-old with a traumatic brain injury b. A 36-year-old who fractured his left femur c. A 58-year-old at risk for aspiration following radiation therapy d. A 66-year-old who is a quadriplegic and has a sacral ulcer e. An 80-year-old who is aphasic after a cerebral vascular accident

ANS: A, C, D, E Risk for asphyxiation related to inspissated oral and nasopharyngeal secretions is caused by poor oral hygiene. Clients at risk include those with altered mental status and level of consciousness (traumatic brain injury), dehydration, an inability to communicate (aphasic) and cough effectively (quadriplegic), and a risk of aspiration (aspiration precautions). The client with a fractured femur is at risk for a pulmonary embolism. DIF: Applying/Application REF: 536 KEY: Medical emergencies MSC: IntegratedProcess:NursingProcess:Assessment NOT: Client Needs Category: Safe and Effective Care Environment: Management of Care

A nurse assesses a client who is experiencing diabetic ketoacidosis (DKA). For which manifestations should the nurse monitor the client? (Select all that apply.) a. Deep and fast respirations b. Decreased urine output c. Tachycardia d. Dependent pulmonary crackles e. Orthostatic hypotension

ANS: A, C, E DKA leads to dehydration, which is manifested by tachycardia and orthostatic hypotension. Usually clients have Kussmaul respirations, which are fast and deep. Increased urinary output (polyuria) is severe. Because of diuresis and dehydration, peripheral edema and crackles do not occur.

7. A nurse cares for a client who is prescribed an intravenous prostacyclin agent. Which actions should the nurse take to ensure the client's safety while on this medication? (Select all that apply.) a. Keep an intravenous line dedicated strictly to the infusion. b. Teach the client that this medication increases pulmonary pressures. c. Ensure that there is always a backup drug cassette available. d. Start a large-bore peripheral intravenous line. e. Use strict aseptic technique when using the drug delivery system.

ANS: A, C, E Intravenous prostacyclin agents should be administered in a central venous catheter with a dedicated intravenous line for this medication. Death has been reported when the drug delivery system is interrupted; therefore, a backup drug cassette should also be available. The nurse should use strict aseptic technique when using the drug delivery system. The nurse should teach the client that this medication decreases pulmonary pressures and increases lung blood flow. DIF: Understanding/Comprehension REF: 571 KEY: Medication administration| safety MSC: Integrated Process: Nursing Process: Implementation NOT: Client Needs Category: Physiological Integrity: Pharmacological and Parenteral Therapies

3. A client is hospitalized in the oliguric phase of acute kidney injury (AKI) and is receiving tube feedings. The nurse is teaching the client's spouse about the kidney-specific formulation for the enteral solution compared to standard formulas. What components should be discussed in the teaching plan? (Select all that apply.) a. Lower sodium b. Higher calcium c. Lower potassium d. Higher phosphorus e. Higher calories

ANS: A, C, E Many clients with AKI are too ill to meet caloric goals and require tube feedings with kidney-specific formulas that are lower in sodium, potassium, and phosphorus, and higher in calories than are standard formulas.

2. A nurse is caring for a postoperative 70-kg client who had major blood loss during surgery. Which findings by the nurse should prompt immediate action to prevent acute kidney injury? (Select all that apply.) a. Urine output of 100 mL in 4 hours b. Urine output of 500 mL in 12 hours c. Large amount of sediment in the urine d. Amber, odorless urine e. Blood pressure of 90/60 mm Hg

ANS: A, C, E The low urine output, sediment, and blood pressure should be reported to the provider. Postoperatively, the nurse should measure intake and output, check the characteristics of the urine, and report sediment, hematuria, and urine output of less than 0.5 mL/kg/hour for 3 to 4 hours. A urine output of 100 mL is low, but a urine output of 500 mL in 12 hours should be within normal limits. Perfusion to the kidneys is compromised with low blood pressure. The amber odorless urine is normal.

2. A nurse is caring for a client who had coronary artery bypass grafting yesterday. What actions does the nurse delegate to the unlicensed assistive personnel (UAP)? (Select all that apply.) a. Assist the client to the chair for meals and to the bathroom. b. Encourage the client to use the spirometer every 4 hours. c. Ensure the client wears TED hose or sequential compression devices. d. Have the client rate pain on a 0-to-10 scale and report to the nurse. e. Take and record a full set of vital signs per hospital protocol.

ANS: A, C, E The nurse can delegate assisting the client to get up in the chair or ambulate to the bathroom, applying TEDs or sequential compression devices, and taking/recording vital signs. The spirometer should be used every hour the day after surgery. Assessing pain using a 0-to-10 scale is a nursing assessment, although if the client reports pain, the UAP should inform the nurse so a more detailed assessment is done. DIF: Applying/Application REF: 768 KEY: Coronary artery disease| coronary artery bypass graft| delegation| activity| unlicensed assistive personnel (UAP) MSC: Integrated Process: Communication and Documentation NOT: Client Needs Category: Safe and Effective Care Environment: Management of Care

A nurse cares for a client with a hypofunctioning anterior pituitary gland. Which hormones should the nurse expect to be affected by this condition? (Select all that apply.) a. Thyroid-stimulating hormone b. Vasopressin c. Follicle-stimulating hormone d. Calcitonin e. Growth hormone

ANS: A, C, E Thyroid-stimulating hormone, follicle-stimulating hormone, and growth hormone all are secreted by the anterior pituitary gland. Vasopressin is secreted from the posterior pituitary gland. Calcitonin is secreted from the thyroid gland.

2. A nurse teaches a client about self-catheterization in the home setting. Which statements should the nurse include in this client's teaching? (Select all that apply.) a. "Wash your hands before and after self-catheterization." b. "Use a large-lumen catheter for each catheterization." c. "Use lubricant on the tip of the catheter before insertion." d. "Self-catheterize at least twice a day or every 12 hours." e. "Use sterile gloves and sterile technique for the procedure." f. "Maintain a specific schedule for catheterization."

ANS: A, C, F The key points in self-catheterization include washing hands, using lubricants, and maintaining a regular schedule to avoid distention and retention of urine that leads to bacterial growth. A smaller rather than a larger lumen catheter is preferred. The client needs to catheterize more often than every 12 hours. Self-catheterization in the home is a clean procedure.

3. A nurse assesses a client who has facial trauma. Which assessment findings require immediate intervention? (Select all that apply.) a. Stridor b. Nasal stuffiness c. Edema of the cheek d. Ecchymosis behind the ear e. Eye pain f. Swollen chin

ANS: A, D Stridor is a sign of airway obstruction and requires immediate intervention. Ecchymosis, or bruising, behind the ear is called "battle sign" and indicates basilar skull fracture. Nasal stuffiness, edema of the cheek or chin, and eye pain do not interfere with respirations or neurologic function, and therefore are not priorities for immediate intervention. DIF: Applying/Application REF: 534 KEY: Trauma| medical emergencies MSC: IntegratedProcess:NursingProcess:Assessment NOT: Client Needs Category: Safe and Effective Care Environment: Management of Care

3. A nurse assesses a client recovering from a cystoscopy. Which assessment findings should alert the nurse to urgently contact the health care provider? (Select all that apply.) a. Decrease in urine output b. Tolerating oral fluids c. Prescription for metformin d. Blood clots present in the urine e. Burning sensation when urinating

ANS: A, D The nurse should monitor urine output and contact the provider if urine output decreases or becomes absent. The nurse should also assess for blood in the client's urine. The urine may be pink-tinged, but gross bleeding or blood clots should not be present. If bleeding is present, the nurse should urgently contact the provider. Tolerating oral fluids is a positive outcome and does not need intervention. Metformin would be a concern if the client received dye; no dye is used in a cystoscopy procedure. The client may experience a burning sensation when urinating after this procedure; this would not require a call to the provider.

4. After teaching a client with congestive heart failure (CHF), the nurse assesses the client's understanding. Which client statements indicate a correct understanding of the teaching related to nutritional intake? (Select all that apply.) a. "I'll read the nutritional labels on food items for salt content." b. "I will drink at least 3 liters of water each day." c. "Using salt in moderation will reduce the workload of my heart." d. "I will eat oatmeal for breakfast instead of ham and eggs." e. "Substituting fresh vegetables for canned ones will lower my salt intake."

ANS: A, D, E Nutritional therapy for a client with CHF is focused on decreasing sodium and water retention to decrease the workload of the heart. The client should be taught to read nutritional labels on all food items, omit table salt and foods high in sodium (e.g., ham and canned foods), and limit water intake to a normal 2 L/day. DIF: Applying/Application REF: 682 KEY: Heart failure| patient education MSC: Integrated Process: Teaching/Learning NOT: Client Needs Category: Physiological Integrity: Basic Care and Comfort

MULTIPLE RESPONSE 1. A nurse cares for a client with congestive heart failure who has a regular cardiac rhythm of 128 beats/min. For which physiologic alterations should the nurse assess? (Select all that apply.) a. Decrease in cardiac output b. Increase in cardiac output c. Decrease in blood pressure d. Increase in blood pressure e. Decrease in urine output f. Increase in urine output

ANS: A, D, E Elevated heart rates in a healthy client initially cause blood pressure and cardiac output to increase. However, in a client who has congestive heart failure or a client with long-term tachycardia, ventricular filling time, cardiac output, and blood pressure eventually decrease. As cardiac output and blood pressure decrease, urine output will fall. DIF: Applying/Application REF: 657 KEY: Cardiac electrical conduction| heart failure MSC: IntegratedProcess:NursingProcess:Assessment NOT: Client Needs Category: Physiological Integrity: Physiological Adaptation

A nurse assesses clients at a health fair. Which clients should the nurse counsel to be tested for diabetes? (Select all that apply.) a. 56-year-old African-American male b. Female with a 30-pound weight gain during pregnancy c. Male with a history of pancreatic trauma d. 48-year-old woman with a sedentary lifestyle e. Male with a body mass index greater than 25 kg/m2 f. 28-year-old female who gave birth to a baby weighing 9.2 pounds

ANS: A, D, E, F Risk factors for type 2 diabetes include certain ethnic/racial groups (African Americans, American Indians, Hispanics), obesity and physical inactivity, and giving birth to large babies. Pancreatic trauma and a 30-pound gestational weight gain are not risk factors.

A nurse cares for a client who is diagnosed with acute rejection 2 months after receiving a simultaneous pancreas-kidney transplant. The client states, "I was doing so well with my new organs, and the thought of having to go back to living on hemodialysis and taking insulin is so depressing." How should the nurse respond? a. "Following the drug regimen more closely would have prevented this." b. "One acute rejection episode does not mean that you will lose the new organs." c. "Dialysis is a viable treatment option for you and may save your life." d. "Since you are on the national registry, you can receive a second transplantation."

ANS: B An episode of acute rejection does not automatically mean that the client will lose the transplant. Pharmacologic manipulation of host immune responses at this time can limit damage to the organ and allow the graft to be maintained. The other statements either belittle the client or downplay his or her concerns. The client may not be a candidate for additional organ transplantation.

A nurse is teaching a client with diabetes mellitus who asks, "Why is it necessary to maintain my blood glucose levels no lower than about 60 mg/dL?" How should the nurse respond? a. "Glucose is the only fuel used by the body to produce the energy that it needs." b. "Your brain needs a constant supply of glucose because it cannot store it." c. "Without a minimum level of glucose, your body does not make red blood cells." d. "Glucose in the blood prevents the formation of lactic acid and prevents acidosis."

ANS: B Because the brain cannot synthesize or store significant amounts of glucose, a continuous supply from the body's circulation is needed to meet the fuel demands of the central nervous system. The nurse would want to educate the client to prevent hypoglycemia. The body can use other sources of fuel, including fat and protein, and glucose is not involved in the production of red blood cells. Glucose in the blood will encourage glucose metabolism but is not directly responsible for lactic acid formation.

After teaching a client with type 2 diabetes mellitus, the nurse assesses the client's understanding. Which statement made by the client indicates a need for additional teaching? a. "I need to have an annual appointment even if my glucose levels are in good control." b. "Since my diabetes is controlled with diet and exercise, I must be seen only if I am sick." c. "I can still develop complications even though I do not have to take insulin at this time." d. "If I have surgery or get very ill, I may have to receive insulin injections for a short time."

ANS: B Clients with diabetes need to be seen at least annually to monitor for long-term complications, including visual changes, microalbuminuria, and lipid analysis. The client may develop complications and may need insulin in the future.

A nurse reviews the medication list of a client with a 20-year history of diabetes mellitus. The client holds up the bottle of prescribed duloxetine (Cymbalta) and states, "My cousin has depression and is taking this drug. Do you think I'm depressed?" How should the nurse respond? a. "Many people with long-term diabetes become depressed after a while." b. "It's for peripheral neuropathy. Do you have burning pain in your feet or hands?" c. "This antidepressant also has anti-inflammatory properties for diabetic pain." d. "No. Many medications can be used for several different disorders."

ANS: B Damage along nerves causes peripheral neuropathy and leads to burning pain along the nerves. Many drugs, including duloxetine (Cymbalta), can be used to treat peripheral neuropathy. The nurse should assess the client for this condition and then should provide an explanation of why this drug is being used. This medication, although it is used for depression, is not being used for that reason in this case. Duloxetine does not have anti-inflammatory properties. Telling the client that many medications are used for different disorders does not provide the client with enough information to be useful.

A nurse cares for a client with diabetes mellitus who asks, "Why do I need to administer more than one injection of insulin each day?" How should the nurse respond? a. "You need to start with multiple injections until you become more proficient at self-injection." b. "A single dose of insulin each day would not match your blood insulin levels and your food intake patterns." c. "A regimen of a single dose of insulin injected each day would require that you eat fewer carbohydrates." d. "A single dose of insulin would be too large to be absorbed, predictably putting you at risk for insulin shock."

ANS: B Even when a single injection of insulin contains a combined dose of different-acting insulin types, the timing of the actions and the timing of food intake may not match well enough to prevent wide variations in blood glucose levels. One dose of insulin would not be appropriate even if the client decreased carbohydrate intake. Additional injections are not required to allow the client practice with injections, nor will one dose increase the client's risk of insulin shock.

A nurse cares for a client who has diabetes mellitus. The nurse administers 6 units of regular insulin and 10 units of NPH insulin at 0700. At which time should the nurse assess the client for potential problems related to the NPH insulin? a. 0800 b. 1600 c. 2000 d. 2300

ANS: B Neutral protamine Hagedorn (NPH) is an intermediate-acting insulin with an onset of 1.5 hours, peak of 4 to 12 hours, and duration of action of 22 hours. Checking the client at 0800 would be too soon. Checking the client at 2000 and 2300 would be too late. The nurse should check the client at 1600.

A nurse assesses a client with diabetes mellitus. Which clinical manifestation should alert the nurse to decreased kidney function in this client? a. Urine specific gravity of 1.033 b. Presence of protein in the urine c. Elevated capillary blood glucose level d. Presence of ketone bodies in the urine

ANS: B Renal dysfunction often occurs in the client with diabetes. Proteinuria is a result of renal dysfunction. Specific gravity is elevated with dehydration. Elevated capillary blood glucose levels and ketones in the urine are consistent with diabetes mellitus but are not specific to renal function.

A nurse develops a dietary plan for a client with diabetes mellitus and new-onset microalbuminuria. Which component of the client's diet should the nurse decrease? a. Carbohydrates b. Proteins c. Fats d. Total calories

ANS: B Restriction of dietary protein to 0.8 g/kg of body weight per day is recommended for clients with microalbuminuria to delay progression to renal failure. The client's diet does not need to be decreased in carbohydrates, fats, or total calories.

A nurse teaches a client about self-monitoring of blood glucose levels. Which statement should the nurse include in this client's teaching to prevent bloodborne infections? a. "Wash your hands after completing each test." b. "Do not share your monitoring equipment." c. "Blot excess blood from the strip with a cotton ball." d. "Use gloves when monitoring your blood glucose."

ANS: B Small particles of blood can adhere to the monitoring device, and infection can be transported from one user to another. Hepatitis B in particular can survive in a dried state for about a week. The client should be taught to avoid sharing any equipment, including the lancet holder. The client should be taught to wash his or her hands before testing. The client would not need to blot excess blood away from the strip or wear gloves.

A nurse reviews laboratory results for a client with diabetes mellitus who is prescribed an intensified insulin regimen: • Fasting blood glucose: 75 mg/dL • Postprandial blood glucose: 200 mg/dL • Hemoglobin A1c level: 5.5% How should the nurse interpret these laboratory findings? a. Increased risk for developing ketoacidosis b. Good control of blood glucose c. Increased risk for developing hyperglycemia d. Signs of insulin resistance

ANS: B The client is maintaining blood glucose levels within the defined ranges for goals in an intensified regimen. Because the client's glycemic control is good, he or she is not at higher risk for ketoacidosis or hyperglycemia and is not showing signs of insulin resistance.

A nurse cares for a client who is prescribed pioglitazone (Actos). After 6 months of therapy, the client reports that his urine has become darker since starting the medication. Which action should the nurse take? a. Assess for pain or burning with urination. b. Review the client's liver function study results. c. Instruct the client to increase water intake. d. Test a sample of urine for occult blood.

ANS: B Thiazolidinediones (including pioglitazone) can affect liver function; liver function should be assessed at the start of therapy and at regular intervals while the client continues to take these drugs. Dark urine is one indicator of liver impairment because bilirubin is increased in the blood and is excreted in the urine. The nurse should check the client's most recent liver function studies. The nurse does not need to assess for pain or burning with urination and does not need to check the urine for occult blood. The client does not need to be told to increase water intake.

A nurse teaches a client with diabetes mellitus about sick day management. Which statement should the nurse include in this client's teaching? a. "When ill, avoid eating or drinking to reduce vomiting and diarrhea." b. "Monitor your blood glucose levels at least every 4 hours while sick." c. "If vomiting, do not use insulin or take your oral antidiabetic agent." d. "Try to continue your prescribed exercise regimen even if you are sick."

ANS: B When ill, the client should monitor his or her blood glucose at least every 4 hours. The client should continue taking the medication regimen while ill. The client should continue to eat and drink as tolerated but should not exercise while sick.

A nurse assesses a client who has diabetes mellitus. Which arterial blood gas values should the nurse identify as potential ketoacidosis in this client? a. pH 7.38, HCO3- 22 mEq/L, PCO2 38 mm Hg, PO2 98 mm Hg b. pH 7.28, HCO3- 18 mEq/L, PCO2 28 mm Hg, PO2 98 mm Hg c. pH 7.48, HCO3- 28 mEq/L, PCO2 38 mm Hg, PO2 98 mm Hg d. pH 7.32, HCO3- 22 mEq/L, PCO2 58 mm Hg, PO2 88 mm Hg

ANS: B When the lungs can no longer offset acidosis, the pH decreases to below normal. A client who has diabetic ketoacidosis would present with arterial blood gas values that show primary metabolic acidosis with decreased bicarbonate levels and a compensatory respiratory alkalosis with decreased carbon dioxide levels.

3. Which teaching point is most important for the client with bacterial pharyngitis? a. Gargle with warm salt water. b. Take all antibiotics as directed. c. Use a humidifier in the bedroom. d. Wash hands frequently.

ANS: B Any client on antibiotics must be instructed to complete the entire course of antibiotics. Not completing them can lead to complications or drug-resistant strains of bacteria. The other instructions are appropriate, just not the most important.

3. Which teaching point is most important for the client with bacterial pharyngitis? a. Gargle with warm salt water. b. Take all antibiotics as directed. c. Use a humidifier in the bedroom. d. Wash hands frequently.

ANS: B Any client on antibiotics must be instructed to complete the entire course of antibiotics. Not completing them can lead to complications or drug-resistant strains of bacteria. The other instructions are appropriate, just not the most important.

3. A nurse is assessing clients on a medical-surgical unit. Which client should the nurse identify as being at greatest risk for atrial fibrillation? a. A 45-year-old who takes an aspirin daily b. A 50-year-old who is post coronary artery bypass graft surgery c. A 78-year-old who had a carotid endarterectomy d. An 80-year-old with chronic obstructive pulmonary disease

ANS: B Atrial fibrillation occurs commonly in clients with cardiac disease and is a common occurrence after coronary artery bypass graft surgery. The other conditions do not place these clients at higher risk for atrial fibrillation. DIF: Applying/Application REF: 666 KEY: Health screening| cardiac electrical conduction MSC: IntegratedProcess:NursingProcess:Assessment NOT: Client Needs Category: Safe and Effective Care Environment: Management of Care

4. A client is on intravenous heparin to treat a pulmonary embolism. The client's most recent partial thromboplastin time (PTT) was 25 seconds. What order should the nurse anticipate? a. Decrease the heparin rate. b. Increase the heparin rate. c. No change to the heparin rate. d. Stop heparin; start warfarin (Coumadin).

ANS: B For clients on heparin, a PTT of 1.5 to 2.5 times the normal value is needed to demonstrate the heparin is working. A normal PTT is 25 to 35 seconds, so this client's PTT value is too low. The heparin rate needs to be increased. Warfarin is not indicated in this situation.

3. After teaching a client with early polycystic kidney disease (PKD) about nutritional therapy, the nurse assesses the client's understanding. Which statement made by the client indicates a correct understanding of the teaching? a. "I will take a laxative every night before going to bed." b. "I must increase my intake of dietary fiber and fluids." c. "I shall only use salt when I am cooking my own food." d. "I'll eat white bread to minimize gastrointestinal gas."

ANS: B Clients with PKD often have constipation, which can be managed with increased fiber, exercise, and drinking plenty of water. Laxatives should be used cautiously. Clients with PKD should be on a restricted salt diet, which includes not cooking with salt. White bread has a low fiber count and would not be included in a high-fiber diet.

21. The nurse is caring for a client on the medical-surgical unit who suddenly becomes unresponsive and has no pulse. The cardiac monitor shows the rhythm below: After calling for assistance and a defibrillator, which action should the nurse take next? a. Perform a pericardial thump. b. Initiate cardiopulmonary resuscitation (CPR). c. Start an 18-gauge intravenous line. d. Ask the client's family about code status.

ANS: B The client's rhythm is ventricular fibrillation. This is a lethal rhythm that is best treated with immediate defibrillation. While the nurse is waiting for the defibrillator to arrive, the nurse should start CPR. A pericardial thump is not a treatment for ventricular fibrillation. If the client does not already have an IV, other members of the team can insert one after defibrillation. The client's code status should already be known by the nurse prior to this event. DIF: Applying/Application REF: 671 KEY: Cardiac electrical conduction| medical emergency MSC: IntegratedProcess:NursingProcess:Assessment NOT: Client Needs Category: Physiological Integrity: Physiological Adaptation

22. A nurse teaches a client with heart failure about energy conservation. Which statement should the nurse include in this client's teaching? a. "Walk until you become short of breath, and then walk back home." b. "Gather everything you need for a chore before you begin." c. "Pull rather than push or carry items heavier than 5 pounds." d. "Take a walk after dinner every day to build up your strength."

ANS: B A client who has heart failure should be taught to conserve energy. Gathering all supplies needed for a chore at one time decreases the amount of energy needed. The client should not walk until becoming short of breath because he or she may not make it back home. Pushing a cart takes less energy than pulling or lifting. Although walking after dinner may help the client, the nurse should teach the client to complete activities when he or she has the most energy. This is usually in the morning. DIF: Applying/Application REF: 696 KEY: Heart failure| functional ability| patient education MSC: IntegratedProcess:Teaching/Learning NOT: Client Needs Category: Health Promotion and Maintenance

7. A client has intra-arterial blood pressure monitoring after a myocardial infarction. The nurse notes the client's heart rate has increased from 88 to 110 beats/min, and the blood pressure dropped from 120/82 to 100/60 mm Hg. What action by the nurse is most appropriate? a. Allow the client to rest quietly. b. Assess the client for bleeding. c. Document the findings in the chart. d. Medicate the client for pain.

ANS: B A major complication related to intra-arterial blood pressure monitoring is hemorrhage from the insertion site. Since these vital signs are out of the normal range, are a change, and are consistent with blood loss, the nurse should assess the client for any bleeding associated with the arterial line. The nurse should document the findings after a full assessment. The client may or may not need pain medication and rest; the nurse first needs to rule out any emergent bleeding. DIF: Applying/Application REF: 771 KEY: Coronary artery disease| intra-arterial blood pressure monitoring| equipment safety| vitalsigns MSC: IntegratedProcess:NursingProcess:Assessment NOT: Client Needs Category: Physiological Integrity: Reduction of Risk Potential

10. A nurse cares for a client who has elevated levels of antidiuretic hormone (ADH). Which disorder should the nurse identify as a trigger for the release of this hormone? a. Pneumonia b. Dehydration c. Renal failure d. Edema

ANS: B ADH increases tubular permeability to water, leading to absorption of more water into the capillaries. ADH is triggered by a rising extracellular fluid osmolarity, as occurs in dehydration. Pneumonia, renal failure, and edema would not trigger the release of ADH.

7. After administering newly prescribed captopril (Capoten) to a client with heart failure, the nurse implements interventions to decrease complications. Which priority intervention should the nurse implement for this client? a. Provide food to decrease nausea and aid in absorption. b. Instruct the client to ask for assistance when rising from bed. c. Collaborate with unlicensed assistive personnel to bathe the client. d. Monitor potassium levels and check for symptoms of hypokalemia.

ANS: B Administration of the first dose of angiotensin-converting enzyme (ACE) inhibitors is often associated with hypotension, usually termed first-dose effect. The nurse should instruct the client to seek assistance before arising from bed to prevent injury from postural hypotension. ACE inhibitors do not need to be taken with food. Collaboration with unlicensed assistive personnel to provide hygiene is not a priority. The client should be encouraged to complete activities of daily living as independently as possible. The nurse should monitor for hyperkalemia, not hypokalemia, especially if the client has renal insufficiency secondary to heart failure. DIF: Applying/Application REF: 685 KEY: Heart failure| angiotensin-converting enzyme (ACE) inhibitor| medication| patient education MSC: Integrated Process: Nursing Process: Implementation NOT: Client Needs Category: Physiological Integrity: Pharmacological and Parenteral Therapies

12. A nurse assists with the cardioversion of a client experiencing acute atrial fibrillation. Which action should the nurse take prior to the initiation of cardioversion? a. Administer intravenous adenosine. b. Turn off oxygen therapy. c. Ensure a tongue blade is available. d. Position the client on the left side.

ANS: B For safety during cardioversion, the nurse should turn off any oxygen therapy to prevent fire. The other interventions are not appropriate for a cardioversion. The client should be placed in a supine position. DIF: Remembering/Knowledge REF: 668 KEY: Assessment/diagnostic examination| safety MSC: Integrated Process: Nursing Process: Implementation NOT: Client Needs Category: Safe and Effective Care Environment: Safety and Infection Control

2. A client received tissue plasminogen activator (t-PA) after a myocardial infarction and now is on an intravenous infusion of heparin. The client's spouse asks why the client needs this medication. What response by the nurse is best? a. "The t-PA didn't dissolve the entire coronary clot." b. "The heparin keeps that artery from getting blocked again." c. "Heparin keeps the blood as thin as possible for a longer time." d. "The heparin prevents a stroke from occurring as the t-PA wears off."

ANS: B After the original intracoronary clot has dissolved, large amounts of thrombin are released into the bloodstream, increasing the chance of the vessel reoccluding. The other statements are not accurate. Heparin is not a "blood thinner," although laypeople may refer to it as such. DIF: Understanding/Comprehension REF: 768 KEY: Coronary artery disease| thrombolytic agents| patient education MSC: IntegratedProcess:Teaching/Learning NOT: Client Needs Category: Physiological Integrity: Pharmacological and Parenteral Therapies

12. A nurse provides health screening for a community health center with a large population of African-American clients. Which priority assessment should the nurse include when working with this population? a. Measure height and weight. b. Assess blood pressure. c. Observe for any signs of abuse. d. Ask about medications.

ANS: B All interventions are important for the visiting nurse to accomplish. However, African Americans have a high rate of hypertension leading to end-stage renal disease. Each encounter that the nurse has with an African-American client provides a chance to detect hypertension and treat it. If the client is already on antihypertensive medication, assessing blood pressure monitors therapy.

2. A nurse reviews the laboratory findings of a client with a urinary tract infection. The laboratory report notes a "shift to the left" in a client's white blood cell count. Which action should the nurse take? a. Request that the laboratory perform a differential analysis on the white blood cells. b. Notify the provider and start an intravenous line for parenteral antibiotics. c. Collaborate with the unlicensed assistive personnel (UAP) to strain the client's urine for renal calculi. d. Assess the client for a potential allergic reaction and anaphylactic shock.

ANS: B An increase in band cells creates a "shift to the left." A left shift most commonly occurs with urosepsis and is seen rarely with uncomplicated urinary tract infections. The nurse will be administering antibiotics, most likely via IV, so he or she should notify the provider and prepare to give the antibiotics. The shift to the left is part of a differential white blood cell count. The nurse would not need to strain urine for stones. Allergic reactions are associated with elevated eosinophil cells, not band cells.

A nurse teaches an older adult with a decreased production of estrogen. Which statement should the nurse include in this client's teaching to decrease injury? a. "Drink at least 2 liters of fluids each day." b. "Walk around the neighborhood for daily exercise." c. "Bathe your perineal area twice a day." d. "You should check your blood glucose before meals."

ANS: B An older adult client with decreased production of estrogen is at risk for decreased bone density and fractures. The nurse should encourage the client to participate in weight-bearing exercises such as walking. Drinking fluids and performing perineal care will decrease vaginal drying but not decrease injury. Older adults often have a decreased glucose tolerance, but this is not related to a decrease in estrogen.

24. A client is having a peritoneal dialysis treatment. The nurse notes an opaque color to the effluent. What is the priority action by the nurse? a. Warm the dialysate solution in a microwave before instillation. b. Take a sample of the effluent and send to the laboratory. c. Flush the tubing with normal saline to maintain patency of the catheter. d. Check the peritoneal catheter for kinking and curling.

ANS: B An opaque or cloudy effluent is the first sign of peritonitis. A sample of the effluent would need to be sent to the laboratory for culture and sensitivity in order to administer the correct antibiotic. Warming the dialysate in a microwave and flushing the tubing are not safe actions by the nurse. Checking the catheter for obstruction is a viable option but will not treat the peritonitis.

2. A nurse cares for a client with arthritis who reports frequent asthma attacks. Which action should the nurse take first? a. Review the client's pulmonary function test results. b. Ask about medications the client is currently taking. c. Assess how frequently the client uses a bronchodilator. d. Consult the provider and request arterial blood gases.

ANS: B Aspirin and other nonsteroidal anti-inflammatory drugs (NSAIDs) can trigger asthma in some people. This results from increased production of leukotriene when aspirin or NSAIDs suppress other inflammatory pathways and is a high priority given the client's history. Reviewing pulmonary function test results will not address the immediate problem of frequent asthma attacks. This is a good intervention for reviewing response to bronchodilators. Questioning the client about the use of bronchodilators will address interventions for the attacks but not their cause. Reviewing arterial blood gas results would not be of use in a client between attacks because many clients are asymptomatic when not having attacks. DIF: Applying/Application REF: 553 KEY: Respiratory distress/failure| medication MSC: IntegratedProcess:NursingProcess:Assessment NOT: Client Needs Category: Physiological Integrity: Pharmacological and Parenteral Therapies

24. A nurse assesses a client who has mitral valve regurgitation. For which cardiac dysrhythmia should the nurse assess? a. Preventricular contractions b. Atrial fibrillation c. Symptomatic bradycardia d. Sinus tachycardia

ANS: B Atrial fibrillation is a clinical manifestation of mitral valve regurgitation and stenosis. Preventricular contractions and bradycardia are not associated with valvular problems. These are usually identified in clients with electrolyte imbalances, myocardial infarction, and sinus node problems. Sinus tachycardia is a manifestation of aortic regurgitation due to a decrease in cardiac output. DIF: Understanding/Comprehension REF: 692 KEY: Valve disorder| cardiac dysrhythmia MSC: IntegratedProcess:NursingProcess:Assessment NOT: Client Needs Category: Physiological Integrity: Reduction of Risk Potential

5. A nurse evaluates prescriptions for a client with chronic atrial fibrillation. Which medication should the nurse expect to find on this client's medication administration record to prevent a common complication of this condition? a. Sotalol (Betapace) b. Warfarin (Coumadin) c. Atropine (Sal-Tropine) d. Lidocaine (Xylocaine)

ANS: B Atrial fibrillation puts clients at risk for developing emboli. Clients at risk for emboli are treated with anticoagulants, such as heparin, enoxaparin, or warfarin. Sotalol, atropine, and lidocaine are not appropriate for this complication. DIF: Applying/Application REF: 667 KEY: Cardiac electrical conduction| medication MSC: Integrated Process: Nursing Process: Analysis NOT: Client Needs Category: Physiological Integrity: Pharmacological and Parenteral Therapies

2. A nurse cares for a client who has a heart rate averaging 56 beats/min with no adverse symptoms. Which activity modification should the nurse suggest to avoid further slowing of the heart rate? a. "Make certain that your bath water is warm." b. "Avoid straining while having a bowel movement." c. "Limit your intake of caffeinated drinks to one a day." d. "Avoid strenuous exercise such as running."

ANS: B Bearing down strenuously during a bowel movement is one type of Valsalva maneuver, which stimulates the vagus nerve and results in slowing of the heart rate. Such a response is not desirable in a person who has bradycardia. The other instructions are not appropriate for this condition. DIF: Applying/Application REF: 663 KEY: Functional ability MSC: Integrated Process: Nursing Process: Implementation NOT: Client Needs Category: Physiological Integrity: Basic Care and Comfort

27. A client is recovering from a kidney transplant. The client's urine output was 1500 mL over the last 12-hour period since transplantation. What is the priority assessment by the nurse? a. Checking skin turgor b. Taking blood pressure c. Assessing lung sounds d. Weighing the client

ANS: B By taking blood pressure, the nurse is assessing for hypotension that could compromise perfusion to the new kidney. The nurse then should notify the provider immediately. Skin turgor, lung sounds, and weight could give information about the fluid status of the client, but they are not the priority assessment.

A nurse cares for a client who has excessive catecholamine release. Which assessment finding should the nurse correlate with this condition? a. Decreased blood pressure b. Increased pulse c. Decreased respiratory rate d. Increased urine output

ANS: B Catecholamines are responsible for the fight-or-flight stress response. Activation of the sympathetic nervous system can be correlated with tachycardia. Catecholamines do not decrease blood pressure or respiratory rate, nor do they increase urine output.

23. A nurse administers medications to a client who has asthma. Which medication classification is paired correctly with its physiologic response to the medication? a. Bronchodilator - Stabilizes the membranes of mast cells and prevents the release of inflammatory mediators b. Cholinergic antagonist - Causes bronchodilation by inhibiting the parasympathetic nervous system c. Corticosteroid - Relaxes bronchiolar smooth muscles by binding to and activating pulmonary beta2 receptors d. Cromone - Disrupts the production of pathways of inflammatory mediators

ANS: B Cholinergic antagonist drugs cause bronchodilation by inhibiting the parasympathetic nervous system. This allows the sympathetic nervous system to dominate and release norepinephrine that actives beta2 receptors. Bronchodilators relax bronchiolar smooth muscles by binding to and activating pulmonary beta2 receptors. Corticosteroids disrupt the production of pathways of inflammatory mediators. Cromones stabilize the membranes of mast cells and prevent the release of inflammatory mediators. DIF: Remembering/Knowledge REF: 554 KEY: Medications MSC: Integrated Process: Nursing Process: Analysis NOT: Client Needs Category: Physiological Integrity: Pharmacological and Parenteral Therapies

14. After teaching a client who is being discharged home after mitral valve replacement surgery, the nurse assesses the client's understanding. Which client statement indicates a need for additional teaching? a. "I'll be able to carry heavy loads after 6 months of rest." b. "I will have my teeth cleaned by my dentist in 2 weeks." c. "I must avoid eating foods high in vitamin K, like spinach." d. "I must use an electric razor instead of a straight razor to shave."

ANS: B Clients who have defective or repaired valves are at high risk for endocarditis. The client who has had valve surgery should avoid dental procedures for 6 months because of the risk for endocarditis. When undergoing a mitral valve replacement surgery, the client needs to be placed on anticoagulant therapy to prevent vegetation forming on the new valve. Clients on anticoagulant therapy should be instructed on bleeding precautions, including using an electric razor. If the client is prescribed warfarin, the client should avoid foods high in vitamin K. Clients recovering from open heart valve replacements should not carry anything heavy for 6 months while the chest incision and muscle heal. DIF: Applying/Application REF: 694 KEY: Valve disorder| patient education| hygiene MSC: IntegratedProcess:Teaching/Learning NOT: Client Needs Category: Physiological Integrity: Reduction of Risk Potential

20. A nurse assesses a client who has a history of heart failure. Which question should the nurse ask to assess the extent of the client's heart failure? a. "Do you have trouble breathing or chest pain?" b. "Are you able to walk upstairs without fatigue?" c. "Do you awake with breathlessness during the night?" d. "Do you have new-onset heaviness in your legs?"

ANS: B Clients with a history of heart failure generally have negative findings, such as shortness of breath. The nurse needs to determine whether the client's activity is the same or worse, or whether the client identifies a decrease in activity level. Trouble breathing, chest pain, breathlessness at night, and peripheral edema are symptoms of heart failure, but do not provide data that can determine the extent of the client's heart failure. DIF: Applying/Application REF: 682 KEY: Heart failure| functional ability| respiratory distress/failure MSC: IntegratedProcess:NursingProcess:Assessment NOT: Client Needs Category: Physiological Integrity: Physiological Adaptation

4. A nurse assesses a client with atrial fibrillation. Which manifestation should alert the nurse to the possibility of a serious complication from this condition? a. Sinus tachycardia b. Speech alterations c. Fatigue d. Dyspnea with activity

ANS: B Clients with atrial fibrillation are at risk for embolic stroke. Evidence of embolic events includes changes in mentation, speech, sensory function, and motor function. Clients with atrial fibrillation often have a rapid ventricular response as a result. Fatigue is a nonspecific complaint. Clients with atrial fibrillation often have dyspnea as a result of the decreased cardiac output caused by the rhythm disturbance. DIF: Applying/Application REF: 666 KEY: Cardiac electrical conduction| vascular perfusion MSC: IntegratedProcess:NursingProcess:Assessment NOT: Client Needs Category: Physiological Integrity: Reduction of Risk Potential

8. An emergency department nurse assesses a client with kidney trauma and notes that the client's abdomen is tender and distended and blood is visible at the urinary meatus. Which prescription should the nurse consult the provider about before implementation? a. Assessing vital signs every 15 minutes b. Inserting an indwelling urinary catheter c. Administering intravenous fluids at 125 mL/hr d. Typing and crossmatching for blood products

ANS: B Clients with blood at the urinary meatus should not have a urinary catheter inserted via the urethra before additional diagnostic studies are done. The urethra could be torn. The nurse should question the provider about the need for a catheter; if one is needed, the provider can insert a suprapubic catheter. The nurse should monitor the client's vital signs closely, send blood for type and crossmatch in case the client needs blood products, and administer intravenous fluids.

20. The nurse is teaching the main principles of hemodialysis to a client with chronic kidney disease. Which statement by the client indicates a need for further teaching by the nurse? a. "My sodium level changes by movement from the blood into the dialysate." b. "Dialysis works by movement of wastes from lower to higher concentration." c. "Extra fluid can be pulled from the blood by osmosis." d. "The dialysate is similar to blood but without any toxins."

ANS: B Dialysis works using the passive transfer of toxins by diffusion. Diffusion is the movement of molecules from an area of higher concentration to an area of lower concentration. The other statements show a correct understanding about hemodialysis.

12. A nurse assesses a client with mitral valve stenosis. What clinical manifestation should alert the nurse to the possibility that the client's stenosis has progressed? a. Oxygen saturation of 92% b. Dyspnea on exertion c. Muted systolic murmur d. Upper extremity weakness

ANS: B Dyspnea on exertion develops as the mitral valvular orifice narrows and pressure in the lungs increases. The other manifestations do not relate to the progression of mitral valve stenosis. DIF: Applying/Application REF: 688 KEY: Valve disorder| respiratory distress/failure MSC: IntegratedProcess:NursingProcess:Assessment NOT: Client Needs Category: Physiological Integrity: Reduction of Risk Potential

18. A student nurse is preparing to administer enoxaparin (Lovenox) to a client. What action by the student requires immediate intervention by the supervising nurse? a. Assessing the client's platelet count b. Choosing an 18-gauge, 2-inch needle c. Not aspirating prior to injection d. Swabbing the injection site with alcohol

ANS: B Enoxaparin is given subcutaneously, so the 18-gauge, 2-inch needle is too big. The other actions are appropriate.

22. A nurse plans care for clients with urinary incontinence. Which client is correctly paired with the appropriate intervention? a. A 29-year-old client after a difficult vaginal delivery - Habit training b. A 58-year-old postmenopausal client who is not taking estrogen therapy - Electrical stimulation c. A 64-year-old female with Alzheimer's-type senile dementia - Bladder training d. A 77-year-old female who has difficulty ambulating - Exercise therapy

ANS: B Exercise therapy and electrical stimulation are used for clients with stress incontinence related to childbirth or low levels of estrogen after menopause. Exercise therapy increases pelvic wall strength; it does not improve ambulation. Physical therapy and a bedside commode would be appropriate interventions for the client who has difficulty ambulating. Habit training is the type of bladder training that will be most effective with cognitively impaired clients. Bladder training can be used only with a client who is alert, aware, and able to resist the urge to urinate.

20. A home health care nurse is visiting an older client who lives alone after being discharged from the hospital after a coronary artery bypass graft. What finding in the home most causes the nurse to consider additional referrals? a. Dirty carpets in need of vacuuming b. Expired food in the refrigerator c. Old medications in the kitchen d. Several cats present in the home

ANS: B Expired food in the refrigerator demonstrates a safety concern for the client and a possible lack of money to buy food. The nurse can consider a referral to Meals on Wheels or another home-based food program. Dirty carpets may indicate the client has no household help and is waiting for clearance to vacuum. Old medications can be managed by the home health care nurse and the client working collaboratively. Having pets is not a cause for concern. DIF: Applying/Application REF: 781 KEY: Home safety| referrals| coronary artery bypass graft MSC: Integrated Process: Communication and Documentation NOT: Client Needs Category: Safe and Effective Care Environment: Safety and Infection Control

7. After delegating care to an unlicensed assistive personnel (UAP) for a client who is prescribed habit training to manage incontinence, a nurse evaluates the UAP's understanding. Which action indicates the UAP needs additional teaching? a. Toileting the client after breakfast b. Changing the client's incontinence brief when wet c. Encouraging the client to drink fluids d. Recording the client's incontinence episodes

ANS: B Habit training is undermined by the use of absorbent incontinence briefs or pads. The nurse should re-educate the UAP on the technique of habit training. The UAP should continue to toilet the client after meals, encourage the client to drink fluids, and record incontinent episodes.

13. A nurse is preparing to admit a client on mechanical ventilation from the emergency department. What action by the nurse takes priority? a. Assessing that the ventilator settings are correct b. Ensuring there is a bag-valve-mask in the room c. Obtaining personal protective equipment d. Planning to suction the client upon arrival to the room

ANS: B Having a bag-valve-mask device is critical in case the client needs manual breathing. The respiratory therapist is usually primarily responsible for setting up the ventilator, although the nurse should know and check the settings. Personal protective equipment is important, but ensuring client safety takes priority. The client may or may not need suctioning on arrival.

A nurse assesses a female client who presents with hirsutism. Which question should the nurse ask when assessing this client? a. "How do you plan to pay for your treatments?" b. "How do you feel about yourself?" c. "What medications are you prescribed?" d. "What are you doing to prevent this from happening?"

ANS: B Hirsutism, or excessive hair growth on the face and body, can result from endocrine disorders. This may cause a disruption in body image, especially for female clients. The nurse should inquire into the client's body image and self-perception. Asking about the client's financial status or current medications does not address the client's immediate problem. The client is not doing anything to herself to cause the problem, nor can the client prevent it from happening.

A nurse cares for a client who is prescribed a drug that blocks a hormone's receptor site. Which therapeutic effect should the nurse expect? a. Greater hormone metabolism b. Decreased hormone activity c. Increased hormone activity d. Unchanged hormone response

ANS: B Hormones cause activity in the target tissues by binding with their specific cellular receptor sites, thereby changing the activity of the cell. When receptor sites are occupied by other substances that block hormone binding, the cell's response is the same as when the level of the hormone is decreased.

14. A nurse is in charge of the coronary intensive care unit. Which client should the nurse see first? a. Client on a nitroglycerin infusion at 5 mcg/min, not titrated in the last 4 hours b. Client who is 1 day post coronary artery bypass graft, blood pressure 180/100 mm Hg c. Client who is 1 day post percutaneous coronary intervention, going home this morning d. Client who is 2 days post coronary artery bypass graft, became dizzy this a.m. while walking

ANS: B Hypertension after coronary artery bypass graft surgery can be dangerous because it puts too much pressure on the suture lines and can cause bleeding. The charge nurse should see this client first. The client who became dizzy earlier should be seen next. The client on the nitroglycerin drip is stable. The client going home can wait until the other clients are cared for. DIF: Analyzing/Analysis REF: 777 KEY: Coronary artery disease| coronary artery bypass graft| collaboration MSC: IntegratedProcess:NursingProcess:Assessment NOT: Client Needs Category: Safe and Effective Care Environment: Management of Care

13. A client has been taking isoniazid (INH) for tuberculosis for 3 weeks. What laboratory results need to be reported to the health care provider immediately? a. Albumin: 5.1 g/dL b. Alanine aminotransferase (ALT): 180 U/L c. Red blood cell (RBC) count: 5.2/mm3 d. White blood cell (WBC) count: 12,500/mm3

ANS: B INH can cause liver damage, especially if the client drinks alcohol. The ALT (one of the liver enzymes) is extremely high and needs to be reported immediately. The albumin and RBCs are normal. The WBCs are slightly high, but that would be an expected finding in a client with an infection.

22. After assessing a client who is receiving an amiodarone intravenous infusion for unstable ventricular tachycardia, the nurse documents the findings and compares these with the previous assessment findings: Vital Signs Nursing Assessment Time: 0800 Temperature: 98° F Heart rate: 68 beats/min Blood pressure: 135/60 mm Hg Respiratory rate: 14 breaths/min Oxygen saturation: 96% Oxygen therapy: 2 L nasal cannula Time: 1000 Temperature: 98.2° F Heart rate: 50 beats/min Blood pressure: 132/57 mm Hg Respiratory rate: 16 breaths/min Oxygen saturation: 95% Oxygen therapy: 2 L nasal cannula Time: 0800 Client alert and oriented. Cardiac rhythm: normal sinus rhythm. Skin: warm, dry, and appropriate for race. Respirations equal and unlabored. Client denies shortness of breath and chest pain. Time: 1000 Client alert and oriented. Cardiac rhythm: sinus bradycardia. Skin: warm, dry, and appropriate for race. Respirations equal and unlabored. Client denies shortness of breath and chest pain. Client voids 420 mL of clear yellow urine. Based on the assessments, which action should the nurse take? a. Stop the infusion and flush the IV. b. Slow the amiodarone infusion rate. c. Administer IV normal saline. d. Ask the client to cough and deep breathe.

ANS: B IV administration of amiodarone may cause bradycardia and atrioventricular (AV) block. The correct action for the nurse to take at this time is to slow the infusion, because the client is asymptomatic and no evidence reveals AV block that might require pacing. Abruptly ceasing the medication could allow fatal dysrhythmias to occur. The administration of IV fluids and encouragement of coughing and deep breathing exercises are not indicated, and will not increase the client's heart rate. DIF: Applying/Application REF: 660 KEY: Cardiac electrical conduction| medication MSC: Integrated Process: Nursing Process: Implementation NOT: Client Needs Category: Physiological Integrity: Pharmacological and Parenteral Therapies

6. A client has hemodynamic monitoring after a myocardial infarction. What safety precaution does the nurse implement for this client? a. Document pulmonary artery wedge pressure (PAWP) readings and assess their trends. b. Ensure the balloon does not remain wedged. c. Keep the client on strict NPO status. d. Maintain the client in a semi-Fowler's position.

ANS: B If the balloon remains inflated, it can cause pulmonary infarction or rupture. The nurse should ensure the balloon remains deflated between PAWP readings. Documenting PAWP readings and assessing trends is an important nursing action related to hemodynamic monitoring, but is not specifically related to safety. The client does not have to be NPO while undergoing hemodynamic monitoring. Positioning may or may not affect readings. DIF: Applying/Application REF: 770 KEY: Coronary artery disease| hemodynamic monitoring| equipment safety MSC: Integrated Process: Nursing Process: Implementation NOT: Client Needs Category: Safe and Effective Care Environment: Safety and Infection Control

19. The nurse is caring for a client with a chest tube after a coronary artery bypass graft. The drainage slows significantly. What action by the nurse is most important? a. Increase the setting on the suction. b. Notify the provider immediately. c. Re-position the chest tube. d. Take the tubing apart to assess for clots.

ANS: B If the drainage in the chest tube decreases significantly and dramatically, the tube may be blocked by a clot. This could lead to cardiac tamponade. The nurse should notify the provider immediately. The nurse should not independently increase the suction, re-position the chest tube, or take the tubing apart. DIF: Applying/Application REF: 778 KEY: Coronary artery bypass graft| critical rescue| chest tubes| cardiovascular system MSC: Integrated Process: Communication and Documentation NOT: Client Needs Category: Safe and Effective Care Environment: Management of Care

10. While assessing a client who is 12 hours postoperative after a thoracotomy for lung cancer, a nurse notices that the lower chest tube is dislodged. Which action should the nurse take first? a. Assess for drainage from the site. b. Cover the insertion site with sterile gauze. \ c. Contact the provider and obtain a suture kit. d. Reinsert the tube using sterile technique.

ANS: B Immediately covering the insertion site helps prevent air from entering the pleural space and causing a pneumothorax. The area will not reseal quickly enough to prevent air from entering the chest. The nurse should not leave the client to obtain a suture kit. An occlusive dressing may cause a tension pneumothorax. The site should only be assessed after the insertion site is covered. The provider should be called to reinsert the chest tube or prescribe other treatment options. DIF: Applying/Application REF: 578 KEY: Drains| surgical care MSC: Integrated Process: Nursing Process: Implementation NOT: Client Needs Category: Physiological Integrity: Physiological Adaptation

6. After teaching a client with nephrotic syndrome and a normal glomerular filtration, the nurse assesses the client's understanding. Which statement made by the client indicates a correct understanding of the nutritional therapy for this condition? a. "I must decrease my intake of fat." b. "I will increase my intake of protein." c. "A decreased intake of carbohydrates will be required." d. "An increased intake of vitamin C is necessary."

ANS: B In nephrotic syndrome, the renal loss of protein is significant, leading to hypoalbuminemia and edema formation. If glomerular filtration is normal or near normal, increased protein loss should be matched by increased intake of protein. The client would not need to adjust fat, carbohydrates, or vitamins based on this disorder.

8. A nurse cares for a client with an intravenous temporary pacemaker for bradycardia. The nurse observes the presence of a pacing spike but no QRS complex on the client's electrocardiogram. Which action should the nurse take next? a. Administer intravenous diltiazem (Cardizem). b. Assess vital signs and level of consciousness. c. Administer sublingual nitroglycerin. d. Assess capillary refill and temperature.

ANS: B In temporary pacing, the wires are threaded onto the epicardial surface of the heart and exit through the chest wall. The pacemaker spike should be followed immediately by a QRS complex. Pacing spikes seen without subsequent QRS complexes imply loss of capture. If there is no capture, then there is no ventricular depolarization and contraction. The nurse should assess for cardiac output via vital signs and level of consciousness. The other interventions would not determine if the client is tolerating the loss of capture. DIF: Applying/Application REF: 664 KEY: Cardiac electrical conduction MSC: IntegratedProcess:NursingProcess:Assessment NOT: Client Needs Category: Physiological Integrity: Reduction of Risk Potential

7. An older adult is brought to the emergency department by a family member, who reports a moderate change in mental status and mild cough. The client is afebrile. The health care provider orders a chest x-ray. The family member questions why this is needed since the manifestations seem so vague. What response by the nurse is best? a. "Chest x-rays are always ordered when we suspect pneumonia." b. "Older people often have vague symptoms, so an x-ray is essential." c. "The x-ray can be done and read before laboratory work is reported." d. "We are testing for any possible source of infection in the client."

ANS: B It is essential to obtain an early chest x-ray in older adults suspected of having pneumonia because symptoms are often vague. Waiting until definitive manifestations are present to obtain the x-ray leads to a costly delay in treatment. Stating that chest x-rays are always ordered does not give the family definitive information. The x-ray can be done while laboratory values are still pending, but this also does not provide specific information about the importance of a chest x-ray in this client. The client has manifestations of pneumonia, so the staff is not testing for any possible source of infection but rather is testing for a suspected disorder.

11. A nurse reviews a female client's laboratory results. Which results from the client's urinalysis should the nurse recognize as abnormal? a. pH 5.6 b. Ketone bodies present c. Specific gravity of 1.020 d. Clear and yellow color

ANS: B Ketone bodies are by-products of incomplete metabolism of fatty acids. Normally no ketones are present in urine. Ketone bodies are produced when fat sources are used instead of glucose to provide cellular energy. A pH between 4.6 and 8, specific gravity between 1.005 and 1.030, and clear yellow urine are normal findings for a female client's urinalysis.

9. The nurse is caring for four clients with chronic kidney disease. Which client should the nurse assess first upon initial rounding? a. Woman with a blood pressure of 158/90 mm Hg b. Client with Kussmaul respirations c. Man with skin itching from head to toe d. Client with halitosis and stomatitis

ANS: B Kussmaul respirations indicate a worsening of chronic kidney disease (CKD). The client is increasing the rate and depth of breathing to excrete carbon dioxide through the lungs. Hypertension is common in most clients with CKD, and skin itching increases with calcium-phosphate imbalances, another common finding in CKD. Uremia from CKD causes ammonia to be formed, resulting in the common findings of halitosis and stomatitis.

3. A nurse cares for a postmenopausal client who has had two episodes of bacterial urethritis in the last 6 months. The client asks, "I never have urinary tract infections. Why is this happening now?" How should the nurse respond? a. "Your immune system becomes less effective as you age." b. "Low estrogen levels can make the tissue more susceptible to infection." c. "You should be more careful with your personal hygiene in this area." d. "It is likely that you have an untreated sexually transmitted disease."

ANS: B Low estrogen levels decrease moisture and secretions in the perineal area and cause other tissue changes, predisposing it to the development of infection. Urethritis is most common in postmenopausal women for this reason. Although immune function does decrease with aging and sexually transmitted diseases are a known cause of urethritis, the most likely reason in this client is low estrogen levels. Personal hygiene usually does not contribute to this disease process.

9. A nurse cares for a client who had a partial laryngectomy 10 days ago. The client states that all food tastes bland. How should the nurse respond? a. "I will consult the speech therapist to ensure you are swallowing properly." b. "This is normal after surgery. What types of food do you like to eat?" c. "I will ask the dietitian to change the consistency of the food in your diet." d. "Replacement of protein, calories, and water is very important after surgery."

ANS: B Many clients experience changes in taste after surgery. The nurse should identify foods that the client wants to eat to ensure the client maintains necessary nutrition. Although the nurse should collaborate with the speech therapist and dietitian to ensure appropriate replacement of protein, calories, and water, the other responses do not address the client's concerns. DIF: Applying/Application REF: 533 KEY: Surgical care| nutrition MSC: Integrated Process: Communication and Documentation NOT: Client Needs Category: Physiological Integrity: Basic Care and Comfort

19. A client is in the family medicine clinic reporting a dry, sore throat. The provider asks the nurse to assess for odynophagia. What assessment technique is most appropriate? a. Ask the client what foods cause trouble swallowing. b. Assess the client for pain when swallowing. c. Determine if the client can swallow saliva. d. Palpate the client's jaw while swallowing.

ANS: B Odynophagia is painful swallowing. The nurse should assess the client for this either by asking or by having the client attempt to drink water. It is not related to specific foods and is not assessed by palpating the jaw. Being unable to swallow saliva is not odynophagia, but it would be a serious situation.

19. A client is in the family medicine clinic reporting a dry, sore throat. The provider asks the nurse to assess for odynophagia. What assessment technique is most appropriate? a. Ask the client what foods cause trouble swallowing. b. Assess the client for pain when swallowing. c. Determine if the client can swallow saliva. d. Palpate the client's jaw while swallowing.

ANS: B Odynophagia is painful swallowing. The nurse should assess the client for this either by asking or by having the client attempt to drink water. It is not related to specific foods and is not assessed by palpating the jaw. Being unable to swallow saliva is not odynophagia, but it would be a serious situation.

5. A client is hospitalized with a second episode of pulmonary embolism (PE). Recent genetic testing reveals the client has an alteration in the gene CYP2C19. What action by the nurse is best? a. Instruct the client to eliminate all vitamin K from the diet. b. Prepare preoperative teaching for an inferior vena cava (IVC) filter. c. Refer the client to a chronic illness support group. d. Teach the client to use a soft-bristled toothbrush.

ANS: B Often clients are discharged from the hospital on warfarin (Coumadin) after a PE. However, clients with a variation in the CYP2C19 gene do not metabolize warfarin well and have higher blood levels and more side effects. This client is a poor candidate for warfarin therapy, and the prescriber will most likely order an IVC filter device to be implanted. The nurse should prepare to do preoperative teaching on this procedure. It would be impossible to eliminate all vitamin K from the diet. A chronic illness support group may be needed, but this is not the best intervention as it is not as specific to the client as the IVC filter. A soft-bristled toothbrush is a safety measure for clients on anticoagulation therapy.

15. A client with coronary artery disease (CAD) asks the nurse about taking fish oil supplements. What response by the nurse is best? a. "Fish oil is contraindicated with most drugs for CAD." b. "The best source is fish, but pills have benefits too." c. "There is no evidence to support fish oil use with CAD." d. "You can reverse CAD totally with diet and supplements."

ANS: B Omega-3 fatty acids have shown benefit in reducing lipid levels, in reducing the incidence of sudden cardiac death, and for stabilizing atherosclerotic plaque. The best source is fish three times a week or some fish oil supplements. The other options are not accurate. DIF: Understanding/Comprehension REF: 761 KEY: Coronary artery disease| lipid-reducing agents| supplements| patient education MSC: IntegratedProcess:Teaching/Learning NOT: Client Needs Category: Health Promotion and Maintenance

1. A nurse assesses a client with polycystic kidney disease (PKD). Which assessment finding should alert the nurse to immediately contact the health care provider? a. Flank pain b. Periorbital edema c. Bloody and cloudy urine d. Enlarged abdomen

ANS: B Periorbital edema would not be a finding related to PKD and should be investigated further. Flank pain and a distended or enlarged abdomen occur in PKD because the kidneys enlarge and displace other organs. Urine can be bloody or cloudy as a result of cyst rupture or infection.

17. A client is in the family practice clinic reporting a severe cough that has lasted for 5 weeks. The client is so exhausted after coughing that work has become impossible. What action by the nurse is most appropriate? a. Arrange for immediate hospitalization. b. Facilitate polymerase chain reaction testing. c. Have the client produce a sputum sample. d. Obtain two sets of blood cultures.

ANS: B Polymerase chain reaction testing is used to diagnose pertussis, which this client is showing manifestations of. Hospitalization may or may not be needed but is not the most important action. The client may or may not be able to produce sputum, but sputum cultures for this disease must be obtained via deep suctioning. Blood cultures will be negative.

17. A client is in the family practice clinic reporting a severe cough that has lasted for 5 weeks. The client is so exhausted after coughing that work has become impossible. What action by the nurse is most appropriate? a. Arrange for immediate hospitalization. b. Facilitate polymerase chain reaction testing. c. Have the client produce a sputum sample. d. Obtain two sets of blood cultures.

ANS: B Polymerase chain reaction testing is used to diagnose pertussis, which this client is showing manifestations of. Hospitalization may or may not be needed but is not the most important action. The client may or may not be able to produce sputum, but sputum cultures for this disease must be obtained via deep suctioning. Blood cultures will be negative.

1. The nurse is assessing a client with a diagnosis of pre-renal acute kidney injury (AKI). Which condition would the nurse expect to find in the client's recent history? a. Pyelonephritis b. Myocardial infarction c. Bladder cancer d. Kidney stones

ANS: B Pre-renal causes of AKI are related to a decrease in perfusion, such as with a myocardial infarction. Pyelonephritis is an intrinsic or intrarenal cause of AKI related to kidney damage. Bladder cancer and kidney stones are post-renal causes of AKI related to urine flow obstruction.

5. The charge nurse on a medical unit is preparing to admit several "clients" who have possible pandemic flu during a preparedness drill. What action by the nurse is best? a. Admit the "clients" on Contact Precautions. b. Cohort the "clients" in the same area of the unit. c. Do not allow pregnant caregivers to care for these "clients." d. Place the "clients" on enhanced Droplet Precautions.

ANS: B Preventing the spread of pandemic flu is equally important as caring for the clients who have it. Clients can be cohorted together in the same set of rooms on one part of the unit to use distancing to help prevent the spread of the disease. The other actions are not appropriate.

14. A nurse teaches a client to use a room humidifier after a laryngectomy. Which statement should the nurse include in this client's teaching? a. "Add peppermint oil to the humidifier to relax the airway." b. "Make sure you clean the humidifier to prevent infection." c. "Keep the humidifier filled with water at all times." d. "Use the humidifier when you sleep, even during daytime naps."

ANS: B Priority teaching related to the use of a room humidifier focuses on infection control. Clients should be taught to meticulously clean the humidifier to prevent the spread of mold or other sources of infection. Peppermint oil should not be added to a humidifier. The humidifier should be refilled with water as needed and should be used while awake and asleep. DIF: Understanding/Comprehension REF: 544 KEY: Surgical care MSC: IntegratedProcess:Teaching/Learning NOT: Client Needs Category: Safe and Effective Care Environment: Safety and Infection Control MULTIPLE RESPONSE

2. A nurse reviews the health history of a client with an oversecretion of renin. Which disorder should the nurse correlate with this assessment finding? a. Alzheimer's disease b. Hypertension c. Diabetes mellitus d. Viral hepatitis

ANS: B Renin is secreted when special cells in the distal convoluted tubule, called the macula densa, sense changes in blood volume and pressure. When the macula densa cells sense that blood volume, blood pressure, or blood sodium levels are low, renin is secreted. Renin then converts angiotensinogen into angiotensin I. This leads to a series of reactions that cause secretion of the hormone aldosterone. This hormone increases kidney reabsorption of sodium and water, increasing blood pressure, blood volume, and blood sodium levels. Inappropriate or excessive renin secretion is a major cause of persistent hypertension. Renin has no impact on Alzheimer's disease, diabetes mellitus, or viral hepatitis.

3. A nurse assesses a client admitted to the cardiac unit. Which statement by the client alerts the nurse to the possibility of right-sided heart failure? a. "I sleep with four pillows at night." b. "My shoes fit really tight lately." c. "I wake up coughing every night." d. "I have trouble catching my breath."

ANS: B Signs of systemic congestion occur with right-sided heart failure. Fluid is retained, pressure builds in the venous system, and peripheral edema develops. Left-sided heart failure symptoms include respiratory symptoms. Orthopnea, coughing, and difficulty breathing all could be results of left-sided heart failure. DIF: Understanding/Comprehension REF: 683 KEY: Heart failure| assessment/diagnostic examination MSC: IntegratedProcess:NursingProcess:Assessment NOT: Client Needs Category: Health Promotion and Maintenance

20. An emergency department nurse assesses a client with a history of urinary incontinence who presents with extreme dry mouth, constipation, and an inability to void. Which question should the nurse ask first? a. "Are you drinking plenty of water?" b. "What medications are you taking?" c. "Have you tried laxatives or enemas?" d. "Has this type of thing ever happened before?"

ANS: B Some types of incontinence are treated with anticholinergic medications such as propantheline (Pro-Banthine). Anticholinergic side effects include dry mouth, constipation, and urinary retention. The nurse needs to assess the client's medication list to determine whether the client is taking an anticholinergic medication. If he or she is taking anticholinergics, the nurse should further assess the client's manifestations to determine if they are related to a simple side effect or an overdose. The other questions are not as helpful to understanding the current situation.

13. A nurse cares for a client recovering from prosthetic valve replacement surgery. The client asks, "Why will I need to take anticoagulants for the rest of my life?" How should the nurse respond? a. "The prosthetic valve places you at greater risk for a heart attack." b. "Blood clots form more easily in artificial replacement valves." c. "The vein taken from your leg reduces circulation in the leg." d. "The surgery left a lot of small clots in your heart and lungs."

ANS: B Synthetic valve prostheses and scar tissue provide surfaces on which platelets can aggregate easily and initiate the formation of blood clots. The other responses are inaccurate. DIF: Applying/Application REF: 696 KEY: Valve disorder| patient education| anticoagulants MSC: IntegratedProcess:Teaching/Learning NOT: Client Needs Category: Physiological Integrity: Reduction of Risk Potential

13. After teaching a client with renal cancer who is prescribed temsirolimus (Torisel), the nurse assesses the client's understanding. Which statement made by the client indicates a correct understanding of the teaching? a. "I will take this medication with food and plenty of water." b. "I shall keep my appointment at the infusion center each week." c. "I'll limit my intake of green leafy vegetables while on this medication." d. "I must not take this medication if I have an infection or am feeling ill."

ANS: B Temsirolimus is administered as a weekly intravenous infusion. This medication blocks protein that is needed for cell division and therefore inhibits cell cycle progression. This medication is not taken orally, and clients do not need to follow a specific diet.

16. A client has presented to the emergency department with an acute myocardial infarction (MI). What action by the nurse is best to meet The Joint Commission's Core Measures outcomes? a. Obtain an electrocardiogram (ECG) now and in the morning. b. Give the client an aspirin. c. Notify the Rapid Response Team. d. Prepare to administer thrombolytics.

ANS: B The Joint Commission's Core Measures set for acute MI require that aspirin is administered when a client with MI presents to the emergency department or when an MI occurs in the hospital. A rapid ECG is vital, but getting another one in the morning is not part of the Core Measures set. The Rapid Response Team is not needed if an emergency department provider is available. Thrombolytics may or may not be needed. DIF: Remembering/Knowledge REF: 766 KEY: Coronary artery disease| Core Measures| The Joint Commission MSC: Integrated Process: Nursing Process: Implementation NOT: Client Needs Category: Safe and Effective Care Environment: Management of Care

10. After teaching a client who has an implantable cardioverter-defibrillator (ICD), a nurse assesses the client's understanding. Which statement by the client indicates a correct understanding of the teaching? a. "I should wear a snug-fitting shirt over the ICD." b. "I will avoid sources of strong electromagnetic fields." c. "I should participate in a strenuous exercise program." d. "Now I can discontinue my antidysrhythmic medication."

ANS: B The client being discharged with an ICD is instructed to avoid strong sources of electromagnetic fields. Clients should avoid tight clothing, which could cause irritation over the ICD generator. The client should be encouraged to exercise but should not engage in strenuous activities that cause the heart rate to meet or exceed the ICD cutoff point because the ICD can discharge inappropriately. The client should continue all prescribed medications. DIF: Applying/Application REF: 674 KEY: Cardiac electrical conduction MSC: Integrated Process: Teaching/Learning NOT: Client Needs Category: Health Promotion and Maintenance

9. After teaching a client with hypertension secondary to renal disease, the nurse assesses the client's understanding. Which statement made by the client indicates a need for additional teaching? a. "I can prevent more damage to my kidneys by managing my blood pressure." b. "If I have increased urination at night, I need to drink less fluid during the day." c. "I need to see the registered dietitian to discuss limiting my protein intake." d. "It is important that I take my antihypertensive medications as directed."

ANS: B The client should not restrict fluids during the day due to increased urination at night. Clients with renal disease may be prescribed fluid restrictions. These clients should be assessed thoroughly for potential dehydration. Increased nocturnal voiding can be decreased by consuming fluids earlier in the day. Blood pressure control is needed to slow the progression of renal dysfunction. When dietary protein is restricted, refer the client to the registered dietitian as needed.

21. A nurse teaches a client who is starting urinary bladder training. Which statement should the nurse include in this client's teaching? a. "Use the toilet when you first feel the urge, rather than at specific intervals." b. "Try to consciously hold your urine until the scheduled toileting time." c. "Initially try to use the toilet at least every half hour for the first 24 hours." d. "The toileting interval can be increased once you have been continent for a week."

ANS: B The client should try to hold the urine consciously until the next scheduled toileting time. Toileting should occur at specific intervals during the training. The toileting interval should be no less than every hour. The interval can be increased once the client becomes comfortable with the interval.

18. A pulmonary nurse cares for clients who have chronic obstructive pulmonary disease (COPD). Which client should the nurse assess first? a. A 46-year-old with a 30-pack-year history of smoking b. A 52-year-old in a tripod position using accessory muscles to breathe c. A 68-year-old who has dependent edema and clubbed fingers d. A 74-year-old with a chronic cough and thick, tenacious secretions

ANS: B The client who is in a tripod position and using accessory muscles is working to breathe. This client must be assessed first to establish how well the client is breathing and provide interventions to minimize respiratory failure. The other clients are not in acute distress. DIF: Applying/Application REF: 559 KEY: Health screening MSC: IntegratedProcess:NursingProcess:Assessment NOT: Client Needs Category: Safe and Effective Care Environment: Management of Care

4. A nurse teaches a client who has open vocal cord paralysis. Which technique should the nurse teach the client to prevent aspiration? a. Tilt the head back as far as possible when swallowing. b. Tuck the chin down when swallowing. c. Breathe slowly and deeply while swallowing. d. Keep the head very still and straight while swallowing.

ANS: B The client with open vocal cord paralysis may aspirate. The nurse should teach the client to tuck in his or her chin during swallowing to prevent aspiration. Tilting the head back would increase the chance of aspiration. Breathing slowly would not decrease the risk of aspiration, but holding the breath would. Keeping the head still and straight would not decrease the risk for aspiration. DIF: Applying/Application REF: 535 KEY: Aspiration precaution MSC: Integrated Process: Teaching/Learning NOT: Client Needs Category: Physiological Integrity: Reduction of Risk Potential

16. A nurse assesses a client with pericarditis. Which assessment finding should the nurse expect to find? a. Heart rate that speeds up and slows down b. Friction rub at the left lower sternal border c. Presence of a regular gallop rhythm d. Coarse crackles in bilateral lung bases

ANS: B The client with pericarditis may present with a pericardial friction rub at the left lower sternal border. This sound is the result of friction from inflamed pericardial layers when they rub together. The other assessments are not related. DIF: Remembering/Knowledge REF: 699 KEY: Inflammatory response| assessment/diagnostic examination MSC: IntegratedProcess:NursingProcess:Assessment NOT: Client Needs Category: Physiological Integrity: Physiological Adaptation

24. A nurse assesses a male client who is recovering from a urologic procedure. Which assessment finding indicates an obstruction of urine flow? a. Severe pain b. Overflow incontinence c. Hypotension d. Blood-tinged urine

ANS: B The most common manifestation of urethral stricture after a urologic procedure is obstruction of urine flow. This rarely causes pain and has no impact on blood pressure. The client may experience overflow incontinence with the involuntary loss of urine when the bladder is distended. Blood in the urine is not a manifestation of the obstruction of urine flow.

17. A nurse is caring for four clients. Which client should the nurse assess first? a. Client with an acute myocardial infarction, pulse 102 beats/min b. Client who is 1 hour post angioplasty, has tongue swelling and anxiety c. Client who is post coronary artery bypass, chest tube drained 100 mL/hr d. Client who is post coronary artery bypass, potassium 4.2 mEq/L

ANS: B The post-angioplasty client with tongue swelling and anxiety is exhibiting manifestations of an allergic reaction that could progress to anaphylaxis. The nurse should assess this client first. The client with a heart rate of 102 beats/min may have increased oxygen demands but is just over the normal limit for heart rate. The two post coronary artery bypass clients are stable. DIF: Analyzing/Analysis REF: 774 KEY: Coronary artery disease| critical rescue| medical emergencies| hypersensitivities| allergic reaction MSC: Integrated Process: Nursing Process: Analysis NOT: Client Needs Category: Safe and Effective Care Environment: Management of Care

12. A nurse assesses a client who is recovering from extracorporeal shock wave lithotripsy for renal calculi. The nurse notes an ecchymotic area on the client's right lower back. Which action should the nurse take? a. Administer fresh-frozen plasma. b. Apply an ice pack to the site. c. Place the client in the prone position. d. Obtain serum coagulation test results.

ANS: B The shock waves from lithotripsy can cause bleeding into the tissues through which the waves pass. Application of ice can reduce the extent and discomfort of the bruising. Although coagulation test results and fresh-frozen plasma are used to assess and treat bleeding disorders, ecchymosis after this procedure is not unusual and does not warrant a higher level of intervention. Changing the client's position will not decrease bleeding.

18. A nurse teaches a client recovering from a heart transplant who is prescribed cyclosporine (Sandimmune). Which statement should the nurse include in this client's discharge teaching? a. "Use a soft-bristled toothbrush and avoid flossing." b. "Avoid large crowds and people who are sick." c. "Change positions slowly to avoid hypotension." d. "Check your heart rate before taking the medication."

ANS: B These agents cause immune suppression, leaving the client more vulnerable to infection. The medication does not place the client at risk for bleeding, orthostatic hypotension, or a change in heart rate. DIF: Applying/Application REF: 703 KEY: Transplant| immune suppressant MSC: Integrated Process: Nursing Process: Implementation NOT: Client Needs Category: Physiological Integrity: Pharmacological and Parenteral Therapies

1. A nurse answers a call light and finds a client anxious, short of breath, reporting chest pain, and having a blood pressure of 88/52 mm Hg on the cardiac monitor. What action by the nurse takes priority? a. Assess the client's lung sounds. b. Notify the Rapid Response Team. c. Provide reassurance to the client. d. Take a full set of vital signs.

ANS: B This client has manifestations of a pulmonary embolism, and the most critical action is to notify the Rapid Response Team for speedy diagnosis and treatment. The other actions are appropriate also but are not the priority.

14. A client seen in the emergency department reports fever, fatigue, and dry cough but no other upper respiratory symptoms. A chest x-ray reveals mediastinal widening. What action by the nurse is best? a. Collect a sputum sample for culture by deep suctioning. b. Inform the client that antibiotics will be needed for 60 days. c. Place the client on Airborne Precautions immediately. d. Tell the client that directly observed therapy is needed.

ANS: B This client has manifestations of early inhalation anthrax. For treatment, after IV antibiotics are finished, oral antibiotics are continued for at least 60 days. Sputum cultures are not needed. Anthrax is not transmissible from person to person, so Standard Precautions are adequate. Directly observed therapy is often used for tuberculosis.

14. A client seen in the emergency department reports fever, fatigue, and dry cough but no other upper respiratory symptoms. A chest x-ray reveals mediastinal widening. What action by the nurse is best? a. Collect a sputum sample for culture by deep suctioning. b. Inform the client that antibiotics will be needed for 60 days. c. Place the client on Airborne Precautions immediately. d. Tell the client that directly observed therapy is needed.

ANS: B This client has manifestations of early inhalation anthrax. For treatment, after IV antibiotics are finished, oral antibiotics are continued for at least 60 days. Sputum cultures are not needed. Anthrax is not transmissible from person to person, so Standard Precautions are adequate. Directly observed therapy is often used for tuberculosis.

23. A nurse is caring for a client on the medical stepdown unit. The following data are related to this client: What action by the nurse is most appropriate? a. Call respiratory therapy for a breathing treatment. b. Facilitate a STAT pulmonary angiography. c. Prepare for immediate endotracheal intubation. d. Prepare to administer intravenous anticoagulants.

ANS: B This client has manifestations of pulmonary embolism (PE); however, many conditions can cause the client's presentation. The gold standard for diagnosing a PE is pulmonary angiography. The nurse should facilitate this test as soon as possible. The client does not have wheezing, so a respiratory treatment is not needed. The client is not unstable enough to need intubation and mechanical ventilation. IV anticoagulants are not given without a diagnosis of PE.

4. A client is admitted with acute kidney injury (AKI) and a urine output of 2000 mL/day. What is the major concern of the nurse regarding this client's care? a. Edema and pain b. Electrolyte and fluid imbalance c. Cardiac and respiratory status d. Mental health status

ANS: B This client may have an inflammatory cause of AKI with proteins entering the glomerulus and holding the fluid in the filtrate, causing polyuria. Electrolyte loss and fluid balance is essential. Edema and pain are not usually a problem with fluid loss. There could be changes in the client's cardiac, respiratory, and mental health status if the electrolyte imbalance is not treated.

3. A client is in the hospital after suffering a myocardial infarction and has bathroom privileges. The nurse assists the client to the bathroom and notes the client's O2 saturation to be 95%, pulse 88 beats/min, and respiratory rate 16 breaths/min after returning to bed. What action by the nurse is best? a. Administer oxygen at 2 L/min. b. Allow continued bathroom privileges. c. Obtain a bedside commode. d. Suggest the client use a bedpan.

ANS: B This client's physiologic parameters did not exceed normal during and after activity, so it is safe for the client to continue using the bathroom. There is no indication that the client needs oxygen, a commode, or a bedpan. DIF: Applying/Application REF: 769 KEY: Coronary artery disease| activity intolerance| vital signs| nursing assessment MSC: IntegratedProcess:NursingProcess:Assessment NOT: Client Needs Category: Physiological Integrity: Reduction of Risk Potential

6. A nurse is caring for four clients on intravenous heparin therapy. Which laboratory value possibly indicates that a serious side effect has occurred? a. Hemoglobin: 14.2 g/dL b. Platelet count: 82,000/L 3 c. Red blood cell count: 4.8/mm 3 d. White blood cell count: 8.7/mm

ANS: B This platelet count is low and could indicate heparin-induced thrombocytopenia. The other values are normal for either gender.

9. A nurse cares for a client with a urine specific gravity of 1.018. Which action should the nurse take? a. Evaluate the client's intake and output for the past 24 hours. b. Document the finding in the chart and continue to monitor. c. Obtain a specimen for a urine culture and sensitivity. d. Encourage the client to drink more fluids, especially water.

ANS: B This specific gravity is within the normal range for urine. There is no need to evaluate the client's intake and output, obtain a urine specimen, or increase fluid intake.

4. After teaching a client how to perform diaphragmatic breathing, the nurse assesses the client's understanding. Which action demonstrates that the client correctly understands the teaching? a. The client lays on his or her side with his or her knees bent. b. The client places his or her hands on his or her abdomen. c. The client lays in a prone position with his or her legs straight. d. The client places his or her hands above his or her head.

ANS: B To perform diaphragmatic breathing correctly, the client should place his or her hands on his or her abdomen to create resistance. This type of breathing cannot be performed effectively while lying on the side or with hands over the head. This type of breathing would not be as effective lying prone. DIF: Applying/Application REF: 562 KEY: Respiratory distress/failure| patient education MSC: IntegratedProcess:Teaching/Learning NOT: Client Needs Category: Physiological Integrity: Physiological Adaptation

6. After teaching a client who is prescribed "voice rest" therapy for vocal cord polyps, a nurse assesses the client's understanding. Which statement indicates the client needs further teaching? a. "I will stay away from smokers to minimize inhalation of secondhand smoke." b. "When I speak, I will whisper rather than use a normal tone of voice." c. "For the next several weeks, I will not lift more than 10 pounds." d. "I will drink at least three quarts of water each day to stay hydrated."

ANS: B Treatment for vocal cord polyps includes no speaking, no lifting, and no smoking. The client has to be educated not to even whisper when resting the voice. It is also appropriate for the client to stay out of rooms where people are smoking, to stay hydrated, and to use stool softeners. DIF: Applying/Application REF: 536 KEY: Cancer| patient education MSC: Integrated Process: Teaching/Learning NOT: Client Needs Category: Physiological Integrity: Reduction of Risk Potential

17. A nurse is teaching a client about warfarin (Coumadin). What assessment finding by the nurse indicates a possible barrier to self-management? a. Poor visual acuity b. Strict vegetarian c. Refusal to stop smoking d. Wants weight loss surgery

ANS: B Warfarin works by inhibiting the synthesis of vitamin K-dependent clotting factors. Foods high in vitamin K thus interfere with its action and need to be eaten in moderate, consistent amounts. A vegetarian may have trouble maintaining this diet. The nurse should explore this possibility with the client. The other options are not related.

16. A client is being discharged soon on warfarin (Coumadin). What menu selection for dinner indicates the client needs more education regarding this medication? a. Hamburger and French fries b. Large chef's salad and muffin c. No selection; spouse brings pizza d. Tuna salad sandwich and chips

ANS: B Warfarin works by inhibiting the synthesis of vitamin K-dependent clotting factors. Foods high in vitamin K thus interfere with its action and need to be eaten in moderate, consistent amounts. The chef's salad most likely has too many leafy green vegetables, which contain high amounts of vitamin K. The other selections, while not particularly healthy, will not interfere with the medication's mechanism of action.

A nurse teaches a client with type 2 diabetes mellitus who is prescribed glipizide (Glucotrol). Which statement should the nurse include in this client's teaching? a. "Change positions slowly when you get out of bed." b. "Avoid taking nonsteroidal anti-inflammatory drugs (NSAIDs)." c. "If you miss a dose of this drug, you can double the next dose." d. "Discontinue the medication if you develop a urinary infection."

ANS: B NSAIDs potentiate the hypoglycemic effects of sulfonylurea agents. Glipizide is a sulfonylurea. The other statements are not applicable to glipizide.

3. A client in the emergency department is taking rifampin (Rifadin) for tuberculosis. The client reports yellowing of the sclera and skin and bleeding after minor trauma. What laboratory results correlate to this condition? (Select all that apply.) a. Blood urea nitrogen (BUN): 19 mg/dL b. International normalized ratio (INR): 6.3 c. Prothrombin time: 35 seconds d. Serum sodium: 130 mEq/L 3 e. White blood cell (WBC) count: 72,000/mm

ANS: B, C Rifampin can cause liver damage, evidenced by the client's high INR and prothrombin time. The BUN and WBC count are normal. The sodium level is low, but that is not related to this client's problem.

5. A nurse assesses a client who has a chest tube. For which manifestations should the nurse immediately intervene? (Select all that apply.) a. Production of pink sputum b. Tracheal deviation c. Sudden onset of shortness of breath d. Pain at insertion site e. Drainage of 75 mL/hr

ANS: B, C Tracheal deviation and sudden onset of shortness of breath are manifestations of a tension pneumothorax. The nurse must intervene immediately for this emergency situation. Pink sputum is associated with pulmonary edema and is not a complication of a chest tube. Pain at the insertion site and drainage of 75 mL/hr are normal findings with a chest tube. DIF: Applying/Application REF: 579 KEY: Drain| respiratory distress/failure MSC: IntegratedProcess:NursingProcess:Assessment NOT: Client Needs Category: Physiological Integrity: Reduction of Risk Potential

2. A nurse assesses a client who has developed epistaxis. Which conditions in the client's history should the nurse identify as potential contributors to this problem? (Select all that apply.) a. Diabetes mellitus b. Hypertension c. Leukemia d. Cocaine use e. Migraine f. Elevated platelets

ANS: B, C, D Frequent causes of nosebleeds include trauma, hypertension, leukemia and other blood dyscrasias, inflammation, tumor, dry air, blowing or picking the nose, cocaine use, and intranasal procedures. Diabetes, migraines, and elevated platelets and cholesterol levels do not cause epistaxis. DIF: Understanding/Comprehension REF: 532 KEY: Respiratory distress/failure MSC: IntegratedProcess:NursingProcess:Assessment NOT: Client Needs Category: Physiological Integrity: Reduction of Risk Potential

2. A nurse assesses clients on the medical-surgical unit. Which clients are at risk for kidney problems? (Select all that apply.) a. A 24-year-old pregnant woman prescribed prenatal vitamins b. A 32-year-old bodybuilder taking synthetic creatine supplements c. A 56-year-old who is taking metformin for diabetes mellitus d. A 68-year-old taking high-dose nonsteroidal anti-inflammatory drugs (NSAIDs) for chronic back pain e. A 75-year-old with chronic obstructive pulmonary disease (COPD) who is prescribed an albuterol nebulizer

ANS: B, C, D Many medications can affect kidney function. Clients who take synthetic creatine supplements, metformin, and high-dose or long-term NSAIDs are at risk for kidney dysfunction. Prenatal vitamins and albuterol nebulizers do not place these clients at risk.

MULTIPLE RESPONSE 1. A nursing student learns about modifiable risk factors for coronary artery disease. Which factors does this include? (Select all that apply.) a. Age b. Hypertension c. Obesity d. Smoking e. Stress

ANS: B, C, D, E Hypertension, obesity, smoking, and excessive stress are all modifiable risk factors for coronary artery disease. Age is a nonmodifiable risk factor. DIF: Remembering/Knowledge REF: 760 KEY: Coronary artery disease| lifestyle factors MSC: Integrated Process: Teaching/Learning NOT: Client Needs Category: Health Promotion and Maintenance

5. A nurse is giving discharge instructions to a client recently diagnosed with chronic kidney disease (CKD). Which statements made by the client indicate a correct understanding of the teaching? (Select all that apply.) a. "I can continue to take antacids to relieve heartburn." b. "I need to ask for an antibiotic when scheduling a dental appointment." c. "I'll need to check my blood sugar often to prevent hypoglycemia." d. "The dose of my pain medication may have to be adjusted." e. "I should watch for bleeding when taking my anticoagulants."

ANS: B, C, D, E In discharge teaching, the nurse must emphasize that the client needs to have an antibiotic prophylactically before dental procedures to prevent infection. There may be a need for dose reduction in medications if the kidney is not excreting them properly (antacids with magnesium, antibiotics, antidiabetic drugs, insulin, opioids, and anticoagulants).

1. A nurse assesses a client who has a family history of polycystic kidney disease (PKD). For which clinical manifestations should the nurse assess? (Select all that apply.) a. Nocturia b. Flank pain c. Increased abdominal girth d. Dysuria e. Hematuria f. Diarrhea

ANS: B, C, E Clients with PKD experience abdominal distention that manifests as flank pain and increased abdominal girth. Bloody urine is also present with tissue damage secondary to PKD. Clients with PKD often experience constipation, but would not report nocturia or dysuria.

9. A nurse assesses a client who is diagnosed with infective endocarditis. Which assessment findings should the nurse expect? (Select all that apply.) a. Weight gain b. Night sweats c. Cardiac murmur d. Abdominal bloating e. Osler's nodes

ANS: B, C, E Clinical manifestations of infective endocarditis include fever with chills, night sweats, malaise and fatigue, anorexia and weight loss, cardiac murmur, and Osler's nodes on palms of the hands and soles of the feet. Abdominal bloating is a manifestation of heart transplantation rejection. DIF: Remembering/Knowledge REF: 697 KEY: Endocarditis MSC: IntegratedProcess:NursingProcess:Assessment NOT: Client Needs Category: Physiological Integrity: Physiological Adaptation

4. A nurse assesses a client with chronic obstructive pulmonary disease. Which questions should the nurse ask to determine the client's activity tolerance? (Select all that apply.) a. "What color is your sputum?" b. "Do you have any difficulty sleeping?" c. "How long does it take to perform your morning routine?" d. "Do you walk upstairs every day?" e. "Have you lost any weight lately?"

ANS: B, C, E Difficulty sleeping could indicate worsening breathlessness, as could taking longer to perform activities of daily living. Weight loss could mean increased dyspnea as the client becomes too fatigued to eat. The color of the client's sputum would not assist in determining activity tolerance. Asking whether the client walks upstairs every day is not as pertinent as determining if the client becomes short of breath on walking upstairs, or if the client goes upstairs less often than previously. DIF: Applying/Application REF: 559 KEY: Functional ability MSC: IntegratedProcess:NursingProcess:Assessment NOT: Client Needs Category: Physiological Integrity: Physiological Adaptation

4. A nurse assesses a client who is recovering from a nephrostomy. Which assessment findings should alert the nurse to urgently contact the health care provider? (Select all that apply.) a. Clear drainage b. Bloody drainage at site c. Client reports headache d. Foul-smelling drainage e. Urine draining from site

ANS: B, D, E After a nephrostomy, the nurse should assess the client for complications and urgently notify the provider if drainage decreases or stops, drainage is cloudy or foul-smelling, the nephrostomy sites leaks blood or urine, or the client has back pain. Clear drainage is normal. A headache would be an unrelated finding.

1. A nurse is caring for five clients. For which clients would the nurse assess a high risk for developing a pulmonary embolism (PE)? (Select all that apply.) a. Client who had a reaction to contrast dye yesterday b. Client with a new spinal cord injury on a rotating bed c. Middle-aged man with an exacerbation of asthma d. Older client who is 1-day post hip replacement surgery e. Young obese client with a fractured femur

ANS: B, D, E Conditions that place clients at higher risk of developing PE include prolonged immobility, central venous catheters, surgery, obesity, advancing age, conditions that increase blood clotting, history of thromboembolism, smoking, pregnancy, estrogen therapy, heart failure, stroke, cancer (particularly lung or prostate), and trauma. A contrast dye reaction and asthma pose no risk for PE.

4. A client is 1 day postoperative after a coronary artery bypass graft. What nonpharmacologic comfort measures does the nurse include when caring for this client? (Select all that apply.) a. Administer pain medication before ambulating. b. Assist the client into a position of comfort in bed. c. Encourage high-protein diet selections. d. Provide complementary therapies such as music. e. Remind the client to splint the incision when coughing.

ANS: B, D, E Nonpharmacologic comfort measures can include positioning, complementary therapies, and splinting the chest incision. Medications are not nonpharmacologic. Food choices are not comfort measures. DIF: Applying/Application REF: 781 KEY: Coronary artery disease| nonpharmacologic comfort measures MSC: Integrated Process: Nursing Process: Implementation NOT: Client Needs Category: Physiological Integrity: Basic Care and Comfort

4. The nurse is teaching a client with diabetes mellitus how to prevent or delay chronic kidney disease (CKD). Which client statements indicate a lack of understanding of the teaching? (Select all that apply.) a. "I need to decrease sodium, cholesterol, and protein in my diet." b. "My weight should be maintained at a body mass index of 30." c. "Smoking should be stopped as soon as I possibly can." d. "I can continue to take an aspirin every 4 to 8 hours for my pain." e. "I really only need to drink a couple of glasses of water each day."

ANS: B, D, E Weight should be maintained at a body mass index (BMI) of 22 to 25. A BMI of 30 indicates obesity. The use of nonsteroidal anti-inflammatory drugs such as aspirin should be limited to the lowest time at the lowest dose due to interference with kidney blood flow. The client should drink at least 2 liters of water daily. Diet adjustments should be made by restricting sodium, cholesterol, and protein. Smoking causes constriction of blood vessels and decreases kidney perfusion, so the client should stop smoking.

2. A nurse assesses a client who has a mediastinal chest tube. Which symptoms require the nurse's immediate intervention? (Select all that apply.) a. Production of pink sputum b. Tracheal deviation c. Pain at insertion site d. Sudden onset of shortness of breath e. Drainage greater than 70 mL/hr f. Disconnection at Y site

ANS: B, D, E, F Immediate intervention is warranted if the client has tracheal deviation because this could indicate a tension pneumothorax. Sudden shortness of breath could indicate dislodgment of the tube, occlusion of the tube, or pneumothorax. Drainage greater than 70 mL/hr could indicate hemorrhage. Disconnection at the Y site could result in air entering the tubing. Production of pink sputum, oxygen saturation less than 95%, and pain at the insertion site are not signs/symptoms that would require immediate intervention. DIF: Applying/Application REF: 579 KEY: Drain| respiratory distress/failure MSC: IntegratedProcess:NursingProcess:Assessment NOT: Client Needs Category: Physiological Integrity: Physiological Adaptation

4. A nurse assesses a client with a fungal urinary tract infection (UTI). Which assessments should the nurse complete? (Select all that apply.) a. Palpate the kidneys and bladder. b. Assess the medical history and current medical problems. c. Perform a bladder scan to assess post-void residual. d. Inquire about recent travel to foreign countries. e. Obtain a current list of medications.

ANS: B, E Clients who are severely immunocompromised or who have diabetes mellitus are more prone to fungal UTIs. The nurse should assess for these factors by asking about medical history, current medical problems, and the current medication list. A physical examination and a post-void residual may be needed, but not until further information is obtained indicating that these examinations are necessary. Travel to foreign countries probably would not be important because, even if exposed, the client needs some degree of compromised immunity to develop a fungal UTI.

A nurse assesses a client with diabetes mellitus 3 hours after a surgical procedure and notes the client's breath has a "fruity" odor. Which action should the nurse take? a. Encourage the client to use an incentive spirometer. b. Increase the client's intravenous fluid flow rate. c. Consult the provider to test for ketoacidosis. d. Perform meticulous pulmonary hygiene care.

ANS: C The stress of surgery increases the action of counterregulatory hormones and suppresses the action of insulin, predisposing the client to ketoacidosis and metabolic acidosis. One manifestation of ketoacidosis is a "fruity" odor to the breath. Documentation should occur after all assessments have been completed. Using an incentive spirometer, increasing IV fluids, and performing pulmonary hygiene will not address this client's problem.

3. A nurse teaches clients about the difference between urge incontinence and stress incontinence. Which statements should the nurse include in this education? (Select all that apply.) a. "Urge incontinence involves a post-void residual volume less than 50 mL." b. "Stress incontinence occurs due to weak pelvic floor muscles." c. "Stress incontinence usually occurs in people with dementia." d. "Urge incontinence can be managed by increasing fluid intake." e. "Urge incontinence occurs due to abnormal bladder contractions."

ANS: B, E Clients who suffer from stress incontinence have weak pelvic floor muscles or urethral sphincter and cannot tighten their urethra sufficiently to overcome the increased detrusor pressure. Stress incontinence is common after childbirth, when the pelvic muscles are stretched and weakened from pregnancy and delivery. Urge incontinence occurs in people who cannot suppress the contraction signal from the detrusor muscle. Abnormal detrusor contractions may be a result of neurologic abnormalities including dementia, or may occur with no known abnormality. Post-void residual is associated with reflex incontinence, not with urge incontinence or stress incontinence. Management of urge incontinence includes decreasing fluid intake, especially in the evening hours.

4. A registered nurse (RN) cares for clients on a surgical unit. Which clients should the RN delegate to a licensed practical nurse (LPN)? (Select all that apply.) a. A 32-year-old who had a radical neck dissection 6 hours ago b. A 43-year-old diagnosed with cancer after a lung biopsy 2 days ago c. A 55-year-old who needs discharge teaching after a laryngectomy d. A 67-year-old who is awaiting preoperative teaching for laryngeal cancer e. An 88-year-old with esophageal cancer who is awaiting gastric tube placement

ANS: B, E The nurse can delegate stable clients to the LPN. The client who had a biopsy 2 days ago and the client who is awaiting gastric tube placement are stable. The client who is 6 hours post-surgery is not yet stable. The RN is the only one who can perform discharge and preoperative teaching; teaching cannot be delegated. DIF: Applying/Application REF: 540 KEY: Interdisciplinary team| delegation MSC: Integrated Process: Teaching/Learning NOT: Client Needs Category: Safe and Effective Care Environment: Management of Care

A nurse assesses a client who is being treated for hyperglycemic-hyperosmolar state (HHS). Which clinical manifestation indicates to the nurse that the therapy needs to be adjusted? a. Serum potassium level has increased. b. Blood osmolarity has decreased. c. Glasgow Coma Scale score is unchanged. d. Urine remains negative for ketone bodies.

ANS: C A slow but steady improvement in central nervous system functioning is the best indicator of therapy effectiveness for HHS. Lack of improvement in the level of consciousness may indicate inadequate rates of fluid replacement. The Glasgow Coma Scale assesses the client's state of consciousness against criteria of a scale including best eye, verbal, and motor responses. An increase in serum potassium, decreased blood osmolality, and urine negative for ketone bodies do not indicate adequacy of treatment.

A nurse assesses a client who has a 15-year history of diabetes and notes decreased tactile sensation in both feet. Which action should the nurse take first? a. Document the finding in the client's chart. b. Assess tactile sensation in the client's hands. c. Examine the client's feet for signs of injury. d. Notify the health care provider.

ANS: C Diabetic neuropathy is common when the disease is of long duration. The client is at great risk for injury in any area with decreased sensation because he or she is less able to feel injurious events. Feet are common locations for neuropathy and injury, so the nurse should inspect them for any signs of injury. After assessment, the nurse should document findings in the client's chart. Testing sensory perception in the hands may or may not be needed. The health care provider can be notified after assessment and documentation have been completed.

5. A nurse cares for a client who has developed esophagitis after undergoing radiation therapy for lung cancer. Which diet selection should the nurse provide for this client? a. Spaghetti with meat sauce, ice cream b. Chicken soup, grilled cheese sandwich c. Omelet, soft whole wheat bread d. Pasta salad, custard, orange juice

ANS: C Side effects of radiation therapy may include inflammation of the esophagus. Clients should be taught that bland, soft, high-calorie foods are best, along with liquid nutritional supplements. Tomato sauce may prove too spicy for a client with esophagitis. A grilled cheese sandwich is too difficult to swallow with this condition, and orange juice and other foods with citric acid are too caustic. DIF: Applying/Application REF: 576 KEY: Cancer| nutrition MSC: IntegratedProcess:NursingProcess:Assessment NOT: Client Needs Category: Physiological Integrity: Basic Care and Comfort

After teaching a client who is newly diagnosed with type 2 diabetes mellitus, the nurse assesses the client's understanding. Which statement made by the client indicates a need for additional teaching? a. "I should increase my intake of vegetables with higher amounts of dietary fiber." b. "My intake of saturated fats should be no more than 10% of my total calorie intake." c. "I should decrease my intake of protein and eliminate carbohydrates from my diet." d. "My intake of water is not restricted by my treatment plan or medication regimen."

ANS: C The client should not completely eliminate carbohydrates from the diet, and should reduce protein if microalbuminuria is present. The client should increase dietary intake of complex carbohydrates, including vegetables, and decrease intake of fat. Water does not need to be restricted unless kidney failure is present.

A nurse cares for a client who is prescribed a serum catecholamine test. Which action should the nurse take when obtaining the sample? a. Discard the first sample and then begin the collection. b. Draw the blood sample after the client eats breakfast. c. Place the sample on ice and send to the laboratory immediately. d. Add preservatives before sending the sample to the laboratory.

ANS: C A blood sample for catecholamine must be placed on ice and taken to the laboratory immediately. This sample is not urine, and therefore the first sample should not be discarded nor should preservatives be added to the sample. The nurse should use the appropriate tube and obtain the sample based on which drugs are administered, not dietary schedules.

1. A client is receiving an infusion of tissue plasminogen activator (t-PA). The nurse assesses the client to be disoriented to person, place, and time. What action by the nurse is best? a. Assess the client's pupillary responses. b. Request a neurologic consultation. c. Stop the infusion and call the provider. d. Take and document a full set of vital signs.

ANS: C A change in neurologic status in a client receiving t-PA could indicate intracranial hemorrhage. The nurse should stop the infusion and notify the provider immediately. A full assessment, including pupillary responses and vital signs, occurs next. The nurse may or may not need to call a neurologist. DIF: Applying/Application REF: 768 KEY: Coronary artery disease| neurologic system| critical rescue| Rapid Response Team| thrombolytic agents MSC: Integrated Process: Nursing Process: Implementation NOT: Client Needs Category: Physiological Integrity: Pharmacological and Parenteral Therapies

7. A telemetry nurse assesses a client with third-degree heart block who has wide QRS complexes and a heart rate of 35 beats/min on the cardiac monitor. Which assessment should the nurse complete next? a. Pulmonary auscultation b. Pulse strength and amplitude c. Level of consciousness d. Mobility and gait stability

ANS: C A heart rate of 40 beats/min or less with widened QRS complexes could have hemodynamic consequences. The client is at risk for inadequate cerebral perfusion. The nurse should assess for level of consciousness, light-headedness, confusion, syncope, and seizure activity. Although the other assessments should be completed, the client's level of consciousness is the priority. DIF: Applying/Application REF: 670 KEY: Cardiac electrical conduction| vascular perfusion MSC: IntegratedProcess:NursingProcess:Assessment NOT: Client Needs Category: Physiological Integrity: Reduction of Risk Potential

3. A client has a pulmonary embolism and is started on oxygen. The student nurse asks why the client's oxygen saturation has not significantly improved. What response by the nurse is best? a. "Breathing so rapidly interferes with oxygenation." b. "Maybe the client has respiratory distress syndrome." c. "The blood clot interferes with perfusion in the lungs." d. "The client needs immediate intubation and mechanical ventilation."

ANS: C A large blood clot in the lungs will significantly impair gas exchange and oxygenation. Unless the clot is dissolved, this process will continue unabated. Hyperventilation can interfere with oxygenation by shallow breathing, but there is no evidence that the client is hyperventilating, and this is also not the most precise physiologic answer. Respiratory distress syndrome can occur, but this is not as likely. The client may need to be mechanically ventilated, but without concrete data on FiO2 and SaO2, the nurse cannot make that judgment.

15. A nurse assesses a client with bladder cancer who is recovering from a complete cystectomy with ileal conduit. Which assessment finding should alert the nurse to urgently contact the health care provider? a. The ileostomy is draining blood-tinged urine. b. There is serous sanguineous drainage present on the surgical dressing. c. The ileostomy stoma is pale and cyanotic in appearance. d. Oxygen saturations are 92% on room air.

ANS: C A pale or cyanotic stoma indicates impaired circulation to the stoma and must be treated to prevent necrosis. Blood-tinged urine and serous sanguineous drainage are expected after this type of surgery. Oxygen saturation of 92% on room air is at the low limit of normal.

24. The provider requests the nurse start an infusion of an inotropic agent on a client. How does the nurse explain the action of these drugs to the client and spouse? a. "It constricts vessels, improving blood flow." b. "It dilates vessels, which lessens the work of the heart." c. "It increases the force of the heart's contractions." d. "It slows the heart rate down for better filling."

ANS: C A positive inotrope is a medication that increases the strength of the heart's contractions. The other options are not correct. DIF: Remembering/Knowledge REF: 772 KEY: Coronary artery disease| inotropic agents| patient education MSC: IntegratedProcess:Teaching/Learning NOT: Client Needs Category: Physiological Integrity: Pharmacological and Parenteral Therapies

10. A client in the cardiac stepdown unit reports severe, crushing chest pain accompanied by nausea and vomiting. What action by the nurse takes priority? a. Administer an aspirin. b. Call for an electrocardiogram (ECG). c. Maintain airway patency. d. Notify the provider.

ANS: C Airway always is the priority. The other actions are important in this situation as well, but the nurse should stay with the client and ensure the airway remains patent (especially if vomiting occurs) while another person calls the provider (or Rapid Response Team) and facilitates getting an ECG done. Aspirin will probably be administered, depending on the provider's prescription and the client's current medications. DIF: Applying/Application REF: 769 KEY: Coronary artery disease| critical rescue| medical emergencies MSC: Integrated Process: Nursing Process: Implementation NOT: Client Needs Category: Safe and Effective Care Environment: Management of Care

15. A nurse cares for a client who is recovering from a closed percutaneous kidney biopsy. The client states, "My pain has suddenly increased from a 3 to a 10 on a scale of 0 to 10." Which action should the nurse take first? a. Reposition the client on the operative side. b. Administer the prescribed opioid analgesic. c. Assess the pulse rate and blood pressure. d. Examine the color of the client's urine.

ANS: C An increase in the intensity of pain after a percutaneous kidney biopsy is a symptom of internal hemorrhage. A change in vital signs can indicate that hemorrhage is occurring. Before other actions, the nurse must assess the client's hemodynamic status.

9. A confused client with pneumonia is admitted with an indwelling catheter in place. During interdisciplinary rounds the following day, which question should the nurse ask the primary health care provider? a. "Do you want daily weights on this client?" b. "Will the client be able to return home?" c. "Can we discontinue the indwelling catheter?" d. "Should we get another chest x-ray today?"

ANS: C An indwelling catheter dramatically increases the risks of urinary tract infection and urosepsis. Nursing staff should ensure that catheters are left in place only as long as they are medically needed. The nurse should inquire about removing the catheter. All other questions might be appropriate, but because of client safety, this question takes priority.

17. After teaching a client who is recovering from a heart transplant to change positions slowly, the client asks, "Why is this important?" How should the nurse respond? a. "Rapid position changes can create shear and friction forces, which can tear out your internal vascular sutures." b. "Your new vascular connections are more sensitive to position changes, leading to increased intravascular pressure and dizziness." c. "Your new heart is not connected to the nervous system and is unable to respond to decreases in blood pressure caused by position changes." d. "While your heart is recovering, blood flow is diverted away from the brain, increasing the risk for stroke when you stand up."

ANS: C Because the new heart is denervated, the baroreceptor and other mechanisms that compensate for blood pressure drops caused by position changes do not function. This allows orthostatic hypotension to persist in the postoperative period. The other options are false statements and do not correctly address the client's question. DIF: Understanding/Comprehension REF: 703 KEY: Transplant| patient education MSC: IntegratedProcess:NursingProcess:Implementation NOT: Client Needs Category: Physiological Integrity: Physiological Adaptation

A nurse evaluates laboratory results for a male client who reports fluid secretion from his breasts. Which hormone value should the nurse assess first? a. Posterior pituitary hormones b. Adrenal medulla hormones c. Anterior pituitary hormones d. Parathyroid hormone

ANS: C Breast fluid and milk production are induced by the presence of prolactin, secreted from the anterior pituitary gland. The other hormones would not cause fluid secretion from the client's breast.

14. A nurse cares for a client who has pyelonephritis. The client states, "I am embarrassed to talk about my symptoms." How should the nurse respond? a. "I am a professional. Your symptoms will be kept in confidence." b. "I understand. Elimination is a private topic and shouldn't be discussed." c. "Take your time. It is okay to use words that are familiar to you." d. "You seem anxious. Would you like a nurse of the same gender to care for you?"

ANS: C Clients may be uncomfortable discussing issues related to elimination and the genitourinary area. The nurse should encourage the client to use language that is familiar to the client. The nurse should not make promises that cannot be kept, like keeping the client's symptoms confidential. The nurse must assess the client and cannot take the time to stop the discussion or find another nurse to complete the assessment.

19. The nurse is teaching a client with chronic obstructive pulmonary disease who has been prescribed continuous oxygen therapy at home. Which statement indicates the client correctly understands the teaching? a. "I plan to wear my oxygen when I exercise and feel short of breath." b. "I will use my portable oxygen when grilling burgers in the backyard." c. "I plan to use cotton balls to cushion the oxygen tubing on my ears." d. "I will only smoke while I am wearing my oxygen via nasal cannula."

ANS: C Cotton balls can decrease pressure ulcers from the oxygen tubing. Continuous oxygen orders mean the client should wear the oxygen at all times. Oxygen fuels a fire. Wearing oxygen while grilling and smoking increases the risk for fire. DIF: Applying/Application REF: 563 KEY: Safety| patient education| oxygen therapy MSC: IntegratedProcess:NursingProcess:Assessment NOT: Client Needs Category: Safe and Effective Care Environment: Safety and Infection Control

6. The nurse is caring for a client with lung cancer who states, "I don't want any pain medication because I am afraid I'll become addicted." How should the nurse respond? a. "I will ask the provider to change your medication to a drug that is less potent." b. "Would you like me to use music therapy to distract you from your pain?" c. "It is unlikely you will become addicted when taking medicine for pain." d. "Would you like me to give you acetaminophen (Tylenol) instead?"

ANS: C Clients should be encouraged to take their pain medications; addiction usually is not an issue with a client in pain. The nurse would not request that the pain medication be changed unless it was not effective. Other methods to decrease pain can be used, in addition to pain medication. DIF: Applying/Application REF: 576 KEY: Cancer| pain| pharmacologic pain management MSC: Integrated Process: Nursing Process: Implementation NOT: Client Needs Category: Physiological Integrity: Pharmacological and Parenteral Therapies 7. After teaching a client who is prescribed salmeterol (Serevent), the nurse assesses the client's understanding. Which statement by the client indicates a need for additional teaching? a. "I will be certain to shake the inhaler well before I use it." b. "It may take a while before I notice a change in my asthma." c. "I will use the drug when I have an asthma attack." d. "I will be careful not to let the drug escape out of my nose and mouth." ANS: C Salmeterol is designed to prevent an asthma attack; it does not relieve or reverse symptoms. Salmeterol has a slow onset of action; therefore, it should not be used as a rescue drug. The drug must be shaken well because it has a tendency to separate easily. Poor technique on the client's part allows the drug to escape through the nose and mouth. DIF: Applying/Application REF: 554 KEY: Medication| patient education MSC: Integrated Process: Teaching/Learning NOT: Client Needs Category: Health Promotion and Maintenance

6. A nurse administers prescribed adenosine (Adenocard) to a client. Which response should the nurse assess for as the expected therapeutic response? a. Decreased intraocular pressure b. Increased heart rate c. Short period of asystole d. Hypertensive crisis

ANS: C Clients usually respond to adenosine with a short period of asystole, bradycardia, hypotension, dyspnea, and chest pain. Adenosine has no conclusive impact on intraocular pressure. DIF: Applying/Application REF: 662 KEY: Cardiac electrical conduction| medication MSC: Integrated Process: Nursing Process: Analysis NOT: Client Needs Category: Physiological Integrity: Pharmacological and Parenteral Therapies

11. A nurse cares for a client with atrial fibrillation who reports fatigue when completing activities of daily living. What interventions should the nurse implement to address this client's concerns? a. Administer oxygen therapy at 2 liters per nasal cannula. b. Provide the client with a sleeping pill to stimulate rest. c. Schedule periods of exercise and rest during the day. d. Ask unlicensed assistive personnel to help bathe the client.

ANS: C Clients who have atrial fibrillation are at risk for decreased cardiac output and fatigue when completing activities of daily living. The nurse should schedule periods of exercise and rest during the day to decrease fatigue. The other interventions will not assist the client with self-care activities. DIF: Applying/Application REF: 658 KEY: Cardiac electrical conduction MSC: Integrated Process: Teaching/Learning NOT: Client Needs Category: Physiological Integrity: Basic Care and Comfort

16. The nurse asks a client who has experienced ventricular dysrhythmias about substance abuse. The client asks, "Why do you want to know if I use cocaine?" How should the nurse respond? a. "Substance abuse puts clients at risk for many health issues." b. "The hospital requires that I ask you about cocaine use." c. "Clients who use cocaine are at risk for fatal dysrhythmias." d. "We can provide services for cessation of substance abuse."

ANS: C Clients who use cocaine or illicit inhalants are particularly at risk for potentially fatal dysrhythmias. The other responses do not adequately address the client's question. DIF: Remembering/Knowledge REF: 657 KEY: Cardiac electrical conduction| substance abuse MSC: IntegratedProcess:NursingProcess:Assessment NOT: Client Needs Category: Psychosocial Integrity

2. A nurse assesses a client in an outpatient clinic. Which statement alerts the nurse to the possibility of left-sided heart failure? a. "I have been drinking more water than usual." b. "I am awakened by the need to urinate at night." c. "I must stop halfway up the stairs to catch my breath." d. "I have experienced blurred vision on several occasions."

ANS: C Clients with left-sided heart failure report weakness or fatigue while performing normal activities of daily living, as well as difficulty breathing, or "catching their breath." This occurs as fluid moves into the alveoli. Nocturia is often seen with right-sided heart failure. Thirst and blurred vision are not related to heart failure. DIF: Understanding/Comprehension REF: 682 KEY: Heart failure| assessment/diagnostic examination MSC: IntegratedProcess:NursingProcess:Assessment NOT: Client Needs Category: Health Promotion and Maintenance

4. A nurse cares for a middle-aged female client with diabetes mellitus who is being treated for the third episode of acute pyelonephritis in the past year. The client asks, "What can I do to help prevent these infections?" How should the nurse respond? a. "Test your urine daily for the presence of ketone bodies and proteins." b. "Use tampons rather than sanitary napkins during your menstrual period." c. "Drink more water and empty your bladder more frequently during the day." d. "Keep your hemoglobin A1c under 9% by keeping your blood sugar controlled."

ANS: C Clients with long-standing diabetes mellitus are at risk for pyelonephritis for many reasons. Chronically elevated blood glucose levels spill glucose into the urine, changing the pH and providing a favorable climate for bacterial growth. The neuropathy associated with diabetes reduces bladder tone and reduces the client's sensation of bladder fullness. Thus, even with large amounts of urine, the client voids less frequently, allowing stasis and overgrowth of microorganisms. Increasing fluid intake (specifically water) and voiding frequently prevent stasis and bacterial overgrowth. Testing urine and using tampons will not help prevent pyelonephritis. A hemoglobin A1c of 9% is too high.

16. A nurse cares for a client with chronic obstructive pulmonary disease (COPD) who appears thin and disheveled. Which question should the nurse ask first? a. "Do you have a strong support system?" b. "What do you understand about your disease?" c. "Do you experience shortness of breath with basic activities?" d. "What medications are you prescribed to take each day?"

ANS: C Clients with severe COPD may not be able to perform daily activities, including bathing and eating, because of excessive shortness of breath. The nurse should ask the client if shortness of breath is interfering with basic activities. Although the nurse should know about the client's support systems, current knowledge, and medications, these questions do not address the client's appearance. DIF: Applying/Application REF: 561 KEY: Functional ability MSC: IntegratedProcess:NursingProcess:Assessment NOT: Client Needs Category: Physiological Integrity

22. A nurse cares for a female client who has a family history of cystic fibrosis. The client asks, "Will my children have cystic fibrosis?" How should the nurse respond? a. "Since many of your family members are carriers, your children will also be carriers of the gene." b. "Cystic fibrosis is an autosomal recessive disorder. If you are a carrier, your children will have the disorder." c. "Since you have a family history of cystic fibrosis, I would encourage you and your partner to be tested." d. "Cystic fibrosis is caused by a protein that controls the movement of chloride. Adjusting your diet will decrease the spread of this disorder."

ANS: C Cystic fibrosis is an autosomal recessive disorder in which both gene alleles must be mutated for the disorder to be expressed. The nurse should encourage both the client and partner to be tested for the abnormal gene. The other statements are not true. DIF: Applying/Application REF: 567 KEY: Gene| allele| health screening MSC: Integrated Process: Teaching/Learning NOT: Client Needs Category: Safe and Effective Care Environment: Management of Care

19. The nurse is taking the vital signs of a client after hemodialysis. Blood pressure is 110/58 mm Hg, pulse 66 beats/min, and temperature is 99.8° F (37.6° C). What is the most appropriate action by the nurse? a. Administer fluid to increase blood pressure. b. Check the white blood cell count. c. Monitor the client's temperature. d. Connect the client to an electrocardiographic (ECG) monitor.

ANS: C During hemodialysis, the dialysate is warmed to increase diffusion and prevent hypothermia. The client's temperature could reflect the temperature of the dialysate. There is no indication to check the white blood cell count or connect the client to an ECG monitor. The other vital signs are within normal limits.

14. A nurse teaches a client who is recovering from a urography. Which instruction should the nurse include in this client's discharge teaching? a. "Avoid direct contact with your urine for 24 hours until the radioisotope clears." b. "You may have some dribbling of urine for several weeks after this procedure." c. "Be sure to drink at least 3 liters of fluids today to help eliminate the dye faster." d. "Your skin may become slightly yellow from the dye used in this procedure."

ANS: C Dyes used in urography are potentially nephrotoxic. A large fluid intake will help the client eliminate the dye rapidly. Dyes used in urography are not radioactive, the client should not experience any dribbling of urine, and the dye should not change the color of the client's skin.

8. A nurse is assisting the health care provider who is intubating a client. The provider has been attempting to intubate for 40 seconds. What action by the nurse takes priority? a. Ensure the client has adequate sedation. b. Find another provider to intubate. c. Interrupt the procedure to give oxygen. d. Monitor the client's oxygen saturation.

ANS: C Each intubation attempt should not exceed 30 seconds (15 is preferable) as it causes hypoxia. The nurse should interrupt the intubation attempt and give the client oxygen. The nurse should also have adequate sedation during the procedure and monitor the client's oxygen saturation, but these do not take priority. Finding another provider is not appropriate at this time.

4. A nurse assesses a client with renal insufficiency and a low red blood cell count. The client asks, "Is my anemia related to the renal insufficiency?" How should the nurse respond? a. "Red blood cells produce erythropoietin, which increases blood flow to the kidneys." b. "Your anemia and renal insufficiency are related to inadequate vitamin D and a loss of bone density." c. "Erythropoietin is usually released from the kidneys and stimulates red blood cell production in the bone marrow." d. "Kidney insufficiency inhibits active transportation of red blood cells throughout the blood."

ANS: C Erythropoietin is produced in the kidney and is released in response to decreased oxygen tension in the renal blood supply. Erythropoietin stimulates red blood cell production in the bone marrow. Anemia and renal insufficiency are not manifestations of vitamin D deficiency. The kidneys do not play a role in the transportation of red blood cells or any other cells in the blood.

2. A client is admitted with a pulmonary embolism (PE). The client is young, healthy, and active and has no known risk factors for PE. What action by the nurse is most appropriate? a. Encourage the client to walk 5 minutes each hour. b. Refer the client to smoking cessation classes. c. Teach the client about factor V Leiden testing. d. Tell the client that sometimes no cause for disease is found.

ANS: C Factor V Leiden is an inherited thrombophilia that can lead to abnormal clotting events, including PE. A client with no known risk factors for this disorder should be referred for testing. Encouraging the client to walk is healthy, but is not related to the development of a PE in this case, nor is smoking. Although there are cases of disease where no cause is ever found, this assumption is premature.

1. A nurse assesses clients on the medical-surgical unit. Which client is at greatest risk for the development of bacterial cystitis? a. A 36-year-old female who has never been pregnant b. A 42-year-old male who is prescribed cyclophosphamide c. A 58-year-old female who is not taking estrogen replacement d. A 77-year-old male with mild congestive heart failure

ANS: C Females at any age are more susceptible to cystitis than men because of the shorter urethra in women. Postmenopausal women who are not on hormone replacement therapy are at increased risk for bacterial cystitis because of changes in the cells of the urethra and vagina. The middle-aged woman who has never been pregnant would not have a risk potential as high as the older woman who is not using hormone replacement therapy.

1. A nurse working in a geriatric clinic sees clients with "cold" symptoms and rhinitis. Which drug would be appropriate to teach these clients to take for their symptoms? a. Chlorpheniramine (Chlor-Trimeton) b. Diphenhydramine (Benadryl) c. Fexofenadine (Allegra) d. Hydroxyzine (Vistaril)

ANS: C First-generation antihistamines are not appropriate for use in the older population. These drugs include chlorpheniramine, diphenhydramine, and hydroxyzine. Fexofenadine is a second-generation antihistamine.

1. A nurse working in a geriatric clinic sees clients with "cold" symptoms and rhinitis. Which drug would be appropriate to teach these clients to take for their symptoms? a. Chlorpheniramine (Chlor-Trimeton) b. Diphenhydramine (Benadryl) c. Fexofenadine (Allegra) d. Hydroxyzine (Vistaril)

ANS: C First-generation antihistamines are not appropriate for use in the older population. These drugs include chlorpheniramine, diphenhydramine, and hydroxyzine. Fexofenadine is a second-generation antihistamine.

13. A client has an intra-arterial blood pressure monitoring line. The nurse notes bright red blood on the client's sheets. What action should the nurse perform first? a. Assess the insertion site. b. Change the client's sheets. c. Put on a pair of gloves. d. Assess blood pressure.

ANS: C For the nurse's safety, he or she should put on a pair of gloves to prevent blood exposure. The other actions are appropriate as well, but first the nurse must don a pair of gloves. DIF: Applying/Application REF: 771 KEY: Standard Precautions| infection control| intra-arterial blood pressure monitoring| staff safety MSC: Integrated Process: Nursing Process: Implementation NOT: Client Needs Category: Safe and Effective Care Environment: Safety and Infection Control

12. A nurse has educated a client on isoniazid (INH). What statement by the client indicates teaching has been effective? a. "I need to take extra vitamin C while on INH." b. "I should take this medicine with milk or juice." c. "I will take this medication on an empty stomach." d. "My contact lenses will be permanently stained."

ANS: C INH needs to be taken on an empty stomach, either 1 hour before or 2 hours after meals. Extra vitamin B needs to be taken while on the drug. Staining of contact lenses commonly occurs while taking rifampin (Rifadin).

12. A nurse has educated a client on isoniazid (INH). What statement by the client indicates teaching has been effective? a. "I need to take extra vitamin C while on INH." b. "I should take this medicine with milk or juice." c. "I will take this medication on an empty stomach." d. "My contact lenses will be permanently stained."

ANS: C INH needs to be taken on an empty stomach, either 1 hour before or 2 hours after meals. Extra vitamin B needs to be taken while on the drug. Staining of contact lenses commonly occurs while taking rifampin (Rifadin).

25. The nurse is teaching a client how to increase the flow of dialysate into the peritoneal cavity during dialysis. Which statement by the client demonstrates a correct understanding of the teaching? a. "I should leave the drainage bag above the level of my abdomen." b. "I could flush the tubing with normal saline if the flow stops." c. "I should take a stool softener every morning to avoid constipation." d. "My diet should have low fiber in it to prevent any irritation."

ANS: C Inflow and outflow problems of the dialysate are best controlled by preventing constipation. A daily stool softener is the best option for the client. The drainage bag should be below the level of the abdomen. Flushing the tubing will not help with the flow. A diet high in fiber will also help with a constipation problem.

16. A nurse obtains a sterile urine specimen from a client's Foley catheter. After applying a clamp to the drainage tubing distal to the injection port, which action should the nurse take next? a. Clamp another section of the tube to create a fixed sample section for retrieval. b. Insert a syringe into the injection port and aspirate the quantity of urine required. c. Clean the injection port cap of the drainage tubing with povidone-iodine solution. d. Withdraw 10 mL of urine and discard it; then withdraw a fresh sample of urine.

ANS: C It is important to clean the injection port cap of the catheter drainage tubing with an appropriate antiseptic, such as povidone-iodine solution or alcohol. This will help prevent surface contamination before injection of the syringe. The urine sample should be collected directly from the catheter; therefore, a second clamp to create a sample section would not be appropriate. Every sample from the catheter is usable; there is the need to discard the first sample.

3. After teaching a client who is prescribed a long-acting beta2 agonist medication, a nurse assesses the client's understanding. Which statement indicates the client comprehends the teaching? a. "I will carry this medication with me at all times in case I need it." b. "I will take this medication when I start to experience an asthma attack." c. "I will take this medication every morning to help prevent an acute attack." d. "I will be weaned off this medication when I no longer need it."

ANS: C Long-acting beta2 agonist medications will help prevent an acute asthma attack because they are long acting. The client will take this medication every day for best effect. The client does not have to always keep this medication with him or her because it is not used as a rescue medication. This is not the medication the client will use during an acute asthma attack because it does not have an immediate onset of action. The client will not be weaned off this medication because this is likely to be one of his or her daily medications. DIF: Applying/Application REF: 554 KEY: Medication| patient education MSC: Integrated Process: Teaching/Learning NOT: Client Needs Category: Physiological Integrity: Pharmacological and Parenteral Therapies

8. A nurse cares for a client with chronic obstructive pulmonary disease (COPD). The client states that he no longer enjoys going out with his friends. How should the nurse respond? a. "There are a variety of support groups for people who have COPD." b. "I will ask your provider to prescribe you with an antianxiety agent." c. "Share any thoughts and feelings that cause you to limit social activities." d. "Friends can be a good support system for clients with chronic disorders."

ANS: C Many clients with moderate to severe COPD become socially isolated because they are embarrassed by frequent coughing and mucus production. They also can experience fatigue, which limits their activities. The nurse needs to encourage the client to verbalize thoughts and feelings so that appropriate interventions can be selected. Joining a support group would not decrease feelings of social isolation if the client does not verbalize feelings. Antianxiety agents will not help the client with social isolation. Encouraging a client to participate in activities without verbalizing concerns also would not be an effective strategy for decreasing social isolation. DIF: Applying/Application REF: 561 KEY: Coping| support MSC: IntegratedProcess:Caring NOT: Client Needs Category: Psychosocial Integrity 9. A nurse is teaching a client who has cystic fibrosis (CF). Which statement should the nurse include in this client's teaching? a. "Take an antibiotic each day." b. "Contact your provider to obtain genetic screening." c. "Eat a well-balanced, nutritious diet." d. "Plan to exercise for 30 minutes every day." ANS: C Clients with CF often are malnourished due to vitamin deficiency and pancreatic malfunction. Maintaining nutrition is essential. Daily antibiotics and daily exercise are not essential actions. Genetic screening would not help the client manage CF better. DIF: Applying/Application REF: 567 KEY: Nutrition| patient education MSC: Integrated Process: Teaching/Learning NOT: Client Needs Category: Health Promotion and Maintenance

12. A nurse is caring for a client who has sleep apnea and is prescribed modafinil (Provigil). The client asks, "How will this medication help me?" How should the nurse respond? a. "This medication will treat your sleep apnea." b. "This sedative will help you to sleep at night." c. "This medication will promote daytime wakefulness." d. "This analgesic will increase comfort while you sleep."

ANS: C Modafinil is helpful for clients who have narcolepsy (uncontrollable daytime sleep) related to sleep apnea. This medication promotes daytime wakefulness. DIF: Remembering/Knowledge REF: 535 KEY: Medication MSC: IntegratedProcess:Teaching/Learning NOT: Client Needs Category: Physiological Integrity: Pharmacological and Parenteral Therapies

10. A nurse is caring for several older clients in the hospital that the nurse identifies as being at high risk for healthcare-associated pneumonia. To reduce this risk, what activity should the nurse delegate to the unlicensed assistive personnel (UAP)? a. Encourage between-meal snacks. b. Monitor temperature every 4 hours. c. Provide oral care every 4 hours. d. Report any new onset of cough.

ANS: C Oral colonization by gram-negative bacteria is a risk factor for healthcare-associated pneumonia. Good, frequent oral care can help prevent this from developing and is a task that can be delegated to the UAP. Encouraging good nutrition is important, but this will not prevent pneumonia. Monitoring temperature and reporting new cough in clients is important to detect the onset of possible pneumonia but do not prevent it.

10. A nurse is caring for several older clients in the hospital that the nurse identifies as being at high risk for healthcare-associated pneumonia. To reduce this risk, what activity should the nurse delegate to the unlicensed assistive personnel (UAP)? a. Encourage between-meal snacks. b. Monitor temperature every 4 hours. c. Provide oral care every 4 hours. d. Report any new onset of cough.

ANS: C Oral colonization by gram-negative bacteria is a risk factor for healthcare-associated pneumonia. Good, frequent oral care can help prevent this from developing and is a task that can be delegated to the UAP. Encouraging good nutrition is important, but this will not prevent pneumonia. Monitoring temperature and reporting new cough in clients is important to detect the onset of possible pneumonia but do not prevent it.

A nurse cares for a client with excessive production of thyrocalcitonin (calcitonin). For which electrolyte imbalance should the nurse assess? a. Potassium b. Sodium c. Calcium d. Magnesium

ANS: C Parafollicular cells produce thyrocalcitonin (calcitonin), which regulates serum calcium levels. Calcitonin has no impact on potassium, sodium, or magnesium balances.

4. After teaching a client with bacterial cystitis who is prescribed phenazopyridine (Pyridium), the nurse assesses the client's understanding. Which statement made by the client indicates a correct understanding of the teaching? a. "I will not take this drug with food or milk." b. "If I think I am pregnant, I will stop the drug." c. "An orange color in my urine should not alarm me." d. "I will drink two glasses of cranberry juice daily."

ANS: C Phenazopyridine discolors urine, most commonly to a deep reddish orange. Many clients think they have blood in their urine when they see this. In addition, the urine can permanently stain clothing. Phenazopyridine is safe to take if the client is pregnant. There are no dietary restrictions or needs while taking this medication.

22. A client had an acute myocardial infarction. What assessment finding indicates to the nurse that a significant complication has occurred? a. Blood pressure that is 20 mm Hg below baseline b. Oxygen saturation of 94% on room air c. Poor peripheral pulses and cool skin d. Urine output of 1.2 mL/kg/hr for 4 hours

ANS: C Poor peripheral pulses and cool skin may be signs of impending cardiogenic shock and should be reported immediately. A blood pressure drop of 20 mm Hg is not worrisome. An oxygen saturation of 94% is just slightly below normal. A urine output of 1.2 mL/kg/hr for 4 hours is normal. DIF: Remembering/Knowledge REF: 772 KEY: Coronary artery disease| critical rescue| nursing assessment MSC: Integrated Process: Nursing Process: Analysis NOT: Client Needs Category: Physiological Integrity: Reduction of Risk Potential

26. A nurse provides phone triage to a pregnant client. The client states, "I am experiencing a burning pain when I urinate." How should the nurse respond? a. "This means labor will start soon. Prepare to go to the hospital." b. "You probably have a urinary tract infection. Drink more cranberry juice." c. "Make an appointment with your provider to have your infection treated." d. "Your pelvic wall is weakening. Pelvic muscle exercises should help."

ANS: C Pregnant clients with a urinary tract infection require prompt and aggressive treatment because cystitis can lead to acute pyelonephritis during pregnancy. The nurse should encourage the client to make an appointment and have the infection treated. Burning pain when urinating does not indicate the start of labor or weakening of pelvic muscles.

23. A client presents to the emergency department with an acute myocardial infarction (MI) at 1500 (3:00 PM). The facility has 24-hour catheterization laboratory abilities. To meet The Joint Commission's Core Measures set, by what time should the client have a percutaneous coronary intervention performed? a. 1530 (3:30 PM) b. 1600 (4:00 PM) c. 1630 (4:30 PM) d. 1700 (5:00 PM)

ANS: C The Joint Commission's Core Measures set for MI includes percutaneous coronary intervention within 90 minutes of diagnosis of myocardial infarction. Therefore, the client should have a percutaneous coronary intervention performed no later than 1630 (4:30 PM). DIF: Remembering/Knowledge REF: 774 KEY: Coronary artery disease| Core Measures| The Joint Commission MSC: Integrated Process: Communication and Documentation NOT: Client Needs Category: Safe and Effective Care Environment: Management of Care

17. A client is placed on fluid restrictions because of chronic kidney disease (CKD). Which assessment finding would alert the nurse that the client's fluid balance is stable at this time? a. Decreased calcium levels b. Increased phosphorus levels c. No adventitious sounds in the lungs d. Increased edema in the legs

ANS: C The absence of adventitious sounds upon auscultation of the lungs indicates a lack of fluid overload and fluid balance in the client's body. Decreased calcium levels and increased phosphorus levels are common findings with CKD. Edema would indicate a fluid imbalance.

5. A nurse assesses clients on the medical-surgical unit. Which client is at greatest risk for development of obstructive sleep apnea? a. A 26-year-old woman who is 8 months pregnant b. A 42-year-old man with gastroesophageal reflux disease c. A 55-year-old woman who is 50 pounds overweight d. A 73-year-old man with type 2 diabetes mellitus

ANS: C The client at highest risk would be the one who is extremely overweight. None of the other clients have risk factors for sleep apnea. DIF: Applying/Application REF: 535 KEY: Respiratory distress/failure MSC: IntegratedProcess:NursingProcess:Assessment NOT: Client Needs Category: Health Promotion and Maintenance

10. The charge nurse of the medical-surgical unit is making staff assignments. Which staff member should be assigned to a client with chronic kidney disease who is exhibiting a low-grade fever and a pericardial friction rub? a. Registered nurse who just floated from the surgical unit b. Registered nurse who just floated from the dialysis unit c. Registered nurse who was assigned the same client yesterday d. Licensed practical nurse with 5 years' experience on this floor

ANS: C The client is exhibiting symptoms of pericarditis, which can occur with chronic kidney disease. Continuity of care is important to assess subtle differences in clients. Therefore, the registered nurse (RN) who was assigned to this client previously should again give care to this client. The float nurses would not be as knowledgeable about the unit and its clients. The licensed practical nurse may not have the education level of the RN to assess for pericarditis.

28. A nurse reviews these laboratory values of a client who returned from kidney transplantation 12 hours ago: What initial intervention would the nurse anticipate? a. Start hemodialysis immediately. b. Discuss the need for peritoneal dialysis. c. Increase the dose of immunosuppression. d. Return the client to surgery for exploration.

ANS: C The client may need a higher dose of immunosuppressive medication as evidenced by the elevated BUN and serum creatinine levels. This increased dose may reverse the possible acute rejection of the transplanted kidney. The client does not need hemodialysis, peritoneal dialysis, or further surgery at this point.

10. A client is on a ventilator and is sedated. What care may the nurse delegate to the unlicensed assistive personnel (UAP)? a. Assess the client for sedation needs. b. Get family permission for restraints. c. Provide frequent oral care per protocol. d. Use nonverbal pain assessment tools.

ANS: C The client on mechanical ventilation needs frequent oral care, which can be delegated to the UAP. The other actions fall within the scope of practice of the nurse.

10. A nurse cares for a client who is scheduled for a total laryngectomy. Which action should the nurse take prior to surgery? a. Assess airway patency, breathing, and circulation. b. Administer prescribed intravenous pain medication. c. Assist the client to choose a communication method. d. Ambulate the client in the hallway to assess gait.

ANS: C The client will not be able to speak after surgery. The nurse should assist the client to choose a communication method that he or she would like to use after surgery. Assessing the client's airway and administering IV pain medication are done after the procedure. Although ambulation promotes health and decreases the complications of any surgery, this client's gait should not be impacted by a total laryngectomy and therefore is not a priority. DIF: Applying/Application REF: 540 KEY: Surgical care MSC: IntegratedProcess:Caring NOT: Client Needs Category: Psychosocial Integrity

21. A nurse cares for a client who tests positive for alpha1-antitrypsin (AAT) deficiency. The client asks, "What does this mean?" How should the nurse respond? a. "Your children will be at high risk for the development of chronic obstructive pulmonary disease." b. "I will contact a genetic counselor to discuss your condition." c. "Your risk for chronic obstructive pulmonary disease is higher, especially if you smoke." d. "This is a recessive gene and should have no impact on your health."

ANS: C The gene for AAT is a recessive gene. Clients with only one allele produce enough AAT to prevent chronic obstructive pulmonary disease (COPD) unless the client smokes. A client with two alleles is at high risk for COPD even if not exposed to smoke or other irritants. The client is a carrier, and children may or may not be at high risk depending on the partner's AAT levels. Contacting a genetic counselor may be helpful but does not address the client's current question. DIF: Applying/Application REF: 558 KEY: Gene| allele| health screening| a1AT (alpha1-antitrypsin) gene MSC: IntegratedProcess:Teaching/Learning NOT: Client Needs Category: Safe and Effective Care Environment: Management of Care

25. A nurse cares for a client with urinary incontinence. The client states, "I am so embarrassed. My bladder leaks like a young child's bladder." How should the nurse respond? a. "I understand how you feel. I would be mortified." b. "Incontinence pads will minimize leaks in public." c. "I can teach you strategies to help control your incontinence." d. "More women experience incontinence than you might think."

ANS: C The nurse should accept and acknowledge the client's concerns, and assist the client to learn techniques that will allow control of urinary incontinence. The nurse should not diminish the client's concerns with the use of pads or stating statistics about the occurrence of incontinence.

21. The charge nurse is orienting a float nurse to an assigned client with an arteriovenous (AV) fistula for hemodialysis in her left arm. Which action by the float nurse would be considered unsafe? a. Palpating the access site for a bruit or thrill b. Using the right arm for a blood pressure reading c. Administering intravenous fluids through the AV fistula d. Checking distal pulses in the left arm

ANS: C The nurse should not use the arm with the AV fistula for intravenous infusion, blood pressure readings, or venipuncture. Compression and infection can result in the loss of the AV fistula. The AV fistula should be monitored by auscultating or palpating the access site. Checking the distal pulse would be an appropriate assessment.

15. A client has been brought to the emergency department with a life-threatening chest injury. What action by the nurse takes priority? a. Apply oxygen at 100%. b. Assess the respiratory rate. c. Ensure a patent airway. d. Start two large-bore IV lines.

ANS: C The priority for any chest trauma client is airway, breathing, circulation. The nurse first ensures the client has a patent airway. Assessing respiratory rate and applying oxygen are next, followed by inserting IVs.

6. "Remove the mouthpiece from your mouth, keep your lips closed, and hold your breath for at least 10 seconds." a. 2, 3, 4, 5, 6, 1 b. 3, 4, 5, 1, 6, 2 c. 4, 3, 5, 1, 2, 6 d. 5, 3, 6, 1, 2, 4

ANS: C The proper order for correctly using an inhaler with a spacer is as follows. Insert the mouthpiece of the inhaler into the nonmouthpiece end of the spacer. Shake the whole unit vigorously three or four times. Place the mouthpiece into the mouth, over the tongue, and seal the lips tightly around it. Press down firmly on the canister of the inhaler to release one dose of medication into the spacer. Breathe in slowly and deeply. Remove the mouthpiece from the mouth, and, keeping the lips closed, hold the breath for at least 10 seconds. Then breathe out slowly. Wait at least 1 minute between puffs. DIF: Applying/Application REF: MSC: IntegratedProcess:Teaching/Learning NOT: Client Needs Category: Physiological Integrity: Pharmacological and Parenteral Therapies

20. A client is admitted with suspected pneumonia from the emergency department. The client went to the primary care provider a "few days ago" and shows the nurse the results of what the client calls "an allergy test," as shown below: What action by the nurse takes priority? a. Assess the client for possible items to which he or she is allergic. b. Call the primary care provider's office to request records. c. Immediately place the client on Airborne Precautions. d. Prepare to begin administration of intravenous antibiotics.

ANS: C This "allergy test" is actually a positive tuberculosis test. The client should be placed on Airborne Precautions immediately. The other options do not take priority over preventing the spread of the disease.

20. A client is admitted with suspected pneumonia from the emergency department. The client went to the primary care provider a "few days ago" and shows the nurse the results of what the client calls "an allergy test," as shown below: What action by the nurse takes priority? a. Assess the client for possible items to which he or she is allergic. b. Call the primary care provider's office to request records. c. Immediately place the client on Airborne Precautions. d. Prepare to begin administration of intravenous antibiotics.

ANS: C This "allergy test" is actually a positive tuberculosis test. The client should be placed on Airborne Precautions immediately. The other options do not take priority over preventing the spread of the disease.

A nurse reviews laboratory results for a client with diabetes mellitus who presents with polyuria, lethargy, and a blood glucose of 560 mg/dL. Which laboratory result should the nurse correlate with the client's polyuria? a. Serum sodium: 163 mEq/L b. Serum creatinine: 1.6 mg/dL c. Presence of urine ketone bodies d. Serum osmolarity: 375 mOsm/kg

ANS: D Hyperglycemia causes hyperosmolarity of extracellular fluid. This leads to polyuria from an osmotic diuresis. The client's serum osmolarity is high. The client's sodium would be expected to be high owing to dehydration. Serum creatinine and urine ketone bodies are not related to the polyuria.

2. A nurse in a family practice clinic is preparing discharge instructions for a client reporting facial pain that is worse when bending over, tenderness across the cheeks, and postnasal discharge. What instruction will be most helpful? a. "Ice packs may help with the facial pain." b. "Limit fluids to dry out your sinuses." c. "Try warm, moist heat packs on your face." d. "We will schedule you for a computed tomography scan this week."

ANS: C This client has rhinosinusitis. Comfort measures for this condition include breathing in warm steam, hot packs, nasal saline irrigations, sleeping with the head elevated, increased fluids, and avoiding cigarette smoke. The client does not need a CT scan.

20. A nurse performs an admission assessment on a 75-year-old client with multiple chronic diseases. The client's blood pressure is 135/75 mm Hg and oxygen saturation is 94% on 2 liters per nasal cannula. The nurse assesses the client's rhythm on the cardiac monitor and observes the reading shown below: Which action should the nurse take first? a. Begin external temporary pacing. b. Assess peripheral pulse strength. c. Ask the client what medications he or she takes. d. Administer 1 mg of atropine.

ANS: C This client is stable and therefore does not require any intervention except to determine the cause of the bradycardia. Bradycardia is often caused by medications. Clients who have multiple chronic diseases are often on multiple medications that can interact with each other. The nurse should assess the client's current medications first. DIF: Applying/Application REF: 658 KEY: Cardiac electrical conduction| medications| adverse effects MSC: IntegratedProcess:NursingProcess:Assessment NOT: Client Needs Category: Physiological Integrity: Pharmacological and Parenteral Therapies

9. An intubated client's oxygen saturation has dropped to 88%. What action by the nurse takes priority? a. Determine if the tube is kinked. b. Ensure all connections are patent. c. Listen to the client's lung sounds. d. Suction the endotracheal tube.

ANS: C When an intubated client shows signs of hypoxia, check for DOPE: displaced tube (most common cause), obstruction (often by secretions), pneumothorax, and equipment problems. The nurse listens for equal, bilateral breath sounds first to determine if the endotracheal tube is still correctly placed. If this assessment is normal, the nurse would follow the mnemonic and assess the patency of the tube and connections and perform suction.

13. A client is diagnosed with chronic kidney disease (CKD). What is an ideal goal of treatment set by the nurse in the care plan to reduce the risk of pulmonary edema? a. Maintaining oxygen saturation of 89% b. Minimal crackles and wheezes in lung sounds c. Maintaining a balanced intake and output d. Limited shortness of breath upon exertion

ANS: C With an optimal fluid balance, the client will be more able to eject blood from the left ventricle without increased pressure in the left ventricle and pulmonary vessels. Other ideal goals are oxygen saturations greater than 92%, no auscultated crackles or wheezes, and no demonstrated shortness of breath.

3. A nurse reviews laboratory results for a client with glomerulonephritis. The client's glomerular filtration rate (GFR) is 40 mL/min as measured by a 24-hour creatinine clearance. How should the nurse interpret this finding? (Select all that apply.) a. Excessive GFR b. Normal GFR c. Reduced GFR d. Potential for fluid overload e. Potential for dehydration

ANS: C, D The GFR refers to the initial amount of urine that the kidneys filter from the blood. In the healthy adult, the normal GFR ranges between 100 and 120 mL/min, most of which is reabsorbed in the kidney tubules. A GFR of 40 mL/min is drastically reduced, with the client experiencing fluid retention and risks for hypertension and pulmonary edema as a result of excess vascular fluid.

2. When working with women who are taking hormonal birth control, what health promotion measures should the nurse teach to prevent possible pulmonary embolism (PE)? (Select all that apply.) a. Avoid drinking alcohol. b. Eat more omega-3 fatty acids. c. Exercise on a regular basis. d. Maintain a healthy weight. e. Stop smoking cigarettes.

ANS: C, D, E Health promotion measures for clients to prevent thromboembolic events such as PE include maintaining a healthy weight, exercising on a regular basis, and not smoking. Avoiding alcohol and eating more foods containing omega-3 fatty acids are heart-healthy actions but do not relate to the prevention of PE.

1. A nurse assesses a client with asthma and notes bilateral wheezing, decreased pulse oxygen saturation, and suprasternal retraction on inhalation. Which actions should the nurse take? (Select all that apply.) 555 KEY: Medication safety Arterial Blood Gas Results Vital Signs pH = 7.32 PaCO2 = 62 mm Hg PaO2 = 46 mm Hg HCO3- = 28 mEq/L Heart rate = 110 beats/min Respiratory rate = 12 breaths/min Blood pressure = 145/65 mm Hg Oxygen saturation = 76% a. Administer prescribed salmeterol (Serevent) inhaler. b. Assess the client for a tracheal deviation. c. Administer oxygen to keep saturations greater than 94%. d. Perform peak expiratory flow readings. e. Administer prescribed albuterol (Proventil) inhaler.

ANS: C, E Suprasternal retraction caused by inhalation usually indicates that the client is using accessory muscles and is having difficulty moving air into the respiratory passages because of airway narrowing. Wheezing indicates a narrowed airway; a decreased pulse oxygen saturation also supports this finding. The asthma is not responding to the medication, and intervention is needed. Administration of a rescue inhaler is indicated, probably along with administration of oxygen. The nurse would not do a peak flow reading at this time, nor would a code be called. Midline trachea is a normal and expected finding. DIF: Applying/Application REF: 575 KEY: Respiratory distress/failure MSC: Integrated Process: Nursing Process: Implementation NOT: Client Needs Category: Physiological Integrity: Pharmacological and Parenteral Therapies

A nurse teaches a client with diabetes mellitus who is experiencing numbness and reduced sensation. Which statement should the nurse include in this client's teaching to prevent injury? a. "Examine your feet using a mirror every day." b. "Rotate your insulin injection sites every week." c. "Check your blood glucose level before each meal." d. "Use a bath thermometer to test the water temperature."

ANS: D Clients with diminished sensory perception can easily experience a burn injury when bathwater is too hot. Instead of checking the temperature of the water by feeling it, they should use a thermometer. Examining the feet daily does not prevent injury, although daily foot examinations are important to find problems so they can be addressed. Rotating insulin and checking blood glucose levels will not prevent injury.

When teaching a client recently diagnosed with type 1 diabetes mellitus, the client states, "I will never be able to stick myself with a needle." How should the nurse respond? a. "I can give your injections to you while you are here in the hospital." b. "Everyone gets used to giving themselves injections. It really does not hurt." c. "Your disease will not be managed properly if you refuse to administer the shots." d. "Tell me what it is about the injections that are concerning you."`

ANS: D Devote as much teaching time as possible to insulin injection and blood glucose monitoring. Clients with newly diagnosed diabetes are often fearful of giving themselves injections. If the client is worried about giving the injections, it is best to try to find out what specifically is causing the concern, so it can be addressed. Giving the injections for the client does not promote self-care ability. Telling the client that others give themselves injections may cause the client to feel bad. Stating that you don't know another way to manage the disease is dismissive of the client's concerns.

After teaching a client who has diabetes mellitus and proliferative retinopathy, nephropathy, and peripheral neuropathy, the nurse assesses the client's understanding. Which statement made by the client indicates a correct understanding of the teaching? a. "I have so many complications; exercising is not recommended." b. "I will exercise more frequently because I have so many complications." c. "I used to run for exercise; I will start training for a marathon." d. "I should look into swimming or water aerobics to get my exercise."

ANS: D Exercise is not contraindicated for this client, although modifications based on existing pathology are necessary to prevent further injury. Swimming or water aerobics will give the client exercise without the worry of having the correct shoes or developing a foot injury. The client should not exercise too vigorously.

A nurse reviews the medication list of a client recovering from a computed tomography (CT) scan with IV contrast to rule out small bowel obstruction. Which medication should alert the nurse to contact the provider and withhold the prescribed dose? a. Pioglitazone (Actos) b. Glimepiride (Amaryl) c. Glipizide (Glucotrol) d. Metformin (Glucophage)

ANS: D Glucophage should not be administered when the kidneys are attempting to excrete IV contrast from the body. This combination would place the client at high risk for kidney failure. The nurse should hold the metformin dose and contact the provider. The other medications are safe to administer after receiving IV contrast.

A nurse teaches a client who is prescribed an insulin pump. Which statement should the nurse include in this client's discharge education? a. "Test your urine daily for ketones." b. "Use only buffered insulin in your pump." c. "Store the insulin in the freezer until you need it." d. "Change the needle every 3 days."

ANS: D Having the same needle remain in place through the skin for longer than 3 days drastically increases the risk for infection in or through the delivery system. Having an insulin pump does not require the client to test for ketones in the urine. Insulin should not be frozen. Insulin is not buffered.

A nurse reviews the laboratory results of a client who is receiving intravenous insulin. Which should alert the nurse to intervene immediately? a. Serum chloride level of 98 mmol/L b. Serum calcium level of 8.8 mg/dL c. Serum sodium level of 132 mmol/L d. Serum potassium level of 2.5 mmol/L

ANS: D Insulin activates the sodium-potassium ATPase pump, increasing the movement of potassium from the extracellular fluid into the intracellular fluid, resulting in hypokalemia. In hyperglycemia, hypokalemia can also result from excessive urine loss of potassium. The chloride level is normal. The calcium and sodium levels are slightly low, but this would not be related to hyperglycemia and insulin administration.

A nurse teaches a client with type 1 diabetes mellitus. Which statement should the nurse include in this client's teaching to decrease the client's insulin needs? a. "Limit your fluid intake to 2 liters a day." b. "Animal organ meat is high in insulin." c. "Limit your carbohydrate intake to 80 grams a day." d. "Walk at a moderate pace for 1 mile daily."

ANS: D Moderate exercise such as walking helps regulate blood glucose levels on a daily basis and results in lowered insulin requirements for clients with type 1 diabetes mellitus. Restricting fluids and eating organ meats will not reduce insulin needs. People with diabetes need at least 130 grams of carbohydrates each day.

After teaching a client with type 2 diabetes mellitus who is prescribed nateglinide (Starlix), the nurse assesses the client's understanding. Which statement made by the client indicates a correct understanding of the prescribed therapy? a. "I'll take this medicine during each of my meals." b. "I must take this medicine in the morning when I wake." c. "I will take this medicine before I go to bed." d. "I will take this medicine immediately before I eat."

ANS: D Nateglinide is an insulin secretagogue that is designed to increase meal-related insulin secretion. It should be taken immediately before each meal. The medication should not be taken without eating as it will decrease the client's blood glucose levels. The medication should be taken before meals instead of during meals.

20. A nurse cares for a client who has a pleural chest tube. Which action should the nurse take to ensure safe use of this equipment? a. Strip the tubing to minimize clot formation and ensure patency. b. Secure tubing junctions with clamps to prevent accidental disconnections. c. Connect the chest tube to wall suction at the level prescribed by the provider. d. Keep padded clamps at the bedside for use if the drainage system is interrupted.

ANS: D Padded clamps should be kept at the bedside for use if the drainage system becomes dislodged or is interrupted. The nurse should never strip the tubing. Tubing junctions should be taped, not clamped. Wall suction should be set at the level indicated by the device's manufacturer, not the provider. DIF: Remembering/Knowledge REF: 578 KEY: Drains| postsurgical care MSC: Integrated Process: Nursing Process: Implementation NOT: Client Needs Category: Safe and Effective Care Environment: Safety and Infection Control

A nurse assesses a client with diabetes mellitus who self-administers subcutaneous insulin. The nurse notes a spongy, swelling area at the site the client uses most frequently for insulin injection. Which action should the nurse take? a. Apply ice to the site to reduce inflammation. b. Consult the provider for a new administration route. c. Assess the client for other signs of cellulitis. d. Instruct the client to rotate sites for insulin injection.

ANS: D The client's tissue has been damaged from continuous use of the same site. The client should be educated to rotate sites. The damaged tissue is not caused by cellulitis or any type infection, and applying ice may cause more damage to the tissue. Insulin can only be administered subcutaneously and intravenously. It would not be appropriate or practical to change the administration route.

A nurse cares for a client experiencing diabetic ketoacidosis who presents with Kussmaul respirations. Which action should the nurse take? a. Administration of oxygen via face mask b. Intravenous administration of 10% glucose c. Implementation of seizure precautions d. Administration of intravenous insulin

ANS: D The rapid, deep respiratory efforts of Kussmaul respirations are the body's attempt to reduce the acids produced by using fat rather than glucose for fuel. Only the administration of insulin will reduce this type of respiration by assisting glucose to move into cells and to be used for fuel instead of fat. The client who is in ketoacidosis may not experience any respiratory impairment and therefore does not need additional oxygen. Giving the client glucose would be contraindicated. The client does not require seizure precautions.

8. A nurse assesses a client after administering isosorbide mononitrate (Imdur). The client reports a headache. Which action should the nurse take? a. Initiate oxygen therapy. b. Hold the next dose of Imdur. c. Instruct the client to drink water. d. Administer PRN acetaminophen.

ANS: D The vasodilating effects of isosorbide mononitrate frequently cause clients to have headaches during the initial period of therapy. Clients should be told about this side effect and encouraged to take the medication with food. Some clients obtain relief with mild analgesics, such as acetaminophen. The client's headache is not related to hypoxia or dehydration; therefore, these interventions would not help. The client needs to take the medication as prescribed to prevent angina; the medication should not be held. DIF: Applying/Application REF: 686 KEY: Heart failure| nitroglycerin/nitrates| medication| pharmacologic pain management MSC: Integrated Process: Nursing Process: Implementation NOT: Client Needs Category: Physiological Integrity: Pharmacological and Parenteral Therapies

A nurse cares for a client who has type 1 diabetes mellitus. The client asks, "Is it okay for me to have an occasional glass of wine?" How should the nurse respond? a. "Drinking any wine or alcohol will increase your insulin requirements." b. "Because of poor kidney function, people with diabetes should avoid alcohol." c. "You should not drink alcohol because it will make you hungry and overeat." d. "One glass of wine is okay with a meal and is counted as two fat exchanges."

ANS: D Under normal circumstances, blood glucose levels will not be affected by moderate use of alcohol when diabetes is well controlled. Because alcohol can induce hypoglycemia, it should be ingested with or shortly after a meal. One alcoholic beverage is substituted for two fat exchanges when caloric intake is calculated. Kidney function is not impacted by alcohol intake. Alcohol is not associated with increased hunger or overeating.

3. A nurse assesses a client who reports waking up feeling very tired, even after 8 hours of good sleep. Which action should the nurse take first? a. Contact the provider for a prescription for sleep medication. b. Tell the client not to drink beverages with caffeine before bed. c. Educate the client to sleep upright in a reclining chair. d. Ask the client if he or she has ever been evaluated for sleep apnea.

ANS: D Clients are usually unaware that they have sleep apnea, but it should be suspected in people who have persistent daytime sleepiness and report waking up tired. Causes of the problem should be assessed before the client is offered suggestions for treatment. DIF: Applying/Application REF: 535 KEY: Respiratory distress/failure MSC: IntegratedProcess:NursingProcess:Assessment NOT: Client Needs Category: Physiological Integrity: Physiological Adaptation

18. A client has the diagnosis of "valley fever" accompanied by myalgias and arthralgias. What treatment should the nurse educate the client on? a. Intravenous amphotericin B b. Long-term anti-inflammatories c. No specific treatment d. Oral fluconazole (Diflucan)

ANS: D "Valley fever," or coccidioidomycosis, is a fungal infection. Many people do not need treatment and the disease resolves on its own. However, the presence of joint and muscle pain indicates a moderate infection that needs treatment with antifungal medications. IV amphotericin is reserved for pregnant women and those with severe infection. Anti-inflammatory medications may be used to treat muscle aches and pain but are not used long term.

13. A nurse cares for a client who had a chest tube placed 6 hours ago and refuses to take deep breaths because of the pain. Which action should the nurse take? a. Ambulate the client in the hallway to promote deep breathing. b. Auscultate the client's anterior and posterior lung fields. c. Encourage the client to take shallow breaths to help with the pain. d. Administer pain medication and encourage the client to take deep breaths.

ANS: D A chest tube is placed in the pleural space and may be uncomfortable for a client. The nurse should provide pain medication to minimize discomfort and encourage the client to take deep breaths. The other responses do not address the client's discomfort and need to take deep breaths to prevent complications. DIF: Applying/Application REF: 580 KEY: Pain| pharmacologic pain management| drain| surgical care MSC: Integrated Process: Nursing Process: Implementation NOT: Client Needs Category: Physiological Integrity: Reduction of Risk Potential

1. A nurse is assessing a client who has suffered a nasal fracture. Which assessment should the nurse perform first? a. Facial pain b. Vital signs c. Bone displacement d. Airway patency

ANS: D A patent airway is the priority. The nurse first should make sure that the airway is patent and then should determine whether the client is in pain and whether bone displacement or blood loss has occurred. DIF: Applying/Application REF: 531 KEY: Trauma| medical emergencies MSC: Integrated Process: Nursing Process: Implementation NOT: Client Needs Category: Safe and Effective Care Environment: Management of Care

14. A client has a long history of hypertension. Which category of medications would the nurse expect to be ordered to avoid chronic kidney disease (CKD)? a. Antibiotic b. Histamine blocker c. Bronchodilator d. Angiotensin-converting enzyme (ACE) inhibitor

ANS: D ACE inhibitors stop the conversion of angiotensin I to the vasoconstrictor angiotensin II. This category of medication also blocks bradykinin and prostaglandin, increases renin, and decreases aldosterone, which promotes vasodilation and perfusion to the kidney. Antibiotics fight infection, histamine blockers decrease inflammation, and bronchodilators increase the size of the bronchi; none of these medications helps slow the progression of CKD in clients with hypertension.

23. A nurse is caring for a client who is scheduled for a dose of cefazolin and vitamins at this time. Hemodialysis for this client is also scheduled in 60 minutes. Which action by the nurse is best? a. Administer cefazolin since the level of the antibiotic must be maintained. b. Hold the vitamins but administer the cefazolin. c. Hold the cefazolin but administer the vitamins. d. Hold all medications since both cefazolin and vitamins are dialyzable.

ANS: D Both the cefazolin and the vitamins should be held until after the hemodialysis is completed because they would otherwise be removed by the dialysis process.

2. A nurse cares for a client with autosomal dominant polycystic kidney disease (ADPKD). The client asks, "Will my children develop this disease?" How should the nurse respond? a. "No genetic link is known, so your children are not at increased risk." b. "Your sons will develop this disease because it has a sex-linked gene." c. "Only if both you and your spouse are carriers of this disease." d. "Each of your children has a 50% risk of having ADPKD."

ANS: D Children whose parent has the autosomal dominant form of PKD have a 50% chance of inheriting the gene that causes the disease. ADPKD is transmitted as an autosomal dominant trait and therefore is not gender specific. Both parents do not need to have this disorder.

10. A nurse teaches a client who has a history of heart failure. Which statement should the nurse include in this client's discharge teaching? a. "Avoid drinking more than 3 quarts of liquids each day." b. "Eat six small meals daily instead of three larger meals." c. "When you feel short of breath, take an additional diuretic." d. "Weigh yourself daily while wearing the same amount of clothing."

ANS: D Clients with heart failure are instructed to weigh themselves daily to detect worsening heart failure early, and thus avoid complications. Other signs of worsening heart failure include increasing dyspnea, exercise intolerance, cold symptoms, and nocturia. Fluid overload increases symptoms of heart failure. The client should be taught to eat a heart-healthy diet, balance intake and output to prevent dehydration and overload, and take medications as prescribed. The most important discharge teaching is daily weights as this provides the best data related to fluid retention. DIF: Applying/Application REF: 687 KEY: Heart failure| patient education MSC: Integrated Process: Teaching/Learning NOT: Client Needs Category: Health Promotion and Maintenance

11. A nurse teaches a client who is recovering from a nephrectomy secondary to kidney trauma. Which statement should the nurse include in this client's teaching? a. "Since you only have one kidney, a salt and fluid restriction is required." b. "Your therapy will include hemodialysis while you recover." c. "Medication will be prescribed to control your high blood pressure." d. "You need to avoid participating in contact sports like football."

ANS: D Clients with one kidney need to avoid contact sports because the kidneys are easily injured. The client will not be required to restrict salt and fluids, end up on dialysis, or have new hypertension because of the nephrectomy.

After teaching a young adult client who is newly diagnosed with type 1 diabetes mellitus, the nurse assesses the client's understanding. Which statement made by the client indicates a correct understanding of the need for eye examinations? a. "At my age, I should continue seeing the ophthalmologist as I usually do." b. "I will see the eye doctor when I have a vision problem and yearly after age 40." c. "My vision will change quickly. I should see the ophthalmologist twice a year." d. "Diabetes can cause blindness, so I should see the ophthalmologist yearly."

ANS: D Diabetic retinopathy is a leading cause of blindness in North America. All clients with diabetes, regardless of age, should be examined by an ophthalmologist (rather than an optometrist or optician) at diagnosis and at least yearly thereafter.

16. A client is being discharged on long-term therapy for tuberculosis (TB). What referral by the nurse is most appropriate? a. Community social worker for Meals on Wheels b. Occupational therapy for job retraining c. Physical therapy for homebound therapy services d. Visiting Nurses for directly observed therapy

ANS: D Directly observed therapy is often utilized for managing clients with TB in the community. Meals on Wheels, job retraining, and home therapy may or may not be appropriate.

19. A nurse teaches a client with functional urinary incontinence. Which statement should the nurse include in this client's teaching? a. "You must clean around your catheter daily with soap and water." b. "Wash the vaginal weights with a 10% bleach solution after each use." c. "Operations to repair your bladder are available, and you can consider these." d. "Buy slacks with elastic waistbands that are easy to pull down."

ANS: D Functional urinary incontinence occurs as the result of problems not related to the client's bladder, such as trouble ambulating or difficulty accessing the toilet. One goal is that the client will be able to manage his or her clothing independently. Elastic waistband slacks that are easy to pull down can help the client get on the toilet in time to void. The other instructions do not relate to functional urinary incontinence.

9. A nurse teaches a client who is prescribed digoxin (Lanoxin) therapy. Which statement should the nurse include in this client's teaching? a. "Avoid taking aspirin or aspirin-containing products." b. "Increase your intake of foods that are high in potassium." c. "Hold this medication if your pulse rate is below 80 beats/min." d. "Do not take this medication within 1 hour of taking an antacid."

ANS: D Gastrointestinal absorption of digoxin is erratic. Many medications, especially antacids, interfere with its absorption. Clients are taught to hold their digoxin for bradycardia; a heart rate of 80 beats/min is too high for this cutoff. Potassium and aspirin have no impact on digoxin absorption, nor do these statements decrease complications of digoxin therapy. DIF: Applying/Application REF: 686 KEY: Heart failure| digoxin| medication| patient education MSC: Integrated Process: Nursing Process: Implementation NOT: Client Needs Category: Physiological Integrity: Pharmacological and Parenteral Therapies

1. A nurse reviews the urinalysis of a client and notes the presence of glucose. Which action should the nurse take? a. Document findings and continue to monitor the client. b. Contact the provider and recommend a 24-hour urine test. c. Review the client's recent dietary selections. d. Perform a capillary artery glucose assessment.

ANS: D Glucose normally is not found in the urine. The normal renal threshold for glucose is about 220 mg/dL, which means that a person whose blood glucose is less than 220 mg/dL will not have glucose in the urine. A positive finding for glucose on urinalysis indicates high blood sugar. The most appropriate action would be to perform a capillary artery glucose assessment. The client needs further evaluation for this abnormal result; therefore, documenting and continuing to monitor is not appropriate. Requesting a 24-hour urine test or reviewing the client's dietary selections will not assist the nurse to make a clinical decision related to this abnormality.

22. A client is assessed by the nurse after a hemodialysis session. The nurse notes bleeding from the client's nose and around the intravenous catheter. What action by the nurse is the priority? a. Hold pressure over the client's nose for 10 minutes. b. Take the client's pulse, blood pressure, and temperature. c. Assess for a bruit or thrill over the arteriovenous fistula. d. Prepare protamine sulfate for administration.

ANS: D Heparin is used with hemodialysis treatments. The bleeding alerts the nurse that too much anticoagulant is in the client's system and protamine sulfate should be administered. Pressure, taking vital signs, and assessing for a bruit or thrill are not as important as medication administration.

8. A nurse plans care for a client with overflow incontinence. Which intervention should the nurse include in this client's plan of care to assist with elimination? a. Stroke the medial aspect of the thigh. b. Use intermittent catheterization. c. Provide digital anal stimulation. d. Use the Valsalva maneuver.

ANS: D In clients with overflow incontinence, the voiding reflex arc is not intact. Mechanical pressure, such as that achieved through the Valsalva maneuver (holding the breath and bearing down as if to defecate), can initiate voiding. Stroking the medial aspect of the thigh or providing digital anal stimulation requires the reflex arc to be intact to initiate elimination. Due to the high risk for infection, intermittent catheterization should only be implemented when other interventions are not successful.

9. A client has been admitted for suspected inhalation anthrax infection. What question by the nurse is most important? a. "Are any family members also ill?" b. "Have you traveled recently?" c. "How long have you been ill?" d. "What is your occupation?"

ANS: D Inhalation anthrax is rare and is an occupational hazard among people who work with animal wool, bone meal, hides, and skin, such as taxidermists and veterinarians. Inhalation anthrax seen in someone without an occupational risk is considered a bioterrorism event and must be reported to authorities immediately. The other questions are appropriate for anyone with an infection.

9. A client has been admitted for suspected inhalation anthrax infection. What question by the nurse is most important? a. "Are any family members also ill?" b. "Have you traveled recently?" c. "How long have you been ill?" d. "What is your occupation?"

ANS: D Inhalation anthrax is rare and is an occupational hazard among people who work with animal wool, bone meal, hides, and skin, such as taxidermists and veterinarians. Inhalation anthrax seen in someone without an occupational risk is considered a bioterrorism event and must be reported to authorities immediately. The other questions are appropriate for anyone with an infection.

14. A nurse cares for a client who has a chest tube. When would this client be at highest risk for developing a pneumothorax? a. When the insertion site becomes red and warm to the touch b. When the tube drainage decreases and becomes sanguineous c. When the client experiences pain at the insertion site d. When the tube becomes disconnected from the drainage system

ANS: D Intrathoracic pressures are less than atmospheric pressures; therefore, if the chest tube becomes disconnected from the drainage system, air can be sucked into the pleural space and cause a pneumothorax. A red, warm, and painful insertion site does not increase the client's risk for a pneumothorax. Tube drainage should decrease and become serous as the client heals. Sanguineous drainage is a sign of bleeding but does not increase the client's risk for a pneumothorax. DIF: Applying/Application REF: 578 KEY: Drain| respiratory distress/failure MSC: Integrated Process: Nursing Process: Implementation NOT: Client Needs Category: Physiological Integrity: Reduction of Risk Potential

17. The nurse is caring for a client who is prescribed a long-acting beta2 agonist. The client states, "The medication is too expensive to use every day. I only use my inhaler when I have an attack." How should the nurse respond? a. "You are using the inhaler incorrectly. This medication should be taken daily." b. "If you decrease environmental stimuli, it will be okay for you to use the inhaler only for asthma attacks." c. "Tell me more about your fears related to feelings of breathlessness." d. "It is important to use this type of inhaler every day. Let's identify potential community services to help you."

ANS: D Long-acting beta2 agonists should be used every day to prevent asthma attacks. This medication should not be taken when an attack starts. Asthma medications can be expensive. Telling the client that he or she is using the inhaler incorrectly does not address the client's financial situation, which is the main issue here. Clients with limited incomes should be provided with community resources. Asking the client about fears related to breathlessness does not address the client's immediate concerns. DIF: Applying/Application REF: 554 KEY: Case management| medication MSC: Integrated Process: Communication and Documentation NOT: Client Needs Category: Psychosocial Integrity

1. A nurse assesses a client's electrocardiograph tracing and observes that not all QRS complexes are preceded by a P wave. How should the nurse interpret this observation? a. The client has hyperkalemia causing irregular QRS complexes. b. Ventricular tachycardia is overriding the normal atrial rhythm. c. The client's chest leads are not making sufficient contact with the skin. d. Ventricular and atrial depolarizations are initiated from different sites.

ANS: D Normal rhythm shows one P wave preceding each QRS complex, indicating that all depolarization is initiated at the sinoatrial node. QRS complexes without a P wave indicate a different source of initiation of depolarization. This finding on an electrocardiograph tracing is not an indication of hyperkalemia, ventricular tachycardia, or disconnection of leads. DIF: Understanding/Comprehension REF: 649 KEY: Cardiac electrical conduction MSC: Integrated Process: Nursing Process: Analysis NOT: Client Needs Category: Physiological Integrity: Physiological Adaptation

5. A client undergoing hemodynamic monitoring after a myocardial infarction has a right atrial pressure of 0.5 mm Hg. What action by the nurse is most appropriate? a. Level the transducer at the phlebostatic axis. b. Lay the client in the supine position. c. Prepare to administer diuretics. d. Prepare to administer a fluid bolus.

ANS: D Normal right atrial pressures are from 1 to 8 mm Hg. Lower pressures usually indicate hypovolemia, so the nurse should prepare to administer a fluid bolus. The transducer should remain leveled at the phlebostatic axis. Positioning may or may not influence readings. Diuretics would be contraindicated. DIF: Applying/Application REF: 770 KEY: Coronary artery disease| hemodynamic monitoring| fluid and electrolyte imbalance MSC: Integrated Process: Nursing Process: Implementation NOT: Client Needs Category: Physiological Integrity: Physiological Adaptation

7. A nurse cares for a client with a urine specific gravity of 1.040. Which action should the nurse take? a. Obtain a urine culture and sensitivity. b. Place the client on restricted fluids. c. Assess the client's creatinine level. d. Increase the client's fluid intake.

ANS: D Normal specific gravity for urine is 1.005 to 1.030. A high specific gravity can occur with dehydration, decreased kidney blood flow (often because of dehydration), and the presence of antidiuretic hormone. Increasing the client's fluid intake would be a beneficial intervention. Assessing the creatinine or obtaining a urine culture would not provide data necessary for the nurse to make a clinical decision.

3. A nurse reviews the urinalysis results of a client and notes a urine osmolality of 1200 mOsm/L. Which action should the nurse take? a. Contact the provider and recommend a low-sodium diet. b. Prepare to administer an intravenous diuretic. c. Obtain a suction device and implement seizure precautions. d. Encourage the client to drink more fluids.

ANS: D Normal urine osmolality ranges from 300 to 900 mOsm/L. This client's urine is more concentrated, indicating dehydration. The nurse should encourage the client to drink more water. Dehydration can be associated with elevated serum sodium levels. Although a low-sodium diet may be appropriate for this client, this diet change will not have a significant impact on urine osmolality. A diuretic would increase urine output and decrease urine osmolality further. Low serum sodium levels, not elevated serum levels, place the client at risk for seizure activity. These options would further contribute to the client's dehydration or elevate the osmolality.

28. A nurse evaluates the following arterial blood gas and vital sign results for a client with chronic obstructive pulmonary disease (COPD): Which action should the nurse take first? a. Administer a short-acting beta2 agonist inhaler. b. Document the findings as normal for a client with COPD. c. Teach the client diaphragmatic breathing techniques. d. Initiate oxygenation therapy to increase saturation to 92%.

ANS: D Oxygen should be administered to a client who is hypoxic even if the client has COPD and is a carbon dioxide retainer. The other interventions do not address the client's hypoxia, which is the priority. DIF: Applying/Application REF: 563 KEY: Oxygen therapy| respiratory distress/failure MSC: Integrated Process: Nursing Process: Implementation NOT: Client Needs Category: Safe and Effective Care Environment: Management of Care MULTIPLE RESPONSE

23. A nurse is caring for a client with acute pericarditis who reports substernal precordial pain that radiates to the left side of the neck. Which nonpharmacologic comfort measure should the nurse implement? a. Apply an ice pack to the client's chest. b. Provide a neck rub, especially on the left side. c. Allow the client to lie in bed with the lights down. d. Sit the client up with a pillow to lean forward on.

ANS: D Pain from acute pericarditis may worsen when the client lays supine. The nurse should position the client in a comfortable position, which usually is upright and leaning slightly forward. Pain is decreased by using gravity to take pressure off the heart muscle. An ice pack and neck rub will not relieve this pain. DIF: Applying/Application REF: 699 KEY: Nonpharmacologic pain management MSC: Integrated Process: Nursing Process: Implementation NOT: Client Needs Category: Physiological Integrity: Basic Care and Comfort

8. A client is in the preoperative holding area prior to an emergency coronary artery bypass graft (CABG). The client is yelling at family members and tells the doctor to "just get this over with" when asked to sign the consent form. What action by the nurse is best? a. Ask the family members to wait in the waiting area. b. Inform the client that this behavior is unacceptable. c. Stay out of the room to decrease the client's stress levels. d. Tell the client that anxiety is common and that you can help.

ANS: D Preoperative fear and anxiety are common prior to cardiac surgery, especially in emergent situations. The client is exhibiting anxiety, and the nurse should reassure the client that fear is common and offer to help. The other actions will not reduce the client's anxiety. DIF: Applying/Application REF: 776 KEY: Coronary artery disease| preoperative nursing| psychosocial response| anxiety| coping| therapeutic communication MSC: Integrated Process: Caring NOT: Client Needs Category: Psychosocial Integrity

22. A student nurse asks for an explanation of "refractory hypoxemia." What answer by the nurse instructor is best? a. "It is chronic hypoxemia that accompanies restrictive airway disease." b. "It is hypoxemia from lung damage due to mechanical ventilation." c. "It is hypoxemia that continues even after the client is weaned from oxygen." d. "It is hypoxemia that persists even with 100% oxygen administration."

ANS: D Refractory hypoxemia is hypoxemia that persists even with the administration of 100% oxygen. It is a cardinal sign of acute respiratory distress syndrome. It does not accompany restrictive airway disease and is not caused by the use of mechanical ventilation or by being weaned from oxygen.

18. A nurse assesses a client's electrocardiogram (ECG) and observes the reading shown below: How should the nurse document this client's ECG strip? a. Ventricular tachycardia b. V entricular fibrillation c. Sinus rhythm with premature atrial contractions (PACs) d. Sinus rhythm with premature ventricular contractions (PVCs)

ANS: D Sinus rhythm with PVCs has an underlying regular sinus rhythm with ventricular depolarization that sometimes precede atrial depolarization. Ventricular tachycardia and ventricular fibrillation rhythms would not have sinus beats present. Premature atrial contractions are atrial contractions initiated from another region of the atria before the sinus node initiates atrial depolarization. DIF: Applying/Application REF: 669 KEY: Cardiac electrical conduction| documentation MSC: Integrated Process: Communication and Documentation NOT: Client Needs Category: Physiological Integrity: Physiological Adaptation

4. A client is in the family practice clinic reporting a severe "cold" that started 4 days ago. On examination, the nurse notes the client also has a severe headache and muscle aches. What action by the nurse is best? a. Educate the client on oseltamivir (Tamiflu). b. Facilitate admission to the hospital. c. Instruct the client to have a flu vaccine. d. Teach the client to sneeze in the upper sleeve.

ANS: D Sneezing and coughing into one's sleeve helps prevent the spread of upper respiratory infections. The client does have manifestations of the flu (influenza), but it is too late to start antiviral medications; to be effective, they must be started within 24 to 48 hours of symptom onset. The client does not need hospital admission. The client should be instructed to have a flu vaccination, but now that he or she has the flu, vaccination will have to wait until next year.

11. A nurse cares for a client who has kidney stones from secondary hyperoxaluria. Which medication should the nurse anticipate administering? a. Phenazopyridine (Pyridium) b. Propantheline (Pro-Banthine) c. Tolterodine (Detrol LA) d. Allopurinol (Zyloprim)

ANS: D Stones caused by secondary hyperoxaluria respond to allopurinol (Zyloprim). Phenazopyridine is given to clients with urinary tract infections. Propantheline is an anticholinergic. Tolterodine is an anticholinergic with smooth muscle relaxant properties.

14. A client is on mechanical ventilation and the client's spouse wonders why ranitidine (Zantac) is needed since the client "only has lung problems." What response by the nurse is best? a. "It will increase the motility of the gastrointestinal tract." b. "It will keep the gastrointestinal tract functioning normally." c. "It will prepare the gastrointestinal tract for enteral feedings." d. "It will prevent ulcers from the stress of mechanical ventilation."

ANS: D Stress ulcers occur in many clients who are receiving mechanical ventilation, and often prophylactic medications are used to prevent them. Frequently used medications include antacids, histamine blockers, and proton pump inhibitors. Zantac is a histamine blocking agent.

1. A nurse assesses several clients who have a history of asthma. Which client should the nurse assess first? a. A 66-year-old client with a barrel chest and clubbed fingernails b. A 48-year-old client with an oxygen saturation level of 92% at rest c. A 35-year-old client who has a longer expiratory phase than inspiratory phase d. A 27-year-old client with a heart rate of 120 beats/min

ANS: D Tachycardia can indicate hypoxemia as the body tries to circulate the oxygen that is available. A barrel chest is not an emergency finding. Likewise, a pulse oximetry level of 92% is not considered an acute finding. The expiratory phase is expected to be longer than the inspiratory phase in someone with airflow limitation. DIF: Applying/Application REF: 552 KEY: Respiratory distress/failure| assessment/diagnostic examination MSC: IntegratedProcess:NursingProcess:Assessment NOT: Client Needs Category: Safe and Effective Care Environment: Management of Care

7. A client has just had a central line catheter placed that is specific for hemodialysis. What is the most appropriate action by the nurse? a. Use the catheter for the next laboratory blood draw. b. Monitor the central venous pressure through this line. c. Access the line for the next intravenous medication. d. Place a heparin or heparin/saline dwell after hemodialysis.

ANS: D The central line should have a heparin or heparin/saline dwell after hemodialysis treatment. The central line catheter used for dialysis should not be used for blood sampling, monitoring central venous pressures, or giving drugs or fluids.

19. A nurse cares for a client with end-stage heart failure who is awaiting a transplant. The client appears depressed and states, "I know a transplant is my last chance, but I don't want to become a vegetable." How should the nurse respond? a. "Would you like to speak with a priest or chaplain?" b. "I will arrange for a psychiatrist to speak with you." c. "Do you want to come off the transplant list?" d. "Would you like information about advance directives?"

ANS: D The client is verbalizing a real concern or fear about negative outcomes of the surgery. This anxiety itself can have a negative effect on the outcome of the surgery because of sympathetic stimulation. The best action is to allow the client to verbalize the concern and work toward a positive outcome without making the client feel as though he or she is crazy. The client needs to feel that he or she has some control over the future. The nurse personally provides care to address the client's concerns instead of pushing the client's issues off on a chaplain or psychiatrist. The nurse should not jump to conclusions and suggest taking the client off the transplant list, which is the best treatment option. DIF: Applying/Application REF: 691 KEY: Transplant| psychosocial response| anxiety MSC: Integrated Process: Nursing Process: Implementation NOT: Client Needs Category: Psychosocial Integrity

A nurse prepares to palpate a client's thyroid gland. Which action should the nurse take when performing this assessment? a. Stand in front of the client instead of behind the client. b. Ask the client to swallow after palpating the thyroid. c. Palpate the right lobe with the nurse's left hand. d. Place the client in a sitting position with the chin tucked down.

ANS: D The client should be in a sitting position with the chin tucked down as the examiner stands behind the client. The nurse feels for the thyroid isthmus while the client swallows and turns the head to the right, and the nurse palpates the right lobe with the right hand. The technique is repeated in the opposite fashion for the left lobe.

17. A nurse cares for a client who is scheduled for the surgical creation of an ileal conduit. The client states, "I am anxious about having an ileal conduit. What is it like to have this drainage tube?" How should the nurse respond? a. "I will ask the provider to prescribe you an antianxiety medication." b. "Would you like to discuss the procedure with your doctor once more?" c. "I think it would be nice to not have to worry about finding a bathroom." d. "Would you like to speak with someone who has an ileal conduit?"

ANS: D The goal for the client who is scheduled to undergo a procedure such as an ileal conduit is to have a positive self-image and a positive attitude about his or her body. Discussing the procedure candidly with someone who has undergone the same procedure will foster such feelings, especially when the current client has an opportunity to ask questions and voice concerns to someone with first-hand knowledge. Medications for anxiety will not promote a positive self-image and a positive attitude, nor will discussing the procedure once more with the physician or hearing the nurse's opinion.

14. A nurse assesses clients on the medical-surgical unit. Which client is at greatest risk for bladder cancer? a. A 25-year-old female with a history of sexually transmitted diseases b. A 42-year-old male who has worked in a lumber yard for 10 years c. A 55-year-old female who has had numerous episodes of bacterial cystitis d. An 86-year-old male with a 50-pack-year cigarette smoking history

ANS: D The greatest risk factor for bladder cancer is a long history of tobacco use. The other factors would not necessarily contribute to the development of this specific type of cancer.

10. A nurse cares for a client who is recovering after a nephrostomy tube was placed 6 hours ago. The nurse notes drainage in the tube has decreased from 40 mL/hr to 12 mL over the last hour. Which action should the nurse take? a. Document the finding in the client's record. b. Evaluate the tube as working in the hand-off report. c. Clamp the tube in preparation for removing it. d. Assess the client's abdomen and vital signs.

ANS: D The nephrostomy tube should continue to have a consistent amount of drainage. If the drainage slows or stops, it may be obstructed. The nurse must notify the provider, but first should carefully assess the client's abdomen for pain and distention and check vital signs so that this information can be reported as well. The other interventions are not appropriate.

26. A client with chronic kidney disease states, "I feel chained to the hemodialysis machine." What is the nurse's best response to the client's statement? a. "That feeling will gradually go away as you get used to the treatment." b. "You probably need to see a psychiatrist to see if you are depressed." c. "Do you need help from social services to discuss financial aid?" d. "Tell me more about your feelings regarding hemodialysis treatment."

ANS: D The nurse needs to explore the client's feelings in order to help the client cope and enter a phase of acceptance or resignation. It is common for clients to be discouraged because of the dependency of the treatment, especially during the first year. Referrals to a mental health provider or social services are possibilities, but only after exploring the client's feelings first. Telling the client his or her feelings will go away is dismissive of the client's concerns.

5. A client with acute kidney injury has a blood pressure of 76/55 mm Hg. The health care provider ordered 1000 mL of normal saline to be infused over 1 hour to maintain perfusion. The client is starting to develop shortness of breath. What is the nurse's priority action? a. Calculate the mean arterial pressure (MAP). b. Ask for insertion of a pulmonary artery catheter. c. Take the client's pulse. d. Slow down the normal saline infusion.

ANS: D The nurse should assess that the client could be developing fluid overload and respiratory distress and slow down the normal saline infusion. The calculation of the MAP also reflects perfusion. The insertion of a pulmonary artery catheter would evaluate the client's hemodynamic status, but this should not be the initial action by the nurse. Vital signs are also important after adjusting the intravenous infusion.

7. A nurse assesses a client who is recovering from a radical nephrectomy for renal cell carcinoma. The nurse notes that the client's blood pressure has decreased from 134/90 to 100/56 mm Hg and urine output is 20 mL for this past hour. Which action should the nurse take? a. Position the client to lay on the surgical incision. b. Measure the specific gravity of the client's urine. c. Administer intravenous pain medications. d. Assess the rate and quality of the client's pulse.

ANS: D The nurse should first fully assess the client for signs of volume depletion and shock, and then notify the provider. The radical nature of the surgery and the proximity of the surgery to the adrenal gland put the client at risk for hemorrhage and adrenal insufficiency. Hypotension is a clinical manifestation associated with both hemorrhage and adrenal insufficiency. Hypotension is particularly dangerous for the remaining kidney, which must receive adequate perfusion to function effectively. Re-positioning the client, measuring specific gravity, and administering pain medication would not provide data necessary to make an appropriate clinical decision, nor are they appropriate interventions at this time.

17. A nurse cares for a client who is having trouble voiding. The client states, "I cannot urinate in public places." How should the nurse respond? a. "I will turn on the faucet in the bathroom to help stimulate your urination." b. "I can recommend a prescription for a diuretic to improve your urine output." c. "I'll move you to a room with a private bathroom to increase your comfort." d. "I will close the curtain to provide you with as much privacy as possible."

ANS: D The nurse should provide privacy to clients who may be uncomfortable or have issues related to elimination or the urogenital area. Turning on the faucet and administering a diuretic will not address the client's concern. Although moving the client to a private room with a private bathroom would be nice, this is not realistic. The nurse needs to provide as much privacy as possible within the client's current room.

25. A nurse is assessing a client who had a myocardial infarction. Upon auscultating heart sounds, the nurse hears the following sound. What action by the nurse is most appropriate? (Click the media button to hear the audio clip.) a. Assess for further chest pain. b. Call the Rapid Response Team. c. Have the client sit upright. d. Listen to the client's lung sounds.

ANS: D The sound the nurse hears is an S3 heart sound, an abnormal sound that may indicate heart failure. The nurse should next assess the client's lung sounds. Assessing for chest pain is not directly related. There is no indication that the Rapid Response Team is needed. Having the client sit up will not change the heart sound. DIF: Applying/Application REF: 762 KEY: Coronary artery disease| respiratory assessment| respiratory system| nursing assessment MSC: IntegratedProcess:NursingProcess:Assessment NOT: Client Needs Category: Physiological Integrity: Reduction of Risk Potential

13. A nurse cares for a client who has packing inserted for posterior nasal bleeding. Which action should the nurse take first? a. Assess the client's pain level. b. Keep the client's head elevated. c. Teach the client about the causes of nasal bleeding. d. Make sure the string is taped to the client's cheek.

ANS: D The string should be attached to the client's cheek to hold the packing in place. The nurse needs to make sure that this does not move because it can occlude the client's airway. The other options are good interventions, but ensuring that the airway is patent is the priority objective. DIF: Applying/Application REF: 533 KEY: Surgical care MSC: Integrated Process: Nursing Process: Implementation NOT: Client Needs Category: Safe and Effective Care Environment: Management of Care

11. A nurse is caring for a client on mechanical ventilation. When double-checking the ventilator settings with the respiratory therapist, what should the nurse ensure as a priority? a. The client is able to initiate spontaneous breaths. b. The inspired oxygen has adequate humidification. c. The upper peak airway pressure limit alarm is off. d. The upper peak airway pressure limit alarm is on.

ANS: D The upper peak airway pressure limit alarm will sound when the airway pressure reaches a preset maximum. This is critical to prevent damage to the lungs. Alarms should never be turned off. Initiating spontaneous breathing is important for some modes of ventilation but not others. Adequate humidification is important but does not take priority over preventing injury.

21. A client is brought to the emergency department after sustaining injuries in a severe car crash. The client's chest wall does not appear to be moving normally with respirations, oxygen saturation is 82%, and the client is cyanotic. What action by the nurse is the priority? a. Administer oxygen and reassess. b. Auscultate the client's lung sounds. c. Facilitate a portable chest x-ray. d. Prepare to assist with intubation.

ANS: D This client has manifestations of flail chest and, with the other signs, needs to be intubated and mechanically ventilated immediately. The nurse does not have time to administer oxygen and wait to reassess, or to listen to lung sounds. A chest x-ray will be taken after the client is intubated.

9. A nurse prepares to defibrillate a client who is in ventricular fibrillation. Which priority intervention should the nurse perform prior to defibrillating this client? a. Make sure the defibrillator is set to the synchronous mode. b. Administer 1 mg of intravenous epinephrine. c. Test the equipment by delivering a smaller shock at 100 joules. d. Ensure that everyone is clear of contact with the client and the bed.

ANS: D To avoid injury, the rescuer commands that all personnel clear contact with the client or the bed and ensures their compliance before delivery of the shock. A precordial thump can be delivered when no defibrillator is available. Defibrillation is done in asynchronous mode. Equipment should not be tested before a client is defibrillated because this is an emergency procedure; equipment should be checked on a routine basis. Epinephrine should be administered after defibrillation. DIF: Applying/Application REF: 668 KEY: Cardiac electrical conduction| safety MSC: Integrated Process: Nursing Process: Implementation NOT: Client Needs Category: Safe and Effective Care Environment: Safety and Infection Control

12. The nurse is preparing to change a client's sternal dressing. What action by the nurse is most important? a. Assess vital signs. b. Don a mask and gown. c. Gather needed supplies. d. Perform hand hygiene.

ANS: D To prevent a sternal wound infection, the nurse washes hands or performs hand hygiene as a priority. Vital signs do not necessarily need to be assessed beforehand. A mask and gown are not needed. The nurse should gather needed supplies, but this is not the priority. DIF: Applying/Application REF: 776 KEY: Coronary artery disease| infection control| hand hygiene MSC: Integrated Process: Nursing Process: Implementation NOT: Client Needs Category: Safe and Effective Care Environment: Safety and Infection Control

p. 818, Prioritization, Delegation, and Supervision The patient is a 52-year-old woman who has undergone an autologous stem cell transplantation for non-Hodgkin's lymphoma. She is recovering, and her white blood cell count is improving but still very low. She remains on neutropenic precautions. The LPN reports that the patient's heart rate, respiratory rate, temperature, and blood pressure are all elevated. 3. A nursing student tearfully reports to you, "I took some flowers into the patient's room to cheer him up, and he told me that he didn't think he was supposed to have flowers. I took them out of the room right away, and then I realized I had made a mistake." How should you respond to this student?

Acknowledge the student for taking responsibility for the error. Helping the student to feel comfortable in reporting errors rather than hiding mistakes is essential for patient safety. Then go with the student to explain the situation to the nursing instructor so that the student can be counseled and procedures reviewed. Suggest to the instructor that the incident be used as a "teachable moment" and carefully shared with the other students. Suggest that the students discuss some of the ways patients with neutropenia can be protected from infections during hospitalization.

p. 802, Ethical/Legal A 27-year-old African-American man in sickle cell crisis is a patient on your unit. During report, one of the nurses from the previous shift mentions that she withheld the regularly scheduled IV opioid pain medication during the night because she had taken care of this patient a year ago and believes that he is a "drug seeker." 2. How should you approach your colleague?

After you have seen to your patient's comfort, remind her that unless there is a physiologic danger, such as severe respiratory depression, she is legally required to administer the prescribed opioid regardless of her beliefs. Determine whether she assessed the patient for pain and what, for this episode, not the patient's behavior a year ago, convinced her that the pain is not real.

1. A nurse assesses a client with hyperthyroidism who is prescribed lithium carbonate. Which assessment finding should alert the nurse to a side effect of this therapy? a. Blurred and double vision b. Increased thirst and urination c. Profuse nausea and diarrhea d. Decreased attention and insomnia

B. Lithium antagonizes antidiuretic hormone and can cause symptoms of diabetes insipidus. This manifests with increased thirst and urination. Lithium has no effect on vision, gastric upset, or level of consciousness.

24. A nurse assesses a male client who is recovering from a urologic procedure. Which assessment finding indicates an obstruction of urine flow? a. Severe pain b. Overflow incontinence c. Hypotension d. Blood-tinged urine

B The most common manifestation of urethral stricture after a urologic procedure is obstruction of urine flow. This rarely causes pain and has no impact on blood pressure. The client may experience overflow incontinence with the involuntary loss of urine when the bladder is distended. Blood in the urine is not a manifestation of the obstruction of urine flow.

The nurse is caring for a group of clients who have sustained myocardial infarction (MI). The nurse observes the client with which type of MI most carefully for the development of left ventricular heart failure? Inferior wall Anterior wall Lateral wall Posterior wall

Anterior wall Due to the large size of the anterior wall, the amount of tissue infarction may be large enough to decrease the force of contraction, leading to heart failure. The client with an inferior wall MI is more likely to develop right ventricular heart failure. Clients with obstruction of the circumflex artery may experience a lateral wall MI and sinus dysrhythmias or a posterior wall MI and sinus dysrhythmias.

Ch.62 p. 1280, Health Promotion and Maintenance The client who is about to have a unilateral adrenalectomy for an adenoma that is causing hypercortisolism asks the nurse if she will have to continue the severe sodium restriction after surgery. What is the nurse's best response? A. "No, once the tumor has been removed and your cortisol levels have normalized, you will not retain excess sodium anymore." B. "No, after surgery you will have to take oral cortisol, which can easily be controlled so that your sodium levels do not rise." C. Yes, the fact that you are retaining sodium and have high blood pressure is related to your age and lifestyle, not the tumor." D. "Yes, sodium is very bad for people and everyone needs to eliminate sodium completely from their diets for the rest of their lives."

Answer: A Rationale: A tumor secreting excessive amounts of cortisol is this patient's reason for needing to severely restrict her sodium. After the tumor is removed, she will not have hypercortisolism but may have to take oral cortisol until the remaining adrenal gland begins to secrete sufficient cortisol. She will no longer experience severe sodium retention. Although people in North America tend to have high-sodium diets and many could stand to reduce their sodium intake, sodium is an essential element and cannot be eliminated from the diet.

Ch.64 p. 1317, Health Promotion and Maintenance Which statement made by a client who is learning about self-injection of insulin indicates to the nurse that clarification is needed about injection site selection and rotation? A. "The abdominal site is best because it is closest to the pancreas." B. "I can reach my thigh best, so I will use different areas of the same thigh." C. "By rotating sites within one area, my chance of having skin changes is less." D. "If I change my injection site from the thigh to an arm, the insulin absorption may be different."

Answer: A Rationale: The abdominal site has the fastest and most consistent rate of absorption because of the blood vessels in the area, not because of its proximity to the pancreas.

p. 806, Health Promotion and Maintenance Which intervention is most important for the nurse to teach the client with polycythemia vera to prevent injury as a result of the increased bleeding tendency? A. Use a soft-bristled toothbrush. B. Drink at least 3 liters of liquids per day. C. Wear gloves and socks outdoors in cool weather. D. Exercise slowly and only on the advice of your physician.

Answer: A Rationale: The other interventions focus on preventing venous stasis, clot formation, and myocardial infarction. Using a soft-bristled toothbrush minimizes trauma to the gums and prevents bleeding.

Ch.63 p. 1288, Physiological Integrity Which manifestations are most often seen in general hyperthyroidism? Select all that apply. A. Increased appetite B. Cold intolerance C. Constipation D. Increased sweating E. Insomnia F. Palpitations G. Tremors H. Weight gain

Answer: A, D, E, F, G Rationale: The person with any type of hyperthyroidism has increased metabolism, which causes an increased appetite, increased sweating, increased nervous system stimulation (tremors and insomnia), and increased cardiovascular responses (palpitations). In most people with hyperthyroidism, all other systems are also stimulated, causing increased bowel movements (not constipation) and an elevated body temperature (not cold intolerance). Even though appetite is increased, most people lose weight.

p. 826, Safe and Effective Care Environment The nurse who just came on duty observes that the client, whose blood type is AB negative, is receiving a transfusion with type O negative packed red blood cells. What is the nurse's best first action? A. Call the blood bank. B. Take and record the client's vital signs. C. Stop the transfusion and keep the IV open. D. Document the observation as the only action.

Answer: B Rationale: Clients with AB negative blood types can receive O negative blood because they do not have antibodies against this type of blood. Therefore, the transfusion does not need to be stopped nor does the blood bank need to be notified. The transfusion can proceed. Because the nurse is seeing the client for the first time since the transfusion was initiated, the client's vital signs need to be assessed rather than just documenting the observation.

Ch.62 p. 1273, Physiological Integrity Which urine properties indicate to the nurse that the client with syndrome of inappropriate (SIADH) antidiuretic hormone is responding to interventions? A. Urine output volume increased; urine specific gravity increased B. Urine output volume increased; urine specific gravity decreased C. Urine output volume decreased; urine specific gravity increased D. Urine output volume decreased; urine specific gravity decreased

Answer: B Rationale: SIADH involves excessive secretion of vasopressin (ADH) when it is not needed. Water is reabsorbed, causing an increase in blood volume and a decrease in urine volume. Blood concentration is diluted, and urine concentration, as measured by specific gravity, is highly increased. When interventions to counter act SIADH are effective, the person slows water reabsorption so that urine output volume increases at the same time that urine concentration decreases, seen as a decreased urine specific gravity.

p. 809, Physiological Integrity The blood of a client who has chronic myelogenous leukemia shows a high percentage of blast cells and promyelocytes. What is the nurse's correct interpretation of this test result? A. The client's risk for infection is decreasing. B. The disease has become more aggressive. C. The drug therapy for the disease is effective. D. The type of leukemia is now lymphocytic rather than myelogenous.

Answer: B Rationale: The leukemia is progressing and the drug therapy is no longer effective. CML has three phases: The chronic phase is often a slowly progressing (indolent) course during which the patient may have mild symptoms and respond to standard treatments. The bone marrow usually shows less than 10% blast cells at this time. The accelerated phase features spleen enlargement and progressive manifestation, such as intermittent fevers, night sweats, and unexplained weight loss. The patient usually does not respond to standard treatment, and the bone marrow may contain 10% to 30% blast cells and promyelocytes. The blast phase indicates transformation to a very aggressive acute leukemia. The bone marrow contains more than 30% blast cells. The promyelocytes and blast cells commonly spread to other tissues and organs. The leukemia becomes more similar to acute leukemia than chronic leukemia but does not change from myelogenous to lymphocytic. With so many blast cells that are immature and do not function properly, the client is now at greatly increased risk for infection.

Ch.64 p. 1328, Health Promotion and Maintenance While assessing the client who has had diabetes for 15 years, the nurse finds that he has decreased sensory perception in both feet. What is the nurse's best first action? A. Document the finding as the only action. B. Examine the feet for manifestations of injury. C. Test the sensory perception of the client's hands. D. Tell the client that he now has peripheral neuropathy.

Answer: B Rationale: When reduced peripheral sensory perception is present, the likelihood of injury is high. Any open area or other problem on the foot of a person with diabetes is at great risk for infection and must be managed carefully and quickly. Checking for sensory perception on the hands and other areas is important but can come after a thorough foot examination.

Ch.63 p. 1291, Safe and Effective Care Environment For which assessment finding in a client who has severe hyperthyroidism does the nurse notify the Rapid Response Team? A. An increase in premature ventricular heart contractions from 4 per minute to 5 per minute B. An increase in or widening of pulse pressure from 40 mm Hg to 46 mm Hg C. An increase in temperature from 99.5° F (37.5° C) to 101.3° F (38.5° C) D. An increase of 20 mL of urine output per hour

Answer: C Rationale: Although all changes listed are concerning, the one most associated with impending thyroid storm (thyroid crisis) is the increase in body temperature. This client requires immediate attention.

Ch.64 p. 1306, Physiological Integrity Which health problems are considered results of microvascular complications from long-term or poorly controlled diabetes mellitus? A. Obesity and hyperglycemia B. Systolic hypertension and heart failure C. Retinal hemorrhage and male erectile dysfunction D. Diabetic ketoacidosis and hyperglycemic-hyperosmolar state

Answer: C Rationale: Both retinal hemorrhage and male erectile dysfunction are caused by microvascular complications. Structural problems in retinal vessels include areas of poor retinal circulation, edema, hard fatty deposits in the eye, and retinal hemorrhages. Microvascular changes cause hypoxia and death of the nerves needed for male erection. Systolic hypertension and heart failure are considered macrovascular complications. Obesity and hyperglycemia are causes of microvascular complications and are not caused by them. Diabetic ketoacidosis and hyperglycemic-hyperosmolar state are problems of hyperglycemia but are not caused by microvascular changes.

p. 814, Safe and Effective Care Environment The client is 3 weeks post-transplant from an allogeneic stem cell transplantation for acute lymphocytic leukemia. There is now some peeling of the client's skin on the palms of the hands and the soles of the feet. Which additional assessment data supports the nurse's suspicion of possible graft-versus-host disease (GVHD)? A. The client's temperature is slightly below normal. B. Today's platelet count is 5,000/mm3 and the WBCs are low. C. The client has had 6 to 10 watery stools daily for 3 days. D. The client's urine output is less than 800 mL in 24 hours.

Answer: C Rationale: GVHD occurs when the immunocompetent cells of the donated marrow recognize the patient's (recipient) cells, tissues, and organs as foreign and start an immunologic attack against them. The tissues most susceptible are the skin, intestinal tract, and liver. The earliest manifestation of gastrointestinal involvement for GVHD is large-volume watery diarrhea. The temperature is unaffected. The fact that the urine output is low is related to dehydration from diarrhea, not kidney damage by GVHD.

Ch.62 p. 1268, Safe and Effective Care Environment For which client does the nurse question the prescription of androgen replacement therapy? A. 35-year-old man who has had a vasectomy B. 48-year old man who takes prednisone for severe asthma C. 62-year-old man who has a history of prostate cancer D. 70-year-old man who has hypertension and type 2 diabetes

Answer: C Rationale: Prostate cancer tends to increase its growth rate in the presence of any type of androgen. Thus, the man who has a history of prostate cancer should avoid exogenous androgen because it could enhance the growth if the previously treated cancer returns. None of the other conditions are contraindicated for androgen replacement therapy.

Ch.61 p. 1264, Health Promotion and Maintenance Which precaution or action is most important for the nurse to teach the client who is to collect a 24-hour urine specimen for endocrine testing? A. Eat a normal diet during the collection period. B. Wear gloves when you urinate to prevent contamination of the specimen. C. Urinate at the end of 24 hours and add that sample to the collection container. D. Avoid walking, running, dancing, or any vigorous exercise during the collection period.

Answer: C Rationale: When a 24-hour urine specimen is started, the specimen should reflect all the urine produced during the specified time. The very first voiding is discarded because the urine has spent some time in the bladder and will not reflect what is happening during the actual 24 hours of the collection. The time of this discard is the beginning of the 24-hour collection period. The test requires that all urine voided after the start time be collected, including the specimen collected by emptying the bladder at end of the 24 hours, which marks the end of the test.

11. A nurse cares for a client who is prescribed magnetic resonance imaging (MRI) of the heart. The clients health history includes a previous myocardial infarction and pacemaker implantation. Which action should the nurse take? a. Schedule an electrocardiogram just before the MRI. b. Notify the health care provider before scheduling the MRI. c. Call the physician and request a laboratory draw for cardiac enzymes. d. Instruct the client to increase fluid intake the day before the MRI.

B The magnetic fields of the MRI can deactivate the pacemaker. The nurse should call the health care provider and report that the client has a pacemaker so the provider can order other diagnostic tests. The client does not need an electrocardiogram, cardiac enzymes, or increased fluids.

p. 1264, Health Promotion and Maintenance Which precaution or action is most important for the nurse to teach the client who is to collect a 24-hour urine specimen for endocrine testing? A. Eat a normal diet during the collection period. B. Wear gloves when you urinate to prevent contamination of the specimen. C. Urinate at the end of 24 hours and add that sample to the collection container. D. Avoid walking, running, dancing, or any vigorous exercise during the collection period.

Answer: C Rationale: When a 24-hour urine specimen is started, the specimen should reflect all the urine produced during the specified time. The very first voiding is discarded because the urine has spent some time in the bladder and will not reflect what is happening during the actual 24 hours of the collection. The time of this discard is the beginning of the 24-hour collection period. The test requires that all urine voided after the start time be collected, including the specimen collected by emptying the bladder at end of the 24 hours, which marks the end of the test.

p. 801, Safe and Effective Care Environment Which new assessment finding in a client with sickle cell disease who currently is in crises does the nurse report immediately to the health care provider? A. Pain in the right hip with limited range of motion B. Slow capillary refill in the toes of the right foot C. Yellow appearance of the roof of the mouth D. Facial drooping on the right side

Answer: D Rationale: All current assessment findings are important. However, the pain in the hip, the slow capillary refill, and the yellow appearance of the roof of the mouth are related to the crises and are expected. The facial drooping as a new finding indicates the possibility of reduced brain perfusion and stroke. This new development requires immediate attention and intervention.

Ch.61 p. 1259, Physiological Integrity What effect on circulating levels of sodium and glucose does the nurse expect in a client who has been taking an oral cortisol preparation for 2 years because of a respiratory problem? A. Decreased sodium; decreased glucose B. Decreased sodium; increased glucose C. Increased sodium; decreased glucose D. Increased sodium; increased glucose

Answer: D Rationale: Any of the glucocorticoids have some mineralocorticoid activity and increase the reabsorption of sodium from the kidney tubules, thus increasing the serum sodium level. Cortisol also increases liver production of glucose (gluconeogenesis) and inhibits peripheral glucose uptake by the cells. Both of these actions increase blood glucose levels.

p. 1259, Physiological Integrity What effect on circulating levels of sodium and glucose does the nurse expect in a client who has been taking an oral cortisol preparation for 2 years because of a respiratory problem? A. Decreased sodium; decreased glucose B. Decreased sodium; increased glucose C. Increased sodium; decreased glucose D. Increased sodium; increased glucose

Answer: D Rationale: Any of the glucocorticoids have some mineralocorticoid activity and increase the reabsorption of sodium from the kidney tubules, thus increasing the serum sodium level. Cortisol also increases liver production of glucose (gluconeogenesis) and inhibits peripheral glucose uptake by the cells. Both of these actions increase blood glucose levels.

35. A patient reports the loss of small amounts of urine during coughing, sneezing, jogging, or lifting. Which type of incontinence do these symptoms describe? a. Urge b. Overflow c. Functional d. Stress

d

Ch.64 p. 1302, Physiological Integrity How is hypoglycemia prevented in the healthy person who does not have diabetes even after fasting for 8 hours? A. Metabolism is so slow when a person sleeps without eating for 8 hours that blood glucose does not enter cells to be used for energy. As a result, hypoglycemia does not occur. B. Fasting for 8 hours triggers conversion of proteins into glycogen (glycogenesis) so that hyperglycemia develops rather than hypoglycemia. C. Lipolysis (fat breakdown) in fat stores occurs, converting fatty acids into glucose to maintain blood glucose levels. D. The secretion of glucagon prevents hypoglycemia by promoting glucose release from liver storage sites.

Answer: D Rationale: Glucagon is a counterregulatory hormone secreted by pancreatic alpha cells when blood glucose levels are low, as they would be during an 8-hour fast. The body's metabolic rate does decrease during sleep (which is not stated in this question) but not sufficiently to prevent hypoglycemia. Glucagon works on the glycogen stored in the liver, breaking it down to glucose (glycogenolysis) molecules that are then released into the blood to maintain blood glucose levels and prevent hypoglycemia. Although proteins can be broken down and converted to glucose, they are not converted to glycogen. Fat breakdown through lipolysis can provide fatty acids for fuel, but this is not glucose, and lipolysis does not occur until all stored glycogen is used.

Ch.63 p. 1298, Safe and Effective Care Environment When taking the blood pressure of a client receiving treatment for hyperparathyroidism, the nurse observes the client's hand to undergo flexion contractions. What is the nurse's interpretation of this observation? A. Hyperphosphatemia B. Hypophosphatemia C. Hypercalcemia D. Hypocalcemia

Answer: D Rationale: Hypocalcemia destabilizes excitable membranes and can lead to muscle twitches, spasms, and tetany. This effect of hypocalcemia is enhanced in the presence of tissue hypoxia. The flexion contractions occurring during blood pressure measurement are indicative of hypocalcemia and referred to as a positive Trousseau's sign.

Ch.62 p. 1276, Patient-Centered Care; Quality Improvement; Safety The patient is a 32-year-old woman admitted to your unit after surgery for fractures of the left arm and leg resulting from a car crash. She is awake and able to verify her medical history of rheumatoid arthritis and her usual daily medications. These are 10 mg of prednisone, naproxen 800 mg twice daily, oral contraceptives, calcium 600 mg, and one multiple vitamin tablet. All of these are prescribed for her to receive during her hospitalization. She is concerned about pain management and how long the recovery will be for the fractures. She is friendly, somewhat anxious, asks many questions, and wants to do "her part" to ensure good recovery. Over the next 4 days, she has become quieter, mumbles that her head and stomach hurt, and now does not recognize the assistant who has been providing her daily care. When she receives her medications, she has difficulty picking them up. The nursing assistant remarks that taking her pulse is difficult because it is so slow and irregular. When you assess her, she is so weak that she is unable to lift her arm for a blood pressure check. Her blood pressure is 92/50, which is down from the 128/84 reading on admission. You also verify that her heart beat is slow and irregular. 5. What could be done to prevent this problem from happening again?

Any patient who routinely takes a corticosteroid should be automatically evaluated on a daily basis for manifestations of adrenal insufficiency. Ideally, the person would receive additional corticosteroid therapy in advance of changes to prevent adrenal insufficiency. At the very least, assessing for early manifestations could have identified this problem earlier and prevented a near tragedy.

An older adult client, 4 hours after coronary artery bypass graft (CABG), has a blood pressure of 80/50 mm Hg. What action does the nurse take? No action is required; low blood pressure is normal for older adults. No action is required for postsurgical CABG clients. Assess pulmonary artery wedge pressure (PAWP). Give ordered loop diuretics.

Assess pulmonary artery wedge pressure (PAWP). Decreased preload as exhibited by decreased PAWP could indicate hypovolemia secondary to hemorrhage or vasodilation; hypotension could cause the graft to collapse. Low blood pressure is not normal in older adults or postoperative clients. The cause of hypotension must be found and treated; further action is needed to determine additional interventions. Hypotension could be caused by hypovolemia; giving loop diuretics increases hypovolemia.

2. A nurse reviews the laboratory findings of a client with a urinary tract infection. The laboratory report notes a shift to the left in a clients white blood cell count. Which action should the nurse take? a. Request that the laboratory perform a differential analysis on the white blood cells. b. Notify the provider and start an intravenous line for parenteral antibiotics. c. Collaborate with the unlicensed assistive personnel (UAP) to strain the clients urine for renal calculi. d. Assess the client for a potential allergic reaction and anaphylactic shock.

B An increase in band cells creates a shift to the left. A left shift most commonly occurs with urosepsis and is seen rarely with uncomplicated urinary tract infections. The nurse will be administering antibiotics, most likely via IV, so he or she should notify the provider and prepare to give the antibiotics. The shift to the left is part of a differential white blood cell count. The nurse would not need to strain urine for stones. Allergic reactions are associated with elevated eosinophil cells, not band cells.

p. 818, Prioritization, Delegation, and Supervision The patient is a 52-year-old woman who has undergone an autologous stem cell transplantation for non-Hodgkin's lymphoma. She is recovering, and her white blood cell count is improving but still very low. She remains on neutropenic precautions. The LPN reports that the patient's heart rate, respiratory rate, temperature, and blood pressure are all elevated. 2. You must assign an unlicensed assistive personnel (UAP) to help care for this patient. Of the four UAPs available, one is newly pregnant and has worked on this unit for 3 years, one has had cold symptoms for 3 days, one has not yet cared for a patient on neutropenic precautions, and one has a fear of people with cancer. Which UAP should you avoid assigning to this patient? Provide a rationale for your choice.

Assign the UAP who is newly pregnant and very experienced to assist with this patient. The patient's possible infection is not harmful to the pregnant UAP because the infection is most likely from overgrowth of the patient's normal flora. Infection in a patient with neutropenia can lead to sepsis and death very quickly. Therefore, an experienced and healthy caregiver is critical. The UAP who fears people with cancer may let his or her fear overshadow the ability to provide appropriate care. If this person fears cancer, he or she may also fear infection. In fact, it may be appropriate to request that this UAP be transferred to a unit that does not include people with cancer in its usual population.

14. A nurse teaches a client with diabetes mellitus and a body mass index of 42 who is at high risk for coronary artery disease. Which statement related to nutrition should the nurse include in this clients teaching? a. The best way to lose weight is a high-protein, low-carbohydrate diet. b. You should balance weight loss with consuming necessary nutrients. c. A nutritionist will provide you with information about your new diet. d. If you exercise more frequently, you wont need to change your diet.

B Clients at risk for cardiovascular diseases should follow the American Heart Association guidelines to combat obesity and improve cardiac health. The nurse should encourage the client to eat vegetables, fruits, unrefined whole-grain products, and fat-free dairy products while losing weight. High-protein food items are often high in fat and calories. Although the nutritionist can assist with client education, the nurse should include nutrition education and assist the client to make healthy decisions. Exercising and eating nutrient-rich foods are both important components in reducing cardiovascular risk.

22. A nurse plans care for clients with urinary incontinence. Which client is correctly paired with the appropriate intervention? a. A 29-year-old client after a difficult vaginal delivery Habit training b. A 58-year-old postmenopausal client who is not taking estrogen therapy Electrical stimulation c. A 64-year-old female with Alzheimers-type senile dementia Bladder training d. A 77-year-old female who has difficulty ambulating Exercise therapy

B Exercise therapy and electrical stimulation are used for clients with stress incontinence related to childbirth or low levels of estrogen after menopause. Exercise therapy increases pelvic wall strength; it does not improve ambulation. Physical therapy and a bedside commode would be appropriate interventions for the client who has difficulty ambulating. Habit training is the type of bladder training that will be most effective with cognitively impaired clients. Bladder training can be used only with a client who is alert, aware, and able to resist the urge to urinate.

7. After delegating care to an unlicensed assistive personnel (UAP) for a client who is prescribed habit training to manage incontinence, a nurse evaluates the UAPs understanding. Which action indicates the UAP needs additional teaching? a. Toileting the client after breakfast b. Changing the clients incontinence brief when wet c. Encouraging the client to drink fluids d. Recording the clients incontinence episodes

B Habit training is undermined by the use of absorbent incontinence briefs or pads. The nurse should re-educate the UAP on the technique of habit training. The UAP should continue to toilet the client after meals, encourage the client to drink fluids, and record incontinent episodes.

3. A nurse cares for a postmenopausal client who has had two episodes of bacterial urethritis in the last 6 months. The client asks, I never have urinary tract infections. Why is this happening now? How should the nurse respond? a. Your immune system becomes less effective as you age. b. Low estrogen levels can make the tissue more susceptible to infection. c. You should be more careful with your personal hygiene in this area. d. It is likely that you have an untreated sexually transmitted disease.

B Low estrogen levels decrease moisture and secretions in the perineal area and cause other tissue changes, predisposing it to the development of infection. Urethritis is most common in postmenopausal women for this reason. Although immune function does decrease with aging and sexually transmitted diseases are a known cause of urethritis, the most likely reason in this client is low estrogen levels. Personal hygiene usually does not contribute to this disease process.

20. An emergency department nurse assesses a client with a history of urinary incontinence who presents with extreme dry mouth, constipation, and an inability to void. Which question should the nurse ask first? a. Are you drinking plenty of water? b. What medications are you taking? c. Have you tried laxatives or enemas? d. Has this type of thing ever happened before?

B Some types of incontinence are treated with anticholinergic medications such as propantheline (Pro-Banthine). Anticholinergic side effects include dry mouth, constipation, and urinary retention. The nurse needs to assess the clients medication list to determine whether the client is taking an anticholinergic medication. If he or she is taking anticholinergics, the nurse should further assess the clients manifestations to determine if they are related to a simple side effect or an overdose. The other questions are not as helpful to understanding the current situation.

13. A nurse cares for a client who has an 80% blockage of the right coronary artery (RCA) and is scheduled for bypass surgery. Which intervention should the nurse be prepared to implement while this client waits for surgery? a. Administration of IV furosemide (Lasix) b. Initiation of an external pacemaker c. Assistance with endotracheal intubation d. Placement of central venous access

B The RCA supplies the right atrium, the right ventricle, the inferior portion of the left ventricle, and the atrioventricular (AV) node. It also supplies the sinoatrial node in 50% of people. If the client totally occludes the RCA, the AV node would not function and the client would go into heart block, so emergency pacing should be available for the client. Furosemide, intubation, and central venous access will not address the primary complication of RCA occlusion, which is AV node malfunction.

21. A nurse teaches a client who is starting urinary bladder training. Which statement should the nurse include in this clients teaching? a. Use the toilet when you first feel the urge, rather than at specific intervals. b. Try to consciously hold your urine until the scheduled toileting time. c. Initially try to use the toilet at least every half hour for the first 24 hours. d. The toileting interval can be increased once you have been continent for a week.

B The client should try to hold the urine consciously until the next scheduled toileting time. Toileting should occur at specific intervals during the training. The toileting interval should be no less than every hour. The interval can be increased once the client becomes comfortable with the interval.

12. A nurse assesses a client who is recovering from extracorporeal shock wave lithotripsy for renal calculi. The nurse notes an ecchymotic area on the clients right lower back. Which action should the nurse take? a. Administer fresh-frozen plasma. b. Apply an ice pack to the site. c. Place the client in the prone position. d. Obtain serum coagulation test results.

B The shock waves from lithotripsy can cause bleeding into the tissues through which the waves pass. Application of ice can reduce the extent and discomfort of the bruising. Although coagulation test results and fresh-frozen plasma are used to assess and treat bleeding disorders, ecchymosis after this procedure is not unusual and does not warrant a higher level of intervention. Changing the clients position will not decrease bleeding.

20. A nurse auscultated heart tones on an older adult client. Which action should the nurse take based on heart tones heard? (Click the media button to hear the audio clip.) a. Administer a diuretic. b. Document the finding. c. Decrease the IV flow rate. d. Evaluate the clients medications.

B The sound heard is an atrial gallop S4. An atrial gallop may be heard in older clients because of a stiffened ventricle. The nurse should document the finding, but no other intervention is needed at this time.

6. A nurse obtains the health history of a client who is newly admitted to the medical unit. Which statement by the client should alert the nurse to the presence of edema? a. I wake up to go to the bathroom at night. b. My shoes fit tighter by the end of the day. c. I seem to be feeling more anxious lately. d. I drink at least eight glasses of water a day.

B Weight gain can result from fluid accumulation in the interstitial spaces. This is known as edema. The nurse should note whether the client feels that his or her shoes or rings are tight, and should observe, when present, an indentation around the leg where the socks end. The other answers do not describe edema.

After teaching a client who is recovering from an endoscopic trans-nasal hypophysectomy, the nurse assesses the clients understanding. Which statement made by the client indicates a correct understanding of the teaching? a. I will wear dark glasses to prevent sun exposure. b. Ill keep food on upper shelves so I do not have to bend over. c. I must wash the incision with peroxide and redress it daily. d. I shall cough and deep breathe every 2 hours while I am awake.

B (After this surgery, the client must take care to avoid activities that can increase intracranial pressure. The client should avoid bending from the waist and should not bear down, cough, or lie flat. With this approach, there is no incision to clean and dress. Protection from sun exposure is not necessary after this procedure.)

A nurse assesses clients for potential endocrine dysfunction. Which client is at greatest risk for a deficiency of gonadotropin and growth hormone? a. A 36-year-old female who has used oral contraceptives for 5 years b. A 42-year-old male who experienced head trauma 3 years ago c. A 55-year-old female with a severe allergy to shellfish and iodine d. A 64-year-old male with adult-onset diabetes mellitus

B (Gonadotropin and growth hormone are anterior pituitary hormones. Head trauma is a common cause of anterior pituitary hypofunction. The other factors do not increase the risk of this condition.)

A nurse cares for a client with adrenal hyperfunction. The client screams at her husband, bursts into tears, and throws her water pitcher against the wall. She then tells the nurse, I feel like I am going crazy. How should the nurse respond? a. I will ask your doctor to order a psychiatric consult for you. b. You feel this way because of your hormone levels. c. Can I bring you information about support groups? d. I will close the door to your room and restrict visitors.

B (Hypercortisolism can cause the client to show neurotic or psychotic behavior. The client needs to know that these behavior changes do not reflect a true psychiatric disorder and will resolve when therapy results in lower and steadier blood cortisol levels. The client needs to understand this effect and does not need a psychiatrist, support groups, or restricted visitors at this time.)

A nurse is caring for a client who was prescribed high-dose corticosteroid therapy for 1 month to treat a severe inflammatory condition. The clients symptoms have now resolved and the client asks, When can I stop taking these medications? How should the nurse respond? a. It is possible for the inflammation to recur if you stop the medication. b. Once you start corticosteroids, you have to be weaned off them. c. You must decrease the dose slowly so your hormones will work again. d. The drug suppresses your immune system, which must be built back up.

B (One of the most common causes of adrenal insufficiency, a life-threatening problem, is the sudden cessation of long-term, high-dose corticosteroid therapy. This therapy suppresses the hypothalamic-pituitary-adrenal axis and must be withdrawn gradually to allow for pituitary production of adrenocorticotropic hormone and adrenal production of cortisol. Decreasing hormone therapy slowly ensures self-production of hormone, not hormone effectiveness. Building the clients immune system and rebound inflammation are not concerns related to stopping high-dose corticosteroids.)

A nurse teaches a client with a cortisol deficiency who is prescribed prednisone (Deltasone). Which statement should the nurse include in this clients instructions? a. You will need to learn how to rotate the injection sites. b. If you work outside in the heat, you may need another drug. c. You need to follow a diet with strict sodium restrictions. d. Take one tablet in the morning and two tablets at night.

B (Steroid dosage adjustment may be needed if the client works outdoors and might be difficult, especially in hot weather, when the client is sweating a great deal more than normal. Clients take prednisone orally, have no need for a salt restriction, and usually start the regimen with two tablets in the morning and one at night.)

The charge nurse at an assisted-living facility receives report from an emergency department (ED) nurse about one of the resident clients. The client was sent to the ED with a fever, chills, muscle aches, and headache. The ED nurse reports the client's rapid influenza report came back from the laboratory positive for influenza A. What action by the nurse at the assisted-living facility is most appropriate? A) Prepare to administer antibiotics. B) Have the resident eat meals in his room. C) Provide oseltamivir (Tamiflu) to the staff. D) Arrange a follow-up chest x-ray in 2 weeks. (Chp. 31; p. 587)

B) Have the resident eat meals in his room. (Chp. 31; p. 587)

A nurse cares for a male client with hypopituitarism who is prescribed testosterone hormone replacement therapy. The client asks, How long will I need to take this medication? How should the nurse respond? a. When your blood levels of testosterone are normal, the therapy is no longer needed. b. When your beard thickens and your voice deepens, the dose is decreased, but treatment will continue forever. c. When your sperm count is high enough to demonstrate fertility, you will no longer need this therapy. d. With age, testosterone levels naturally decrease, so the medication can be stopped when you are 50 years old

B (Testosterone therapy is initiated with high-dose testosterone derivatives and is continued until virilization is achieved. The dose is then decreased, but therapy continues throughout life. Therapy will continue throughout life; therefore, it will not be discontinued when blood levels are normal, at the age of 50 years, or when sperm counts are high.)

A nurse cares for a client who possibly has syndrome of inappropriate antidiuretic hormone (SIADH). The clients serum sodium level is 114 mEq/L. Which action should the nurse take first? a. Consult with the dietitian about increased dietary sodium. b. Restrict the clients fluid intake to 600 mL/day. c. Handle the client gently by using turn sheets for re-positioning. d. Instruct unlicensed assistive personnel to measure intake and output.

B (With SIADH, clients often have dilutional hyponatremia. The client needs a fluid restriction, sometimes to as little as 500 to 600 mL/24 hr. Adding sodium to the clients diet will not help if he or she is retaining fluid and diluting the sodium. The client is not at increased risk for fracture, so gentle handling is not an issue. The client should be on intake and output; however, this will monitor only the clients intake, so it is not the best answer. Reducing intake will help increase the clients sodium.)

A client is being discharged home with active tuberculosis. Which information does the nurse include in the discharge teaching plan? A) "You are not contagious unless you stop taking your medication." B) "You will not be contagious to the people you have been living with." C) "You will have to take these medications for at least 1 year." D) "Your sputum may turn a rust color as your condition gets better." (Chp. 31; elsevier resources)

B) "You will not be contagious to the people you have been living with." (Chp. 31; elsevier resources)

Which of these clients should the charge nurse assign to the LPN/LVN working on the medical-surgical unit? A) Client with group A beta-hemolytic streptococcal pharyngitis who has stridor B) Client with pulmonary tuberculosis who is receiving multiple medications C) Client with sinusitis who has just arrived after having endoscopic sinus surgery D) Client with tonsillitis who has a thick-sounding voice and difficulty swallowing (Chp. 31; elsevier resources)

B) Client with pulmonary tuberculosis who is receiving multiple medications (Chp. 31; elsevier resources)

A client with pneumonia caused by aspiration after alcohol intoxication has just been admitted. The client is febrile and agitated. Which health care provider order should the nurse implement first? A) Administer levofloxacin (Levaquin) 500 mg IV. B) Draw aerobic and anaerobic blood cultures. C) Give lorazepam (Ativan) as needed for agitation. D) Refer to social worker for alcohol counseling. (Chp. 31; elsevier resources)

B) Draw aerobic and anaerobic blood cultures. (Chp. 31; elsevier resources)

Which method is the best way to prevent outbreaks of pandemic influenza? A) Avoiding public gatherings at all times B) Early recognition and quarantine C) Vaccinating everyone with pneumonia vaccine D) Widespread distribution of antiviral drugs (Chp. 31; elsevier resources)

B) Early recognition and quarantine (Chp. 31; elsevier resources)

Which symptom of pneumonia may present differently in the older adult than in the younger adult? A) Crackles on auscultation B) Fever C) Headache D) Wheezing (Chp. 31; elsevier resources)

B) Fever (Chp. 31; elsevier resources)

An older client presents to the emergency department with a 2-day history of cough, pain on inspiration, shortness of breath, and dyspnea. The client never had a pneumococcal vaccine. The client's chest x-ray shows density in both bases. The client has wheezing upon auscultation of both lungs. Would a bronchodilator be beneficial for this client? A) It would not be beneficial for this client. B) It would help decrease the bronchospasm. C) It would clear up the density in the bases of the client's lungs. D) It would decrease the client's pain on inspiration. (Chp. 31; elsevier resources)

B) It would help decrease the bronchospasm. (Chp. 31; elsevier resources)

2. An emergency room nurse assesses a female client. Which assessment findings should alert the nurse to request a prescription for an electrocardiogram? (Select all that apply.) a. Hypertension b. Fatigue despite adequate rest c. Indigestion d. Abdominal pain e. Shortness of breath

B, C, E Women may not have chest pain with myocardial infarction, but may feel discomfort or indigestion. They often present with a triad of symptomsindigestion or feeling of abdominal fullness, feeling of chronic fatigue despite adequate rest, and feeling unable to catch their breath. Frequently, women are not diagnosed and therefore are not treated adequately. Hypertension and abdominal pain are not associated with acute coronary syndrome.

5. A nurse prepares a client for a pharmacologic stress echocardiogram. Which actions should the nurse take when preparing this client for the procedure? (Select all that apply.) a. Assist the provider to place a central venous access device. b. Prepare for continuous blood pressure and pulse monitoring. c. Administer the clients prescribed beta blocker. d. Give the client nothing by mouth 3 to 6 hours before the procedure. e. Explain to the client that dobutamine will simulate exercise for this examination.

B, D, E Clients receiving a pharmacologic stress echocardiogram will need peripheral venous access and continuous blood pressure and pulse monitoring. The client must be NPO 3 to 6 hours prior to the procedure. Education about dobutamine, which will be administered during the procedure, should be performed. Beta blockers are often held prior to the procedure.

3. A nurse assesses a client who is recovering after a coronary catheterization. Which assessment findings in the first few hours after the procedure require immediate action by the nurse? (Select all that apply.) a. Blood pressure of 140/88 mm Hg b. Serum potassium of 2.9 mEq/L c. Warmth and redness at the site d. Expanding groin hematoma e. Rhythm changes on the cardiac monitor

B, D, E In the first few hours postprocedure, the nurse monitors for complications such as bleeding from the insertion site, hypotension, acute closure of the vessel, dye reaction, hypokalemia, and dysrhythmias. The clients blood pressure is slightly elevated but does not need immediate action. Warmth and redness at the site would indicate an infection, but this would not be present in the first few hours.

A nurse cares for a client who is prescribed vasopressin (DDAVP) for diabetes insipidus. Which assessment findings indicate a therapeutic response to this therapy? (Select all that apply.) a. Urine output is increased. b. Urine output is decreased. c. Specific gravity is increased. d. Specific gravity is decreased. e. Urine osmolality is increased. f. Urine osmolality is decreased.

B, D, E (Diabetes insipidus causes urine output to be greatly increased, with a low urine osmolality, as evidenced by a low specific gravity. Effective treatment results in decreased urine output that is more concentrated, as evidenced by an increased specific gravity.)

A nurse teaches a client with Cushings disease. Which dietary requirements should the nurse include in this clients teaching? (Select all that apply.) a. Low calcium b. Low carbohydrate c. Low protein d. Low calories e. Low sodium

B, D, E (The client with Cushings disease has weight gain, muscle loss, hyperglycemia, and sodium retention. Dietary modifications need to include reduction of carbohydrates and total calories to prevent or reduce the degree of hyperglycemia. Sodium retention causes water retention and hypertension. Clients are encouraged to restrict their sodium intake moderately. Clients often have bone density loss and need more calcium. Increased protein intake will help decrease muscle loss.)

an older client has a history of coronary artery disease. which modifiable risk factors will the nurse assess to guide the client's teaching plan? select all that apply. A. older age B. tobacco use C. female D. high-fat diet E. family history F. obesity

B, D, F

1. A nurse evaluates the following laboratory results for a client who has hypoparathyroidism: Calcium 7.2 mg/dl Sodium 144 mEq/L Magnesium 1.2 mEq/L Potassium 5.7 mEq/L Based on these results, which medications should the nurse anticipate administering? (Select all that apply.) a. Oral potassium chloride b. Intravenous calcium chloride c. 3% normal saline IV solution d. 50% magnesium sulfate e. Oral calcitriol (Rocaltrol)

B, D. The client has hypocalcemia (treated with calcium chloride) and hypomagnesemia (treated with magnesium sulfate). The potassium level is high, so replacement is not needed. The client's sodium level is normal, so hypertonic IV solution is not needed. No information about a vitamin D deficiency is evident, so calcitriol is not needed.

4. A nurse assesses a client with a fungal urinary tract infection (UTI). Which assessments should the nurse complete? (Select all that apply.) a. Palpate the kidneys and bladder. b. Assess the medical history and current medical problems. c. Perform a bladder scan to assess post-void residual. d. Inquire about recent travel to foreign countries. e. Obtain a current list of medications.

B, E Clients who are severely immunocompromised or who have diabetes mellitus are more prone to fungal UTIs. The nurse should assess for these factors by asking about medical history, current medical problems, and the current medication list. A physical examination and a post-void residual may be needed, but not until further information is obtained indicating that these examinations are necessary. Travel to foreign countries probably would not be important because, even if exposed, the client needs some degree of compromised immunity to develop a fungal UTI.

A client has been diagnosed with hypothyroidism. What medication is usually prescribed to treat this disorder? A. Atenolol (Tenormin) B. Levothyroxine sodium (Synthroid) C. Methimazole (Tapazole) D. Propylthiouracil

B. Levothyroxine is a synthetic form of thyroxine (T4) that is used to treat hypothyroidism. Atenolol is a beta blocker that is used to treat cardiovascular disease. Methimazole and propylthiouracil are used to treat hyperthyroidism.

3. A nurse teaches clients about the difference between urge incontinence and stress incontinence. Which statements should the nurse include in this education? (Select all that apply.) a. Urge incontinence involves a post-void residual volume less than 50 mL. b. Stress incontinence occurs due to weak pelvic floor muscles. c. Stress incontinence usually occurs in people with dementia. d. Urge incontinence can be managed by increasing fluid intake. e. Urge incontinence occurs due to abnormal bladder contractions.

B, E Clients who suffer from stress incontinence have weak pelvic floor muscles or urethral sphincter and cannot tighten their urethra sufficiently to overcome the increased detrusor pressure. Stress incontinence is common after childbirth, when the pelvic muscles are stretched and weakened from pregnancy and delivery. Urge incontinence occurs in people who cannot suppress the contraction signal from the detrusor muscle. Abnormal detrusor contractions may be a result of neurologic abnormalities including dementia, or may occur with no known abnormality. Post-void residual is associated with reflex incontinence, not with urge incontinence or stress incontinence. Management of urge incontinence includes decreasing fluid intake, especially in the evening hours.

The client is taking fludrocortisone (Florinef) for adrenal hypofunction. The nurse instructs the client to report which symptom while taking this drug? A. Anxiety B. Headache C. Nausea D. Weight loss

B. A side effect of fludrocortisone is hypertension. New onset of headache should be reported, and the client's blood pressure should be monitored. Anxiety is not a side effect of fludrocortisone and is not associated with adrenal hypofunction. Nausea is associated with adrenal hypofunction; it is not a side effect of fludrocortisone. Sodium-related fluid retention and weight gain, not loss, are possible with fludrocortisone therapy.

A client recently admitted with hyperparathyroidism has a very high urine output. Of these actions, what does the nurse do next? A. Calls the health care provider B. Monitors intake and output C. Performs an immediate cardiac assessment D. Slows the rate of IV fluids

B. Diuretic and hydration therapies are used most often for reducing serum calcium levels in clients with hyperparathyroidism. Usually, a diuretic that increases kidney excretion of calcium is used together with IV saline in large volumes to promote renal calcium excretion. The health care provider does not need to be notified in this situation, given the information available in the question. Cardiac assessment is part of the nurse's routine evaluation of the client. Slowing the rate of IV fluids is contraindicated because the client will become dehydrated due to the use of diuretics to increase kidney excretion of calcium.

A client had a parathyroidectomy 18 hours ago. Which finding requires immediate attention? A. Edema at the surgical site B. Hoarseness C. Pain on moving the head Incorrect D. Sore throat

B. Hoarseness or stridor is an indication of respiratory distress and requires immediate attention. Edema at the surgical site of any surgery is an expected finding. Pain when the client moves the head or attempts to lift the head off the bed is an expected finding after a parathyroidectomy. Any time a client has been intubated for surgery, a sore throat is a common occurrence in the postoperative period. This is especially true for clients who have had surgery involving the neck.

A client being treated for hyperthyroidism calls the home health nurse and mentions that his heart rate is slower than usual. What is the nurse's best response? A. Advise the client to go to a calming environment. B. Ask whether the client has increased cold sensitivity or weight gain. C. Instruct the client to see his health care provider immediately. D. Tell the client to check his pulse again and call back later.

B. Increased sensitivity to cold and weight gain are symptoms of hypothyroidism, indicating an overcorrection by the medication. The client must be assessed further because he may require a lower dose of medication. A calming environment will not have any effect on the client's heart rate. The client will want to notify the health care provider about the change in heart rate. If other symptoms such as chest pain, shortness of breath, or confusion accompany the slower heart rate, then the client should see the health care provider immediately. If the client was concerned enough to call because his heart rate was slower than usual, the nurse needs to stay on the phone with the client while he re-checks his pulse. This time could also be spent providing education about normal ranges for that client.

A client presents to the emergency department with acute adrenal insufficiency and the following vital signs: P 118 beats/min, R 18 breaths/min, BP 84/44 mm Hg, pulse oximetry 98%, and T 98.8° F oral. Which nursing intervention is the highest priority for this client? A. Administering furosemide (Lasix) B. Providing isotonic fluids C. Replacing potassium losses D. Restricting sodium

B. Providing isotonic fluid is the priority intervention because this client's vital signs indicate volume loss that may be caused by nausea and vomiting and may accompany acute adrenal insufficiency. Isotonic fluids will be needed to administer IV medications such as hydrocortisone. Furosemide is a loop diuretic, which this client does not need. Potassium is normally increased in acute adrenal insufficiency, but potassium may have been lost if the client has had diarrhea; laboratory work will have to be obtained. Any restrictions, including sodium, should not be started without obtaining laboratory values to establish the client's baseline.

A client has undergone a transsphenoidal hypophysectomy. Which intervention does the nurse implement to avoid increasing intracranial pressure (ICP) in the client? A. Encourages the client to cough and deep-breathe B. Instructs the client not to strain during a bowel movement C. Instructs the client to blow the nose for postnasal drip D. Places the client in the Trendelenburg position

B. Straining during a bowel movement increases ICP and must be avoided. Laxatives may be given and fluid intake encouraged to help with this. Although deep breathing is encouraged, the client must avoid coughing early after surgery because this increases pressure in the incision area and may lead to a cerebrospinal fluid (CSF) leak. If the client has postnasal drip, he or she must inform the nurse and not blow the nose; postnasal drip may indicate leakage of CSF. The head of the bed must be elevated after surgery.

Family members of a client diagnosed with hyperthyroidism are alarmed at the client's frequent mood swings. What is the nurse's response? A. "How does that make you feel?" B. "The mood swings should diminish with treatment." C. "The medications will make the mood swings disappear completely." D. "Your family member is sick. You must be patient."

B. Telling the family that the client's mood swings should diminish over time with treatment will provide information to the family, as well as reassurance. Asking how the family feels is important; however, the response should focus on the client. Any medications or treatment may not completely remove the mood swings associated with hyperthyroidism. The family is aware that the client is sick; telling them to be patient introduces guilt and does not address the family's concerns.

A client admitted with hyperthyroidism is fidgeting with the bedcovers and talking extremely fast. What does the nurse do next? A. Calls the provider B. Encourages the client to rest C. Immediately assesses cardiac status D. Tells the client to slow down

B. The client with hyperthyroidism often has wide mood swings, irritability, decreased attention span, and manic behavior. The nurse should accept the client's behavior and provide a calm, quiet, and comfortable environment. Because the client's behavior is expected, there is no need to call the provider. Monitoring the client's cardiac status is part of the nurse's routine assessment. Telling the client to slow down is unsupportive and unrealistic.

14. After teaching a client who is recovering from a complete thyroidectomy, the nurse assesses the client's understanding. Which statement made by the client indicates a need for additional instruction? a. "I may need calcium replacement after surgery." b. "After surgery, I won't need to take thyroid medication." c. "I'll need to take thyroid hormones for the rest of my life." d. "I can receive pain medication if I feel that I need it."

B. After the client undergoes a thyroidectomy, the client must be given thyroid replacement medication for life. He or she may also need calcium if the parathyroid is damaged during surgery, and can receive pain medication postoperatively.

12. A nurse cares for a client newly diagnosed with Graves' disease. The client's mother asks, "I have diabetes mellitus. Am I responsible for my daughter's disease?" How should the nurse respond? a. "The fact that you have diabetes did not cause your daughter to have Graves' disease. No connection is known between Graves' disease and diabetes." b. "An association has been noted between Graves' disease and diabetes, but the fact that you have diabetes did not cause your daughter to have Graves' disease." c. "Graves' disease is associated with autoimmune diseases such as rheumatoid arthritis, but not with a disease such as diabetes mellitus." d. "Unfortunately, Graves' disease is associated with diabetes, and your diabetes could have led to your daughter having Graves' disease."

B. An association between autoimmune diseases such as rheumatoid arthritis and diabetes mellitus has been noted. The predisposition is probably polygenic, and the mother's diabetes did not cause her daughter's Graves' disease. The other statements are inaccurate.

9. A nurse assesses a client who is recovering after a left-sided cardiac catheterization. Which assessment finding requires immediate intervention? a. Urinary output less than intake b. Bruising at the insertion site c. Slurred speech and confusion d. Discomfort in the left leg

C A left-sided cardiac catheterization specifically increases the risk for a cerebral vascular accident. A change in neurologic status needs to be acted on immediately. Discomfort and bruising are expected at the site. If intake decreases, a client can become dehydrated because of dye excretion. The second intervention would be to increase the clients fluid status. Neurologic changes would take priority.

15. A nurse assesses a client with bladder cancer who is recovering from a complete cystectomy with ileal conduit. Which assessment finding should alert the nurse to urgently contact the health care provider? a. The ileostomy is draining blood-tinged urine. b. There is serous sanguineous drainage present on the surgical dressing. c. The ileostomy stoma is pale and cyanotic in appearance. d. Oxygen saturations are 92% on room air.

C A pale or cyanotic stoma indicates impaired circulation to the stoma and must be treated to prevent necrosis. Blood-tinged urine and serous sanguineous drainage are expected after this type of surgery. Oxygen saturation of 92% on room air is at the low limit of normal.

9. A confused client with pneumonia is admitted with an indwelling catheter in place. During interdisciplinary rounds the following day, which question should the nurse ask the primary health care provider? a. Do you want daily weights on this client? b. Will the client be able to return home? c. Can we discontinue the indwelling catheter? d. Should we get another chest x-ray today?

C An indwelling catheter dramatically increases the risks of urinary tract infection and urosepsis. Nursing staff should ensure that catheters are left in place only as long as they are medically needed. The nurse should inquire about removing the catheter. All other questions might be appropriate, but because of client safety, this question takes priority.

16. A nurse cares for a client who is recovering from a myocardial infarction. The client states, I will need to stop eating so much chili to keep that indigestion pain from returning. How should the nurse respond? a. Chili is high in fat and calories; it would be a good idea to stop eating it. b. The provider has prescribed an antacid for you to take every morning. c. What do you understand about what happened to you? d. When did you start experiencing this indigestion?

C Clients who experience myocardial infarction often respond with denial, which is a defense mechanism. The nurse should ask the client what he or she thinks happened, or what the illness means to him or her. The other responses do not address the clients misconception about recent pain and the cause of that pain.

1. A nurse assesses clients on the medical-surgical unit. Which client is at greatest risk for the development of bacterial cystitis? a. A 36-year-old female who has never been pregnant b. A 42-year-old male who is prescribed cyclophosphamide c. A 58-year-old female who is not taking estrogen replacement d. A 77-year-old male with mild congestive heart failure

C Females at any age are more susceptible to cystitis than men because of the shorter urethra in women. Postmenopausal women who are not on hormone replacement therapy are at increased risk for bacterial cystitis because of changes in the cells of the urethra and vagina. The middle-aged woman who has never been pregnant would not have a risk potential as high as the older woman who is not using hormone replacement therapy.

7. A nurse assesses an older adult client who is experiencing a myocardial infarction. Which clinical manifestation should the nurse expect? a. Excruciating pain on inspiration b. Left lateral chest wall pain c. Disorientation and confusion d. Numbness and tingling of the arm

C In older adults, disorientation or confusion may be the major manifestation of myocardial infarction caused by poor cardiac output. Pain manifestations and numbness and tingling of the arm could also be related to the myocardial infarction. However, the nurse should be more concerned about the new onset of disorientation or confusion caused by decreased perfusion.

8. A nurse assesses a client 2 hours after a cardiac angiography via the left femoral artery. The nurse notes that the left pedal pulse is weak. Which action should the nurse take? a. Elevate the leg and apply a sandbag to the entrance site. b. Increase the flow rate of intravenous fluids. c. Assess the color and temperature of the left leg. d. Document the finding as left pedal pulse of +1/4.

C Loss of a pulse distal to an angiography entry site is serious, indicating a possible arterial obstruction. The pulse may be faint because of edema. The left pulse should be compared with the right, and pulses should be compared with previous assessments, especially before the procedure. Assessing color (pale, cyanosis) and temperature (cool, cold) will identify a decrease in circulation. Once all peripheral and vascular assessment data are acquired, the primary health care provider should be notified. Simply documenting the findings is inappropriate. The leg should be positioned below the level of the heart or dangling to increase blood flow to the distal portion of the leg. Increasing intravenous fluids will not address the clients problem.

4. A nurse assesses an older adult client who has multiple chronic diseases. The clients heart rate is 48 beats/min. Which action should the nurse take first? a. Document the finding in the chart. b. Initiate external pacing. c. Assess the clients medications. d. Administer 1 mg of atropine.

C Pacemaker cells in the conduction system decrease in number as a person ages, resulting in bradycardia. The nurse should check the medication reconciliation for medications that might cause such a drop in heart rate, then should inform the health care provider. Documentation is important, but it is not the priority action. The heart rate is not low enough for atropine or an external pacemaker to be needed.

3. A nurse assesses clients on a medical-surgical unit. Which client should the nurse identify as having the greatest risk for cardiovascular disease? a. An 86-year-old man with a history of asthma b. A 32-year-old Asian-American man with colorectal cancer c. A 45-year-old American Indian woman with diabetes mellitus d. A 53-year-old postmenopausal woman who is on hormone therapy

C The incidence of coronary artery disease and hypertension is higher in American Indians than in whites or Asian Americans. Diabetes mellitus increases the risk for hypertension and coronary artery disease in people o any race or ethnicity. Asthma, colorectal cancer, and hormone therapy do not increase risk for cardiovascular disease.

4. After teaching a client with bacterial cystitis who is prescribed phenazopyridine (Pyridium), the nurse assesses the clients understanding. Which statement made by the client indicates a correct understanding of the teaching? a. I will not take this drug with food or milk. b. If I think I am pregnant, I will stop the drug. c. An orange color in my urine should not alarm me. d. I will drink two glasses of cranberry juice daily.

C Phenazopyridine discolors urine, most commonly to a deep reddish orange. Many clients think they have blood in their urine when they see this. In addition, the urine can permanently stain clothing. Phenazopyridine is safe to take if the client is pregnant. There are no dietary restrictions or needs while taking this medication.

26. A nurse provides phone triage to a pregnant client. The client states, I am experiencing a burning pain when I urinate. How should the nurse respond? a. This means labor will start soon. Prepare to go to the hospital. b. You probably have a urinary tract infection. Drink more cranberry juice. c. Make an appointment with your provider to have your infection treated. d. Your pelvic wall is weakening. Pelvic muscle exercises should help.

C Pregnant clients with a urinary tract infection require prompt and aggressive treatment because cystitis can lead to acute pyelonephritis during pregnancy. The nurse should encourage the client to make an appointment and have the infection treated. Burning pain when urinating does not indicate the start of labor or weakening of pelvic muscles.

25. A nurse cares for a client with urinary incontinence. The client states, I am so embarrassed. My bladder leaks like a young childs bladder. How should the nurse respond? a. I understand how you feel. I would be mortified. b. Incontinence pads will minimize leaks in public. c. I can teach you strategies to help control your incontinence. d. More women experience incontinence than you might think.

C The nurse should accept and acknowledge the clients concerns, and assist the client to learn techniques that will allow control of urinary incontinence. The nurse should not diminish the clients concerns with the use of pads or stating statistics about the occurrence of incontinence.

18. A nurse prepares a client for coronary artery bypass graft surgery. The client states, I am afraid I might die. How should the nurse respond? a. This is a routine surgery and the risk of death is very low. b. Would you like to speak with a chaplain prior to surgery? c. Tell me more about your concerns about the surgery. d. What support systems do you have to assist you?

C The nurse should discuss the clients feelings and concerns related to the surgery. The nurse should not provide false hope or push the clients concerns off on the chaplain. The nurse should address support systems after addressing the clients current issue.

A nurse cares for a client who is recovering from a hypophysectomy. Which action should the nurse take first? a. Keep the head of the bed flat and the client supine. b. Instruct the client to cough, turn, and deep breathe. c. Report clear or light yellow drainage from the nose. d. Apply petroleum jelly to lips to avoid dryness.

C (A light yellow drainage or a halo effect on the dressing is indicative of a cerebrospinal fluid leak. The client should have the head of the bed elevated after surgery. Although deep breathing is important postoperatively, coughing should be avoided to prevent cerebrospinal fluid leakage. Although application of petroleum jelly to the lips will help with dryness, this instruction is not as important as reporting the yellowish drainage.)

After teaching a client with acromegaly who is scheduled for a hypophysectomy, the nurse assesses the clients understanding. Which statement made by the client indicates a need for additional teaching? a. I will no longer need to limit my fluid intake after surgery. b. I am glad no visible incision will result from this surgery. c. I hope I can go back to wearing size 8 shoes instead of size 12. d. I will wear slip-on shoes after surgery to limit bending over.

C (Although removal of the tissue that is oversecreting hormones can relieve many symptoms of hyperpituitarism, skeletal changes and organ enlargement are not reversible. It will be appropriate for the client to drink as needed postoperatively and avoid bending over. The client can be reassured that the incision will not be visible.)

A nurse plans care for a client with Cushings disease. Which action should the nurse include in this clients plan of care to prevent injury? a. Pad the siderails of the clients bed. b. Assist the client to change positions slowly. c. Use a lift sheet to change the clients position. d. Keep suctioning equipment at the clients bedside.

C (Cushings syndrome or disease greatly increases the serum levels of cortisol, which contributes to excessive bone demineralization and increases the risk for pathologic bone fracture. Padding the siderails and assisting the client to change position may be effective, but these measures will not protect him or her as much as using a lift sheet. The client should not require suctioning.)

A nurse plans care for a client with a growth hormone deficiency. Which action should the nurse include in this clients plan of care? a. Avoid intramuscular medications. b. Place the client in protective isolation. c. Use a lift sheet to re-position the client. d. Assist the client to dangle before rising.

C (In adults, growth hormone is necessary to maintain bone density and strength. Adults with growth hormone deficiency have thin, fragile bones. Avoiding IM medications, using protective isolation, and assisting the client as he or she moves from sitting to standing will not serve as safety measures when the client is deficient in growth hormone.)

A nurse assesses a client who is recovering from a transsphenoidal hypophysectomy. The nurse notes nuchal rigidity. Which action should the nurse take first? a. Encourage range-of-motion exercises. b. Document the finding and monitor the client. c. Take vital signs, including temperature. d. Assess pain and administer pain medication.

C (Nuchal rigidity is a major manifestation of meningitis, a potential postoperative complication associated with this surgery. Meningitis is an infection; usually the client will also have a fever and tachycardia. Range-of-motion exercises are inappropriate because meningitis is a possibility. Documentation should be done after all assessments are completed and should not be the only action. Although pain medication may be a palliative measure, it is not the most appropriate initial action.)

A local hunter is admitted to the intensive care unit with a diagnosis of inhalation anthrax. Which medications does the RN anticipate the health care provider will order? A) Amoxicillin (Amoxil, Triamox) 500 mg orally every 8 hours B) Ceftriaxone (Rocephin) 2 g IV every 8 hours C) Ciprofloxacin (Cipro) 400 mg IV every 12 hours D) Pyrazinamide (Zinamide) 1000 to 2000 mg orally every day (Chp. 31; elsevier resources)

C) Ciprofloxacin (Cipro) 400 mg IV every 12 hours (Chp. 31; elsevier resources)

The medical-surgical unit has one negative-airflow room. Which of these four clients who have just arrived on the unit should the charge nurse admit to this room? A) Client with bacterial pneumonia and a cough productive of green sputum B) Client with neutropenia and pneumonia caused by Candida albicans C) Client with possible pulmonary tuberculosis who currently has hemoptysis D) Client with right empyema who has a chest tube and a fever of 103.2° F (Chp. 31; elsevier resources)

C) Client with possible pulmonary tuberculosis who currently has hemoptysis (Chp. 31; elsevier resources)

The nurse is preparing a client for discharge who has undergone percutaneous needle aspiration of a peritonsillar abscess. Which is most important to teach the client about follow-up care? A) Completing the antibiotic medication regimen B) Taking pain medications every 4 to 6 hours C) Contacting the provider if the throat feels more swollen D) Using warm saline gargles and irrigations (Chp. 31; elsevier resources)

C) Contacting the provider if the throat feels more swollen (Chp. 31; elsevier resources)

A client who has recently traveled to Vietnam comes to the emergency department with fatigue, lethargy, night sweats, and a low-grade fever. What is the nurse's first action? A) Contact the health care provider for tuberculosis (TB) medications. B) Perform a TB skin test. C) Place a respiratory mask on the client. D) Test all family members for TB. (Chp. 31; elsevier resources)

C) Place a respiratory mask on the client. (Chp. 31; elsevier resources)

A client is admitted to the emergency department (ED) with a possible diagnosis of avian influenza ("bird flu"). Which of these actions included in the hospital protocol for avian influenza will the nurse take first? A) Ensure that ED staff members receive oseltamivir (Tamiflu). B) Obtain specimens for the H5 polymerase chain reaction test. C) Place the client in a negative air pressure room. D) Start an IV line and administer rehydration therapy. (Chp. 31; elsevier resources)

C) Place the client in a negative air pressure room. (Chp. 31; elsevier resources)

Which action does the postanesthesia care unit (PACU) nurse perform first when caring for a client who has just arrived after a total thyroidectomy? A. Assess the wound dressing for bleeding. B. Give morphine sulfate 4 to 8 mg IV for pain. C. Monitor oxygen saturation using pulse oximetry. Correct D. Support the head and neck with sandbags.

C. Airway assessment and management is always the first priority with every client. This is especially important for a client who has had surgery that involves potential bleeding and edema near the trachea. Assessing the wound dressing for bleeding is a high priority, although this is not the first priority. Pain control and supporting the head and neck with sandbags are important priorities, but can be addressed after airway assessment.

A client diagnosed with hyperpituitarism resulting from a prolactin-secreting tumor has been prescribed bromocriptine mesylate (Parlodel). As a dopamine agonist, what effect does this drug have by stimulating dopamine receptors in the brain? A. Decreases the risk for cerebrovascular disease B. Increases the risk for depression C. Inhibits the release of some pituitary hormones D. Stimulates the release of some pituitary hormones

C. Bromocriptine mesylate inhibits the release of both prolactin and growth hormone. It does not decrease the risk for cerebrovascular disease leading to stroke. Increased risk for depression is not associated with the use of bromocriptine mesylate; however, hallucinations have been reported as a side effect. Bromocriptine mesylate does not stimulate the release of any hormones.

Which client does the nurse identify as being at highest risk for acute adrenal insufficiency resulting from corticosteroid use? A. Client with hematemesis, upper epigastric pain for the past 3 days not relieved with food, and melena B. Client with right upper quadrant pain unrelieved for the past 2 days, dark-brown urine, and clay-colored stools C. Client with shortness of breath and chest tightness, nasal flaring, audible wheezing, and oxygen saturation of 85% for the second time this week D. Client with three emergency department visits in the past month for edema, shortness of breath, weight gain, and jugular venous distention

C. Corticosteroids may be used to treat signs and symptoms of asthma, such as shortness of breath and chest tightness, nasal flaring, audible wheezing, and oxygen saturation of 85%. This places the client at risk for adrenal insufficiency. Corticosteroids are not used to treat signs and symptoms of GI bleeding or peptic ulcer disease (hematemesis, upper epigastric pain for the past 3 days not relieved with food, and melena), gallbladder disease (right upper quadrant pain unrelieved for the past 2 days, dark brown urine, and clay-colored stools), or congestive heart failure (edema, shortness of breath, weight gain, and jugular venous distention).

These data are obtained by the RN who is assessing a client who had a transsphenoidal hypophysectomy yesterday. What information has the most immediate implications for the client's care? A. Dry lips and oral mucosa on examination B. Nasal drainage that tests negative for glucose C. Client report of a headache and stiff neck D. Urine specific gravity of 1.016

C. Headache and stiff neck (nuchal rigidity) are symptoms of meningitis that have immediate implications for the client's care. Dry lips and mouth are not unusual after surgery. Frequent oral rinses and the use of dental floss should be encouraged because the client cannot brush the teeth. Any nasal drainage should test negative for glucose; nasal drainage that tests positive for glucose indicates the presence of a cerebrospinal fluid leak. A urine specific gravity of 1.016 is within normal limits.

Which laboratory result indicates that fluid restrictions have been effective in treating syndrome of inappropriate antidiuretic hormone (SIADH)? A. Decreased hematocrit B. Decreased serum osmolality C. Increased serum sodium D. Increased urine specific gravity

C. Increased serum sodium due to fluid restriction indicates effective therapy. Hemoconcentration is a result of hypovolemic hyponatremia caused by SIADH and diabetes insipidus. Plasma osmolality is decreased as a result of SIADH. Urine specific gravity is decreased with diabetes insipidus and is increased with SIADH.

A client with hypothyroidism is being discharged. Which environmental change may the client experience in the home? A. Frequent home care B. Handrails in the bath C. Increased thermostat setting D. Strict infection-control measures

C. Manifestations of hypothyroidism include cold intolerance. Increased thermostat settings or additional clothing may be necessary. A client with a diagnosis of hypothyroidism can be safely managed at home with adequate discharge teaching regarding medications and instructions on when to notify the health care provider or home health nurse. In general, hypothyroidism does not cause mobility issues. Activity intolerance and fatigue may be an issue, however. A client with hypothyroidism is not immune-compromised or contagious, so no environmental changes need to be made to the home.

An older client with an elevated serum calcium level is receiving IV furosemide (Lasix) and an infusion of normal saline at 150 mL/hr. Which nursing action can the RN delegate to unlicensed assistive personnel (UAP)? A. Ask the client about any numbness or tingling. B. Check for bone deformities in the client's back. C. Measure the client's intake and output hourly. D. Monitor the client for shortness of breath.

C. Measuring intake and output is a commonly delegated nursing action that is within the UAP scope of practice. Numbness and tingling is part of the client assessment that needs to be completed by a licensed nurse. Bony deformities can be due to pathologic fractures; physical assessment is a complex task that cannot be delegated. An older client receiving an IV at 150 mL/hr is at risk for congestive heart failure; careful monitoring for shortness of breath is the responsibility of the RN.

An RN and LPN/LVN are caring for a group of clients on the medical-surgical unit. Which client will be the best to assign to the LPN/LVN? A. Client with Graves' disease who needs discharge teaching after a total thyroidectomy B. Client with hyperparathyroidism who is just being admitted for a parathyroidectomy C. Client with infiltrative ophthalmopathy who needs administration of high-dose prednisone (Deltasone) D. Newly diagnosed client with hypothyroidism who needs education about the use of thyroid supplements

C. Medication administration for the client with infiltrative ophthalmopathy is within the scope of practice of the LPN/LVN. Discharge teaching is a complex task that cannot be delegated to the LPN/LVN. A client being admitted for a parathyroidectomy needs preoperative teaching, which must be provided by the RN. A client who has a new diagnosis will have questions about the disease and prescribed medications; teaching is a complex task that is appropriate for the RN.

Which type of thyroid cancer often occurs as part of multiple endocrine neoplasia (MEN) type II? A. Anaplastic B. Follicular C. Medullary D. Papillary

C. Medullary carcinoma commonly occurs as part of MEN type II, which is a familial endocrine disorder. Anaplastic carcinoma is an aggressive tumor that invades surrounding tissue. Follicular carcinoma occurs more frequently in older clients and may metastasize to bone and lung. Papillary carcinoma is the most common type of thyroid cancer. It is slow growing and, if the tumor is confined to the thyroid gland, the outlook for a cure is good with surgical management.

The nurse is providing discharge instructions to a client on spironolactone (Aldactone) therapy. Which comment by the client indicates a need for further teaching? A. "I must call the provider if I am more tired than usual." B. "I need to increase my salt intake." C. "I should eat a banana every day." D. "This drug will not control my heart rate."

C. Spironolactone increases potassium levels, so potassium supplements and foods rich in potassium, such as bananas, should be avoided to prevent hyperkalemia. While taking spironolactone, symptoms of hyponatremia such as drowsiness and lethargy must be reported; the client may need increased dietary sodium. Spironolactone will not have an effect on the client's heart rate.

A client with Cushing's disease begins to laugh loudly and inappropriately, causing the family in the room to be uncomfortable. What is the nurse's best response? A. "Don't mind this. The disease is causing this." B. "I need to check the client's cortisol level." C. "The disease can sometimes affect emotional responses." D. "Medication is available to help with this."

C. The client may have neurotic or psychotic behavior as a result of high blood cortisol levels. Being honest with the family helps them to understand what is happening. Telling the family not to mind the laughter and that the disease is causing it is vague and minimizes the family's concern. This is the perfect opportunity for the nurse to educate the family about the disease. Cushing's disease is the hypersecretion of cortisol, which is abnormally elevated in this disease and, because the diagnosis has already been made, blood levels do not need to be redrawn. Telling the family that medication is available to help with inappropriate laughing does not assist them in understanding the cause of or the reason for the client's behavior.

A client has hyperparathyroidism. Which incident witnessed by the nurse requires the nurse's intervention? A. The client eating a morning meal of cereal and fruit B. The physical therapist walking with the client in the hallway C. Unlicensed assistive personnel pulling the client up in bed by the shoulders D. Visitors talking with the client about going home

C. The client with hyperparathyroidism is at risk for pathologic fracture. All members of the health care team must move the client carefully. A lift sheet should be used to re-position the client. The client with hyperparathyroidism is not restricted from eating and should maintain a balanced diet. The client can benefit from moderate exercise and physical therapy, and is not restricted from having visitors.

The nurse is teaching a client about thyroid replacement therapy. Which statement by the client indicates a need for further teaching? A. "I should have more energy with this medication." B. "I should take it every morning." C. "If I continue to lose weight, I may need an increased dose." D. "If I gain weight and feel tired, I may need an increased dose."

C. Weight loss indicates a need for a decreased dose, not an increased dose. One of the symptoms of hypothyroidism is lack of energy; thyroid replacement therapy should help the client have more energy. The correct time to take thyroid replacement therapy is in the morning. If the client is gaining weight and continues to feel tired, that is an indication that the dose may need to be increased.

To validate that a client has had a myocardial infarction (MI), the nurse assesses for positive findings on which tests? Creatine kinase-MB fraction (CK-MB) and alkaline phosphatase Homocysteine and C-reactive protein Total cholesterol, low-density lipoprotein cholesterol, and high-density lipoprotein cholesterol CK-MB and troponin

CK-MB and troponin CK-MB and troponin are the cardiac markers used to determine whether MI has occurred. Alkaline phosphatase is often elevated in liver disease. Homocysteine and C-reactive protein are markers of inflammation, which may represent risk for MI, but they are not diagnostic for MI. Elevated cholesterol levels are risks for MI, but they do not validate that an MI has occurred.

Ch.64 p. 1335, Prioritization, Delegation, and Supervision The patient, a 21-year-old college student, was brought to the emergency department (ED) by his roommate. He reports abdominal pain, polyuria for the past 2 days, vomiting several times prior to arrival, and extreme thirst. He appears flushed, and his lips and mucous membranes are dry and cracked. His skin turgor is poor. He demonstrates deep rapid respirations; there is a fruit odor to his breath. He has type 1 diabetes and "may have skipped a few doses of insulin because of cramming for final exams." He is alert and talking but is having trouble focusing on your questions. Vital signs: Blood pressure 110/60 Pulse 110/min Respirations 32/min Temperature 100.8° F Fingerstick glucose 485 mg/dL Oxygen saturation 99% 1. You have completed triage assessment and history, should you now notify his parents for permission to treat him? Why or why not?

Calling the parents is not necessary because as the patient is old enough to sign consent for himself. (If he were under age, the treatment would not be delayed if the parents were unavailable in an emergency situation.) Calling the primary health care provider is usually done by the ED physician after the preliminary workup is completed. (Policies for calling private physicians may vary among institutions. Be sure to check the policy at your facility.)

Ch.64 p. 1335, Prioritization, Delegation, and Supervision The patient, a 21-year-old college student, was brought to the emergency department (ED) by his roommate. He reports abdominal pain, polyuria for the past 2 days, vomiting several times prior to arrival, and extreme thirst. He appears flushed, and his lips and mucous membranes are dry and cracked. His skin turgor is poor. He demonstrates deep rapid respirations; there is a fruit odor to his breath. He has type 1 diabetes and "may have skipped a few doses of insulin because of cramming for final exams." He is alert and talking but is having trouble focusing on your questions. Vital signs: Blood pressure 110/60 Pulse 110/min Respirations 32/min Temperature 100.8° F Fingerstick glucose 485 mg/dL Oxygen saturation 99% 3. Should you call his primary health provider? Why or why not?

Calling the primary health care provider is usually done by the ED physician after the preliminary work-up is completed and interventions have started. (Policies for calling private physicians may vary among institutions. Be sure to check the policy at your facility.)

Ch.64 p. 1332, Safety; Quality Improvement; Teamwork and Collaboration The patient is a 60 year-old-woman who is 1 day postoperative after a total knee replacement. She has type 2 diabetes and just recently was switched from oral antidiabetic drugs to an insulin regimen. She let her nurse know that her on-demand lunch has been ordered. The nurse tests her blood and gives her the prescribed short-acting insulin dose. An hour later, the physical therapist finds her pale, confused, and clammy. Her lunch tray is on her table and appears totally untouched. 2. What is your first action? Provide a rationale.

Check her blood glucose level immediately because the methods to increase her blood glucose level are dependent on how low the current level is. a. As an alternative, if there is an easily digestible carbohydrate on her tray and she is able to swallow, you could give that to her immediately and then obtain a blood glucose measurement. However, this is less precise.

Ch.63 p. 1295, Patient-Centered Care; Safety The patient, a 45-year-old former school teacher, is residing in a skilled nursing facility to recover from a tibia-fibula fracture that is being managed with an external fixation system. On admission 2 weeks ago, she told you that she felt she was "getting old too fast." She explained that she had gained 54 pounds in the previous 6 months, had no energy, was often constipated, and was always cold. She teared up and said that her ability to concentrate was so bad that not only could she no longer help her high school children with their homework, but that she didn't recognize the step hazard that caused her to fall and break her ankle. Today the nursing assistant assigned to her care reports that the patient's pulse is only 42 beats per minute and that her temperature was 96° F even with two blankets. When you enter her room, she is sleeping with an untouched breakfast tray on her table. 1. What are the priority assessment data you should obtain? Provide a rationale for your choices.

Check her respiratory rate and effectiveness (use pulse oximetry). Then recheck her heart rate (apically) and blood pressure. She has indications of severe hypothyroidism. Respiratory arrest is a potential cause of death for patients with this problem. Assess her level of consciousness. Coma is possible, especially if she received any opioid drugs for pain control.

A client has just returned from coronary artery bypass graft surgery. For which finding does the nurse contact the surgeon? Temperature 98.2° F Chest tube drainage 175 mL last hour Serum potassium 3.9 mEq/L Incisional pain 6 on a scale of 0 to 10

Chest tube drainage 175 mL last hour Some bleeding is expected after surgery; however, the nurse should report chest drainage over 150 mL/hr to the surgeon. Although hypothermia is a common problem after surgery, a temperature of 98.2° F is a normal finding. Serum potassium of 3.9 mEq/L is a normal finding. Incisional pain of 6 on a scale of 0 to 10 is expected immediately after major surgery; the nurse should administer prescribed analgesics.

Ch.64 p. 1332, Safety; Quality Improvement; Teamwork and Collaboration The patient is a 60 year-old-woman who is 1 day postoperative after a total knee replacement. She has type 2 diabetes and just recently was switched from oral antidiabetic drugs to an insulin regimen. She let her nurse know that her on-demand lunch has been ordered. The nurse tests her blood and gives her the prescribed short-acting insulin dose. An hour later, the physical therapist finds her pale, confused, and clammy. Her lunch tray is on her table and appears totally untouched. 3. What is the most likely cause leading to this problem?

Clearly, there was a delay in eating after receiving the insulin. The tray may have been delayed longer than expected from food service, or perhaps she decided she was not hungry when it first arrived. She could have been interrupted (possible phone call or visitor) before she had a chance to eat it. In addition, it is possible because she has only recently been started on insulin that she did not understand the necessity of eating soon after receiving insulin.

The nurse in the coronary care unit is caring for a group of clients who have had myocardial infarction. Which client does the nurse see first? Client with dyspnea on exertion when ambulating to the bathroom Client with third-degree heart block on the monitor Client with normal sinus rhythm and PR interval of 0.28 second Client who refuses to take heparin or nitroglycerin

Client with third-degree heart block on the monitor Third-degree heart block is a serious complication that indicates that a large portion of the left ventricle and conduction system are involved, so the client with the third-degree heart block should be seen first. Third-degree heart block usually requires pacemaker insertion. A normal rhythm with prolonged PR interval indicates first-degree heart block, which usually does not require treatment. The client with dyspnea on exertion when ambulating to the bathroom is not at immediate risk. The client's uncooperative behavior when refusing to take heparin or nitroglycerin may indicate fear or denial; he should be seen after emergency situations have been handled.

The nurse is concerned that a client who had myocardial infarction (MI) has developed cardiogenic shock. Which findings indicate shock? (Select all that apply.) Bradycardia Cool, diaphoretic skin Crackles in the lung fields Respiratory rate of 12 breaths/min Anxiety and restlessness Temperature of 100.4° F

Cool, diaphoretic skin Crackles in the lung fields Anxiety and restlessness The client with shock has cool, moist skin. Because of extensive tissue necrosis, the left ventricle cannot forward blood adequately, resulting in pulmonary congestion and crackles. Because of poor tissue perfusion, a change in mental status, anxiety, and restlessness are expected. All types of shock (except neurogenic) present with tachycardia, not bradycardia. Due to pulmonary congestion, a client with cardiogenic shock typically has tachypnea. Cardiogenic shock does not present with low-grade fever; this would be more likely to occur in pericarditis.

The client in the cardiac care unit has had a large myocardial infarction. How does the nurse recognize onset of left ventricular failure? Urine output of 1500 mL on the preceding day Crackles in the lung fields Pedal edema Expectoration of yellow sputum

Crackles in the lung fields Manifestations of left ventricular failure and pulmonary edema are noted by listening for crackles and identifying their locations in the lung fields. A urine output of 1500 mL is normal. Edema is a sign of right ventricular heart failure. Yellow sputum indicates the presence of white blood cells and possible infection.

19. An emergency department nurse triages clients who present with chest discomfort. Which client should the nurse plan to assess first? a. A 42-year-old female who describes her pain as a dull ache with numbness in her fingers b. A 49-year-old male who reports moderate pain that is worse on inspiration c. A 53-year-old female who reports substernal pain that radiates to her abdomen d. A 58-year-old male who describes his pain as intense stabbing that spreads across his chest

D All clients who have chest pain should be assessed more thoroughly. To determine which client should be seen first, the nurse must understand common differences in pain descriptions. Intense stabbing, vise-like substernal pain that spreads through the clients chest, arms, jaw, back, or neck is indicative of a myocardial infarction. The nurse should plan to see this client first to prevent cardiac cell death. A dull ache with numbness in the fingers is consistent with anxiety. Pain that gets worse with inspiration is usually related to a pleuropulmonary problem. Pain that spreads to the abdomen is often associated with an esophageal-gastric problem, especially when this pain is experienced by a male client. Female clients may experience abdominal discomfort with a myocardial event. Although clients with anxiety, pleuropulmonary, and esophageal-gastric problems should be seen, they are not a higher priority than myocardial infarction.

3. A patient is scheduled to undergo diagnostic testing for sickle cell anemia. For which diagnostic test does the nurse provide patient teaching? a. Bone marrow biopsy b. Platelet count c. Philadelphia chromosome analysis d. Hemoglobin S

d

10. A nurse assesses a client who is scheduled for a cardiac catheterization. Which assessment should the nurse complete prior to this procedure? a. Clients level of anxiety b. Ability to turn self in bed c. Cardiac rhythm and heart rate d. Allergies to iodine-based agents

D Before the procedure, the nurse should ascertain whether the client has an allergy to iodine-containing preparations, such as seafood or local anesthetics. The contrast medium used during the procedure is iodine based. This allergy can cause a life-threatening reaction, so it is a high priority. Second, it is important for the nurse to assess anxiety, mobility, and baseline cardiac status.

2. A nurse assesses a client after administering a prescribed beta blocker. Which assessment should the nurse expect to find? a. Blood pressure increased from 98/42 mm Hg to 132/60 mm Hg b. Respiratory rate decreased from 25 breaths/min to 14 breaths/min c. Oxygen saturation increased from 88% to 96% d. Pulse decreased from 100 beats/min to 80 beats/min

D Beta blockers block the stimulation of beta1-adrenergic receptors. They block the sympathetic (fight-or-flight) response and decrease the heart rate (HR). The beta blocker will decrease HR and blood pressure, increasing ventricular filling time. It usually does not have effects on beta2-adrenergic receptor sites. Cardiac output will drop because of decreased HR.

19. A nurse teaches a client with functional urinary incontinence. Which statement should the nurse include in this clients teaching? a. You must clean around your catheter daily with soap and water. b. Wash the vaginal weights with a 10% bleach solution after each use. c. Operations to repair your bladder are available, and you can consider these. d. Buy slacks with elastic waistbands that are easy to pull down.

D Functional urinary incontinence occurs as the result of problems not related to the clients bladder, such as trouble ambulating or difficulty accessing the toilet. One goal is that the client will be able to manage his or her clothing independently. Elastic waistband slacks that are easy to pull down can help the client get on the toilet in time to void. The other instructions do not relate to functional urinary incontinence.

8. A nurse plans care for a client with overflow incontinence. Which intervention should the nurse include in this clients plan of care to assist with elimination? a. Stroke the medial aspect of the thigh. b. Use intermittent catheterization. c. Provide digital anal stimulation. d. Use the Valsalva maneuver.

D In clients with overflow incontinence, the voiding reflex arc is not intact. Mechanical pressure, such as that achieved through the Valsalva maneuver (holding the breath and bearing down as if to defecate), can initiate voiding. Stroking the medial aspect of the thigh or providing digital anal stimulation requires the reflex arc to be intact to initiate elimination. Due to the high risk for infection, intermittent catheterization should only be implemented when other interventions are not successful.

11. A nurse cares for a client who has kidney stones from secondary hyperoxaluria. Which medication should the nurse anticipate administering? a. Phenazopyridine (Pyridium) b. Propantheline (Pro-Banthine) c. Tolterodine (Detrol LA) d. Allopurinol (Zyloprim)

D Stones caused by secondary hyperoxaluria respond to allopurinol (Zyloprim). Phenazopyridine is given to clients with urinary tract infections. Propantheline is an anticholinergic. Tolterodine is an anticholinergic with smooth muscle relaxant properties.

15. A nurse cares for a client who has advanced cardiac disease and states, I am having trouble sleeping at night. How should the nurse respond? a. I will consult the provider to prescribe a sleep study to determine the problem. b. You become hypoxic while sleeping; oxygen therapy via nasal cannula will help. c. A continuous positive airway pressure, or CPAP, breathing mask will help you breathe at night. d. Use pillows to elevate your head and chest while you are sleeping.

D The client is experiencing orthopnea (shortness of breath while lying flat). The nurse should teach the client to elevate the head and chest with pillows or sleep in a recliner. A sleep study is not necessary to diagnose this client. Oxygen and CPAP will not help a client with orthopnea.

17. A nurse cares for a client who is scheduled for the surgical creation of an ileal conduit. The client states, I am anxious about having an ileal conduit. What is it like to have this drainage tube? How should the nurse respond? a. I will ask the provider to prescribe you an antianxiety medication. b. Would you like to discuss the procedure with your doctor once more? c. I think it would be nice to not have to worry about finding a bathroom. d. Would you like to speak with someone who has an ileal conduit?

D The goal for the client who is scheduled to undergo a procedure such as an ileal conduit is to have a positive self-image and a positive attitude about his or her body. Discussing the procedure candidly with someone who has undergone the same procedure will foster such feelings, especially when the current client has an opportunity to ask questions and voice concerns to someone with first-hand knowledge. Medications for anxiety will not promote a positive self-image and a positive attitude, nor will discussing the procedure once more with the physician or hearing the nurses opinion.

14. A nurse assesses clients on the medical-surgical unit. Which client is at greatest risk for bladder cancer? a. A 25-year-old female with a history of sexually transmitted diseases b. A 42-year-old male who has worked in a lumber yard for 10 years c. A 55-year-old female who has had numerous episodes of bacterial cystitis d. An 86-year-old male with a 50pack-year cigarette smoking history

D The greatest risk factor for bladder cancer is a long history of tobacco use. The other factors would not necessarily contribute to the development of this specific type of cancer.

A nurse cares for a client after a pituitary gland stimulation test using insulin. The clients post-stimulation laboratory results indicate elevated levels of growth hormone (GH) and adrenocorticotropic hormone (ACTH). How should the nurse interpret these results? a. Pituitary hypofunction b. Pituitary hyperfunction c. Pituitary-induced diabetes mellitus d. Normal pituitary response to insulin

D (Some tests for pituitary function involve administering agents that are known to stimulate the secretion of specific pituitary hormones and then measuring the response. Such tests are termed stimulation tests. The stimulation test for GH or ACTH assessment involves injecting the client with regular insulin (0.05 to 1 unit/kg of body weight) and checking circulating levels of GH and ACTH. The presence of insulin in clients with normal pituitary function causes increased release of GH and ACTH.)

The nurse has taught a client about influenza infection control. Which client statement indicates the need for further teaching? A) "Handwashing is the best way to prevent transmission." B) "I should avoid kissing and shaking hands." C) "It is best to cough and sneeze into my upper sleeve." D) "The intranasal vaccine can be given to everybody in the family." (Chp. 31; elsevier resources)

D) "The intranasal vaccine can be given to everybody in the family." (Chp. 31; elsevier resources)

The nurse is preparing to admit an adult client with pertussis. Which symptom does the nurse anticipate finding in this client? A) "Whooping" after a cough B) Hemoptysis C) Mild cold-like symptoms D) Post-cough emesis (Chp. 31; elsevier resources)

D) Post-cough emesis (Chp. 31; elsevier resources)

Community health nurses are tasked with providing education on prevention of respiratory infection for diseases such as the flu. Which target audience is given the highest priority? A) Homeless people B) Hospital staff C) Politicians D) Prison staff and inmates (Chp. 31; elsevier resources)

D) Prison staff and inmates (Chp. 31; elsevier resources)

The nurse notices a visitor walking into the room of a client on airborne isolation with no protective gear. What does the nurse do? A) Ensures that the client is wearing a mask B) Tells the visitor that the client cannot receive visitors at this time C) Provides a particulate air respirator to the visitor D) Provides a mask to the visitor (Chp. 31; elsevier resources)

D) Provides a mask to the visitor (Chp. 31; elsevier resources)

A client comes to the emergency department with a sore throat. Examination reveals redness and swelling of the pharyngeal mucous membranes. Which diagnostic test does the nurse expect will be requested first? A) Chest x-ray B) Complete blood count (CBC) C) Tuberculosis (TB) skin test D) Throat culture (Chp. 31; elsevier resources)

D) Throat culture (Chp. 31; elsevier resources)

The nurse is caring for a client with severe acute respiratory syndrome. What is the most important precaution the nurse should take when preparing to suction this client? A) Keeping the head of the bed elevated 30 to 45 degrees B) Performing oral care after suctioning the oropharynx C) Washing hands and donning gloves prior to the procedure D) Wearing a disposable particulate mask respirator and protective eyewear (Chp. 31; elsevier resources)

D) Wearing a disposable particulate mask respirator and protective eyewear (Chp. 31; elsevier resources)

35. After a thyroidectomy, a patient reports tingling around the mouth and muscle twitching. Which complication do these assessment findings indicate to the nurse? a. Hemorrhage b. Respiratory distress c. Thyroid storm d. Hypocalcemia

d

The RN has just received change-of-shift report on the medical-surgical unit. Which client will need to be assessed first? A. Client with Hashimoto's thyroiditis and a large goiter B. Client with hypothyroidism and an apical pulse of 51 beats/min C. Client with parathyroid adenoma and flank pain due to a kidney stone D. Client who had a parathyroidectomy yesterday and has muscle twitching

D. A client who is 1 day postoperative for parathyroidectomy and has muscle twitching is showing signs of hypocalcemia and is at risk for seizures. Rapid assessment and intervention are needed. Clients with Hashimoto's thyroiditis are usually stable; this client does not need to be assessed first. Although an apical pulse of 51 is considered bradycardia, a low heart rate is a symptom of hypothyroidism. A client with a kidney stone will be uncomfortable and should be asked about pain medication as soon as possible, but this client does not need to be assessed first.

The nurse is caring for a client with hypercortisolism. The nurse begins to feel the onset of a cold but still has 4 hours left in the shift. What does the nurse do? A. Asks another nurse to care for the client B. Monitors the client for cold-like symptoms C. Refuses to care for the client D. Wears a facemask when caring for the client

D. A client with hypercortisolism will be immune-suppressed. Anyone with a suspected upper respiratory infection who must enter the client's room must wear a mask to prevent the spread of infection. Although asking another nurse to care for the client might be an option in some facilities, it is not generally realistic or practical. The nurse, not the client, feels the onset of the cold, so monitoring the client for cold-like symptoms is part of good client care for a client with hypercortisolism. Refusing to care for the client after starting care would be considered abandonment.

The charge nurse is making client assignments for the medical-surgical unit. Which client will be best to assign to an RN who has floated from the pediatric unit? A. Client in Addisonian crisis who is receiving IV hydrocortisone B. Client admitted with syndrome of inappropriate antidiuretic hormone (SIADH) secondary to lung cancer C. Client being discharged after a unilateral adrenalectomy to remove an adrenal tumor D. Client with Cushing's syndrome who has elevated blood glucose and requires frequent administration of insulin

D. An RN who works with pediatric clients would be familiar with glucose monitoring and insulin administration. A client in Addisonian crisis would best be monitored by an RN from the medical-surgical floor. Although the float RN could complete the admission history, the client with SIADH secondary to lung cancer might require teaching and orientation to the unit that a nurse more familiar with that area would be better able to provide. Discharge teaching specific to adrenalectomy should be provided by the RN who is regularly assigned to the medical-surgical floor and is more familiar with taking care of postoperative adult clients with endocrine disorders.

A client is referred to a home health agency after a transsphenoidal hypophysectomy. Which action does the RN case manager delegate to the home health aide who will see the client daily? A. Document symptoms of incisional infection or meningitis. B. Give over-the-counter laxatives if the client is constipated. C. Set up medications as prescribed for the day. D. Test any nasal drainage for the presence of glucose.

D. Cerebrospinal fluid (CSF) will test positive using a glucose "dipstick." Nasal drainage that is positive for glucose after a transsphenoidal hypophysectomy would indicate a CSF leak that would require immediate notification of the health care provider. Home health aides can be taught the correct technique to perform this procedure. Assessing for symptoms of infection and documenting them in the record, medication administration, and setting up medication are not within the scope of practice of the home health aide.

A client with thyroid cancer has just received 131I ablative therapy. Which statement by the client indicates a need for further teaching? A. "I cannot share my toothpaste with anyone." B. "I must flush the toilet three times after I use it." C. "I need to wash my clothes separately from everyone else's clothes." D. "I'm ready to hold my newborn grandson now."

D. Clients undergoing 131I therapy should avoid close contact with pregnant women, infants, and young children for 1 week after treatment. Clients should remain at least 1 meter (39 inches, or roughly 3 feet) away, and limit exposure to less than 1 hour per day. Some radioactivity will remain in the client's salivary glands for up to 1 week after treatment. Care should be taken to avoid exposing others to the saliva. Flushing the toilet three times after use will ensure that all urine has been diluted and removed. Clothing needs to be washed separately and the washing machine then needs to be run empty for a full cycle before it is used to wash the clothing of others.

How does the drug desmopressin (DDAVP) decrease urine output in a client with diabetes insipidus (DI)? A. Blocks reabsorption of sodium B. Increases blood pressure C. Increases cardiac output D. Works as an antidiuretic hormone (ADH) in the kidneys

D. Desmopressin is a synthetic form of ADH that binds to kidney receptors and enhances reabsorption of water, thus reducing urine output. Desmopressin does not have any effect on sodium reabsorption. It may cause a slight increase or a transient decrease in blood pressure, but this does not affect urine output. Desmopressin does not increase cardiac output.

A client with a possible adrenal gland tumor is admitted for testing and treatment. Which nursing action is most appropriate for the charge nurse to delegate to the nursing assistant? A. Assess skin turgor and mucous membranes for hydration status. B. Discuss the dietary restrictions needed for 24-hour urine testing. C. Plan ways to control the environment that will avoid stimulating the client. D. Remind the client to avoid drinking coffee and changing position suddenly.

D. Drinking caffeinated beverages and changing position suddenly are not safe for a client with a potential adrenal gland tumor because of the effects of catecholamines. Reminding the client about previous instructions is an appropriate role for a nursing assistant who may observe the client doing potentially risky activities. Client assessment, client teaching, and environment planning are higher-level skills that require the experience and responsibility of the RN, and are not within the scope of practice of the nursing assistant.

A client with Cushing's disease says that she has lost 1 pound. What does the nurse do next? A. Auscultates the lungs for crackles B. Checks urine for specific gravity C. Forces fluids D. Weighs the client

D. Fluid retention with weight gain is more of a problem than weight loss in clients with Cushing's disease. Weighing the client with Cushing's disease is part of the nurse's assessment. Crackles in the lungs indicate possible fluid retention, which would cause weight gain, not weight loss. Urine specific gravity will help assess hydration status, but this would not be the next step in the client's assessment. Forcing fluids is not appropriate because usually excess water and sodium reabsorption cause fluid retention in the client with Cushing's disease.

What effect can starting a dose of levothyroxine sodium (Synthroid) too high or increasing a dose too rapidly have on a client? A. Bradycardia and decreased level of consciousness B. Decreased respiratory rate C. Hypotension and shock D. Hypertension and heart failure

D. Hypertension and heart failure are possible if the levothyroxine sodium dose is started too high or raised too rapidly, because levothyroxine would essentially put the client into a hyperthyroid state. The client would be tachycardic, not bradycardic. The client may have an increased respiratory rate. Shock may develop, but only as a late effect and as the result of "pump failure."

The nurse reviews the vital signs of a client diagnosed with Graves' disease and sees that the client's temperature is up to 99.6° F. After notifying the health care provider, what does the nurse do next? A. Administers acetaminophen B. Alerts the Rapid Response Team C. Asks any visitors to leave D. Assesses the client's cardiac status completely

D. If the client's temperature has increased by even 1°, the nurse's first action is to notify the provider. Continuous cardiac monitoring should be the next step. Administering a nonsalicylate antipyretic such as acetaminophen is appropriate, but is not a priority action for this client. Alerting the Rapid Response Team is not needed at this time. Asking visitors to leave would not be the next action, and if visitors are providing comfort to the client, this would be contraindicated.

A client is taking methimazole (Tapazole) for hyperthyroidism and would like to know how soon this medication will begin working. What is the nurse's best response? A. "You should see effects of this medication immediately." B. "You should see effects of this medication within 1 week." C. "You should see full effects from this medication within 1 to 2 days." D. "You should see some effects of this medication within 2 weeks."

D. Methimazole is an iodine preparation that decreases blood flow through the thyroid gland. This action reduces the production and release of thyroid hormone. The client should see some effects within 2 weeks; however, it may take several more weeks before metabolism returns to normal. Although onset of action is 30 to 40 minutes after an oral dose, the client will not see effects immediately. Effects will take longer than 1 week to become apparent when methimazole is used. Methimazole needs to be taken every 8 hours for an extended period of time. Levels of triiodothyronine (T3) and thyroxine (T4) will be monitored and dosages adjusted as levels fall.

11. A nurse cares for a client who is recovering from a parathyroidectomy. When taking the client's blood pressure, the nurse notes that the client's hand has gone into flexion contractions. Which laboratory result does the nurse correlate with this condition? a. Serum potassium: 2.9 mEq/L b. Serum magnesium: 1.7 mEq/L c. Serum sodium: 122 mEq/L d. Serum calcium: 6.9 mg/dL

D. Hypocalcemia destabilizes excitable membranes and can lead to muscle twitches, spasms, and tetany. This effect of hypocalcemia is enhanced in the presence of tissue hypoxia. The flexion contractions (Trousseau's sign) that occur during blood pressure measurement are indicative of hypocalcemia, not the other electrolyte imbalances, which include hypokalemia, hyponatremia, and hypomagnesemia.

7. A nurse assesses a client who is prescribed levothyroxine (Synthroid) for hypothyroidism. Which assessment finding should alert the nurse that the medication therapy is effective? a. Thirst is recognized and fluid intake is appropriate. b. Weight has been the same for 3 weeks. c. Total white blood cell count is 6000 cells/mm3. d. Heart rate is 70 beats/min and regular.

D. Hypothyroidism decreases body functioning and can result in effects such as bradycardia, confusion, and constipation. If a client's heart rate is bradycardic while on thyroid hormone replacement, this is an indicator that the replacement may not be adequate. Conversely, a heart rate above 100 beats/min may indicate that the client is receiving too much of the thyroid hormone. Thirst, fluid intake, weight, and white blood cell count do not represent a therapeutic response to this medication.

2. A nurse assesses a client who is recovering from a total thyroidectomy and notes the development of stridor. Which action should the nurse take first? a. Reassure the client that the voice change is temporary. b. Document the finding and assess the client hourly. c. Place the client in high-Fowler's position and apply oxygen. d. Contact the provider and prepare for intubation.

D. Stridor on exhalation is a hallmark of respiratory distress, usually caused by obstruction resulting from edema. One emergency measure is to remove the surgical clips to relieve the pressure. This might be a physician function. The nurse should prepare to assist with emergency intubation or tracheostomy while notifying the provider or the Rapid Response Team. Stridor is an emergency situation; therefore, reassuring the client, documenting, and reassessing in an hour do not address the urgency of the situation. Oxygen should be applied, but this action will not keep the airway open.

Ch.62 p. 1276, Patient-Centered Care; Quality Improvement; Safety The patient is a 32-year-old woman admitted to your unit after surgery for fractures of the left arm and leg resulting from a car crash. She is awake and able to verify her medical history of rheumatoid arthritis and her usual daily medications. These are 10 mg of prednisone, naproxen 800 mg twice daily, oral contraceptives, calcium 600 mg, and one multiple vitamin tablet. All of these are prescribed for her to receive during her hospitalization. She is concerned about pain management and how long the recovery will be for the fractures. She is friendly, somewhat anxious, asks many questions, and wants to do "her part" to ensure good recovery. Over the next 4 days, she has become quieter, mumbles that her head and stomach hurt, and now does not recognize the assistant who has been providing her daily care. When she receives her medications, she has difficulty picking them up. The nursing assistant remarks that taking her pulse is difficult because it is so slow and irregular. When you assess her, she is so weak that she is unable to lift her arm for a blood pressure check. Her blood pressure is 92/50, which is down from the 128/84 reading on admission. You also verify that her heart beat is slow and irregular. 3. How could this problem been avoided?

Daily assessment of her salivary or serum cortisol levels could have indicated a need for a higher dose. Also, because she was receiving prednisone, daily blood glucose levels should have been performed. Examining these parameters would provide data to determine the adequacy of her therapy, as well as its potential side effects.

A nurse assesses clients who are at risk for diabetes mellitus. Which client is at greatest risk? a. A 29-year-old Caucasian b. A 32-year-old African-American c. A 44-year-old Asian d. A 48-year-old American Indian

Diabetes is a particular problem among African Americans, Hispanics, and American Indians. The incidence of diabetes increases in all races and ethnic groups with age. Being both an American Indian and middle-aged places this client at highest risk.

The nurse is teaching a group of teens about prevention of heart disease. Which point should the nurse emphasize? Reduce abdominal fat. Avoid stress. Do not smoke or chew tobacco. Avoid alcoholic beverages.

Do not smoke or chew tobacco. Tobacco exposure, including secondhand smoke, reduces coronary blood flow; causes vasoconstriction, endothelial dysfunction, and thickening of the vessel walls; increases carbon monoxide; and decreases oxygen. Because it is highly addicting, beginning smoking in the teen years may lead to decades of exposure. Teens are not likely to experience metabolic syndrome from obesity, but are very likely to use tobacco. Avoiding stress is a less modifiable risk factor, which is less likely to cause heart disease in teens. The risk of smoking outweighs the risk of alcohol use.

14. A patient reports intense urgency, freuqnecy, and bladder pain. Urinalysis results show white blood cells (WBCs) and red blood cells (RBCs) and urine culture results are negative for infection. How does the nurse interpret these findings? a. Interstitial cystitis b. Urethritis c. Bacteriuria d. Infectious cystitis

a

14. An elderly patient has been in bed for several days after a fall. The nurse encourage ambulation to stimulate the movement of urine through the ureter by what phenomenon? a. Peristalsis b. Gravity c Pelvic pressure d. Backflow

a

Which atypical symptoms may be present in a female client experiencing myocardial infarction (MI)? (Select all that apply.) Sharp, inspiratory chest pain Dyspnea Dizziness Extreme fatigue Anorexia

Dyspnea Dizziness Extreme fatigue Many women who experience an MI present with dyspnea, light-headedness, and fatigue. Sharp, pleuritic pain is more consistent with pericarditis or pulmonary embolism. Anorexia is neither a typical nor an atypical sign of MI.

Ch.64 p. 1329, Patient-Centered Care During a clinic visit, you are reviewing the records of a 39-year-old patient who was diagnosed 5 years ago with type 2 diabetes. You discover that, although he has always been extremely near-sighted, he has not seen an ophthalmologist for 4 years. He has gained 12 lbs since his last visit a year ago. His laboratory values show a fasting blood glucose level of 96 mg/dL, an A1C of 8.2%, a total cholesterol of 322 mg/dL, and an LDL of 190 mg/dL. When you ask him about ophthalmology follow-up and point out his laboratory values, he replies that because he is taking prescribed antidiabetic medication, he believes that he won't have all the diabetes complications that his father had. He further tells you that he did have his eyes checked by an optometrist to make sure his prescription was accurate but that because he was younger than 40 years old, he does not need intraocular pressure measurements. 3. Is he correct in thinking that an ophthalmologist visit is not necessary at this time? Explain your response.

Even if he did not have a vision problem, his risk for ophthalmic complications leading to blindness is high. Just having diabetes is a reason to been seen by an ophthalmologist rather than an optometrist (who is not a medical doctor). Depending on the control of his disease (which right now is not very controlled), coupled with long-standing vision problems, he should be seen yearly by an ophthalmologist. The ophthalmologist can determine whether a less frequent evaluation cycle would be appropriate.

After thrombolytic therapy, the nurse working in the cardiac catheterization laboratory would be alarmed to notice which sign? A 1-inch backup of blood in the IV tubing Facial drooping Partial thromboplastin time (PTT) 68 seconds Report of chest pressure during dye injection

Facial drooping During and after thrombolytic administration, the nurse observes for any indications of bleeding, including changes in neurologic status, which may indicate intracranial bleeding. A 1-inch backup of blood in the IV tubing may be related to IV positioning. If heparin is used, PTT reflects a therapeutic value. Reports of chest pressure during dye injection or stent deployment are considered an expected result of the procedure.

Ch.64 p. 1329, Patient-Centered Care During a clinic visit, you are reviewing the records of a 39-year-old patient who was diagnosed 5 years ago with type 2 diabetes. You discover that, although he has always been extremely near-sighted, he has not seen an ophthalmologist for 4 years. He has gained 12 lbs since his last visit a year ago. His laboratory values show a fasting blood glucose level of 96 mg/dL, an A1C of 8.2%, a total cholesterol of 322 mg/dL, and an LDL of 190 mg/dL. When you ask him about ophthalmology follow-up and point out his laboratory values, he replies that because he is taking prescribed antidiabetic medication, he believes that he won't have all the diabetes complications that his father had. He further tells you that he did have his eyes checked by an optometrist to make sure his prescription was accurate but that because he was younger than 40 years old, he does not need intraocular pressure measurements. 4. Is he correct in believing that taking antidiabetic medication will prevent complications of diabetes? Explain your response.

He is not correct in his thinking. Diabetes is a complex disorder and can only be controlled with a combination of antidiabetic medications and life-style changes that include nutrition therapy, maintenance of a healthy weight, blood pressure control, blood lipid control, and physical activity. The drugs are only part of the management plan. The fact that he was diagnosed at an earlier age and is taking medications is helpful, but without proper management, the complications of diabetes will not even be delayed let alone prevented.

Ch.64 p. 1332, Safety; Quality Improvement; Teamwork and Collaboration The patient is a 60 year-old-woman who is 1 day postoperative after a total knee replacement. She has type 2 diabetes and just recently was switched from oral antidiabetic drugs to an insulin regimen. She let her nurse know that her on-demand lunch has been ordered. The nurse tests her blood and gives her the prescribed short-acting insulin dose. An hour later, the physical therapist finds her pale, confused, and clammy. Her lunch tray is on her table and appears totally untouched. 1. Is her condition consistent with hyperglycemia or hypoglycemia? Explain your choice.

Her condition is consistent with hypoglycemia, especially because she received insulin about an hour ago. Manifestations of hypoglycemia include weakness; difficulty thinking; confusion; sweating; and cool, pale skin, and manifestations of hyperglycemia include warm, moist skin and possible fruity breath odor. Hyperglycemia does not change level of consciousness until it is severe.

Ch.64 p. 1329, Patient-Centered Care During a clinic visit, you are reviewing the records of a 39-year-old patient who was diagnosed 5 years ago with type 2 diabetes. You discover that, although he has always been extremely near-sighted, he has not seen an ophthalmologist for 4 years. He has gained 12 lbs since his last visit a year ago. His laboratory values show a fasting blood glucose level of 96 mg/dL, an A1C of 8.2%, a total cholesterol of 322 mg/dL, and an LDL of 190 mg/dL. When you ask him about ophthalmology follow-up and point out his laboratory values, he replies that because he is taking prescribed antidiabetic medication, he believes that he won't have all the diabetes complications that his father had. He further tells you that he did have his eyes checked by an optometrist to make sure his prescription was accurate but that because he was younger than 40 years old, he does not need intraocular pressure measurements. 1. How should you interpret his laboratory values in terms of his personal glucose regulation?

His fasting blood glucose level is acceptable and indicates that he has controlled his diabetes during the past 24 hours. However, his hemoglobin A1C is high, indicating that his overall control for the past several months is poor. It is possible that the current medication regimen is not sufficient to manage his disease. The fact that his weight is increasing rather than decreasing and that his blood lipid levels are quite high hint that his nutrition therapy is probably not being followed.

p. 813, Safety; Teamwork and Collaboration The patient is a 44-year-old chemical plant foreman who developed acute myelogenous leukemia 6 months ago. His initial therapy was successful, and he is scheduled to have a stem cell transplant (bone marrow) with his identical twin brother as the donor. His brother lives in the same city and is a professor at a local university. The patient is very grateful that his brother will donate bone marrow and states that he is certain that he has no risk for infection during the procedure because his brother is his identical twin. 4. Which, if any, complications (and why) are still possible even with a donor who is an identical sibling?

In addition to infection, other pancytopenia problems can occur until full engraftment. The patient will be at risk for bleeding complications and may need both red blood cell and platelet transfusions after transplantation. It is possible that engraftment may not occur for many reasons, the most common of which is insufficient stem cells transplanted. Although HSCT can help cure leukemia, it is still possible that not all cancerous bone marrow cells were killed and could come back.

Which characteristics place women at high risk for myocardial infarction (MI)? (Select all that apply.) Premenopausal Increasing age Family history Abdominal obesity Breast cancer

Increasing age Family history Abdominal obesity Increasing age is a risk factor, especially after 70 years. Family history is a significant risk factor in both men and women. A large waist size and/or abdominal obesity are risk factors for both metabolic syndrome and MI. Premenopausal women are not at higher risk for MI, and breast cancer is not a risk factor for MI.

p. 802, Ethical/Legal A 27-year-old African-American man in sickle cell crisis is a patient on your unit. During report, one of the nurses from the previous shift mentions that she withheld the regularly scheduled IV opioid pain medication during the night because she had taken care of this patient a year ago and believes that he is a "drug seeker." 3. Can a patient with sickle cell disease become addicted to opioids?

It is possible for any patient with chronic pain that may have acute exacerbations to become addicted to opioids. However, this should never be assumed. Even if addiction did occur, the pain is real and must be appropriately managed, including the use of opioid analgesics.

Ch.62 p. 1276, Patient-Centered Care; Quality Improvement; Safety The patient is a 32-year-old woman admitted to your unit after surgery for fractures of the left arm and leg resulting from a car crash. She is awake and able to verify her medical history of rheumatoid arthritis and her usual daily medications. These are 10 mg of prednisone, naproxen 800 mg twice daily, oral contraceptives, calcium 600 mg, and one multiple vitamin tablet. All of these are prescribed for her to receive during her hospitalization. She is concerned about pain management and how long the recovery will be for the fractures. She is friendly, somewhat anxious, asks many questions, and wants to do "her part" to ensure good recovery. Over the next 4 days, she has become quieter, mumbles that her head and stomach hurt, and now does not recognize the assistant who has been providing her daily care. When she receives her medications, she has difficulty picking them up. The nursing assistant remarks that taking her pulse is difficult because it is so slow and irregular. When you assess her, she is so weak that she is unable to lift her arm for a blood pressure check. Her blood pressure is 92/50, which is down from the 128/84 reading on admission. You also verify that her heart beat is slow and irregular. 1. What other assessment data should you obtain immediately and why?

Listen to her apical pulse to assess the true heart rate. With some dysrhythmias, especially if she is having premature contraction, the radial pulse can be very different from the apical pulse. Assess her oxygen saturation to determine whether cardiac function is adequate for the moment or whether the rapid response team is needed now. (Cardiac arrest is possible because of hyperkalemia.) Perform a finger stick blood glucose analysis immediately to determine whether she is hypoglycemic.

Ch.63 p. 1295, Patient-Centered Care; Safety The patient, a 45-year-old former school teacher, is residing in a skilled nursing facility to recover from a tibia-fibula fracture that is being managed with an external fixation system. On admission 2 weeks ago, she told you that she felt she was "getting old too fast." She explained that she had gained 54 pounds in the previous 6 months, had no energy, was often constipated, and was always cold. She teared up and said that her ability to concentrate was so bad that not only could she no longer help her high school children with their homework, but that she didn't recognize the step hazard that caused her to fall and break her ankle. Today the nursing assistant assigned to her care reports that the patient's pulse is only 42 beats per minute and that her temperature was 96° F even with two blankets. When you enter her room, she is sleeping with an untouched breakfast tray on her table. 4. What manifestations of hypothyroidism are in her history and present during this assessment?

Many of the problems that she has listed over the past 6 months are manifestations of hypothyroidism that is slowly getting worse. These include her weight gain, having no energy, being constipated frequently, always feeling cold, and having difficulty with concentration. It is also possible that her ankle fractured more easily because of hypothyroidism-induced bone thinning.

14. An elderly patient has been in bed for several days after a fall. The nurse encourage ambulation to stimulate the movement of urine through the ureter by what phenomenon? a. Peristalsis b. Gravity c. Pelvic pressure d. Backflow

a

The visiting nurse is seeing a client postoperative for coronary artery bypass graft. Which nursing action should be performed first? Assess coping skills. Assess for postoperative pain at the client's incision site. Monitor for dysrhythmias. Monitor mental status.

Monitor for dysrhythmias. Dysrhythmias are the leading cause of prehospital death; the nurse should monitor the client's heart rhythm. Assessing mental status, coping skills, or postoperative pain is not the priority for this client.

When planning care for a client in the emergency department, which interventions are needed in the acute phase of myocardial infarction? (Select all that apply.) Morphine sulfate Oxygen Nitroglycerin Naloxone Acetaminophen Verapamil (Calan, Isoptin)

Morphine sulfate Oxygen Nitroglycerin Morphine is needed to reduce oxygen demand, preload, pain, and anxiety, and nitroglycerin is used to reduce preload and chest pain. Administering oxygen will increase available oxygen for the ischemic myocardium. Naloxone is a narcotic antagonist that is used for overdosage of opiates, not for MI. Acetaminophen may be used for headache related to nitroglycerin. Because of negative inotropic action, calcium channel blockers such as verapamil are used for angina, not for MI.

p. 813, Safety; Teamwork and Collaboration The patient is a 44-year-old chemical plant foreman who developed acute myelogenous leukemia 6 months ago. His initial therapy was successful, and he is scheduled to have a stem cell transplant (bone marrow) with his identical twin brother as the donor. His brother lives in the same city and is a professor at a local university. The patient is very grateful that his brother will donate bone marrow and states that he is certain that he has no risk for infection during the procedure because his brother is his identical twin. 2. Is the patient correct in assuming that he has no risk for infection because the donor is his twin brother? Provide a rationale for your response.

No, the patient is not correct. The patient will still have to undergo a conditioning regimen to clear his own, potentially leukemic, cells. Thus he will still be profoundly immunosuppressed and at great risk for infection unless and until his brother's stem cells properly engraft and grow in his body.

Ch.64 p. 1335, Prioritization, Delegation, and Supervision The patient, a 21-year-old college student, was brought to the emergency department (ED) by his roommate. He reports abdominal pain, polyuria for the past 2 days, vomiting several times prior to arrival, and extreme thirst. He appears flushed, and his lips and mucous membranes are dry and cracked. His skin turgor is poor. He demonstrates deep rapid respirations; there is a fruit odor to his breath. He has type 1 diabetes and "may have skipped a few doses of insulin because of cramming for final exams." He is alert and talking but is having trouble focusing on your questions. Vital signs: Blood pressure 110/60 Pulse 110/min Respirations 32/min Temperature 100.8° F Fingerstick glucose 485 mg/dL Oxygen saturation 99% 6. What intravenous (IV) solution do you anticipate the ED physician will order for initial fluid replacement?

Normal saline (0.9% sodium chloride) is the first fluid used to correct dehydration in most adults with diabetic ketoacidosis.

A client comes to the emergency department with chest discomfort. Which action does the nurse perform first? Administers oxygen therapy Obtains the client's description of the chest discomfort Provides pain relief medication Remains calm and stays with the client

Obtains the client's description of the chest discomfort A description of the chest discomfort must be obtained first, before further action can be taken. Neither oxygen therapy nor pain medication is the first priority in this situation; an assessment is needed first. Remaining calm and staying with the client are important, but are not matters of highest priority.

Ch.64 p. 1335, Prioritization, Delegation, and Supervision The patient, a 21-year-old college student, was brought to the emergency department (ED) by his roommate. He reports abdominal pain, polyuria for the past 2 days, vomiting several times prior to arrival, and extreme thirst. He appears flushed, and his lips and mucous membranes are dry and cracked. His skin turgor is poor. He demonstrates deep rapid respirations; there is a fruit odor to his breath. He has type 1 diabetes and "may have skipped a few doses of insulin because of cramming for final exams." He is alert and talking but is having trouble focusing on your questions. Vital signs: Blood pressure 110/60 Pulse 110/min Respirations 32/min Temperature 100.8° F Fingerstick glucose 485 mg/dL Oxygen saturation 99% 2. Should you apply oxygen at this time? Why or why not?

Oxygen is not needed. Although his respiratory rate is above normal, he is not hypoxemic. Applying oxygen would serve no useful purpose.

Prompt pain management with myocardial infarction is essential for which reason? The discomfort will increase client anxiety and reduce coping. Pain relief improves oxygen supply and decreases oxygen demand. Relief of pain indicates that the MI is resolving. Pain medication should not be used until a definitive diagnosis has been established.

Pain relief improves oxygen supply and decreases oxygen demand. The focus of pain relief is on reducing myocardial oxygen demand. Chest discomfort will increase anxiety, but it may not affect coping. Relief of pain is secondary to the use of opiates or indicates that the tissue infarction is complete. Although it used to be true that pain medication was not to be used for undiagnosed abdominal pain, this does not relate to MI.

14. In caring for a patient with acute leukemia, what is the priority collaborative problem? a. Protecting the patient from infection b. Minimizing the side effects of chemotherapy c. Controlling the patient's pain d. Assisting the patient to cope with fatigue

a

The nurse is caring for a client in phase 1 cardiac rehabilitation. Which activity does the nurse suggest? The need to increase activities slowly at home Planning and participating in a walking program Placing a chair in the shower for independent hygiene Consultation with social worker for disability planning

Placing a chair in the shower for independent hygiene Phase 1 begins with the acute illness and ends with discharge from the hospital; it focuses on promoting rest and allowing clients to improve their activities of daily living based on their abilities. Phase 2 begins after discharge and continues through convalescence at home, including consultation with a social worker for long-term planning; it consists of achieving and maintaining a vital and productive life while remaining within the limits of the heart's ability to respond to increases in activity and stress. Phase 3 refers to long-term conditioning, such as a walking program.

The nurse is caring for a client 36 hours after coronary artery bypass grafting, with a priority problem of intolerance for activity related to imbalance of myocardial oxygen supply and demand. Which finding causes the nurse to terminate an activity and return the client to bed? Pulse 60 beats/min and regular Urinary frequency Incisional discomfort Respiratory rate 28 breaths/min

Respiratory rate 28 breaths/min Tachypnea and tachycardia reflect activity intolerance; activity should be terminated. Pulse 60 beats/min and regular is a normal finding. Urinary frequency may indicate infection or diuretic use, but not activity intolerance. Pain with activity after surgery is anticipated; pain medication should be available.

Ch.63 p. 1295, Patient-Centered Care; Safety The patient, a 45-year-old former school teacher, is residing in a skilled nursing facility to recover from a tibia-fibula fracture that is being managed with an external fixation system. On admission 2 weeks ago, she told you that she felt she was "getting old too fast." She explained that she had gained 54 pounds in the previous 6 months, had no energy, was often constipated, and was always cold. She teared up and said that her ability to concentrate was so bad that not only could she no longer help her high school children with their homework, but that she didn't recognize the step hazard that caused her to fall and break her ankle. Today the nursing assistant assigned to her care reports that the patient's pulse is only 42 beats per minute and that her temperature was 96° F even with two blankets. When you enter her room, she is sleeping with an untouched breakfast tray on her table. 3. What indications do you have that the changes in her health status are not related to complications of her fractured ankle?

Some of her health status changes could be related to her fracture, specifically sepsis and pulmonary embolism. However, because she has had some of these changes gradually over several weeks, they are more likely to be related to reduced thyroid function. By assessing her cardiac and respiratory status, you are also assessing for manifestations of sepsis. Usually, respirations are elevated for either pulmonary embolism or for sepsis. Heart rate is usually fast and thready with these two problems.

p. 802, Ethical/Legal A 27-year-old African-American man in sickle cell crisis is a patient on your unit. During report, one of the nurses from the previous shift mentions that she withheld the regularly scheduled IV opioid pain medication during the night because she had taken care of this patient a year ago and believes that he is a "drug seeker." 4. What can you do to prevent an incident like this one from happening again?

Speak to the nurse manager and the patient's health care provider about the incident. It is likely that this nurse does not understand the pathophysiology of sickle cell crisis or its management. There may be other nurses who have similar views. Arrange for a sickle cell disease and pain specialist to hold mandatory inservices on this topic for the entire staff on this unit.

p. 802, Ethical/Legal A 27-year-old African-American man in sickle cell crisis is a patient on your unit. During report, one of the nurses from the previous shift mentions that she withheld the regularly scheduled IV opioid pain medication during the night because she had taken care of this patient a year ago and believes that he is a "drug seeker." 1. What is your first action?

Stop report and go and assess the patient for pain and administer the prescribed scheduled opioid. The pain during crisis is intense. Pain medication, including opioids, must be given on a scheduled basis and not as needed, and they should not be withheld.

The nurse is assessing a client with chest pain to evaluate whether the client is suffering from angina or myocardial infarction (MI). Which symptom is indicative of an MI? Chest pain brought on by exertion or stress Substernal chest discomfort occurring at rest Substernal chest discomfort relieved by nitroglycerin or rest Substernal chest pressure relieved only by opioids

Substernal chest pressure relieved only by opioids Substernal chest pressure relieved only by opioids is typically indicative of MI. Substernal chest discomfort that occurs at rest is not necessarily indicative of MI; it could be a sign of unstable angina. Both chest pain brought on by exertion or stress and substernal chest discomfort relieved by nitroglycerin or rest are indicative of angina.

After receiving change-of-shift report in the coronary care unit, which client does the nurse assess first? The client with acute coronary syndrome who has a 3-pound weight gain and dyspnea The client with percutaneous coronary angioplasty who has a dose of heparin scheduled The client who had bradycardia after a myocardial infarction and now has a paced heart rate of 64 beats/min A client who has first-degree heart block, rate 68 beats/min, after having an inferior myocardial infarction

The client with acute coronary syndrome who has a 3-pound weight gain and dyspnea Dyspnea and weight gain are symptoms of left ventricular failure and pulmonary edema; this client needs prompt intervention. A scheduled heparin dose does not take priority over dyspnea; it can be administered after the client with dyspnea is taken care of. The client with a pacemaker and a normal heart rate is not in danger. First-degree heart block is rarely symptomatic, and the client has a normal heart rate; the client with dyspnea should be seen first.

Ch.62 p. 1276, Patient-Centered Care; Quality Improvement; Safety The patient is a 32-year-old woman admitted to your unit after surgery for fractures of the left arm and leg resulting from a car crash. She is awake and able to verify her medical history of rheumatoid arthritis and her usual daily medications. These are 10 mg of prednisone, naproxen 800 mg twice daily, oral contraceptives, calcium 600 mg, and one multiple vitamin tablet. All of these are prescribed for her to receive during her hospitalization. She is concerned about pain management and how long the recovery will be for the fractures. She is friendly, somewhat anxious, asks many questions, and wants to do "her part" to ensure good recovery. Over the next 4 days, she has become quieter, mumbles that her head and stomach hurt, and now does not recognize the assistant who has been providing her daily care. When she receives her medications, she has difficulty picking them up. The nursing assistant remarks that taking her pulse is difficult because it is so slow and irregular. When you assess her, she is so weak that she is unable to lift her arm for a blood pressure check. Her blood pressure is 92/50, which is down from the 128/84 reading on admission. You also verify that her heart beat is slow and irregular. 2. What is the most likely cause of the changes in this patient's physical and mental status?

The most likely cause is acute adrenal insufficiency as a result of increased cortisol needs related to the stress of surgery and injury. Because she has been on prednisone long term, she has some degree of adrenal suppression and cannot increase the extra cortisol needed during the additional stress. Although she is receiving 10 mg of prednisone daily, it is not enough for her current needs.

24. The nurse is counseling a patient with recurrent symptomatic UTIs about dietary therapy. What information does the nurse give to the patient? a. Drink 50 mL of concentrated cranberry juice every day. b. Low consumption of protein may prevent recurrent UTIs. c. Caffeine, carbonated beverages, and tomato products cause UTI. d. Cranberry tablets are more effective than juice or fluids.

a

Ch.64 p. 1335, Prioritization, Delegation, and Supervision The patient, a 21-year-old college student, was brought to the emergency department (ED) by his roommate. He reports abdominal pain, polyuria for the past 2 days, vomiting several times prior to arrival, and extreme thirst. He appears flushed, and his lips and mucous membranes are dry and cracked. His skin turgor is poor. He demonstrates deep rapid respirations; there is a fruit odor to his breath. He has type 1 diabetes and "may have skipped a few doses of insulin because of cramming for final exams." He is alert and talking but is having trouble focusing on your questions. Vital signs: Blood pressure 110/60 Pulse 110/min Respirations 32/min Temperature 100.8° F Fingerstick glucose 485 mg/dL Oxygen saturation 99% 5. In caring for this patient, what immediate intervention do you anticipate the ED physician will order to be performed first? Provide a rationale for your choice.

The patient needs IV fluids to correct fluid deficit that places him at risk for hypovolemic shock. Also, he needs carefully regulated insulin therapy at this time, which is best accomplished by the IV route. Subcutaneous insulin does not absorb fast enough and is inappropriate for emergency situations.

Ch.64 p. 1332, Safety; Quality Improvement; Teamwork and Collaboration The patient is a 60 year-old-woman who is 1 day postoperative after a total knee replacement. She has type 2 diabetes and just recently was switched from oral antidiabetic drugs to an insulin regimen. She let her nurse know that her on-demand lunch has been ordered. The nurse tests her blood and gives her the prescribed short-acting insulin dose. An hour later, the physical therapist finds her pale, confused, and clammy. Her lunch tray is on her table and appears totally untouched. 4. What could be done on this nursing care unit to prevent such an incident from happening again?

The patient should receive more education about the relationship between insulin and eating. The unit needs to establish guidelines or policies about premeal insulin administration. Perhaps it should not be administered until the tray is actually in the patient's possession and the patient is ready to eat it. Also, whenever short-acting insulin is given, the nurse giving it should evaluate the patient within 20 minutes.

Ch.64 p. 1329, Patient-Centered Care During a clinic visit, you are reviewing the records of a 39-year-old patient who was diagnosed 5 years ago with type 2 diabetes. You discover that, although he has always been extremely near-sighted, he has not seen an ophthalmologist for 4 years. He has gained 12 lbs since his last visit a year ago. His laboratory values show a fasting blood glucose level of 96 mg/dL, an A1C of 8.2%, a total cholesterol of 322 mg/dL, and an LDL of 190 mg/dL. When you ask him about ophthalmology follow-up and point out his laboratory values, he replies that because he is taking prescribed antidiabetic medication, he believes that he won't have all the diabetes complications that his father had. He further tells you that he did have his eyes checked by an optometrist to make sure his prescription was accurate but that because he was younger than 40 years old, he does not need intraocular pressure measurements. 5. How do you propose to assist this patient in managing his diabetes?

The patient's comments and the laboratory data indicate that he does not understand the disease, its consequences, management techniques, and his role in the management plan. His issues are going to require more than your intervention, although you can get this started. Patient-centered evaluation and management with the entire diabetes management team is needed as soon as possible. He will need further testing to assess for early-stage complications and possible changes to his medication regimen. You should start by asking what he knows about the disease and correct any misunderstandings. Bringing in a diabetes educator, the health care provider and registered dietitian is really needed now. If classes are available, he should be strongly encouraged to start them. If he has a partner, try to include her or him in this process. Essentially, this patient requires that the team treat him as if he had just been newly diagnosed with diabetes.

p. 813, Safety; Teamwork and Collaboration The patient is a 44-year-old chemical plant foreman who developed acute myelogenous leukemia 6 months ago. His initial therapy was successful, and he is scheduled to have a stem cell transplant (bone marrow) with his identical twin brother as the donor. His brother lives in the same city and is a professor at a local university. The patient is very grateful that his brother will donate bone marrow and states that he is certain that he has no risk for infection during the procedure because his brother is his identical twin. 3. Which, if any, complications of stem cell transplantation are reduced or eliminated by having an identical sibling donate the stem cells?

The risk for GVHD is greatly reduced, although not completely eliminated, because the transplanted stem cells are identical to his own cells. The risk for veno-occlusive disease is likewise greatly reduced.

p. 818, Prioritization, Delegation, and Supervision The patient is a 52-year-old woman who has undergone an autologous stem cell transplantation for non-Hodgkin's lymphoma. She is recovering, and her white blood cell count is improving but still very low. She remains on neutropenic precautions. The LPN reports that the patient's heart rate, respiratory rate, temperature, and blood pressure are all elevated. 1. Which vital sign finding would you report to the health care provider immediately and why?

The temperature elevation, no matter how slight, in a patient with neutropenia indicates infection until it has been ruled out. This elevation should be reported immediately, and you need to take the standard neutropenic interventions of full assessment, obtaining appropriate specimens for culture, obtaining chest radiography, and starting antibiotic therapy.

Ch.62 p. 1276, Patient-Centered Care; Quality Improvement; Safety The patient is a 32-year-old woman admitted to your unit after surgery for fractures of the left arm and leg resulting from a car crash. She is awake and able to verify her medical history of rheumatoid arthritis and her usual daily medications. These are 10 mg of prednisone, naproxen 800 mg twice daily, oral contraceptives, calcium 600 mg, and one multiple vitamin tablet. All of these are prescribed for her to receive during her hospitalization. She is concerned about pain management and how long the recovery will be for the fractures. She is friendly, somewhat anxious, asks many questions, and wants to do "her part" to ensure good recovery. Over the next 4 days, she has become quieter, mumbles that her head and stomach hurt, and now does not recognize the assistant who has been providing her daily care. When she receives her medications, she has difficulty picking them up. The nursing assistant remarks that taking her pulse is difficult because it is so slow and irregular. When you assess her, she is so weak that she is unable to lift her arm for a blood pressure check. Her blood pressure is 92/50, which is down from the 128/84 reading on admission. You also verify that her heart beat is slow and irregular. 4. What specifically would be the nurse's role in preventing this problem?

This type of adrenal insufficiency develops over a period of days. Monitoring trends for level of consciousness, blood pressure, and heart rate and rhythm should be something all nurses do on every patient. It is very likely that changes were present earlier and not recognized.

24. Which disorder could be a complication from AKI? a. Heart failure b. Diabetes mellitus c. Kidney cancer d. Compartment syndrome

a

The nurse is preparing to teach a client that metabolic syndrome can increase the risk for myocardial infarction (MI). Which signs of metabolic syndrome should the nurse include in the discussion? (Select all that apply.) Truncal obesity Hypercholesterolemia Elevated homocysteine levels Glucose intolerance Client taking losartan (Cozaar)

Truncal obesity Hypercholesterolemia Glucose intolerance Client taking losartan (Cozaar) A large waist size (excessive abdominal fat causing central obesity)—40 inches (102 cm) or greater for men, 35 inches (88 cm) or greater for women—is a sign of metabolic syndrome. Decreased high-density lipoprotein cholesterol (HDL-C) (usually with high low-density lipoprotein cholesterol)—HDL-C less than 40 mg/dL for men or less than 50 mg/dL for women—or taking an anticholesterol drug is a sign of metabolic syndrome. Increased fasting blood glucose (caused by diabetes, glucose intolerance, or insulin resistance) is included in the constellation of metabolic syndrome. Blood pressure greater than 130/85 mm Hg or taking antihypertensive medication indicates metabolic syndrome. Although elevated homocysteine levels may predispose to atherosclerosis, they are not part of metabolic syndrome.

Ch.64 p. 1335, Prioritization, Delegation, and Supervision The patient, a 21-year-old college student, was brought to the emergency department (ED) by his roommate. He reports abdominal pain, polyuria for the past 2 days, vomiting several times prior to arrival, and extreme thirst. He appears flushed, and his lips and mucous membranes are dry and cracked. His skin turgor is poor. He demonstrates deep rapid respirations; there is a fruit odor to his breath. He has type 1 diabetes and "may have skipped a few doses of insulin because of cramming for final exams." He is alert and talking but is having trouble focusing on your questions. Vital signs: Blood pressure 110/60 Pulse 110/min Respirations 32/min Temperature 100.8° F Fingerstick glucose 485 mg/dL Oxygen saturation 99% 4. Your work plan includes checking hourly vital signs, assessing blood glucose levels, updating the roommate about the patient's condition, and measuring the patient's emesis. Which task(s) is (are) appropriate to assign to the new nursing assistant? Provide a rationale for your choices.

Vital signs and measuring and recording output are within the scope of duties for the nursing assistant. Releasing information should not be done by the nursing assistant because of confidentiality issues. The RN should decide how to convey information to friends and family who are waiting keeping HIPAA standards in mind. Checking blood glucose is usually accomplished with a fingerstick, which is usually within the scope of practice for the experienced nursing assistant but not for one who is new.

The nurse is interviewing a patient who has iron deficiency anemia. Which symptom is the patient most likely to report? a. Fatigue b. Nights sweats c. Calf pain d. Blood in urine

a

p. 813, Safety; Teamwork and Collaboration The patient is a 44-year-old chemical plant foreman who developed acute myelogenous leukemia 6 months ago. His initial therapy was successful, and he is scheduled to have a stem cell transplant (bone marrow) with his identical twin brother as the donor. His brother lives in the same city and is a professor at a local university. The patient is very grateful that his brother will donate bone marrow and states that he is certain that he has no risk for infection during the procedure because his brother is his identical twin. 1. What type of class of stem cell transplant would this procedure be considered?

When the donor is an identical sibling, all major and minor tissue antigens are a perfect match, not just the six standard major ones. Therefore, this is considered a syngeneic transplant.

Ch.63 p. 1295, Patient-Centered Care; Safety The patient, a 45-year-old former school teacher, is residing in a skilled nursing facility to recover from a tibia-fibula fracture that is being managed with an external fixation system. On admission 2 weeks ago, she told you that she felt she was "getting old too fast." She explained that she had gained 54 pounds in the previous 6 months, had no energy, was often constipated, and was always cold. She teared up and said that her ability to concentrate was so bad that not only could she no longer help her high school children with their homework, but that she didn't recognize the step hazard that caused her to fall and break her ankle. Today the nursing assistant assigned to her care reports that the patient's pulse is only 42 beats per minute and that her temperature was 96° F even with two blankets. When you enter her room, she is sleeping with an untouched breakfast tray on her table. 2. Should oxygen be applied? Why or why not?

Yes, oxygen should be applied. Her heart rate is so low that she is not able to maintain good oxygenation of vital organs. Applying additional oxygen can help oxygenate those tissues. Death from hypoxia is a strong possibility.

Ch.64 p. 1329, Patient-Centered Care During a clinic visit, you are reviewing the records of a 39-year-old patient who was diagnosed 5 years ago with type 2 diabetes. You discover that, although he has always been extremely near-sighted, he has not seen an ophthalmologist for 4 years. He has gained 12 lbs since his last visit a year ago. His laboratory values show a fasting blood glucose level of 96 mg/dL, an A1C of 8.2%, a total cholesterol of 322 mg/dL, and an LDL of 190 mg/dL. When you ask him about ophthalmology follow-up and point out his laboratory values, he replies that because he is taking prescribed antidiabetic medication, he believes that he won't have all the diabetes complications that his father had. He further tells you that he did have his eyes checked by an optometrist to make sure his prescription was accurate but that because he was younger than 40 years old, he does not need intraocular pressure measurements. 2. Should you address his weight gain? Why or why not?

Yes, you should address his weight. A major pathophysiological problem with type 2 diabetes is insulin resistance. Increasing weight correlates to greater insulin resistance. Even modest weight loss can improve the sensitivity of insulin receptors to insulin. The excess weight is contributing even more to his risk for cardiovascular events, as evidenced by the high blood lipid levels.

12. The nurse is caring for several patients on a medical-surgical unit. None of the patient currently has any acute or chronic kidney problems. Which patient has the greatest risk to develop AKI? a. 73-year-old male who has hypertension and peripheral vascular disease b. 32-year-old female who is pregnant and has gestational diabetes c. 49-year-old male who is obese and has a history of skin cancer d. 23-year-old female who has been treated for a urinary tract infection

a

10. A patient has polycythemia vera. Which action by the UAP requires intervention by the supervising nurse? a. Assisting the patient to floss his teeth b. Using an electric shaver on the patient c. Using a soft-bristled toothbrush on the patient d. Assisting the patient to don support hose

a

11. The nurse is teaching a woman how to prevent UTIs. What information does the nurse include? a. Clean the perineal area from front to back. b. Always use a condom if spermicides are used for contraception. c. Obtain prescription for estrogen for vaginal dryness. d. Avoid urinary stasis by urinating every 6 to 8 hours.

a

13. Mastering voluntary micturition is a normal developmental task for which person? a. A healthy 20-month-old toddler b. A 56-year-old woman with stress incontinence c. A healthy 8-year-old child d. A 25-year-old with a spinal cord injury

a

14. The nurse is caring for a postoperative patient and is evaluating the patient's intake and output as a measure to prevent AKI. The patient weighs 60 kilograms and has produced 180 mL of urine in the past 4 hours. What should the nurse do? a. Perform other assessments related to fluid status and record the output. b. Call the health care provider and obtain an order for a fluid bolus. c. Encourage the patient to drink more fluid, so that the output is increased. d. Compare the patient's weight to baseline to determine fluid retention.

a

15. A patient with PKD usually experiences constipation. What does the nurse recommend? a. Increased dietary fiber and increased fluids b. Decreased dietary fiber and laxatives c. Daily laxatives and increased exercise e. Ta-water enemas and fiber supplements

a

15. Which renal change associated with aging does the nurse expect an older adult patient to report? a. Nocturnal polyuria b. Micturition c. Hematuria d. Dysuria

a

16. A patient with PKD has nocturia. What does the nurse encourage the patient to do? a. Drink at least 2 liters of fluid daily. b. Restrict fluid in the evening. c. Drink 1000 mL early in the morning d. Add a pinch of salt to water in the evenings.

a

16. According to the RIFLE classification (Risk, Injury, Failure, Loss, End-stage kidney failure). How would the nurse interpret the following data? Serum creatinine increased x 1.5 or glomerular filtration rate (GFR) decrease > 25 %; Urine output is <0.5 mL/kg/hr for ≥6 hours. a. Risk stage b. Injury stage c. Failure stage d. End-stage kidney disease (ESKD)

a

17. A patient reports symptoms indicating a UTI. Results from which diagnostic test will verify a UTI? a. Urinalysis to test for leukocyte esterase and nitrate b. Urinalysis for glucose and red blood cells c. Urinalysis to test for ketones and protein d. Urinalysis for pH and specific gravity

a

17. For the patient with PKD, which antihypertensive medication may be used because it helps control the cell growth aspects of PKD and reduce microalbuminuria? a. Angiotensin-converting enzyme inhibitors b. Beta blockers c. Calcium channel blockers d. Vasodilators

a

17. Impairment in the thirst mechanism associated with aging makes an older adult patient more vulnerable to which disorder? a. Hypernatremia b. Hypocalcemia c. Hyperkalemia d. Hypoglycemia

a

17. Which disorder creates the highest risk for the patient to develop infection? a. Sickle cell crisis b. Vitamin B12 deficiency anemia c. Polycythemia vera d. Thrombocytopenia

a

18. The nurse is talking to a group of older women about changes in the urinary system related to aging. What symptom is likely to be the common concern for this group? a. Incontinence b. Hematuria c. Retention d. Dysuria

a

18. The patient who is prescribed methimazole (Tapazole) 4 mg orally every 8 hours tells the nurse that his heart rate is slow (60/minute), he has gained 7 pounds, and he wears a sweater even on warm days. What does the nurse suspect? a. Indications of hypothyroidism will require a lower dosage. b. Indications of hypothyroidism will require a higher dosage. c. Indications of hyperthyroidism will require a lower dosage. d. Indications of hyperthyroidism will require a higher dosage.

a

18. Which medication increases the risk for the patient to develop infection? a. Steroids b. Aspirin c. Iron solutions d. Heparin

a

19. The nurse is teaching a patient about self-care measures to prevent UTIs. Which daily fluid intake does the nurse recommend to the patient to prevent a bladder infection? a. 2 to 3 L of water b. 3 to 6 glasses of iced tea c. 4 to 6 cups of electrolyte fluid d. 3 to 4 glasses of juice

a

2. The community health nurse is designing programs to reduce kidney problems and kidney injury among the general public. In order to do so, the nurse targets health promotion and compliance with therapy for people with which conditions? a. Diabetes mellitus and hypertension b. Frequent episodes of sexually transmitted disease c. Osteoporosis and other bone diseases d. Gastroenteritis and poor eating habits

a

20. The health care team is using a collaborative and interdisciplinary approach to design a treatment plan for a patient with PKD. What is the top priority? a. Controlling hypertension b. Preventing rupture of cysts c. Providing genetic counseling d. Identifying community resources

a

20. The nurse is caring for a patient with AKI and notes a trend of increasingly elevated BUN levels. HOw does the nurse interpret this information? a. Breakdown of muscle for protein which leads to an increase in azotemia b. Sign of urinary retention and decreased urinary output c. Expected trend that can be reversed by increasing dietary d. Ominous sign of impending irreversible kidney failure

a

21. A patient with a hypophysectomy can postoperatively experience transient DI. Which manifestation alerts the nurse to this problem? a. Output much greater than intake b. Change in mental status indicating confusion c. Laboratory results indicating hyponatremia d. Nonpitting edema

a

21. The home care nurse is visiting a patient who had a stem cell transplant. Which observation by the nurse requires immediate action? a. The patient's grandson is visiting after receiving a MMR vaccine. b. The patient bumps his toe on a chair and applies pressure to the toe for 10 minutes. c. The patient with a platelet count of 48,000/mm3 follows platelet precautions. d. The patient avoids going out to grocery shop in the winter months.

a

22. A patient sustained extensive burns and depletion of vascular volume. The nurse expects which changes in vital signs and urinary function? a. Decreased urine output, hypotension, tachycardia b. Increased urine output, hypertension, tachycardia c. Bradycardia, hypotension, polyuria d. Dysrhythmias, hypertension, oliguria

a

24. The new registered nurse is giving a blood transfusion to a patient. Which statement by the new nurse indicates the need for action by the supervising nurse? a. "I will complete red blood cell transfusion within 6 hours." b. "I will check the patient verification with another registered nurse." c. "I will use normal saline solution to dilute the blood." d. "I will remain with the patient for the first 15 to 30 minutes of the infusion."

a

25. A patient reports straining to pass very small amounts of urine today, despite a normal fluid intake, and reports having the urge to urinate. The nurse palpates the bladder and finds that it is distended. Which condition is most likely to be associated with these findings? a. Urethral stricture b. Hydroureter c. Hydronephrosis d. PKD

a

25. After a visit to the health care provider's office, a patient is diagnosed with general thyroid enlargement and elevated thyroid hormone level. Which condition do these findings indicate? a. Hyperthyroidism and goiter b. Hypothyroidism and goiter c. Nodules on the parathyroid gland d. Thyroid or parathyroid cancer

a

26. A patient is diagnosed with hydronephrosis. What is a complication that could result from this condition? a. Damage to the nephrons b. Kidney cancer c. Kidney stone d. Structural defects

a

26. A patient's recurrent cystitis appears to be related to sexual intercourse. The patient seems uncomfortable talking about the situation. What communication technique does the nurse use to assist the patient? a. Have a frank and sensitive discussion with the patient. b. Give the patient reading material with instructions to call with any questions. c. Call the patient's partner and invite the partner to discuss the problem. d. Talk about other topics until the patient feels more comfortable.

a

26. The nurse is caring for a patient with AKI who does not have signs or symptoms of fluid overload. A fluid challenge is performed to promote kidney perfusion by doing what? a. Administering normal saline 500 to 1000 mL infused over 1 hour b. Administering drugs to suppress aldosterone release c. Instilling warm, sterile normal saline into the bladder d. Having the patient drink several large glasses or water

a

27. The nurse is caring for a patient with DI. What is the priority goal of collaborative care? a. Correct the water metabolism problem. b. Control blood sugar and blood pH. c. Measure urine output, specific gravity, and osmolality hourly. d. Monitor closely for respiration distress.

a

27. Which clinical manifestation in a patient with an obstruction in the urinary system is associated specifically with a hydronephrosis? a. Flank asymmetry b. Chills and fever c. Urge incontinence d. Decreased urine volume

a

28. Which medication is used to treat DI? a. Desmopressin acetate (DDAVP) b. Lithium (Eskalith) c. Vasopressin (Pitressin) d. Demeclocycline (Declomycin)

a

29. A patient has the following assessment findings: elevated TSH level, low T3 and T4 levels, difficulty with memory, lethargy, and muscle stiffness. These are clinical manifestations of which disorder? a. Hypothyroidism b. Hyperthyroidism c. Hypoparathyroidism d. Hyperparathyroidism

a

29. A patient with AKI has a high rate of catabolism. What is this related to? a. Increased levels of catecholamines, corticosol, and glucagon b. Inability to excrete excess electrolytes c. Conversion of body fat into glucose d. Presence of retained nitrogenous wastes

a

30. A patient had a nephrostomy and a nephrostomy tube is in place. What is included in the postoperative care of this patient? a. Assess the amount of drainage in the collection bag. b. Irrigate the tube to ensure patency. c. Keep the patient NPO for 6 to 8 hours. d. Review the results of the clotting studies.

a

32. A patient with AKI is receiving total parenteral nutriton (TPN). What is the therapeutic goal of using TPN? a. Preserve lean body mass b. Promote tubular reabsorption c. Create a negative nitrogen balance d. Prevent infection

a

32. The nurse hears in report that the patient is having renal colic pain. When performing the physical assessment of this patient during a severe pain episode, what additional sign/symptom may the nurse expect o observe? a. Diaphoresis b. Redness over the flank c. Jaundice d. Bruit in the renal artery

a

32. The nurse hears in report that the patient is having renal colic pain. When performing the physical assessment of this patient during a severe pain episode, what additional sign/symptom may the nurse expect to observe? a. Diaphoresis b. Redness over the flank c. Jaundice d. Bruit in the renal artery

a

32. The off-going nurse is giving shift report to the oncoming nurse about the care of a patient who had a nephrostomy tube placed 3 days ago and it is to remain in place until the urinary obstruction is resolved. What is the most important point to clearly communicate about the urine drainage? a. "Urine is draining only into the collection bag, not the baldder; therefore the minimum expected drainage is 30 mL/hour." b. "For the first 24 hours postoperatively, the amount of urinary drainage was assessed every hour." c. "The surgeon placed ureteral tubes so all the urine may pass through the bladder or all of the urine might go directly into the collection bag." d. "The nephrostomy site has not been leaking any blood or urine and you should continue to monitor the site for leakage."

a

33. Which nursing intervention is the priority for a patient with SIADH? a. Restrict fluid intake. b. Monitor neurologic status at least every 2 hours. c. Offer ice chips frequently to ease discomfort by dry mouth . d. Monitor urine tests for decreased sodium levels and low specific gravity.

a

34. A patient is diagnosed with a urethral stricture. The nurse prepares the patient for which temporary treatment? a. Dilation of the urethra b. Antibiotic therapy c. Fluid restriction d. Urinary diversion

a

35. A nursing student asks the registered nurse why D5W is contraindicated when transfusing blood. How does the nurse respond? a. "It causes hemolysis of blood cells." b. "It dilutes the cells." c. "It shrinks the blood cells." d. "It is in the procedure manual."

a

36. A patient is receiving a blood transfusion through a single-lumen peripherally inserted central catheter. The patient has two other peripheral IVs: one is capped and the other has D5/.45 NS running at a rate of 50 mL/hr. What can be given concurrently through the line that is selected for the blood product? a. Normal saline b. Piggyback of 10 mEq potassium chloride c. Total parenteral nutrition d. Furosemide (Lasix) 5 mg IV push

a

The nurse is performing a cardiac assessment on an older adult. What is a common assessment finding for this patient? a. S4 heart sound b. leg edema c. Pericardial friction rubs d. Change in point of maximum impulse location

a

37. The nurse is assessing a patient for bladder distention. What technique does the nurse use? a. Gently palpate for the outline of the bladder, percuss the lower abdomen, continue toward the umbilicus until dull sounds are no longer produced. b. Gently palpate for the outline of the bladder, auscultate for sounds in the lower abdomen. c. Place on hand under the back and palpate with the other hand over the bladder, percuss the lower abdomen until tympanic sounds are no longer produced. d. Use the hand to depress the bladder as the patient takes a deep breath, then percuss.

a

37. Which patient is mostly likely to have mixed incontinence? a. 54-year-old woman who had four full-term pregnancies b. 52-year-old man who had a stroke with neurologic deficits c. 76-year-old man with benign prosthetic hyperplasia d. 25-year-old woman who has a pelvic fracture

a

39. An older patient has been receiving frequent blood transfusions without any complications or adverse reactions; however, the nurse carefully monitors the patient during the current transfusion. Which signs/symptoms suggest that the patient is experiencing circulatory overload? a. Hypertension, bounding pulse, and distended neck veins b. Fever, chills, and tachycardia c. Urticaria, itching, and bronchospasm d. Headache, chest pain, and hemoglobinuria

a

4. The student nurse is caring for a patient in sickle cell crisis. Which action by the student nurse warrants intervention by the supervising nurse? a. Keeping the patient's room cool b. Using distraction and relaxation techniques c. Positioning painful areas of the patient with support d. Using therapeutic touch

a

40. A patient has come to the clinic for follow-up of acute pyelonephritis. Which action does the nurse reinforce to the patient? a. Complete all antibiotic regimens. b. Report episodes of nocturia. c. Stop talking the antibiotic when pain is relieved. d. Avoid taking nay over-the-counter drugs.

a

40. A patient with CKD has a potassium level of 9 mEq/L. The nurse notifies the health care provider after assessing for which sign/symptom? a. Cardiac dysrhythmias b. Respiratory depression c. Hypercalcemia d. Hypokalemia

a

40. The advanced-practice nurse is performing a digital rectal examination (DRE) and notes that the rectal sphincter contracts on digital insertion. How does the nurse interpret this finding? a. Nerve supply to the bladder is most likely intact. b. There is adequate strength in the pelvic floor. c. A rectocele is placing pressure on the bladder. d. Abnormal function for the bladder is unlikely.

a

41. A patient is admitted for acute glomerulonephritis. In reviewing the patient's past medical history, which systemic conditions does the nurse suspect may have caused acute glomerulonephritis and will include in the overall plan of care? a. Systemic lupus erythematosus and diabetic nephropathy b. Myocardial infarction and atrial fibrillation c. Ischemic stroke and hemiparesis d. Blunt trauma to the kidney with hematuria

a

41. Experienced nurse A is supervising new nurse B. In which circumstance would nurse A intervene? a. Nurse B prepares to use blood administration tubing to infuse stems cells. b. Nurse B obtains Y-tubing with a blood filter to administer packed red blood cells. c. Nurse B uses a special shorter tubing with a smaller filter to deliver platelets. d. Nurse B rapidly delivers fresh frozen plasma through regular straight filtered tubing.

a

43. A patient with CKD develops severe chest pain, an increased pulse, low-grade fever, and a pericardial friction rub with a cardiac dysrhythmia and muffled heart tones. The nurse immediately alerts the health care provider and prepares for which emergency procedure? a. Pericardiocentesis b. CVVH c. Kidney dialysis d. Endotracheal intubation

a

43. Production of which hormone causes lower levels of calcium? a. Calcitonin b. PTH c. T4 d. TSH

a

43. The home health nurse is assessing an older adult patient who refuses to leave the house to see friends or participate in usual activities. She reports taking a bath several times a day and becomes very upset when she has an incontinent episode. What is the priority problem for this patient? a. Negative self-image b. Stress urinary incontinence c. Social isolation d. Potential for skin breakdown

a

44. Bone changes in the older adult are often seen with endocrine dysfunction and increased secretion of which substance? a. PTH b. Calcitonin c. Insulin d. Testosterone

a

44. What does the BUN test measure? a. Kidney excretion of urea nitrogen b. Urine osmolality c. Creatinine clearance d. Urine output

a

45. The nurse is reviewing urinalysis results for a patient who is in the early stages of CKD, what results might the nurse expect to see? a. Excessive protein, glucose, red blood cells, and white blood cells b. Increased specific gravity with a dark amber discoloration c. Dramatically increased urine osmolarity d. Pink-tinged urine with obvious small blood clots

a

47. Which patient is most likely to have a decreased calcium level? a. Patient with kidney disease b. Patient with cystitis c. Patient with a Foley catheter d. Patient with urinary retention

a

48. The nurse determines a priority patient problem of altered self-concept in a female patient with Cushing's syndrome who expresses concern about the changes in her general appearance. What is the expected outcome for this patient? a. To verbalize an understanding that treatment will reverse many of the problems b. To ventilate about the frustration of these lifelong physical changes c. To verbalize ways to cope with the changes such as joining a support group or changing style of dress d. To achieve a personal desired level of sexual functioning

a

48. The nurse is designing a habit training bladder program for an older adult patient who is alert but mildly confused. What task associated with the training program is delegated to the UAP? a. Tell the patient it is time to go to the toilet and assist him to go on a regular schedule. b. Help the patient record the incidents of incontinence in a bladder diary. c. Change the patient's incontinence pants (or pad) every 4 hours. d. Gradually encourage independence and increase the intervals between voiding.

a

48. The nurse is reviewing the laboratory results for a patient with chronic glomerulonephritis. The serum albumin level is low. What else does the nurse expect to see? a. Proteinuria b. Elevated hematocrit c. High specific gravity d. Low white blood cell count

a

48. The patient with CKD reports chronic fatigue and lethargy with weakness and mild shortness of breath with dizziness when rising to a standing position. In addition, the nurse note pale mucous membranes. Based on the patient's illness and the presenting symptoms, which laboratory result does the nurse expect to see? a. Low hemoglobin and hematocrit b. Low white cell count c. Low blood glucose d. Low oxygen saturation

a

49. Which drug is an adrenal cytotoxic agent used for inoperable adrenal tumors? a. Mitotane (Lysodren) b. Aminoglutethimide (Cytadren) c. Cyprohepatadine (Periactin) d. Fludrocortisone (Florinef)

a

5. The nurse is caring for a patient with an indwelling catheter. What intervention does the nurse use to minimize catheter-related infections? a. Assess the patient daily to determine need for catheter. b. Irrigate the catheter daily with sterile solution to remove debris. c. Use sterile technique when opening system to obtain urine samples. d. Apply antiseptic solutions or antibiotic ointments to the perineal area.

a

50. A patient has hyperparathyroidism and high levels of serum calcium. Which initial treatment does the nurse prepare to administer to the patient? a. Furosemide (Lasix) with IV saline b. Calcitonin c. Oral phosphates d. Mmithramycin

a

50. The nurse is reviewing the laboratory results of a patient with chronic glomerulonephritis. The phosphorus level is 5.3 mg/dL. What else does the nurse expect to see? a. Serum calcium level below the normal range. b. Serum potassium level below the normal range. c. Falsely elevated serum sodium level. d. Elevated serum levels for all other elctrolytes.

a

51. The health care provider has recommended intermittent self-catheterization for a patient with long-term problems of incomplete bladder emptying. Which information does the nurse give the patient about the procedure? a. Perform proper handwashing and cleaning of the catheter to reduce the risk for infection. b. Use a large-lumen catheter and good lubrication for rapid emptying of the bladder. c. Catheterize yourself whenever the bladder gets distended. d. Use sterile technique, especially if catheterization is done by a family member.

a

51. The nurse is reviewing arterial blood gas results of a patient with acute glomerulonephritis. The pH of the sample is 7.35. As acidosis is likely to be present because of hydrogen ion retention and loss of bicarbonate, how does the nurse interpret this data? a. Normal pH with respiratory compensation b. Acidosis with failure of respiratory compensation c. Alkalosis with failure of metabolic compensation d. Normal pH with metabolic compensation

a

52. The nurse is reviewing a care plan for a patient who has functional incontinence. There is a note that containment is recommended, especially at night. What is the major concern with this approach? a. Skin integrity b. Cost of care and materials c. Self-esteem of the patient d. Fall risk

a

52. Which patient is most likely to produce urine with a specific gravity of less than 1.005? a. Takes diuretic medication everyday b. Has dehydration secondary to vomiting c. Is hypovolemic due to blood loss d. Has syndrome of inappropriate antidiuretic hormone

a

53. In order to assist a patient in the prevention of osteodystrophy, which intervention does the nurse perform? a. Administer phosphate binders with meals. b. Encourage high-quality protein foods. c. Administer iron supplements. d. Encourage extra milk at mealtimes.

a

54. Which patient is at risk for developing secondary adrenal insufficiency? a. Patient who suddenly stops taking high-dose steroid therapy b. Patient who tapers the dosages of steroid therapy c. Patient deficient in ADH d. Patient with an adrenal tumor causing excessive secretion of ACTH

a

54. Which urine characteristic listed on a urinalysis report arouses the nurse's suspicion of a problem in the urinary tract? a. Cloudiness b. Straw color c. Ammonia odor d. One cast per high-powered field

a

55. A patient with acute glomerulonephritis is required to provide a 24-hour urine specimen. What does the nurse expect to see when looking at the specimen? a. Smoky or cola-colored urine b. Clear and very dilute urine c. Urine that is full of pus and very thick d. Bright orange-colored urine

a

55. An ACTH stimulation test is the most definitive test for which disorder? a. Adrenal insufficiency b. Cushing's syndrome c. Pheochromocytoma d. Acromegaly

a

56. The nurse monitors a CKD patient's daily weights because of the risk for fluid retention. What instructions does the nurse give to the UAP? a. Weigh the patient daily at the same time each day, same scale, with the same amount of clothing. b. Weigh the patient daily and add 1 kilogram of weight for the intake of each liter of fluid. c. Weigh the patient in the morning before breakfast and weigh the patient at night just before bedtime. d. Ask the patient what his or her normal weight is and them weigh the patient before and after each voiding.

a

57. Teaching intermittent self-catheterization for incontinence is appropriate for which patient? a. 25-year-old male patient with paraplegia b. 35-year-old female patient with stress incontinence c. 70-year-old patient who wears absorbent briefs d. 18-year-old patient with a severe head injury

a

59. A patient is scheduled for a computed tomography (CT) with iodinated contrast medium. Which medication is discontinued 24 hours before the procedure and for at least 48 hours until kidney function has been reevaluated? a. Glucophage (Metformin) b. Morphine (MS Contin) c. Furosemide (Lasix) d. Oral acetylcystein (Mucomyst)

a

59. The nurse is evaluating a patient's treatment response to erythropoietin (Epogen). Which hemoglobin reading indicates that the goal is being met? a. Around 10 g/dL b. Greater than 20 g/dL c. Upward trend d. At baseline for gender

a

6. Which patient is most likely to have severe manifestations of sickle cell disease even when triggering conditions are mild? a. Both parents have hemoglobin S gene alleles b. Mother has hemoglobin S gene alleles and father has hemoglobin A gene alleles c. Mother has sickle cell trait and father has hemoglobin A gene alleles d. Both parents have hemoglobin A gene alleles

a

6. Which task related to care of patients who have indwelling catheters can be delegated to unlicensed assistive personnel (UAP)? a. Perform daily catheter care by washing the perineum and proximal portion of the catheter with soap and water. b. Use sterile technique when inserting the urinary catheter or when opening the system to obtain urine samples. c. Determine whether use of condom catheters is appropriate for male patients and apply the devices accordingly. d. Keep urine collection bag in a place that is readily visible to the patient, so that the patient is reassured of kidney function.

a

60. A patient has been receiving erythropoietin (Epogen). Which statement by the patient indicates that the therapy is producing the desired effect? a. "I can do my housework with less fatigue." b. "I have been passing more urine than I was before." c. "I have less pain and discomfort now." d. "I can swallow and eat much better than before."

a

63. A patient returns to the medical-surgical unit after having shock wave lithotripsy (SWL). What is an appropriate nursing intervention for the postprocedural care of this patient? a. Strain the urine to monitor the passage of stone fragments. b. Report bruising that occurs on the flank of the affected side. c. Continuously monitor electrocardiogram (ECG) for dysrhythmias. d. Apply a local anesthetic cream to the skin of the affected sie.

a

63. The nurse is reviewing the results of a patient's ultrasound of the kidney. The report reveals an enlarged kidney which suggests which possible problem? a. Polycystic kidney b. Kidney infection c. Renal carcinoma d. Chronic kidney disease

a

67. A patient had a cystoscopy. After the procedure, what does the nurse expect to see in this patient? a. Pink-tinged urine b. Bloody urine c. Very dilute urine d. Decreased urine output

a

67. The urine output of a patient with a kidney stone has decreased from 40 mL/hr to 5 mL/hr. What is the nurse's priority action? a. Ensure IV access and notify the health care provider. b. Perform the Crede maneuver on the patient's bladder. c. Test the urine for ketone bodies. d. Document the finding and continue monitoring.

a

67. Which patient has the greatest risk of developing a kidney abscess? a. Patient is diagnosed with acute pyelonephritis b. Patient has flank asymmetry related to hydronephrosis c. Patient developed a urinary tract infection secondary to a urinary catheter d. Patient is diagnosed with hypertension and nephrosclerosis

a

68. A 53-year-old patient is newly diagnosed with renal artery stenosis. What clinical manifestations the nurse most likely to observe when the patient first seeks health care? a. Sudden onset of hypertension b. Urinary frequency and dysuria c. Nausea and vomiting d. Flank pain and hematuria

a

7. Why may a patient with PKD experience constipation? a. Polycystic kidneys enlarge and put pressure on the large intestine. b. Patient becomes dehydrated because the kdineys are dysfunctional. c. Constipation is a side effect from the medications given to treat PKD. d. Patients with PKD have special dietary restrictions that cause constipation.

a

70. The home health nurse is evaluating the home setting for a patient who wishes to have inhome hemodialysis. What is important to have in the home setting to support this therapy? a. Specialized water treatment system to provide a safe, purified water supply b. Large dust-free space to accommodate and store the dialysis equipment c. Modified electrical system to provide high voltage to power the equipment d. Specialized cooling system to maintain strict temperature control

a

71. A patient is newly diagnosed with type 2 diabetes mellitus. Which screening recommendation does the nurse give to the patient regarding the early diabetic of diabetic kidney disease? a. Urine should be tested annually for protein and microalbuminuria. b. Blood urea nitrogen and serum creatinine should tested within 5 years. c. Urine should be tested within 5 years for protein and microalbuminuria. d. Urine should be tested annually for protein, glucose, and blood.

a

72. Which data set indicates that the patient with diabetes is achieving the goals of care to prevent the development of microalbuminuria and delay the progression to end-stage kidney disease? a. A1c <7%, BP is 125/75 mm Hg, LDL cholesterol is 90 mg/dL b. A1c >7%, BP is 140/80 mm Hg, LDL cholesterol is 200 mg/dL c. A1c <7%, BP is 130/80 mm Hg, LDL cholesterol is 300 mg/dL d. A1c >7%, BP is 120/70 mm Hg, LDL cholesterol is 300 mg/dL

a

73. A patient is undergoing a dialysis treatment and exhibits a progression of symptoms which include headache, nausea, and vomiting; and fatigue. How does the nurse interpret these symptoms? a. Mild dialysis disequilibrium syndrome b. Expected manifestations in ESKD c. Transient symptoms in a new dialysis patient d. Adverse reaction to the dialysate

a

73. Limiting fluid intake would have what effect on urine? a. Increases the concentration of urine b. Makes the urine less irritating c. Decreases the risk for urine infection d. Decreases the pH of urine

a

74. A patient has had one kidney removed as a treatment for kidney cancer. The patient's spouse asks, "Does the good kidney take over immediately? I know a person can live with just one kidney." What is the nurse's best response? a. "The other kidney will provide adequate function, but this may take days or weeks." b. "The other kidney alone isn't able to provide adequate function, so supplemental therapies will be needed." c. "That's a good question. Remember to ask your doctor next time he or she comes in." d. "It varies a lot, but within a few days we expect everything to normalize."

a

76. The nurse is providing postdialysis care for a patient. In comparing vital signs and weight measurements to the predialysis data, what does the nurse expect to find? a. Blood pressure and weight are reduced. b. Blood pressure is increased and weight is reduced. c. Blood pressure and weight are slightly increased. d. Blood pressure is low and weight is the same.

a

77. The nurse is caring for a patient with kidney cell carcinoma. What does the nurse expect to find documented about the patient's initial assessment? a. flank pain, gross hematuria, palpable kidney mass, and renal bruit b. Gross hematuria, hypertension, diabetes, and oliguria c. Dysuria, polyuria, dehydration, and palpable kidney mass d. Nocturia and urinary retention with difficulty starting stream

a

79. A patient has returned to the medical-surgical unit after having a dialysis treatment. The nurse notes that the patient is also scheduled for an invasive procedure on the same day. What is the primary rationale for delaying the procedure for 4 to 6 hours? a. The patient was heparinized during dialysis. b. The patient will have cardiac dysrhythmias after dialysis. c. The patient will be incoherent and unable to give consent. d. The patient needs routine medications that were delayed.

a

80. The nurse is talking to a patient with ESKD. The patient frequently displays weight gain and increased blood pressure beyond the baseline measurements. Which question is the nurse most likely to ask to determine if the patient is doing something that is contributing to these assessment findings? a. "Are you controlling your salt intake?" b. "Are you following the protein restrictions?" c. "Have you been eating a lot of sweets?" d. "Have you been exercising regularly?"

a

84. What is the best description of CAPD? a. Daily infusion of four 2 L exchanges of dialysate every 4 to 6 hours while awake. b. Is a form of automated dialysis that uses an automated cycling machine. c. Functions of the cycling machine are programmed to the patient's needs. d. This form decreases the risk of peritonitis and poor dialysate flow.

a

85. The home health nurse is visiting a patient who independently performs PD. Which question does the nurse ask the patient to assess for the major complication associated with PD? a. "Have you noticed any signs or symptoms of infection?" b. "Are you having any pain during the dialysis treatment?" c. "Is the dialysate fluid slow or sluggish?" d. "Have you noticed any leakage around the catheter?"

a

89. The nurse is caring for a patient requiring PD. In order to monitor the patient's weight, what does the nurse do? a. Check the weight after a drain and before the next fill to monitor the patient's "dry weight." b. Calculate the "dry weight" by weighing the patient every day and comparing the patient's weight to a standard weight chart based on height and age. d. Weigh the patient each day and count fluid intake and dialysate volume to determine the patient's "dry weight."

a

9. The clinical manifestations of hyperthyroidism are known as which condition? a. Thyrotoxicosis b. Euthyroid function c. Graves' disease d. Hypermetabolism

a

9. The nurse is caring for a patient who had hypovolemic shock secondary to trauma in the emergency department (ED) 2 days ago. Based on the pathophysiology of hypovolemia and prerenal azotemia, what does the nurse assess at least every hour? a. Urinary output b. Presence of edema c. Urine color d. Presence of pain

a

A patient has the following assessment findings: elevated TSH level, low T3 and T4 levels, difficulty with memory, lethargy, and muscle stiffness. These are clinical manifestations of which disorder? a. Hypothyroidism b. Hyperthyroidism c. Hypoparathyroidism d. Hyperparathyroidism

a

A patient in the ED with chest pain has a possible MI. Which laboratory test is done to determine this diagnosis? a. Troponin T and I b. Serum potassium c. Homocysteine d. Highly sensitive C-reactive protein

a

A patient is being discharged with a prescription for warfarin (Coumadin). Which test does the nurse instruct the patient to routinely have done for follow-up monitoring? a. PT and INR b. PTT c. Complete blood count and platelet count d. Sodium and potassium levels

a

A patient's chart notes that the examiner has heard S1 and S2 on auscultation of the heart. What does this documentation refer to? a. First and second heart sounds b. Pericardial friction rub c. Murmur d. Gallop

a

After a visit to the health care provider's office, a patient is diagnosed with general thyroid enlargement and elevated thyroid hormone level. Which condition do these findings indicate? a. Hyperthyroidism and goiter b. Hypothyroidism and goiter c. Nodules on the parathyroid gland d. Thyroid or parathyroid cancer

a

Emergency personnel discovered a patient lying outside in the cool evening air for an unknown length of time. The patient is in a hypothermic state and the metabolic needs of the tissues are decreased. What other assessment finding does the nurse expect to see? a. Blood pressure and heart rate lower than normal b. Heart rate and respiratory rate higher than normal c. Normal vital signs due to compensatory mechanisms d. Gradually improved vital signs with enteral nutrition

a

In assessing a patient, the nurse finds that the PMI appears in more than one intercostal space, and has shifted laterally to the midclavicular line. How does the nurse interpret this data? a. Left ventricular hypertrophy b. Superior vena cava obstruction c. Pulmonary hypertension d. Constrictive pericarditis

a

In the bone marrow of an older adult, what would be considered a normal physiologic change related to aging? a. Fatty tissue replaces bone marrow. b. Bone marrow cells become smaller. c. Weakened bones absorb bone marrow. d. Bone marrow cells fail to function.

a

The advanced-practice nurse is assessing the vascular status of a patient's lower extremities using the angle-brachial index. What is the correct technique for this assessment method? a. A blood pressure cuff is applied to the lower extremities and the systolic pressure is measured by Doppler ultrasound at both the dorsalis pedis and posterior tibial pulses b. The dorsalis pedis and posterior tibial pulses are manually palpated and compared bilaterally for strength and equality and compared to the standard of index c. A blood pressure cuff is applied to the lower extremities to observe for an exaggerated decrease in systolic pressure by more than 10 mm Hg during inspiration d. Measure blood pressure on the legs when the patient is supine; then have the patient stand fro several minutes and repeat blood pressure measurement in the arms

a

The clinical manifestations of hyperthyroidism are known as which condition? a. Thyrotoxicosis b. Euthyroid function c. Graves' disease d. Hypermetabolism

a

The nurse is caring for a patient at risk fro MI. For what primary reason does the nurse plan interventions to prevent anxiety or overexertion? a. An increase in heart rate increases myocardial oxygen demand b. A release of epinephrine and norepinephrine causes MI c. AN increase in activity or emotion affects preload and afterload d. Cardiac output is decreased by anxiety or physical stress

a

The nurse is reviewing the patient's medication list and sees that the patient is receiving parenteral enoxaparin (Lovenox). Which outcome statement is the target of the enoxaparin therapy? a. Patient will not develop signs/symptoms of a blood clot. b. Patient will report a decrease in fatigue and dizziness. c. Patient will not develop signs/symptoms of infection. d. Patient will demonstrate no shortness of breath on exertion.

a

The nurse is talking to a patient who has been trying to quit smoking. HWich statement by the patient indicates an understanding of cigarette usage as it relates to reducing cardiovascular risks? a. "I need to be completely cigarette-free for at least 3 years" b. "I don't smoke as much as I used to I'm down to one pack a day." c. "I started smoking a while ago, but I'll quit in a couple of years" d. "I only smoke to relax, when I drink, or when I go out with friends"

a

The nurse performing a physical assessment on a patient with a history of CVD observes that the patient has ascites, jaundice, and anasarca. How does the nurse interpret these findings? a. Late signs of severe right-sided heart failure b. Early signs of mild right-sided heart failure c. Late signs of mild left-sided heart failure d. Early signs of left- and right-sided heart failure

a

The nurse taking a medical history of a patient makes a special notation to follow up on valvular abnormalities of the heart. Which recurrent condition in the patient history causes the nurse to make this negotiation? a. Streptococcal infections of the throat b. Staphylococcal infections of the skin c. Vaginal yeast infections d. Fungal infections of the feet or inner thighs

a

The patient is scheduled for an exercise stress test. Which medications does the nurse expect the cardiologist will want held before the procedure? a. Atenolol and Cardizem b. Vitamins and potassium c. Colace and enteric-coated aspirin d. Acetaminophen and metered-dose bronchodilator

a

The patient who is prescribed methimazole (Tapazole) 4 mg orally every 8 hours tells the nurse that his heart rate is slow (60/minute), he has gained 7 pounds, and he wears a sweater even on warm days. What does the nurse suspect? a. Indications of hypothyroidism will require a lower dosage. b. Indications of hypothyroidism will require a higher dosage. c. Indications of hyperthyroidism will require a lower dosage. d. Indications of hyperthyroidism will require a higher dosage.

a

What is the most significant laboratory cardiac marker in a patient who has had an MI? a. Presence of troponin T and I b. Elevation of myoglobin levels c. Creatine kinase levels d. Elevation of the white blood cell count

a

When the nurse assesses a patient with CVD, there is difficulty auscultating the first heart sound (S1). What is the nurse's best action? a. Ask the patient to lean forward or roll to his or her left side b. Instruct the patient to take a deep breath and hold it c. Auscultate with the bell instead of the diaphragm d. Ask the UAP to complete a 12-lead ECG immediately

a

Which category of cardiovascular drug blocks the sympathetic stimulation to the heart and decreases the heart rate? a. Beta blockers b. Catecholamines c. Steroids d. Benzodiazepines

a

Which statement about the peripheral vascular system is true? a. Veins are equipped with valves that direct blood flow to the heart and prevent backflow b. The velocity of blood flow depends on the diameter of the vessel lumen c. Blood flow decreases and blood tends to clot as the viscosity decreases d. The parasympathetic nervous system has the largest effect on blood flow to organs

a

Which test is performed to determine valve disease of the mitral valve, left atrium, or aortic arch? a. Transesophageal echocardiogram b. ECG c. Myocardial nuclear perfusion imaging (MNPI) d. Phonocardiography

a

Question 37 of 41 The nurse is teaching a client about induction therapy for acute leukemia. Which client statement indicates a need for additional education? a. "After this therapy, I will not need to have any more." b. "I will need to avoid people with a cold or flu." c. "I will probably lose my hair during this therapy." d. "The goal of this therapy is to put me in remission."

a "After this therapy, I will not need to have any more." Induction therapy is not a cure for leukemia, it is a treatment; therefore, the client needs more education to understand this. Because of infection risk, clients with leukemia should avoid people with a cold or flu. Induction therapy will most likely cause the client with leukemia to lose his or her hair. The goal of induction therapy is to force leukemia into remission.

Ch.61 Question 9 of 16 A client with an endocrine disorder says, "I can't, you know, satisfy my wife anymore." What is the nurse's best response? a. "Can you please tell me more?" b. "Don't worry. That is normal." c. "How does she feel?" d. "Should I make an appointment with a counselor?"

a "Can you please tell me more?" Asking the client to explain his concerns in an open-ended question allows the nurse to explore his feelings more thoroughly. Telling a client that something is "normal" is dismissive; this is new to the client and is a concern for him. The focus of the nurse's response needs to be on the client, not on the wife initially. Referring the client to a counselor is not an appropriate first step; this dismisses the client's concerns and does not allow him to express his frustrations at the moment.

Question 9 of 16 A client with an endocrine disorder says, "I can't, you know, satisfy my wife anymore." What is the nurse's best response? a. "Can you please tell me more?" b. "Don't worry. That is normal." c. "How does she feel?" d. "Should I make an appointment with a counselor?"

a "Can you please tell me more?" Asking the client to explain his concerns in an open-ended question allows the nurse to explore his feelings more thoroughly. Telling a client that something is "normal" is dismissive; this is new to the client and is a concern for him. The focus of the nurse's response needs to be on the client, not on the wife initially. Referring the client to a counselor is not an appropriate first step; this dismisses the client's concerns and does not allow him to express his frustrations at the moment.

20. A patient who has been diagnosed with Graves' disease is to receive RAI in the oral form of 131 I as a treatment. What instructions does the nurse include in the teaching plan about preventing radiation exposure to others? (Select all that apply.) a. Do not share a toilet with others for 2 weeks after treatment. b. Flush the toilet three times after each use. c. Wash clothing separately from others in the household. d. Limit contact with pregnant women, infants, and children. e. Do not use a laxative within 12 weeks of having the treatment.

a, b, c, d

Ch.61 Question 7 of 16 The nurse is instructing a client who will undergo a suppression test. Which statement by the client indicates that teaching was effective? a. "I am being tested to see whether my hormone glands are hyperactive." b. "I am being tested to see whether my hormone glands are hypoactive." c. "I am being tested to see whether my kidneys work at all." d. "I will be given more hormones as a trigger."

a "I am being tested to see whether my hormone glands are hyperactive." Suppression tests are used when hormone levels are high or in the upper range of normal. Failure of suppression of hormone production during testing indicates hyperfunction. A stimulation test assesses whether hormone glands are hypoactive. The adrenal glands are endocrine glands that are located on the kidneys; a suppression test does not measure kidney function. Hormones are given as a trigger in a stimulation test.

Question 7 of 16 The nurse is instructing a client who will undergo a suppression test. Which statement by the client indicates that teaching was effective? a."I am being tested to see whether my hormone glands are hyperactive." b. "I am being tested to see whether my hormone glands are hypoactive." c. "I am being tested to see whether my kidneys work at all." d. "I will be given more hormones as a trigger."

a "I am being tested to see whether my hormone glands are hyperactive." Suppression tests are used when hormone levels are high or in the upper range of normal. Failure of suppression of hormone production during testing indicates hyperfunction. A stimulation test assesses whether hormone glands are hypoactive. The adrenal glands are endocrine glands that are located on the kidneys; a suppression test does not measure kidney function. Hormones are given as a trigger in a stimulation test.

Ch.64 Question 20 of 25 The nurse is providing discharge teaching to a client with diabetes about injury prevention for peripheral neuropathy. Which statement by the client indicates a need for further teaching? a. "I can break in my shoes by wearing them all day." b. "I need to monitor my feet daily for blisters or skin breaks." c. "I should never go barefoot." d. "I should quit smoking."

a "I can break in my shoes by wearing them all day." Shoes should be properly fitted and worn for a few hours a day to break them in, with frequent inspection for irritation or blistering. People with diabetes have decreased peripheral circulation, so even small injuries to the feet must be managed early. Going barefoot is contraindicated. Tobacco use further decreases peripheral circulation in a client with diabetes.

Ch.63Question 6 of 21 A client is being discharged with propylthiouracil (PTU). Which statement by the client indicates a need for further teaching by the nurse? a. "I can return to my job at the nursing home." b. "I must call if my urine is dark." c. "I must faithfully take the drug every 8 hours." d. "I need to report weight gain."

a "I can return to my job at the nursing home." The client should avoid large crowds and people who are ill because PTU reduces blood cell counts and the immune response, which increases the risk for infection. The client does not, however, need to remain completely at home. Dark urine may indicate liver toxicity or failure, and the client must notify the provider immediately. Taking PTU regularly at the same time each day provides better drug levels and ensures better drug action. The client must notify the provider of weight gain because this may indicate hypothyroidism; a lower drug dose may be required.

Ch.64 Question 21 of 25 The nurse is teaching a client with type 2 diabetes about the importance of weight control. Which comment by the client indicates a need for further teaching? a. "I should begin exercising for at least an hour a day." b. "I should monitor my diet." c. "If I lose weight, I may not need to use the insulin anymore." d. "Weight loss can be a sign of diabetic ketoacidosis."

a "I should begin exercising for at least an hour a day." For long-term maintenance of major weight loss, large amounts of exercise (7 hr/wk) or moderate or vigorous aerobic physical activity may be helpful, but the client must start slowly. Monitoring the diet is key to type 2 diabetes management. Weight loss can minimize the need for insulin and can also be a sign of diabetic ketoacidosis.

9. The binding of a hormone to a specific receptor site is an example of which endocrine process? a. "Lock and key" manner b. Negative feedback mechanism c. Neuroendocrine regulation d. "Fight-or-flight" response

a "Lock and key" manner

Question 25 of 41 Which client is at greatest risk for having a hemolytic transfusion reaction? a. A 34-year-old client with type O blood b. A 42-year-old client with allergies c. A 58-year-old immune-suppressed client d. A 78-year-old client

a A 34-year-old client with type O blood Hemolytic transfusion reactions are caused by blood type or Rh incompatibility. When blood that contains antigens different from the client's own antigens is infused, antigen-antibody complexes are formed in the client's blood. Type O is considered the universal donor, but not the universal recipient. The client with allergies would be most susceptible to an allergic transfusion reaction. The immune-suppressed client would be most susceptible to a transfusion-associated graft-versus-host disease. The older adult client would be most susceptible to circulatory overload.

Question 18 of 41 An RN from pediatrics has "floated" to the medical-surgical unit. Which client is assigned to the float nurse? a. A 42-year-old with sickle cell disease receiving a transfusion of packed red blood cells b. A 50-year-old with pancytopenia needing assessment of risk factors for aplastic anemia c. A 55-year-old with folic acid deficiency anemia caused by alcohol abuse who needs counseling d. A 60-year-old with newly diagnosed polycythemia vera who needs teaching about the disease

a A 42-year-old with sickle cell disease receiving a transfusion of packed red blood cells Because sickle cell disease is commonly diagnosed during childhood, the pediatric nurse will be familiar with the disease and with red blood cell transfusion; therefore, he or she should be assigned to the client with sickle cell disease. Aplastic anemia, folic acid deficiency, and polycythemia vera are problems more commonly seen in adult clients who should be cared for by nurses who are more experienced in caring for adults.

A patient who has been diagnosed with Graves' disease is to receive RAI in the oral form of 131I as a treatment. What instructions does the nurse include in the teaching plan about preventing radiation exposure to others? (Select all that apply.) a. Do not share a toilet with others for 2 weeks after treatment. b. Flush the toilet three times after each use. c. Wash clothing separately from others in the household. d. Limit contact with pregnant women, infants, and children. e. Do not use a laxative within 2 weeks of having the treatment.

a, b, c, d

Ch.62 Question 9 of 23 A client with syndrome of inappropriate antidiuretic hormone is admitted with a serum sodium level of 105 mEq/L. Which request by the health care provider does the nurse address first? a. Administer infusion of 150 mL of 3% NaCl over 3 hours. b. Draw blood for hemoglobin and hematocrit. c. Insert retention catheter and monitor urine output. d. Weigh the client on admission and daily thereafter.

a Administer infusion of 150 mL of 3% NaCl over 3 hours. The client with a sodium level of 105 mEq/L is at high risk for seizures and coma. The priority intervention is to increase the sodium level to a more normal range. Ideally, 3% NaCl should be infused through a central line or with a small needle through a large vein to prevent irritation. Monitoring laboratory values for fluid balance and monitoring urine output are important, but are not the top priority. Monitoring client weight will help in the assessment of fluid balance; however, this is also not the top priority.

Ch.62 Question 8 of 23 A client presents to the emergency department with a history of adrenal insufficiency. The following laboratory values are obtained: Na+ 130 mEq/L, K+ 5.6 mEq/L, and glucose 72 mg/dL. Which is the first request that the nurse anticipates? a. Administer insulin and dextrose in normal saline to shift potassium into cells. b. Give spironolactone (Aldactone) 100 mg orally. c. Initiate histamine2 (H2) blocker therapy with ranitidine for ulcer prophylaxis. d. Obtain arterial blood gases to assess for peaked T waves.

a Administer insulin and dextrose in normal saline to shift potassium into cells. This client is hyperkalemic. The nurse should anticipate a request to administer 20 to 50 units of insulin with 20 to 50 mg of dextrose in normal saline as an IV infusion to shift potassium into the cells. Spironolactone is a potassium-sparing diuretic that helps the body keep potassium, which the client does not need. Although H2 blocker therapy would be appropriate for this client, it is not the first priority. Arterial blood gases are not used to assess for peaked T waves associated with hyperkalemia; an electrocardiogram needs to be obtained instead.

Ch.61 Question 8 of 16 Which gland releases catecholamines? a. Adrenal b. Pancreas c. Parathyroid d. Thyroid

a Adrenal The adrenal medulla releases catecholamines in response to stimulation of the sympathetic nervous system. The principal hormones of the pancreas are insulin, glucagon, and somatostatin. Parathyroid hormone is the principal hormone of the parathyroid gland. Triiodothyronine (T3), thyroxine (T4), and calcitonin are the principal hormones of the thyroid.

Question 8 of 16 Which gland releases catecholamines? a. Adrenal b. Pancreas c. Parathyroid d. Thyroid

a Adrenal The adrenal medulla releases catecholamines in response to stimulation of the sympathetic nervous system. The principal hormones of the pancreas are insulin, glucagon, and somatostatin. Parathyroid hormone is the principal hormone of the parathyroid gland. Triiodothyronine (T3), thyroxine (T4), and calcitonin are the principal hormones of the thyroid.

Ch.64 Question 19 of 25 Which is the best referral that the nurse can suggest to a client who has been newly diagnosed with diabetes? a. American Diabetes Association b. Centers for Disease Control and Prevention c. Health care provider office d. Pharmaceutical representative

a American Diabetes Association The American Diabetes Association can provide national and regional support and resources to clients with diabetes and their families. The Centers for Disease Control and Prevention does not focus on diabetes. The client's health care provider's office is not the best resource for diabetes information and support. A pharmaceutical representative is not an appropriate resource for diabetes information and support.

Ch.61 Question 10 of 16 The nurse should encourage fluids every 2 hours for older adult clients because of a decrease in which factor? a. Antidiuretic hormone (ADH) production b. General metabolism c. Glucose tolerance d. Ovarian production of estrogen

a Antidiuretic hormone (ADH) production A decrease in ADH production causes urine to be more dilute, so urine might not concentrate when fluid intake is low. The older adult is at greater risk for dehydration as a result of urine loss. A decrease in general metabolism causes decreased tolerance to cold, decreased appetite, and decreased heart rate and blood pressure; it is not related to fluid intake or hydration. A decrease in glucose tolerance does not affect fluid intake or hydration. A decrease in estrogen production causes a decrease in bone density and is not related to fluid intake and hydration.

Question 10 of 16 The nurse should encourage fluids every 2 hours for older adult clients because of a decrease in which factor? a. Antidiuretic hormone (ADH) production b. General metabolism c. Glucose tolerance d. Ovarian production of estrogen

a Antidiuretic hormone (ADH) production A decrease in ADH production causes urine to be more dilute, so urine might not concentrate when fluid intake is low. The older adult is at greater risk for dehydration as a result of urine loss. A decrease in general metabolism causes decreased tolerance to cold, decreased appetite, and decreased heart rate and blood pressure; it is not related to fluid intake or hydration. A decrease in glucose tolerance does not affect fluid intake or hydration. A decrease in estrogen production causes a decrease in bone density and is not related to fluid intake and hydration.

Ch.62 Question 19 of 23 A client with iatrogenic Cushing's syndrome is a resident in a long-term care facility. Which nursing action included in the client's care would be best to delegate to unlicensed assistive personnel (UAP)? a. Assist with personal hygiene and skin care. b. Develop a plan of care to minimize risk for infection. c. Instruct the client on the reasons to avoid overeating. d. Monitor for signs and symptoms of fluid retention.

a Assist with personal hygiene and skin care. Assisting a client with bathing and skin care is included in UAP scope of practice. It is not within their scope of practice to develop a plan of care, although they will play a very important role in following the plan of care. Client teaching requires a broad education and should not be delegated to UAP. Monitoring for signs and symptoms of fluid retention is part of client assessment, which requires a higher level of education and clinical judgment.

In assessing the patient's hematologic status, which questions would the nurse include? (Select all that apply.) a. "Have you had unusual or increased fatigue?" b. "Have you ever had any radiation therapy?" c. "Have you ever had a job that exposed you to chemicals?" d. "Do you have a personal or family history of blood disorders?" e. "What drugs have you used in the past 3 days?"

a, b, c, d

The nurse is performing a hematologic assessment of an older adult patient. Which findings does the nurse identify as normal changes in the older adult? (Select all that apply.) a. Progressive loss of body hair b. Thickened or discolored nails c. Yellowing of the skin d. Dryness of the skin e. Ecchymosis

a, b, c, d

Ch.61 Question 6 of 23 A client with pheochromocytoma is admitted for surgery. What does the nurse do for the admitting assessment? a. Avoids palpating the abdomen b. Monitors for pulmonary edema with a chest x-ray c. Obtains a 24-hour urine specimen on admission d. Places the client in a room with a roommate for distraction

a Avoids palpating the abdomen The abdomen must not be palpated in a client with pheochromocytoma because this action could cause a sudden release of catecholamines and severe hypertension. The tumor on the adrenal gland causes sympathetic hyperactivity, increasing blood pressure and heart rate, not pulmonary edema. A 24-hour urine collection will already have been completed to determine the diagnosis of pheochromocytoma. A client diagnosed with a pheochromocytoma may feel anxious as part of the disease process; providing a roommate for distraction will not reduce the client's anxiety.

Question 14 of 41 A 56-year-old client admitted with a diagnosis of acute myelogenous leukemia is prescribed IV cytosine arabinoside for 7 days and an infusion of daunorubicin for the first 3 days. What is the major side effect of this therapy? a. Bone marrow suppression b. Liver toxicity c. Nausea d. Stomatitis

a Bone marrow suppression Intravenous cytosine arabinoside and daunorubicin are a commonly prescribed course of aggressive chemotherapy, and bone marrow suppression is a major side effect. The client is even more at risk for infection than before treatment began. Liver toxicity, nausea, and stomatitis are not the major problems with this therapy.

Ch.64 Question 8 of 25 A client with type 1 diabetes arrives in the emergency department breathing deeply and stating, "I can't catch my breath." The client's vital signs are: T 98.4° F (36.9° C), P 112 beats/min, R 38 breaths/min, BP 91/54 mm Hg, and O2 saturation 99% on room air. Which action does the nurse take first? a. Check the blood glucose. b. Administer oxygen. c. Offer reassurance. d. Attach a cardiac monitor.

a Check the blood glucose. The client's clinical presentation is consistent with diabetic ketoacidosis, so the nurse should initially check the client's glucose level. Based on the oxygen saturation, oxygen administration is not necessary. The nurse provides support, but it is early in the course of assessment and intervention to offer reassurance without more information. Cardiac monitoring may be implemented, but the first action should be to obtain the glucose level.

Ch.61 Question 14 of 16 The charge nurse on the medical-surgical unit is making client assignments for the shift. Which client is the most appropriate to assign to an LPN/LVN? a. Client with Cushing's syndrome who requires orthostatic vital signs assessments b. Client with diabetes mellitus who was admitted with a blood glucose of 45 mg/dL c. Client with exophthalmos who has many questions about endocrine function d. Client with possible pituitary adenoma who has just arrived on the nursing unit

a Client with Cushing's syndrome who requires orthostatic vital signs assessments An LPN/LVN will be familiar with Cushing's syndrome and the method for assessment of orthostatic vital signs. The client with a blood glucose of 45 mg/dL, the client with questions about endocrine function, and the client with a possible pituitary adenoma all have complex needs that require the experience and scope of practice of an RN.

Ch.63 Question 4 of 21 The nurse manager for the medical-surgical unit is making staff assignments. Which client will be most appropriate to assign to a newly graduated RN who has completed a 6-week unit orientation? a. Client with chronic hypothyroidism and dementia who takes levothyroxine (Synthroid) daily b. Client with follicular thyroid cancer who has vocal hoarseness and difficulty swallowing c. Client with Graves' disease who is experiencing increasing anxiety and diaphoresis d. Client with hyperparathyroidism who has just arrived on the unit after a parathyroidectomy

a Client with Hashimoto's thyroiditis and a large goiter The client with chronic hypothyroidism and dementia is the most stable of the clients described and would be most appropriate to assign to an inexperienced RN. A client with vocal hoarseness and difficulty swallowing is at higher risk for complications and requires close observation by a more experienced nurse. Increasing anxiety and diaphoresis in a client with Graves' disease can be an indication of impending thyroid storm, which is an emergency; this is not a situation to be managed by a newly graduated RN. A client who has just arrived on the unit after a parathyroidectomy requires close observation for bleeding and airway compromise and requires assessment by an experienced nurse.

Ch.62 Question 23 of 23 After receiving change-of-shift report about these four clients, which client does the nurse attend to first? a. Client with acute adrenal insufficiency who has a blood glucose of 36 mg/dL b. Client with diabetes insipidus who has a dose of desmopressin (DDAVP) due c. Client with hyperaldosteronism who has a serum potassium of 3.4 mEq/L d. Client with pituitary adenoma who is reporting a severe headache

a Client with acute adrenal insufficiency who has a blood glucose of 36 mg/dL A glucose level of 36 mg/dL is considered an emergency; this client must be assessed and treated immediately. Although it is important to maintain medications on schedule, the client requiring a dose of desmopressin is not the first client who needs to be seen. A serum potassium of 3.4 mEq/L in the client with hyperaldosteronism may be considered normal (or slightly hypokalemic), based on specific hospital levels. The client reporting a severe headache needs to be evaluated as soon as possible after the client with acute adrenal insufficiency. As an initial measure, the RN could delegate obtaining vital signs to unlicensed assistive personnel.

Ch.64 Question 18 of 25 The nurse has just received change-of-shift report on the endocrine unit. Which client does the nurse see first? a. Client with type 1 diabetes whose insulin pump is beeping "occlusion" b. Newly diagnosed client with type 1 diabetes who is reporting thirst c. Client with type 2 diabetes who has a blood glucose of 150 mg/dL d. Client with type 2 diabetes with a blood pressure of 150/90 mm Hg

a Client with type 1 diabetes whose insulin pump is beeping "occlusion" Because glucose levels will increase quickly in clients who use continuous insulin pumps, the nurse should assess this client and the insulin pump first to avoid diabetic ketoacidosis. Thirst is a symptom of hyperglycemia and, although important, is not a priority; the nurse could delegate a fingerstick blood glucose to unlicensed assistive personnel while assessing the client whose insulin pump is beeping. Although a blood glucose reading of 150 mg/dL is mildly elevated, this is not an emergency. Mild hypertension is also not an emergency.

11. Which hormone is directly suppressed when circulating levels of cortisol are above normal? a. Corticotropin-releasing hormone (CRH) b. ADH c. Adrenocorticotropic hormone (ACTH) d. Growth hormone-releasing hormone (GH-RH)

a Corticotropin-releasing hormone (CRH)

Ch.62 Question 15 of 23 The nurse is teaching a client about how to monitor therapy effectiveness for syndrome of inappropriate antidiuretic hormone. What does the nurse tell the client to look for? a. Daily weight gain of less than 2 pounds b. Dry mucous membranes c. Increasing heart rate d. Muscle spasms

a Daily weight gain of less than 2 pounds The client must monitor daily weights because this assesses the degree of fluid restriction needed. A weight gain of 2 pounds or more daily or a gradual increase over several days is cause for concern. Dry mucous membranes are a sign of dehydration and an indication that therapy is not effective. An increased heart rate indicates increased fluid retention or dehydration and hypovolemia, and either condition is an indication that therapy is not effective. Muscle spasms are associated with hyponatremia and are an indication of a change in the client's neurologic status. Untreated hyponatremia can lead to seizures and coma.

Question 22 of 41 The nurse is teaching a client with vitamin B12 deficiency anemia about dietary intake. Which type of food does the nurse encourage the client to eat? a. Dairy products b. Grains c. Leafy vegetables d. Starchy vegetables

a Dairy products Dairy products such as milk, cheese, and eggs will provide the vitamin B12 that the client needs. Grains, leafy vegetables, and starchy vegetables are not a source of vitamin B12.

34. In the older adult female, which physiologic changes occur as a result of decreased function of the ovaries? a. Decreased bone density, decreased production of estrogen. b. Decreased sensitivity of peripheral tissues to the effects of insulin c. Decreased urine-concentrating ability of the kidneys d. Decreased metabolic rate

a Decreased bone density, decreased production of estrogen.

Ch.62 Question 17 of 23 A client has been admitted to the medical intensive care unit with a diagnosis of diabetes insipidus (DI) secondary to lithium overdose. Which medication is used to treat the DI? a. Desmopressin (DDAVP) b. Dopamine hydrochloride (Intropin) c. Prednisone d. Tolvaptan (Samsca)

a Desmopressin (DDAVP) Desmopressin is the drug of choice for treatment of severe DI. It may be administered orally, nasally, or by intramuscular or intravenous routes. Dopamine hydrochloride is a naturally occurring catecholamine and inotropic vasopressor; it would not be used to treat DI. Prednisone would not be used to treat DI. Tolvaptan is a selective competitive arginine vasopressin receptor 2 antagonist and is not used with DI.

5. A patient with a PRL-secreting tumor is likely to be treated with which medications? a. Dopamine agonists b. Vasopressin c. Steroids d. Growth hormone (GH)

a Dopamine agonists

Ch.62 Question 1 of 23 A client with diabetes insipidus (DI) has dry lips and mucous membranes and poor skin turgor. Which intervention does the nurse provide first? a. Force fluids b. Offer lip balm c. Perform a 24-hour urine test d. Withhold desmopressin acetate (DDAVP)

a Force fluids Dry lips and mucous membranes and poor skin turgor are indications of dehydration, which can occur with DI. This is a serious condition that must be treated rapidly. Encouraging fluids is the initial step, provided the client is able to tolerate oral intake. Lip balm may make the client more comfortable, but does not address the problem of dehydration. A 24-hour urine test will identify loss of electrolytes and adrenal androgen metabolites, but will not correct the dehydration that this client is experiencing. Desmopressin acetate is a synthetic form of antidiuretic hormone that is given to reduce urine production; it improves DI and should not be withheld.

9. When analyzing laboratory values, the nurse expects to find which value as a direct result of overproduction of GH? a. Hyperglycemia b. Hyperphosphatemia c. Hypocalcemia d. Hypercalcemia

a Hyperglycemia

35. An older adult reports a lack of energy and not being able to do the usual daily activities without several naps during the day. Which problem may these symptoms indicate that is often seen in the older adult? a. Hypothyroidism b. Hyperparathyroidism c. Overproduction of cortisol d. Underproduction of glucagon

a Hypothyroidism

Ch.61 Question 3 of 16 The nurse is teaching a client about maintaining a proper diet to prevent an endocrine disorder. Which food does the nurse suggest after the client indicates a dislike of fish? a. Iodized salt for cooking b. More red meat c. More green vegetables d. Salt substitute for cooking

a Iodized salt for cooking Dietary deficiencies in iodide-containing foods may be a cause of an endocrine disorder. For clients who do not eat saltwater fish on a regular basis, teach them to use iodized salt in food preparation. The client should eat a well-balanced diet that includes less animal fat. Eating vegetables contributes to a proper diet; however, this does not prevent an endocrine disorder. Using a salt substitute does not prevent an endocrine disorder; in addition, salt substitutes may contain high levels of potassium, which may lead to electrolyte imbalances.

Question 3 of 16 The nurse is teaching a client about maintaining a proper diet to prevent an endocrine disorder. Which food does the nurse suggest after the client indicates a dislike of fish? a. Iodized salt for cooking b. More red meat c. More green vegetables d. Salt substitute for cooking

a Iodized salt for cooking Dietary deficiencies in iodide-containing foods may be a cause of an endocrine disorder. For clients who do not eat saltwater fish on a regular basis, teach them to use iodized salt in food preparation. The client should eat a well-balanced diet that includes less animal fat. Eating vegetables contributes to a proper diet; however, this does not prevent an endocrine disorder. Using a salt substitute does not prevent an endocrine disorder; in addition, salt substitutes may contain high levels of potassium, which may lead to electrolyte imbalances.

Question 24 of 41 The nurse is assessing a newly admitted client with thrombocytopenia. Which factor needs immediate intervention? a. Nosebleed b. Reports of pain c. Decreased urine output d. Increased temperature

a Nosebleed The client with thrombocytopenia has a high risk for bleeding. The nosebleed should be attended to immediately. The client's report of pain, decreased urine output, and increased temperature are not the highest priority.

22. Which hormone responds to a low serum calcium blood level by increasing bone resorption? a. Parathyroid hormone (PTH) b. T4 c. T3 d. Calcitonin

a Parathyroid hormone (PTH)

34. The nurse is preparing for a patient to return from thyroid surgery. What priority equipment does the nurse ensure is immediately available? (Select all that apply.) a. Tracheostomy equipment b. Calcium gluconate or calcium chloride for IV administration c. Mechanical ventilator d. Humidified oxygten e. Suction equipment f. Pillow

a, b, d, e, f

Ch.64 Question 17 of 25 The nurse caring for four diabetic clients has all of these activities to perform. Which is appropriate to delegate to unlicensed assistive personnel (UAP)? a. Perform hourly bedside blood glucose checks for a client with hyperglycemia. b. Verify the infusion rate on a continuous infusion insulin pump. c. Monitor a client with blood glucose of 68 mg/dL for tremors and irritability. d. Check on a client who is reporting palpitations and anxiety.

a Perform hourly bedside blood glucose checks for a client with hyperglycemia. Performing bedside glucose monitoring is an activity that may be delegated because it does not require extensive clinical judgment to perform; the nurse will follow up with the results. Intravenous therapy and medication administration are not within the scope of practice for UAP. The client with blood glucose of 68 mg/dL will need further monitoring, assessment, and intervention not within the scope of practice for UAP. The client reporting palpitations and anxiety may have hypoglycemia, requiring further intervention; this client must be assessed by licensed nursing staff.

14. The anterior pituitary gland secretes tropic hormones in response to which hormones from the hypothalamus? a. Releasing hormones b. Target tissue hormones c. Growth hormones d. Demand hormones

a Releasing hormones

22. A patient has been taught how to care for his central venous catheter at home. Which statements by the patient indicate that further instruction is necessary? (Select all that apply.) a. "I will flush the catheter with heparin three times a day." b. "I will change the Luer-Lok cap on each catheter daily." c. "I will tape the catheter to my skin." d. "If the catheter lumen breaks or punctures, I will immediately clamp the catheter between myself and the opening." e. "I will wash my hands before working with the catheter."

a, b

22. The nurse is assessing a patient with a diagnosis of Hashimoto's disease. What are the primary manifestations of this disease? (Select all that apply.) a. Dysphagia b. Painless enlargement of the thyroid gland c. Painful enlargement of the thyroid gland d. Weight loss e. Intolerance to heat

a, b

26. A hospital patient is prescribed desmopressin acetate metered dose spray as a replacement hormone for ADH. Which is an indication for another dose? (Select all that apply.) a. Excessive urination b. Specific gravity of 1.003 c. Dark, concentrated urine d. Edema in the legs e. Decreased urination

a, b

51. Which are assessment findings of hypocalcemia? (Select all that apply.) a. Numbness and tingling around the mouth b. Muscle cramping c. Bone fractures d. Fever e. Tachycardia

a, b

The nurse is assessing a patient with a diagnosis of Hashimoto's disease. What are the primary manifestations of this disease? (Select all that apply.) a. Dysphagia b. Painless enlargement of the thyroid gland c. Painful enlargement of the thyroid gland d. Weight loss e. Intolerance to heat

a, b

15. The nurse is caring for a patient receiving gentamicin. Because this drug has potential for nephrotoxicity, which laboratory results does the nurse monitor? (Select all that apply.) a. Blood urea nitrogen b. Creatinine c. Drug peak and trough levels d. Prothrombin time (PT) e. Platelet count f. Hemoglobin and hematocrit

a, b, c

15. Which statements about hyperthyroidism are accurate? (Select all that apply.) a. It is most commonly caused by Graves' disease . b. It can be caused by overuse of thyroid replacement medication. c. It occurs more often in men between the ages of 20-40. d. Weight gain is a common manifestation. e. Serum T3 and T4 results will be elevated.

a, b, c

5. Based on the nurse's knowledge of the normal function of the kidney, which large particles are not found in the urine because they are too large to filter through the glomerular capillary walls? (Select all that apply.) a. Blood cells b. Albumin c. Other proteins d. E.ectrolytes e. water

a, b, c

5. Based on the nurse's knowledge of the normal function of the kidney, which large particles are not found in the urine because they are too large to filter through the glomerular capillary walls? (Select all that apply.) a. Blood cells b. Albumin c. Other proteins d. Electrolytes e. water

a, b, c

57. Which diagnostic tests and results does the nurse expect to see with acute glomerulonephritis? (Select all that apply.) a. Urinalysis revealing hematuria b. Urinalysis revealing proteinuria c. Microscopic red blood cell casts d. Serum albumin levels increased e. Serum potassium decreased

a, b, c

60. The nurse should instruct a patient who is taking hydrocortisone to report which symptoms to the health care provider for possible does adjustment? (Select all that apply.) a. Rapid weight gain b. Round face c. Fluid retnetion d. Gastrointestinal irritation e. Urinary incontinence

a, b, c

66. Which are the most accurate ways to monitor kidney function in the patient with CKD? (Select all that apply.) a. Monitoring intake and output b. Checking urine specific gravity c. Reviewing BUN and serum creatinine levels d. Reviewing x-ray reports e. Consulting the dietitian's notes

a, b, c

17. Following a hypophysectomy, the patient requires instruction on hormone replacement for which hormones? (Select all that apply.) a. Cortisol b. Thyroid c. Gonadal d. Vasopressin e. PRL

a, b, c, d

57. A patient with CKD is taking digoxin (Lanoxin). Which signs of digoxin toxicity does the nurse vigilantly monitor for? (Select all that apply.) a. Nausea and vomiting b. Visual changes c. Respiratory depression d. Restlessness or confusion e. Headache or fatigue f. Tachycardia

a, b, d, e, f

Question 35 of 41 What are the risk factors for the development of leukemia? Select all that apply. a. Bone marrow hypoplasia b. Chemical exposure c. Down syndrome d. Ionizing radiation e. Multiple blood transfusions f. Prematurity at birth

a, b, c, d Bone marrow hypoplasia, Chemical exposure, Down syndrome, Ionizing radiation Reduced production of blood cells in the bone marrow is one of the risk factors for developing leukemia. Exposure to chemicals through medical need or by environmental events can also contribute. Certain genetic factors contribute to the development of leukemia; Down syndrome is one such condition. Radiation therapy for cancer or other exposure to radiation, perhaps through the environment, also contributes. There is no indication that multiple blood transfusions are connected to clients who have leukemia. Although some genetic factors may influence the incidence of leukemia, prematurity at birth is not one of them.

38. Which are diagnostic methods to measure patient hormone levels? (Select all that apply.) a. Stimulation testing b. Suppression testing c. 24-hour urine testing d. Chromatographic assay e. Needle biopsy

a, b, c, d Stimulation testing, Suppression testing, 24-hour urine testing, Chromatographic assay

12. Which statements about the etiology of hypopituitarism are correct? (Select all that apply.) a. Dysfunction can result from radiation treatment to the head or brain. b. Dysfunction can result from infection or a brain tumor. c. Infarction following systemic shock can result in hypopituitarism. d. Severe malnutrition and body fat depletion can depress pituitary gland function. e. There is always an underlying cause of hypopituitarism.

a, b, c, d Dysfunction can result from radiation treatment to the head or brain. Dysfunction can result from infection or a brain tumor. Infarction following systemic shock can result in hypopituitarism. Severe malnutrition and body fat depletion can depress pituitary gland function.

13. The nurse is caring for a patient with acute leukemia. Which signs/symptoms is the nurse most likely to observe during the assessment? (Select all that apply.) a. Hematuria b. Orthostatic hypotension c. Bone pain d. Joint swelling e. Fatigue f. Weight gain

a, b, c, d, e

23. The nurse has instructed a patient at risk for bleeding about techniques to manage this condition. Which statements by the patient indicate that teaching has been successful? (Select all that apply.) a. "I will take a stool softener to prevent straining during a bowel movement." b. "I won't take aspirin or aspirin-containing products." c. "I won't participate in any contact sports.." d. "I will report a headache that is not responsive to acetaminophen." e. "I will avoid bending over at the waist." f. "If I am bumped, I will apply ice to the site for at least 10 minutes."

a, b, c, d, e

68. The nurse is assessing a patient with uremia. Which gastrointestinal changes does the nurse expect to find? (Select all that apply.) a. Halitosis b. Hiccups c. Anorexia d. Nausea e. Vomiting f. Salivation

a, b, c, d, e

41. A middle-aged woman has urinary stress incontinence related to weak pelvic muscles. The patient is highly motivated to participate in self-care. Which interventions does the nurse include in the treatment plan? (Select all that apply.) a. Suggest keeping a detailed diary of urine leakage, activities, and foods eaten. b. Suggest wearing absorbent undergarments during the assessment process. c. Teach pelvic floor (Kegel) exercise therapy. d. Teach about vaginal cone therapy. e. Encourage drinking orange juice every day for 4 to 6 weeks. f. Refer to a nutritionist for diet therapy for weight reduction.

a, b, c, d, f

83. A patient is brought to the emergency department (ED) after being invovled in a fight in which the patient was kicked and punched repeatedly in the back. What does the nurse include in the initial physical assessment? (Select all that apply.) a. Take complete vital signs. b. Check apical and peripheral pulses. c. Inspect both flanks for asymmetry or penetrating injuries of the lower chest or back. d. Inspect the abdomen for bruising or penetrating wounds. e. Deeply palpate the abdomen for signs of rigidity. f. Inspect the urethra for gross bleeding.

a, b, c, d, f

1. Which problems occur with acute kidney injury (AKI)? (Select all that apply.) a. decreased peristalsis b. Anemia c. Metabolic acidosis d. Hypokalemia e. Peripehral edema

a, b, c, e

16. Which factors are associated with an increased risk for lymphoma? (Select all that apply.) a. Immunosuppressive disorders b. Chronic infection from Helicobacter pylori c. Epstein-Barr viral infection d. Chronic alcoholism e. Pesticides and insecticides

a, b, c, e

28. The nurse is caring for a patient who has donated bone marrow. In addition to having the aspiration sites monitored, the nurse will anticipate the need for which intervention? (Select all that apply.) a. Fluid for hydration b. Pain management c. Possible RBC infusion d. Prophylactic antibiotic therapy e. Assessment for complications of anesthesia

a, b, c, e

51. In collaboration with the registered dietitian, the nurse teaches the patient about which diet recommendations for management of CKD? (Select all that apply.) a. Controlling protein intake b. Limiting fluid intake c. Restricting potassium d. Increasing sodium e. Restricting phosphorus f. Reducing calories

a, b, c, e

52. Which medication therapies does the nurse expect patients with hypoparathyroidism to receive? (Select all that apply.) a. Calcium chlroide b. Calcium gluconate c. Calitrol (Rocaltrol) d. Propranolol (Inderal) e. Ergocalciferol

a, b, c, e

64. The nurse is teaching self-care measures to a patient who had lithotripsy for kidney stones. What information does the nurse include? (Select all that apply.) a. Finish the entire prescription of antibiotics to prevent UTIs. b. Balance regular exercise with sleep and rest. c. Drink at least 3 L of fluid a day. d. Watch for and immediately report bruising after lithotripsy. e. Urine may be bloody for several days. f. Pain in the region of the kidneys or bladder is expected.

a, b, c, e

Management of the patient with hyperthyroidism focuses on which goals? (Select all that apply.) a. Blocking the effects of excessive thyroid secretion b. Treating the signs and symptoms the patient experiences c. Establishing euthyroid function d. Preventing spread of the disease e. Maintaining an environment of reduced stimulation

a, b, c, e

20. Which assessment findings does the nurse monitor in response to catecholamines released by the adrenal medulla? (Select all that apply.) a. Increased heart rate related to vasoconstriction b. Increased blood pressure related to vasoconstriction c. Increased perspiration d. Constriction of pupils e. Increased blood glucose in response to glycogenolysis

a, b, c, e Increased heart rate related to vasoconstriction. Increased blood pressure related to vasoconstriction. Increased perspiration. Increased blood glucose in response to glycogenolysis.

24. Which statements about T3 and T4 hormones are correct? (Select all that apply.) a. The basal metabolic rate is affected. b. Hypothalamus is stimulated by cold and stress to secret thyrotropin-releasing hormone (TRH). c. These hormones need intake of protein and iodine for production. d. Circulating hormone in the blood directly affects the production of TSH. e. T3 and T4 increase oxygen use in tissue.

a, b, c, e The basal metabolic rate is affected. Hypothalamus is stimulated by cold and stress to secret thyrotropin-releasing hormone (TRH). These hormones need intake of protein and iodine for production. T3 and T4 increase oxygen use in tissue.

27. Patients with sickle cell disease are more susceptible to infections. Which actions help prevent infection? (Select all that apply.) a. Perform consistent thorough handwashing b. Encourage yearly flu vaccination c. Administer twice-daily oral penicillin d. Administer NSAIDs three times a day e. Monitor CBC and differential white cell count f. Assess vital signs at least every 4 hours

a, b, c, e, f

10. In caring for a patient with hyperpituirarism, which symptoms does the nurse expect the patient to report? a. Joint pain b. Visual disturbances c. Changes in menstruation d. Increase libido e. Headache f. Fatigue

a, b, c, e, f Joint pain, Visual disturbances, Changes in menstruation, Headache, Fatigue

4. A patient can develop intrarenal kidney injury from which causes? (Select all that apply.) a. Vasculitis b. Pyelonephritis c. Strenuous exercise d. Exposure to nephrotoxins e. Bladder cancer

a, b, d

49. Which disorders/conditions can cause hyperparathyroidism? (Select all that apply.) a. Chronic kidney disease b. Neck trauma c. Thyroidectomy d. Vitamin D deficiency e. Parathyroidectomy

a, b, d

8. The nurse is developing a teaching plan for a patient with PKD. Which topics does the nurse include? (Select all that apply.) a. Teach how to measure and record blood pressure. b. Assist to develop a schedule for self-monitoring drugs. c. Instruct to take and record weight twice a month. d. Explain the potential side effects of the drugs. e. Review high-protein, low-fat diet plan.

a, b, d

Question 28 of 41 A distant family member arrives to visit a female client recently diagnosed with leukemia. The family member asks the nurse, "What should I say to her?" Which responses does the nurse suggest? Select all that apply. a. "Ask her how she is feeling." b. "Ask her if she needs anything." c. "Tell her to be brave and to not cry." d. "Talk to her as you normally would when you haven't seen her for a long time." e. "Tell her what you know about leukemia."

a, b, d "Ask her how she is feeling.", "Ask her if she needs anything.", "Talk to her as you normally would when you haven't seen her for a long time." Asking the client how she is feeling is a broad general opening and would be nonthreatening to the client. Asking if she needs anything is a therapeutic communication of offering self and would be considered to be therapeutic and helpful to the client. The family member should talk to her as she normally would when she hasn't seen her in a long time. There is no need to act differently with the client. If she wants to offer her feelings, keeping a normal atmosphere facilitates that option. Acting as if things are "different" because she has cancer takes the control of the situation from the client. Telling her to be brave and not to cry is callous and unfeeling; if the client is feeling vulnerable and depressed, telling her to "be brave" shuts off any opportunity for her to express her feelings. There is no need to inform the client about her disease, unless she asks about it. Opening the conversation with discussion about leukemia should be the client's prerogative.

21. Which statements about hypothyroidism are accurate? (Select all that apply.) a. It occurs more often in women. b. It can be caused by iodine deficiency. c. Weight loss is a common manifestation. d. It can be caused by autoimmune thyroid destruction. e. Myxedema coma is a rare but serious complication.

a, b, d, e

35. Which characteristics are associated with ESKD? (Select all that apply.) a. Severe fluid overload b. Renal osteodystrophy c. Nephrons compensate d. Dialysis or transplant needed to maintain homeostasis e. Excessive waste products

a, b, d, e

39. Which statements about thyroiditis are accurate? (Select all that apply.) a. It is an inflammation of the thyroid gland. b. Hashimoto's disease is the most common type. c. It always resolves with antibiotic therapy. d. There are three types: acute, subacute, and chronic. e. The patient must take thyroid hormones.

a, b, d, e

Which statements about hypothyroidism are accurate? (Select all that apply.) a. It occurs more often in women. b. It can be caused by iodine deficiency. c. Weight loss is a common manifestation. d. It can be caused by autoimmune thyroid destruction. e. Myxedema coma is a rare but serious complication.

a, b, d, e

3. A malfunctioning anterior pituitary gland can result in which disorder? (Select all that apply.) a. Pituitary hypofunction b. Pituitary hyperfunction c. DI d. Hypothyroidism e. Osteoporosis

a, b, d, e Pituitary hypofunction Pituitary hyperfunction Hypothyroidism Osteoporosis

41. Which are the types of radiographic tests that may be used for an endocrine assessment? (Select all that apply.) a. Ultrasonography b. Skull x-ray c. Chest x-ray d. Magnetic resonance imaging (MRI) e. Computed tomography (CT)

a, b, d, e Ultrasonography, Skull x-ray, Magnetic resonance imaging (MRI), Computed tomography (CT)

20. A patient undergoing hematopoietic stem cell transplantation reports severe fatigue. To assist the patient with energy management, what does the nurse encourage the patient to do? (Select all that apply.) a. Verbalize feelings about limitations. b. Monitor nutritional intake to ensure adequate energy resources. c. Avoid napping throughout the day. d. Limit the number of visitors as appropriate. e. Plan activities for periods when the patient has the most energy. f. Monitor overall response to self-care activities.

a, b, d, e, f

The nurse is preparing for a patient to return from thyroid surgery. What priority equipment does the nurse ensure is immediately available? (Select all that apply.) a. Tracheostomy equipment b. Calcium gluconate or calcium chloride for IV administration c. Mechanical ventilator d. Humidified oxygen e. Suction equipment f. Pillows

a, b, d, e, f

Question 13 of 41 The nurse is teaching a client with newly diagnosed anemia about conserving energy. What does the nurse tell the client? Select all that apply. a. "Allow others to perform your care during periods of extreme fatigue." b. "Drink small quantities of protein shakes and nutritional supplements daily." c. "Perform a complete bath daily to reduce your chance of getting an infection." d. "Provide yourself with four to six small, easy-to-eat meals daily." e. "Perform your care activities in groups to conserve your energy." f. "Stop activity when shortness of breath or palpitations are present."

a, b, d, f "Allow others to perform your care during periods of extreme fatigue.", "Drink small quantities of protein shakes and nutritional supplements daily.", "Provide yourself with four to six small, easy-to-eat meals daily.", "Stop activity when shortness of breath or palpitations are present." It is critical to have others help the anemic client who is extremely tired. Although it may be difficult for him or her to ask for help, this practice should be stressed to the client. Drinking small protein or nutritional supplements will help rebuild the client's nutritional status. Having four to six small meals daily is preferred over three large meals; this practice conserves the body's expenditure of energy used in digestion and assimilation of nutrients. Stopping activities when strain on the cardiac or respiratory system is noted is critical. A complete bath should be performed only every other day; on days in between, the client can be taught to take a "mini" sponge bath, which will conserve energy and still be safe in preventing the risks for infection. Care activities should be spaced every hour or so rather than in groups to conserve energy; the time just before and after meals should be avoided.

15. Which statements about hyperthyroidism are accurate? (Select all that apply.) a. It is most commonly caused by Graves' disease . b. It can be caused by overuse of thyroid replacement medication. c. It occurs more often in men between the ages of 20-40. d. Weight gain is a common manifestation. e. Serum T3 and T4 results will be elevated.

a, b, e

40. Which statement about acute thyroiditis are accurate? (Select all that apply.) a. It is caused by a bacterial infection of the thyroid gland. b. It is treated with antibiotic therapy. c. It results from a viral infection of the thyroid gland. d. Subtotal thyroidectomy is a form of treatment. e. Manifestations include neck tenderness, fever, and dysphagia.

a, b, e

6. When shock or other problems cause an acute reduction in blood flow to the kidneys, how do the kidneys compensate? (Select all that apply.) a. Constrict blood vessels in the kidneys. b. Activate the renin-angiotensin-aldosterone pathway. c. Release beta blockers. d. Dilate blood vessels throughout the body. e. Release antidiuretic hormones.

a, b, e

87. What laboratory values would the nurse interpret for a patient experiencing problems with urinary elimination as a result of acute peylonephritis? (Select all that apply.) a. Observe complete blood count for elevation of differentials. b. Observe for elevation of BUN and serum creatinine levels. c. Observe for electrolyte imbalances, such as hypokalemia. d. Observe arterial blood gases for alkalosis and respiratory compensation. e. Observe urinalysis for bacteria, leukocyte esterase, nitrate, or red blood cells.

a, b, e

96. What might the nurse notice if the patient is experiencing reduced perfusion and altered urinary elimination related to AKI? (Select all that apply.) a. Hemodynamic instability, especially persistent hypotension and tachycardia. b. Urine output of less than 0.5 mL/kg/hour for 6 or more hours c. Serum creatinine below baseline or admission values d. Urine may be clear or have a pale yellow color e. Abnormal serum and urine potassium and sodium values

a, b, e

A deficiency in any of the anticlotting factors, such as protein C, protein S, and antithrombin III increases the patient's risk for which disorder(s)? (Select all that apply.) a. Pulmonary embolism b. Myocardial infarction c. Sepsis d. Pernicious anemia e. Stroke

a, b, e

Which statements about hyperthyroidism are accurate? (Select all that apply.) a. It is most commonly caused by Graves' disease. b. It can be caused by overuse of thyroid replacement medication. c. It occurs more often in men between the ages of 20- 40. d. Weight gain is a common manifestation. e. Serum T3 and T4 results will be elevated.

a, b, e

13. Which statements about the pituitary glands are correct? (Select all that apply.) a. The main role of the anterior pituitary is to secrete tropic hormones. b. The posterior pituitary gland stores hormones produced by the hypothalamus. c. The anterior pituitary is connected to the thalamus gland. d. The anterior pituitary releases stored hormones produced by the hypothalamus. e. The anterior pituitary gland secretes gonadotropins.

a, b, e The main role of the anterior pituitary is to secrete tropic hormones. The posterior pituitary gland stores hormones produced by the hypothalamus. The anterior pituitary gland secretes gonadotropins.

8. Which statements about hormones and the endocrine system are accurate? (Select all that apply.) a. There are specific normal blood levels of each hormone. b. Hormones exert their effects on specific target tissues. c. Each hormone can bind with multiple receptor sites. d. The endocrine system works independently to regulate homeostasis. e. More than one hormone can be stimulated before the target tissue is affected.

a, b, e There are specific normal blood levels of each hormone. Hormones exert their effects on specific target tissues. More than one hormone can be stimulated before the target tissue is affected.

Ch.63 Question 8 of 21 The nurse is preparing the room for the client returning from a thyroidectomy. Which items are important for the nurse to have available for this client? Select all that apply. a. Calcium gluconate b. Emergency tracheotomy kit c. Furosemide (Lasix) d. Hypertonic saline e. Oxygen f. Suction

a, b, e, f Calcium gluconate, Emergency tracheotomy kit, Oxygen, Suction Calcium gluconate should be available at the bedside to treat hypocalcemia and tetany that might occur if the parathyroid glands have been injured during the surgery. Equipment for an emergency tracheotomy must be kept at the bedside in the event that hemorrhage or edema should occlude the airway. Respiratory distress can result from swelling or damage to the laryngeal nerve leading to spasm, so it is important that the nurse work with respiratory therapy to have oxygen ready at the bedside for the client on admission. Because of the potential for increased secretions, it is important that a working suction device is present at the bedside for admission of the client from the operating room. Furosemide might be useful in the postoperative client to assist with urine output; however, this is not of added importance for this client. Hypertonic saline would not be of benefit to this client as the client is not hyponatremic.

53. Discharge planning for a patient with chronic hypoparathyroidism includes which instructions? (Select all that apply.) a. Prescribed medications must be taken for the patient's entire life. b. Eat foods low in vitamin D and high in phosphorus. c. Eat foods high in calcium, but low in phosphorus. d. After several weeks, medications can be discontinued. e. Kidney stones are no longer a risk to the patient.

a, c

24. Which statements about DI are accurate? (Select all that apply.) a. It is caused by ADH deficiency. b. It is characterized by a decrease in urination. c. Urine output of greater than 4 L/24 hours is the first diagnostic indication. d. The water loss increases plasma osmolarity. e. Nephrogenic DI can be caused by lithium (Eskalith).

a, c, d

27. Which signs/symptoms does the nurse expect to see in the patient with AKI that has progressed in severity? (Select all that apply.) a. Oliguria b. Hypotension c. Shortness of breath d. Pulmonary crackles e. Weight loss

a, c, d

28. Which factors are considered to be triggers for thyroid storm? (Select all that apply.) a. Infection b. Cold temperatures c. Vigorous palpation of a goiter d. Diabetic ketoacidosis e. Extremely warm temperatures

a, c, d

31. The nurse is caring for a patient in the intensive care unit who sustained blood loss during a traumatic accident. For early identification of signs and symptoms that would suggest the development of kidney dysfunction, what does the nurse observe for? (Select all that apply.) a. Hypotension b. Bradycardia c. Decreased urine output d. Decreased cardiac output e. Increased central venous pressure

a, c, d

32. In SIADH, which laboratory value does the nurse expect to find? (Select all that apply.) a. Increased sodium in urine b. Elevated serum sodium level c. Increased specific gravity (concentrated urine) d. Decreased serum osmolarity e. Decreased urine specific gravity

a, c, d

56. Which conditions may precipitate myxedema coma? (Select all that apply.) a. Rapid withdrawal of thyroid medication b. Vitamin D deficiency c. Untreated hypothyroidism d. Surgery e. Excessive exposure to iodine

a, c, d

56. Which interventions are necessary for a patient with acute adrenal insufficiency (Addisonian crisis)? (Select all that apply.) a. IV infusion of normal saline b. IV infusion of 3% saline c. Hourly glucose monitoring d. Insulin administration e. IV potassium therapy

a, c, d

Which factors are considered to be triggers for thyroid storm? (Select all that apply.) a. Infection b. Cold temperatures c. Vigorous palpation of a goiter d. Diabetic ketoacidosis e. Extremely warm temperatures

a, c, d

2. A patient with sickle cell crisis is admitted to the hospital. Which questions does the nurse ask the patient to elicit information about the cause of the current crisis? (Select all that apply.) a. "Have you recently traveled on an airplane?" b. "Have you ever had radiation therapy?" c. "In the past 24 hours, has any activity made you short of breath?" d. "Have you recently consumed alcohol or used recreational drugs?" e. "Have you had any symptoms of infection, such as fever?"

a, c, d, e

30. A patient is at high risk for the development of venoocclusive disease (VOD). What assessments does the nurse perform for early detection of this disorder? (Select all that apply.) a. Jaundice b. Weight loss c. Hepatomegaly d. Right upper quadrant abdominal pain e. Ascites

a, c, d, e

31. Which signs and symptoms are assessment findings indicative of thyroid storm? (Select all that apply.) a. Abdominal pain and nausea b. Hypothermia c. Elevated temperature d. Tachycardia e. Elevated systolic blood pressure f. Bradycardia

a, c, d, e

4. The nurse is interviewing a patient with suspected PKD. What questions does the nurse ask the patient? (Select all that apply.) a. "Is there any family history of PKD or kidney disease?" b. "Do you have a history of sexually transmitted disease?" c. "Have you had any constipation or abdominal discomfort?" d. "Have you noticed a change in urine color or frequency?" e. "Have you had any problems with headaches?" f. " Is there a family history of sudden death from a myocardial infarction?"

a, c, d, e

69. Which patients with CKD are candidates for intermittent hemodialysis? (Select all that apply.) a. Patient with fluid overload who does not respond to diuretics b. Patient with injury stage according to the RIFLE classification c. Patient with symptomatic toxin ingestion d. Patient with uremic manifestations, such as decreased cognition e. Patient with symptomatic hyperkalemia and calciphylaxis

a, c, d, e

Which signs and symptoms are assessment findings indicative of thyroid storm? (Select all that apply.) a. Abdominal pain and nausea b. Hypothermia c. Elevated temperature d. Tachycardia e. Elevated systolic blood pressure f. Bradycardia

a, c, d, e

1. Which glands are parts of the endocrine system? (Select all that apply.) a. Thyroid b. Occipital c. Parathyroid d. Adrenal e. Pituitary

a, c, d, e Thyroid, Parathyroid, Adrenal, Pituitary

68. What does the nurse include in the care plan for a patient who had pyelolithotomy? (Select all that apply.) a. Monitor the amount of bleeding from incisions. b. Restrict fluids to prevent edema and fluid overload. c. Strain the urine to monitor the passage of stone fragments. d. Encourage fluids to avoid dehydration and supersaturation e. Monitor changes in urine output. f. Administer antibiotics to eliminate or prevent infections.

a, c, d, e ,f

2. Which assessment findings indicate hyperthyroidism? (Select all that apply.) a. Weight loss with increased appetite b. Constipation c. Increased heart rate d. Insomnia e. Decreased libido f. Heat intolerance

a, c, d, f

23. The nurse is taking a history of a patient at risk for kidney failure. What does the nurse ask the patient about during the interview? (Select all that apply.) a. Exposure to nephrotoxic chemicals b. Increased appetite c. History of diabetes meliitus, hypertension, systemic lupus erythematosus d. Recent surgery, trauma, or transfusions e. Leakage of urine when coughing or laughing f. Recent or prolonged use of antibiotics and NSAIDs

a, c, d, f

Which assessment findings indicate hyperthyroidism? (Select all that apply.) a. Weight loss with increased appetite b. Constipation c. Increased heart rate d. Insomnia e. Decreased libido f. Heat intolerance

a, c, d, f

35. In addition to IV fluids, a patient with SIADH is on a fluid restriction as low as 500 to 600 mL/24 hours. Which serum and urine results demonstrate effectiveness of this treatment? (Select all that apply.) a. Decreased urine specific gravity b. Decreased serum sodium c. Increased urine output d. Increased urine specific gravity e. Increased serum sodium f. Decreased urine output

a, c, e

46. Kidney tissue changes in chronic glomerulonephritis are caused by which factors? (Select all that apply.) a. Ischemia b. Fluid overload c. Hypertension d. Obstruction e. Infection

a, c, e

7. The nurse is caring for a patient in sickle cell crisis. What are priority interventions for this patient? (Select all that apply.) a. Managing pain b. Managing nutrition c. Ensuring hydration d. Administering platelets e. Assessing oxygen saturation

a, c, e

The nurse is interviewing a patient who reports dizziness and lightheadedness, and bleeding gums every time she brushes her teeth. Which questions does the nurse ask the patient in order to focus in on the problem? (Select all that apply.) a. "How often do you take aspirin or any other nonsteroidal antiinflammatory drug?" b. "Do you have swollen glands or a sore throat?" c. "How much meat do you eat in a week?" d. "Are you having trouble swallowing?" e. "Does your heart ever seem to pound?"

a, c, e

28. Which statements about insulin secretion are correct? (Select all that apply.) a. Insulin levels increase following the ingestion of a meal. b. Insulin is stimulated primarily by fat ingestion. c. Basal levels are secreted continuously. d. Insulin promotes glycogenolysis and gluconeogenesis. e. Carbohydrate intake is the main trigger for insulin secretion.

a, c, e Insulin levels increase following the ingestion of a meal. Basal levels are secreted continuously. Carbohydrate intake is the main trigger for insulin secretion.

46. Which are physical findings of Cushing's disease? (Select all that apply.) a. "Moon-faced" appearance b. Decreased amount of body hair c. Truncal obesity d. Coarse facial features e. Thin, easily damaged skin f. Extremity muscle wasting

a, c, e, f

36. Which medications are used in SIADH to promote water excretion without causing sodium loss? (Select all that apply.) a. Tolvaptan (Samsca) b. Demeclocycline (Declomycin) c. Furosemide (Lasix) d. Conivaptan (Vaprisol) e. Spironolactone (Aldactone)

a, d

32. A young woman tells the nurse that she gets frequent UTIs that seem to follow sexual intercourse. Which questions would the nurse ask? (Select all that apply.) a. "Do you use a diaphragm or spermicides for contraception?" b. "Do you feel guilty or embarassed about your secual activities?" c. "Have you considered abstaining from intercourse?" d. "Do you and your partner(s) wash the perineal area before intercourse?" e. "Some positions cause more irritation during sex. Have you noticed that?"

a, d, e

1. The nurse is interviewing a patient who is newly admitted to the unit with a diagnosis of anemia. Which assessment findings does the nurse expect? (Select all that apply.) a. Dyspnea on exertion b. Systolic hypertension c. Intolerance to heat d. Concave appearance of the nails e. Pallor of the ears f. Headache

a, d, e, f

16. A patient is recovering from a transphenoidal hypophysectomy. What postoperative nursing interventions apply to this patient? (Select all that apply.) a. Encouraging the patient to perform deep-breathing exercises b. Vigorous coughing and deep-breathing exercises c. Instructing on the use of a soft-bristled toothbrush for brushing the teeth d. Struct monitoring of fluid balance e. Hourly neurologic checks for first 24 hours f. Instructing the patient to alert the nurse regarding postnasal drip

a, d, e, f

54. The home health nurse is reviewing the medication list of a patient with CKD. The nurse calls the health care provider as a reminder that the patient might need which nutritional supplements? (Select all that apply.) a. Iron b. Magnesium c. Phospohrus d. Calcium e. Vitamin D f. Water-soluble vitamins

a, d, e, f

72. A patient has undergone a kidney biopsy. In the immediate post-procedural period, the nurse notifies the health care provider about which findings? (Select all that apply.) a. Hematuria with blood clots b. Localized pain at the site c. "Tampnade effect" d. Decreasing urine output e. Flank pain f. Decreasing blood pressure

a, d, e, f

72. A patient has undergone a kidney biopsy. In the immediate post-procedural period, the nurse notifies the health care provider about which findings? (Select all that apply.) a. Hematuria with blood clots b. Localized pain at the site c. "Tamponade effect" d. Decreasing urine output e. Flank pain f. Decreasing blood pressure

a, d, e, f

The health care provider tells the nurse that a client is to be started on a platelet inhibitor. About what drug does the nurse plan to teach the client? a. Clopidogrel (Plavix) b. Enoxaparin (Lovenox) c. Reteplase (Retavase) d. Warfarin (Coumadin)

a. Clopidogrel (Plavix) ANS: A Clopidogrel is a platelet inhibitor. Enoxaparin is an indirect thrombin inhibitor. Reteplase is a fibrinolytic agent. Warfarin is a vitamin K antagonist.

5. Which hormones are secreted by the thyroid gland? (Select all that apply.) a. Calcitonin b. Somatostatin c. Glucagon d. Thyroxine (T4) e. Aldosterone f. Triiodothyronine (T3)

a, d, f Calcitonin, Thyroxine (T4), Triiodothyronine (T3)

6. A patient is prescribed bromocriptine mesylate (Parlodel). Which information does the nurse teach the patient? (Select all that apply.) a. Get up slowly from a lying position. b. Take medication on an empty stomach. c. Take daily for purposes of raising GH levels to reduce symptom of acromegaly. d. Begin therapy with a maintenance level dose. e. Report watery nasal discharge to the health care provider immediately.

a, e Get up slowly from a lying position. Report watery nasal discharge to the health care provider immediately.

17. Which statements about the adrenal glands are correct? (Select all that apply.) a. The cortex secretes androgens in men and women. b. Catecholamines are secreted from the cortex. c. Glucocorticoids are secreted by the medulla. d. The medulla secretes hormones essential for life. e. The cortex secrets aldosterone that maintains extracellular fluid volume.

a, e The cortex secretes androgens in men and women. The cortex secrets aldosterone that maintains extracellular fluid volume.

Question 14 of 16 The charge nurse on the medical-surgical unit is making client assignments for the shift. Which client is the most appropriate to assign to an LPN/LVN? a. Client with Cushing's syndrome who requires orthostatic vital signs assessments b. Client with diabetes mellitus who was admitted with a blood glucose of 45 mg/dL c. Client with exophthalmos who has many questions about endocrine function d. Client with possible pituitary adenoma who has just arrived on the nursing unit

a. Client with Cushing's syndrome who requires orthostatic vital signs assessments An LPN/LVN will be familiar with Cushing's syndrome and the method for assessment of orthostatic vital signs. The client with a blood glucose of 45 mg/dL, the client with questions about endocrine function, and the client with a possible pituitary adenoma all have complex needs that require the experience and scope of practice of an RN.

An older client asks the nurse why "people my age" have weaker immune systems than younger people. What responses by the nurse are best? (Select all that apply.) a. "Bone marrow produces fewer blood cells." b. "You may have decreased levels of circulating platelets." c. "You have lower levels of plasma proteins in the blood." d. "Lymphocytes become more reactive to antigens." e. "Spleen function declines after age 60."

a. "Bone marrow produces fewer blood cells." c. "You have lower levels of plasma proteins in the blood." ANS: A, C The aging adult has bone marrow that produces fewer cells and decreased blood volume with fewer plasma proteins. Platelet numbers remain unchanged, lymphocytes become less reactive, and spleen function stays the same.

A client is receiving rivaroxaban (Xarelto) and asks the nurse to explain how it works. What response by the nurse is best? a. "It inhibits thrombin." b. "It inhibits fibrinogen." c. "It thins your blood." d. "It works against vitamin K."

a. "It inhibits thrombin." ANS: A Rivaroxaban is a direct thrombin inhibitor. It does not work on fibrinogen or vitamin K. It is not a "blood thinner," although many clients call anticoagulants by this name.

A client is having a bone marrow biopsy and is extremely anxious. What action by the nurse is best? a. Assess client fears and coping mechanisms. b. Reassure the client this is a common test. c. Sedate the client prior to the procedure. d. Tell the client he or she will be asleep.

a. Assess client fears and coping mechanisms. ANS: A Assessing the client's specific fears and coping mechanisms helps guide the nurse in providing holistic care that best meets the client's needs. Reassurance will be helpful but is not the best option. Sedation is usually used. The client may or may not be totally asleep during the procedure.

A nurse is assessing a dark-skinned client for pallor. What action is best? a. Assess the conjunctiva of the eye. b. Have the client open the hand widely. c. Look at the roof of the client's mouth. d. Palpate for areas of mild swelling.

a. Assess the conjunctiva of the eye. ANS: A To assess pallor in dark-skinned people, assess the conjunctiva of the eye or the mucous membranes. Looking at the roof of the mouth can reveal jaundice. Opening the hand widely is not related to pallor, nor is palpating for mild swelling.

A student nurse learns that the spleen has several functions. What functions do they include? (Select all that apply.) a. Breaks down hemoglobin b. Destroys old or defective red blood cells (RBCs) c. Forms vitamin K for clotting d. Stores extra iron in ferritin e. Stores platelets not circulating

a. Breaks down hemoglobin b. Destroys old or defective red blood cells (RBCs) e. Stores platelets not circulating ANS: A, B, E Functions of the spleen include breaking down hemoglobin released from RBCs, destroying old or defective RBCs, and storing the platelets that are not in circulation. Forming vitamin K for clotting and storing extra iron in ferritin are functions of the liver.

A client with unstable angina has received education about acute coronary syndrome. Which statement indicates that the client has understood the teaching? a. "This is a big warning; I must modify my lifestyle or risk having a heart attack in the next year." b. "Angina is just a temporary interruption of blood flow to my heart." c. "I need to tell my wife I've had a heart attack." d. "Because this was temporary, I will not need to take any medications for my heart."

a. "This is a big warning; I must modify my lifestyle or risk having a heart attack in the next year." Among people who have unstable angina, 10% to 30% have a myocardial infarction within 1 year. Although anginal pain is temporary, it reflects underlying coronary artery disease (CAD), which requires attention, including lifestyle modifications. Unstable angina reflects tissue ischemia, but infarction represents tissue necrosis. Clients with underlying CAD may need medications such as aspirin, lipid-lowering agents, anti-anginals, or antihypertensives.

A client in sickle cell crisis is dehydrated and in the emergency department. The nurse plans to start an IV. Which fluid choice is best? a. 0.45% normal saline b. 0.9% normal saline c. Dextrose 50% (D50) d. Lactated Ringer's solution

a. 0.45% normal saline

A client presents to the emergency department in sickle cell crisis. What intervention by the nurse takes priority? a. Administer oxygen. b. Apply an oximetry probe. c. Give pain medication. d. Start an IV line.

a. Administer oxygen.

A client has heparin-induced thrombocytopenia (HIT). The student nurse asks how this is treated. About what drugs does the nurse instructor teach? (Select all that apply.) a. Argatroban (Argatroban) b. Bivalirudin (Angiomax) c. Clopidogrel (Plavix) d. Lepirudin (Refludan) e. Methylprednisolone (Solu-Medrol)

a. Argatroban (Argatroban) b. Bivalirudin (Angiomax) d. Lepirudin (Refludan)

A nurse is preparing to administer a blood transfusion to an older adult. Understanding age-related changes, what alterations in the usual protocol are necessary for the nurse to implement? (Select all that apply.) a. Assess vital signs more often. b. Hold other IV fluids running. c. Premedicate to prevent reactions. d. Transfuse smaller bags of blood. e. Transfuse each unit over 8 hours.

a. Assess vital signs more often. b. Hold other IV fluids running.

A client has frequent hospitalizations for leukemia and is worried about functioning as a parent to four small children. What action by the nurse would be most helpful? a. Assist the client to make "sick day" plans for household responsibilities. b. Determine if there are family members or friends who can help the client. c. Help the client inform friends and family that they will have to help out. d. Refer the client to a social worker in order to investigate respite child care.

a. Assist the client to make "sick day" plans for household responsibilities.

A client has been admitted after sustaining a humerus fracture that occurred when picking up the family cat. What test result would the nurse correlate to this condition? a. Bence-Jones protein in urine b. Epstein-Barr virus: positive c. Hemoglobin: 18 mg/dL d. Red blood cell count: 8.2/mm3

a. Bence-Jones protein in urine

A client has a platelet count of 9000/mm3. The nurse finds the client confused and mumbling. What action takes priority? a. Calling the Rapid Response Team b. Delegating taking a set of vital signs c. Instituting bleeding precautions d. Placing the client on bedrest

a. Calling the Rapid Response Team

A student studying leukemias learns the risk factors for developing this disorder. Which risk factors does this include? (Select all that apply.) a. Chemical exposure b. Genetically modified foods c. Ionizing radiation exposure d. Vaccinations e. Viral infections

a. Chemical exposure c. Ionizing radiation exposure e. Viral infections

A nurse is caring for four clients with leukemia. After hand-off report, which client should the nurse see first? a. Client who had two bloody diarrhea stools this morning b. Client who has been premedicated for nausea prior to chemotherapy c. Client with a respiratory rate change from 18 to 22 breaths/min d. Client with an unchanged lesion to the lower right lateral malleolus

a. Client who had two bloody diarrhea stools this morning

A nurse caring for a client with sickle cell disease (SCD) reviews the client's laboratory work. Which finding should the nurse report to the provider? a. Creatinine: 2.9 mg/dL b. Hematocrit: 30% c. Sodium: 147 mEq/L d. White blood cell count: 12,000/mm3

a. Creatinine: 2.9 mg/dL

A nurse working with clients with sickle cell disease (SCD) teaches about self-management to prevent exacerbations and sickle cell crises. What factors should clients be taught to avoid? (Select all that apply.) a. Dehydration b. Exercise c. Extreme stress d. High altitudes e. Pregnancy

a. Dehydration c. Extreme stress d. High altitudes e. Pregnancy

The nurse is caring for a client with leukemia who has the priority problem of fatigue. What action by the client best indicates that an important goal for this problem has been met? a. Doing activities of daily living (ADLs) using rest periods b. Helping plan a daily activity schedule c. Requesting a sleeping pill at night d. Telling visitors to leave when fatigued

a. Doing activities of daily living (ADLs) using rest periods

A student nurse is learning about blood transfusion compatibilities. What information does this include? (Select all that apply.) a. Donor blood type A can donate to recipient blood type AB. b. Donor blood type B can donate to recipient blood type O. c. Donor blood type AB can donate to anyone. d. Donor blood type O can donate to anyone. e. Donor blood type A can donate to recipient blood type B.

a. Donor blood type A can donate to recipient blood type AB. d. Donor blood type O can donate to anyone.

A client has Crohn's disease. What type of anemia is this client most at risk for developing? a. Folic acid deficiency b. Fanconi's anemia c. Hemolytic anemia d. Vitamin B12 anemia

a. Folic acid deficiency

A client hospitalized with sickle cell crisis frequently asks for opioid pain medications, often shortly after receiving a dose. The nurses on the unit believe the client is drug seeking. When the client requests pain medication, what action by the nurse is best? a. Give the client pain medication if it is time for another dose. b. Instruct the client not to request pain medication too early. c. Request the provider leave a prescription for a placebo. d. Tell the client it is too early to have more pain medication.

a. Give the client pain medication if it is time for another dose.

A student nurse is helping a registered nurse with a blood transfusion. Which actions by the student are most appropriate? (Select all that apply.) a. Hanging the blood product using normal saline and a filtered tubing set b. Taking a full set of vital signs prior to starting the blood transfusion c. Telling the client someone will remain at the bedside for the first 5 minutes d. Using gloves to start the client's IV if needed and to handle the blood product e. Verifying the client's identity, and checking blood compatibility and expiration time

a. Hanging the blood product using normal saline and a filtered tubing set b. Taking a full set of vital signs prior to starting the blood transfusion d. Using gloves to start the client's IV if needed and to handle the blood product

The nurse is providing discharge instructions for a patient who had a cardiac catheterization. Which instructions must the nurse include? (SATA) a. Notify the health care provider for increased swelling, redness, warmth, or pain b. Leave the dressing in place for the first day c. Limit activity for at least 2 to 3 weeks d. Avoid lifting and exercise for a few days e. Report any bruise or hematoma to the health care provider

abd

When planning care for a client in the emergency department, which interventions are needed in the acute phase of myocardial infarction? (Select all that apply.) a. Morphine sulfate b. Oxygen c. Nitroglycerin d. Naloxone e. Acetaminophen f. Verapamil (Calan, Isoptin)

a. Morphine sulfate b. Oxygen c. Nitroglycerin Morphine is needed to reduce oxygen demand, preload, pain, and anxiety, and nitroglycerin is used to reduce preload and chest pain. Administering oxygen will increase available oxygen for the ischemic myocardium. Naloxone is a narcotic antagonist that is used for overdosage of opiates, not for MI. Acetaminophen may be used for headache related to nitroglycerin. Because of negative inotropic action, calcium channel blockers such as verapamil are used for angina, not for MI.

After receiving change-of-shift report in the coronary care unit, which client does the nurse assess first? a. The client with acute coronary syndrome who has a 3-pound weight gain and dyspnea b. The client with percutaneous coronary angioplasty who has a dose of heparin scheduled c. The client who had bradycardia after a myocardial infarction and now has a paced heart rate of 64 beats/min d. A client who has first-degree heart block, rate 68 beats/min, after having an inferior myocardial infarction

a. The client with acute coronary syndrome who has a 3-pound weight gain and dyspnea Dyspnea and weight gain are symptoms of left ventricular failure and pulmonary edema; this client needs prompt intervention. A scheduled heparin dose does not take priority over dyspnea; it can be administered after the client with dyspnea is taken care of. The client with a pacemaker and a normal heart rate is not in danger. First-degree heart block is rarely symptomatic, and the client has a normal heart rate; the client with dyspnea should be seen first.

Which interventions and actions does the nurse perform to detect and prevent kidney toxicity when caring for a patient after cardiac catheterization? (SATA) a. Provide IV and oral fluids for 12 to 24 hours b. Monitor intake and output c. Check the catheterization site every hour d. Administer acetylcysteine if ordered e. Keep the catheterized extremity straight for 6 hours

abd

Which laboratory tests are used to predict a patient's risk for CAD? (SATA) a. Cholesterol level b. Triglyceride level c. Prothrombin time d. Low-density lipoprotein level e. Albumin level

abd

The nurse is preparing to teach a client that metabolic syndrome can increase the risk for myocardial infarction (MI). Which signs of metabolic syndrome should the nurse include in the discussion? (Select all that apply.) a. Truncal obesity b. Hypercholesterolemia c. Elevated homocysteine levels d. Glucose intolerance e. Client taking losartan (Cozaar)

a. Truncal obesity b. Hypercholesterolemia d. Glucose intolerance e. Client taking losartan (Cozaar) A large waist size (excessive abdominal fat causing central obesity)—40 inches (102 cm) or greater for men, 35 inches (88 cm) or greater for women—is a sign of metabolic syndrome. Decreased high-density lipoprotein cholesterol (HDL-C) (usually with high low-density lipoprotein cholesterol)—HDL-C less than 40 mg/dL for men or less than 50 mg/dL for women—or taking an anticholesterol drug is a sign of metabolic syndrome. Increased fasting blood glucose (caused by diabetes, glucose intolerance, or insulin resistance) is included in the constellation of metabolic syndrome. Blood pressure greater than 130/85 mm Hg or taking antihypertensive medication indicates metabolic syndrome. Although elevated homocysteine levels may predispose to atherosclerosis, they are not part of metabolic syndrome.

28. When patients have problems with the kidneys or urinary tract, what is the most common symptom that prompts them to seek medical attention? a. Change in the frequency or amount of urination b. Pain in flank or abdomen or pain when urinating c. Noticing a change in the color or odor of the urine d. Exposure to a nephrotoxic substance

b

48. The nurse performs a dipstick urine test for a patient being evaluated for kidney problems. Glucose is present in the urine. How does the nurse interpret this result? a. Blood glucose level is greater than 200 mg/dL. b. The kidneys are failing to filter any glucose. c. The patient is at risk for hypoglycemia. d. The renal threshold has not been exceeded.

a?

Microalbuminuria has been shown to be a clear marker of widespread endothelial dysfunction in CVD. Which conditions should prompt patients to be tested annually for microalbuminuria? (SATA) a. Hypertension b. Metabolic syndrome c. Smoker d. Diabetes mellitus e. Use of anticoagulant therapy

abd

The nurse is giving a community presentation about heart disease in women. What information does the nurse include in the presentation? (SATA) a. Dyspnea on exertion may be the first and only symptoms of heart failure b. Symptoms are subtle or atypical c. Pain is often relieved by rest d. Having waist and abdominal obesity is a higher risk factor than having fat in buttocks and thighs e. Pain always responds to nitroglycerin f. Common symptoms include back pain, indigestion, nausea, vomiting, and anorexia

abdf

Which blood pressure reading require further assessment? (SATA) a. 90 mm Hg systolic b. 139 mm Hg Hg systolic c. 115 mm Hg systolic d 66 mm Hg diastolic e. 100 mm Hg diastolic

abe

The nurse is instructing a patient with CHF on what signs to look for when experiencing an exacerbation of CHF> Which are appropriate teaching points for this patient? (SATA) a. "It is possible to gain up to 10 or 15 lbs before edema develops" b. "Notify the provider of a weight loss of 3 to 5 lbs within 2 weeks" c. "Notify the provider of a weight gain of 2 lbs within 1 to 2 days" d. "Notify the provider if you notice that your shoes or rings feel tight" e. "Notify the provider if your skin becomes dry and scaly."

acd

The patient has a diagnosis of angina. Which assessment findings would the nurse expect to find? (SATA) a. Sudden onset of pain b. Intermittent pain relieved with sitting upright c. Substernal pain which may spread across chest, back, arms d. Pain usually lasts less than 15 minutes e. Sharp, stabbing pain which is moderate to severe

acd

Which are risk factors for CVD in women? (SATA) a. Waist and abdominal obesity b. Excess fat in the buttocks, hips, and thighs c. Postmenopausal d. Diabetes mellitus e. Asian ethnicity

acd

Which statements about blood pressure are accurate? (SATA) a. Pulse pressure is the difference between the systolic and diastolic pressures b. The right ventricle of the heart generates the greatest amount of blood pressure c. Diastolic blood pressure is primarily determined by the amount fo peripheral vasoconstriction d. To maintain adequate blood flow through the coronary arteries, mean arterial pressure (MAP) must be at least 60 mm Hg. e. Diastolic blood pressure is the highest pressure during contraction of the ventricles

acd

A patient comes to the ED reporting chest pain. In evaluating the patient's pain, which questions does the nurse ask the patient? (SATA) a. "How long does the pain last and how often does it occur?" b. "How do you feel about the pain?" c. "Is the pain different from any other episode of pain you've had?" d. "What activities were you doing when the pain first occurred?" e. "Where is the chest pain? What does it feel like?" f. "Have you had other signs and symptoms that occur at the same time?"

acdef

The nurse interprets a patient's serum lipid test. Which results suggest an increased risk for CVD? (SATA) a. LDL 160 mg/dL b HDL 60 mg/dL c. Total cholesterol 180 mg/dL d Triglycerides 175 mg/dL e Lp (a) 45 mg/dL

ade

The nurse is assessing a patient's nicotine dependency. Which questions does the nurse ask for an accurate assessment? (SATA) a. "How soon after you wake up in the morning do you smoke?" b. "What kind of cigarettes do you smoke?" c. "Do you inhale deeply when you smoke?" d. "Do you find it difficult not to smoke in places where smoking is prohibited?" e. "Do you smoke when you are ill?" f. "What happened the last time you tried to quit smoking"

adef

1. A patient has sustained a minor kidney injury. Which structure must remain functional in order to form urine from blood? a. Medulla b. Nephron c. Calyx d. Capsule

b

10. The nurse is talking to an older adult male patient who is reasonably healthy for his age, but has benign prostatic hyperplasia (BPH). Which conditioin does the BPH potentially place him at risk for? a. Prerenal acute kidney injury b. Postrenal acute kidney injury c. Polycystic kidney disease d. Acute glomerulonephritis

b

10. What is the most common cause of hyperthyroidism? a. Radiation to thyroid b. Graves' disease c. Thyroid cancer d. Thyroiditis

b

10. Which patient is most likely to exceed the renal threshold if there is noncompliance with the prescribed therapeutic regimen? a. Has recurrent kidney stone formation b. Has type 2 diabetes mellitus c. Has functional urinary incontinence d. Has biliary obstruction

b

11. Which personal action is most likely to cause the kidneys to produce and release erythropoietin? a. Person moves to a low desert area where the humidity is very low. b. Person moves to a high-altitude area where atmospheric oxygen is low. c. Person drinks an excessive amount of fluid that results in fluid overload. d.d Person eats a large high-protein meal after a rigorous exercise workout.

b

12. The nurse is assessing a patient diagnosed with hyperthyroidism and observes dry, waxy swelling of the front surfaces of the lower legs. How does the nurse interpret this finding? a. Globe lag b. Pretibial myxedema c. Exophthalmos d. Goiter

b

12. Vitamin D is converted to its active from in the kidney. If this function fails, which electrolyte imbalance will occur? a. Hyperkalemia b. Hypocalcemia c. Hypernatremia d. Hypoglycemia

b

15. The nurse is helping a patient prepare for induction therapy for acute leukemia. What information will the nurse give to the patient? a. A donor is needed for hematopoietic stem cell transplantation. b. Prolonged hospitalization is common to protect against infection. c. The therapy may last from months to years to maintain remission. d. Success of the therapy results in remission and the intent is to cure.

b

16. An older adult male patient has a history of an enlarged prostate. The patient is most likely to report which symptom associated with this condition? a. Inability to sense the urge to void b. Difficulty starting the urine stream c. Excreting large amounts of very dilute urine d. Burning sensation when urinating

b

17. The nurse is providing instructions to a patient who is taking the anti-thyroid medication propylthiouracil (PTU). The nurse instructs the patient to notify the health care provider immediately if which sign/symptom occurs? a. Weight gain b. Dark-colored urine c. Cold intolerance d. Headache

b

18. A patient is in the diuretic phase of AKI. During this phase, what is the nurse mainly concerned about? a. Assessing for hypertension and fluid overload b. Monitoring for hypovolemia and electrolyte loss c. Adjusting the dosage of diuretic medications d. Balancing diuretic therapy with intake

b

18. After the nurse instructs a patient with PKD on home care, the patient knows to contact the health care provider immediately when what sign/symptom occurs? a. Urine is a clear, pale yellow color. b. Weight has increased by 3 pounds in 2 days. c. Two days have passed since the last bowel movement. d. Morning systolic blood pressure has decreased by 5 mm Hg.

b

39. The nurse is reviewing a patient's laboratory results. In the early phase of CKD, the patient is at risk for which electrolyte abnormality? a. Hyperkalemia b. Hyponatremia c. Hypercalcemia d. Hypokalemia

b

19. A patient who has been diagnosed with Graves' disease is going to receive radioactive iodine (RAI) in the oral form of 131 I. What does the nurse teach the patient about how this drug works? a. It destroys the hormones T3 and T4. b. It destroys the tissue that produces thyroid hormones. c. It blocks thyroid hormone production. d. It prevents T4 from being converted to T3.

b

19. A patient with prerenal azotemia is administered a fluid challenge. In evaluating response to the therapy, which outcome indicates that the goal was met? a. Patient reports feeling better and indicates an eagerness to go home. b. Patient produces urine soon after the initial bolus. c. The therapy is completed without adverse effects. d. The health care provider orders a diuretic when the challenge is completed.

b

19. Preoperative care for a patient who has had a transphenoidal hypophysectomy includes which intervention? a. Encouraging coughing and deep-breathing to decrease pulmonary complications b. Testing nasal drainage for glucose to determine whether it contains CSF c. Keeping the bed flat to decrease central CSF leakage d. Assisting the patient with brushing the teeth to reduce risk of infection

b

19. Which ethnic group has the highest risk for kidney failure and needs special attention for patient teaching related to hypertension and sodium intake? a. Caucasian Americans b. African Americans c. Asian Americans d. Native Americans

b

2. Which group has the highest prevalence of urinary tract infections (UTIs)? a. Young men b. Older women c. Older men d. School-aged girls

b

20. The nursing student sees an order for a urinalysis for a patient with frequency, urgency, and dysuria. In order to collect the specimen, what does the student do? a. Use sterile technique to insert a small-diameter (6 Fr) catheter. b. Instruct the patient on how to collect a clean-catch specimen. c. Tell the patient to urinate approximately 10 mL into a specimen cup. d. Take the urine from a bedpan and transfer it into a specimen cup.

b

20. While caring for a postoperative patient following a transsphenoidal hypophysectomy, the nurse observes nasal drainage that is clear with yellow color at the edge. This "halo sign" is indicative of which condition? a. Worsening neurologic status of the patient b. Drainage of CSF from the patient's nose c. Onset of postoperative infection d. An expected finding following this surgery

b

21. The nurse is caring for a patient with AKI that developed after a severe anaphylactic reaction. What is a primary treatment goal of the initial phase that will help to prevent permanent kidney damage for this patient? a. Correct fluid volume by administering IV normal saline. b. Maintain a mean arterial pressure (MAP) of 65 mm Hg. c. Prevent kidney infections by administering antibiotics d. Give antihistamines to prevent allergic response.

b

21. The nurse is reviewing the laboratory results for an older adult patient with an indwelling catheter. The urine culture is pending, but the urinalysis shows greater than 10^5 colony-forming units, and the differential WBC count shows a "left shift." How does the nurse interpret these findings? a. Interstitial cystitis b. Urosepsis c. Complicated cystitis d. Radiation-induced cystitis

b

22. In PKD, the effect on the renin-antiotensin system in the kidney has which result? a. Adrenal insufficiency b. Increased blood pressure c. Increased urine output d. Oliguria

b

22. The nurse is interviewing a 35-year-old woman who needs evaluation for a potential kidney problem. The woman reports she has been pregnant twice and has two healthy children. What would the nurse ask about health problems that occurred during pregnancy? a. "How much weight did you gain during the pregnancy" b. "Were you treated for gestational diabetes?" c. "Did both of your pregnancies go to full-term? d. "Did you have a urinary catheter inserted during labor?"

b

23. The nurse tells the patient that the health care provider recommends a fluid intake of at least 2 liter per day. The nurse then asks the patient to report on fluid intake over the past 24 hours to assess typical intake. The patient reports 15 ounces of coffee and 10 ounces of juice for breakfast; 10 ounces of skim milk for a mid-morning snack, 12 ounces of protein shake for lunch, 1/2 liter of sports drink in the afternoon and 3 ounces of wine for dinner. After calculating the 24-hour fluid intake, what does the nurse tell the patient? a. Fluid consumption should be increased by at least 2 more servings. b. Fluid consumption is meeting the 2 liter/day recommendation. c. Fluid consumption exceeds recommendation, therefore eliminate the wine. d. Fluid consumption only includes liquids such as water, juice, or milk.

b

23. What is the disorder that results from a deficiency of vasopressin (ADH) from the posterior pituitary gland called? a. SIADH b. DI c. Cushing's syndrome d. Addison's disease

b

24. The nurse is taking a history on a 55-year-old patient who denies any serious chronic health problems. Which sudden onset sign/symptom suggests possible kidney disease in this patient? a. Weakness b. Hypertension c. Confusion d. Dysrhythmia

b

25. A patient received an antibiotic prescription several hours ago and has started the medication, but requests "some relief from the burning." What comfort measures does the nurse suggest to the patient? a. Take over-the-counter acetaminophen. b. Sit in a sitz bath and urinate into the warm water. c. Place a cold pack over the perineal area. d. Rest in a recumbent positioin with legs elevated.

b

26. A patient with lymphoma requires a hematopoietic stem cell transplant and a donor is being sought. which type of transplant is likely to yield the best results? a. Synthetic human leukocyte antigen (HLA) b. HLA-identical twin sibling c. HLA-matched first-degree relative d. HLA-matched stem cells from an umbilical cord of a related donor

b

28. A male college student comes to the clinic reporting burning or difficulty with urination and a discharge from the urethral meatus. Based on the patient's chief complaint, what is the most logical question for the nurse to ask about the patient's pas medical history? a. "Do you have a history of a narrow urethra or a stricture?" b. "Could you have been exposed to a sexually transmitted disease (STD)?" c. "Do you have a history of kidney stones?" d. "Have you been drinking an adequate amount of fluids?"

b

29. The` nurse is reviewing the laboratory results for a patient being evaluated for trouble with passing urine. The urinalysis shows tubular epithelial cells on microscopic examination. How does the nurse interpret this finding? a. The obstruction is resolving. b. The obstruction is prolonged. c. Glomerular filtration rate is reduced. d. Glomerular filtration rate is adequate.

b

3. In which circumstance is the regulatory role of aldosterone most important in order for the person to maintain homeostasis? a. Person is having pain related to a kidney stone. b. Person has been hiking in the desert for several hours. c. Person experiences stress incontinence when coughing. d. Person experiences a burning sensation during urination.

b

3. The nurse assesses a patient in the emergency department (ED) and finds the following: constipation, fatigue with increased sleeping time, impaired memory, facial puffiness, and weight gain. Which deficiency does the nurse recognize? a. Hyperthyroidism b. Hypothyroidism c. Hyperparathyroidism d. Hypoparathyroidism

b

3. Which patient has the highest risk for developing a complicated UTI? a. 26-year-old woman who is sexually active, but not currently pregnant b. 22-year-old man who has a neurogenic bladder due to spinal cord injury c. 35-year-old woman who had three full-term pregnancies and a miscarriage d. 53-year-old woman who is having some menstrual irregularities

b

30. A patient has been prescribed thyroid hormone for treatment of hypothyroidism. Within what time frame does the patient expect improvement in mental awareness with this treatment? a. A few days b. 2 weeks c. 1 month d. 3 months

b

30. The nurse is performing an assessment of the renal system. What is the first step in the assessment process? a. Percuss the lower abdomen; continue toward the umbilicus b. Observe the flank region for asymmetry or dicoloration c. Listen for a bruit over each renal artery d. Lightly palpate the abdomen in all quadrants

b

30. The nurse requests a dietary consult to address the patient's high rate of catabolism. Which nutritional element is directly related to this metabolic process? a. Carbohydrates b. Inability to excrete excess electrolytes c. Conversion of body fat into glucose d. Presence of retained nitrogenous wastes

b

30. Which statement about the pathophysiology of SIADH is correct? a. ADH secretion is inhibited in the presence of low plasma osmolality. b. Water retention results in dilutional hyponatremia and expanded extracellular fluid (ECF) volume. c. The glumerulus is unable to increase its filtration rate to reduce the excess plasma volume. d. Renin and aldosterone are released and help decrease the loss of urinary sodium.

b

31. The nurse is caring for a patient with a nephrostomy. The nurse notifies the health care provider about which assessment finding? a. Urine drainage is red-tinged 4 hours post-surgery. b. Amount of drainage decreases and the patient has back pain. c. There is a small steady drainage for the first 4 hours post-surgery.

b

31. While being interviewed for admission, a patient tells the nurse that he has Christmas disease. What does the nurse document this as? a. Hemophilia A b. Hemophilia B c. Thrombocytopenia d. Sickle cell disease

b

33. The nurse is inwerting an intravenous needle into an older patient for the purpose of administering a blood transfusion. Which size needle should the nurse select? a. 22-gauge needle b. 20-gauge needle c. 19-gauge needle d. 23-gauge butterfly needle

b

34. A patient is receiving a blood transfusion. Which solution does the nurse administer with the blood? a. Ringer's lactate b. Normal saline c. Dextrose in water d. Dextrose in saline

b

34. A ptient is diagnosed with renal artery stenosis. Which sound does the nurse expect to hear by auscultation when a bruit is present in a renal artery? a. Quiet, pulsating sound b. Swishing sound c. Faint wheezing d. No sound at all

b

34. Which factor/manifestation is primarily associated with acute pyelonephritis? a. Obstruction caused by hydroureter b. Active bacterial infection c. Decreased urine specific gravity d. Alcohol abuse

b

35. The nurse is assessing a patient who reports chills, high fever, and flank pain with urinary urgency and frequency. On physical examination, the patient has costovertebral angle (CVA) tenderness, pulse is 110 beats/min, and respirations are 28/min. How does the nurse interpret these findings? a. Complicated cystitis b. Acute peylonephritis c. Chronic pyelonephritis d. Acute glomerulonephritis

b

36. The nurse reads in the assessment note made by the advanced-practice nurse that the "left kidney cannot be palpated." How does the nurse interpret this notation? a. The left kidney is smaller than normal, which indicates CKD. b. The left kidney is normally deeper and often cannot be palpated. c. The palpation of kidneys should be repeated by another provider. d. The patient is too obese for this type of examination.

b

37. Which statement about pheochromocytoma is correct? a. It is most often malignant. b. It is a catecholamine-producing tumor. c. It is found only in the adrenal medulla. d. It is manifested by hypotension.

b

38. A patient in the emergency department is diagnosed with possible pheochromcytoma. What is the priority nursing intervention for this patient? a. Monitor the patient's intake and output and urine specific gravity. b. Monitor blood pressure for severe hypertension. c. Monitor blood pressure for severe hypotension. d. Administer medication to increase cardiac output.

b

38. After hospitalization for myxedema, a patient is prescribed thyroid replacement medication. Which statement by the patient demonstrates a correct understanding of this therapy? a. "I'll be taking this medication until my symptoms are completely resolved." b. "I'll be taking thyroid medication for the rest of my life." c. "Now that I'm feeling better, no changes in my medication will be necessary." d. "I'm taking this medication to prevent symptoms of an overactive thyroid gland."

b

39. The nurse is preparing to assess a female patient's urethra prior to the insertion of a Foley catheter. In addition to gloves, which equipment does the nurse obtain to perform the initial assessment? a. Glass slide b. Good light source c. Speculum d. Cotton ball

b

40. Which intervention applies to a patient with pheochromocytoma? a. Assist to sit in a chair for blood pressure monitoring. b. Instruct not to smoke, drink coffee, or change positions suddenly. c. Encourage to maintain an active exercise schedule including activity such as running. d. Encourage one glass of red wine nightly to promote rest.

b

41. What is the hallmark of thyroid cancer? a. Aggressive tumors b. Elevated serum thyroglobulin level c. Metastasis to other organs d. Invasion of blood vessels

b

41. Which intervention is contraindicated for a patient with pheochromocytoma? a. Monitoring blood pressure b. Palpating the abdomen c. Collecting 24-hour urine specimens d. Instructing the patient to limit activity

b

42. A patient has been performing Kegel exercises for 2 months. How does the nurse know whether the exercises are working? a. Incontinence is still present, but the patient states that it is less. b. The patient is able to stop the urine stream. c. There are no complaints of urgency from the patient. d. The patient is using absorbent undergarments for protection.

b

42. The nurse is assessing a patient with possible acute glomerulonephritis. During the inspection of the hands, face, and eyelids, what is the nurse primarily observing for? a. Redness b. Edema c. Rashes d. Dryness

b

42. Which outcome indicates that engraftment of transplanted cells in the patient's bone marrow has been successful? a. There is no evidence of graft-versus-host disease. b. WBC, RBC, and platelet counts begin to rise. c. Laboratory results indicate probable regressive chimerism. d. Laboratory results show decreasing percentage of donor cells.

b

43. The nurse is caring for a patient with dehydration. Which laboratory test results does the nurse anticipate to see for this patient? a. BUN and creatinine ratio stay the same. b. BUN rises faster than creatinine level. c. Creatinine rises faster than BUN. d. BUN and creatinine have a direct relationship.

b

44. A patient reports fatigue, bone pain, and frequent bacterial infections. Further investigation reveals anemia, hypercalcemia, and x-ray findings show bone thinning with areas of bone loss that resemble Swiss cheese. The signs/symptoms and diagnostic findings are consistent with which disorder? a. Acute leukemia b. Multiple myeloma c. Non-Hodgkin's lymphoma d. Sickle cell anemia

b

45. The nurse is teaching a patient with urge incontinence about dietary modifications. What is the best information the nurse gives to the patient about fluid intake? a. Drink at least 2000 mL per day unless contraindicated. b. Drink 120 mL every hour or 240 mL every 2 hours and limit fluids after dinner. c. Drink fluid freely in the morning hours, but limit intake before going to bed. d. Drinking water is especially good for bladder health.

b

46. A patient has agreed to try a bladder training program. What is the priority nursing intervention in starting this therapy? a. Start a schedule for voiding (e.g., every 2-3 hours). b. Teach the patient how to be alert, aware, and able to resist the urge to urinate. c. Convince the patient that the bladder issues are controlling his/her lifestyle. d. Give a thorough explanation of the problem of stress incontinence.

b

46. A patient has positive Trousseau's and Chvostek's signs resulting from hypoparathyrodism. What condition does this assessment finding indicate? a. Hypercalcemia b. Hypocalcemia c. Hyperphosphatemia d. Hypophosphatemia

b

46. The night shift nurse sees a patient with kidney failure sitting up in bed. The patient states, "I feel a little short of breath at night or when I get up to walk to the bathroom." What assessment does the nurse do? a. Check for orthostatic hypotension because of potential volume depletion. b. Auscultate the lungs for crackles, which indicate fluid overload. c. Check the pulse and blood pressure for possible decreased cardiac output. d. Assess for normal sleep pattern and need for a prn sedative.

b

46. The nurse sees that an older patient has a blood osmolarity of 303 mOsm/L. Which additional assessment will the nurse make before notifying the health care provider about the laboratory results? a. Patient's mental status b. Signs of dehydration c. Patient's temperature d. Odor of the urine

b

47. A patient is diagnosed with chronic glomerulonephritis. The patient's spouse reports that the patient is irritable, forgetful, and has trouble concentrating. Which assessment finding does the nurse expect on further examination? a. Increased respiratory rate b. Elevated blood urea nitrogen c. High white count with a left shift d. Low blood pressure and bradycardia

b

49. A patient has chronic glomerulonephritis. In order to assess for uremic symptoms, what does the nurse do? a. Evaluate the blood urea nitrogen (BUN). b. Ask the patient to extend the arms and hyperextend the wrists. c. Gently palpate the flank for asymmetry and tenderness. d. Auscultate for the presence of an S3 heart sound.

b

49. Which patient with incontinence is most likely to benefit from a surgical intervention? a. patient with vaginal atrophy and altered urethral competency b. Patient with reflex (overflow) incontinence caused by obstruction c. Patient with functional incontinence related to musculoskeletal weakness d. Patient with urge incontinence or overactive bladder

b

5. Which factor is a main assessment finding that signifies hypothyroidism? a. Irritability b. Cold intolerance c. Diarrhea d. Fatigue

b

50. Which drug decreases cortisol production? a. Mitotane (Lysodren) b. Aminoglutethimide (Cytadren) c. Cyproheptadine (Periactin) d. Hydrocortisone (Cortef)

b

51. A patient is scheduled for bilateral adrenalectomy. Before surgery, steroids are to be given. Which is the reasoning behind the administration of this drug? a. To promote glycogen storage by the liver for body energy reserves b. To compensate for sudden lack of adrenal hormones following surgery c. To increase the body's inflammatory response to promote scar formation d. To enhance urinary excretion of salt and water following surgery

b

51. Which test is the best indicator of kidney function? a. Urine osmolarity b. Serum creatinine c. Urine pH d. BUN

b

52. A patient receives dialysis therapy and the health care provider has ordered sodium restriction to 3 g daily. What does the nurse teach the patient? a. Add smaller amounts of salt at the table or during cooking. b. Identify foods that are high in sodium (e.g., bacon, potato chips, fast foods). c. Avoid foods that have a metallic, salty, or bitter taste. d. Eat larger amounts of bland foods with very minimal amounts of spicing.

b

53. Which patient history factor is considered causative for acute glomerulonephritis? a. Urinary incontinence 6 months ago b. Strep throat 3 weeks ago c. Kidney stones 2 years ago d. Milk hypertension diagnosed 1 year ago

b

53. Which statement about a patient with hyperaldosteronism after a successful unilateral adrenalectomy is correct? a. The low-sodium diet must be continued postoperatively. b. Glucocorticoid replacement therapy is temporary. c. Spironolactone (Aldactone) must be taken for life. d. Additional measures are needed to control hypertension.

b

54. In older adults, assessment findings of fatigue, altered thought processes, dry skin, and constipation are often mistaken for signs of aging rather than assessment findings for which endocrine disorder? a. Hyperthyroidism b. Hypothyroidism c. Hyperparathyroidism d. Hypoparathyroidism

b

55. The nurse is caring for a patient with ESKD and dialysis has been initiated. Which drug order does the nurse question? a. Erythropoietin b. Diuretic c. ACE inhibitor d. Calcium channel blocker

b

56. A patient with urinary incontinence is prescribed oxybutynin (Ditropan). What precautions or instructions does the nurse provide related to this therapy? a. Avoid aspirin or aspirin-containing products. b. Increase fluids and dietary fiber intake. c. Report any unusual vaginal bleeding. d. Change positions slowly, especially in the morning.

b

56. During the day, the nursing student is measuring urine output and observing for urine characteristics in a patient. Which abnormal finding is the most urgent, which must be reported to the supervising nurse? a. Specific gravity is decreased b. Output is decreased c. pH is decreased d. Color has changed

b

56. during the day, the nursing student is measuring urine output and observing for urine characteristics in a patient. Which abnormal finding is the most urgent, which must be reported to the supervising nurse? a. Specific gravity is decreased b. Output is decreased c. pH is decreaed d. Color has changed

b

57. A patient in the emergency department who reports lethargy, muscle weakness, nausea, vomiting, and weight loss over the past weeks is diagnosed with Addisonian crisis (acute adrenal insufficiency). Which drug(s) does the nurse expect to administer to this patient? a. Beta blocker to control the hypertension and dysrhythmias b. Solu-Cortef IV along with IM injections of hydrocortisone c. IV fluids of D5NS with KCl added for dehydration d. Spironolactone (Aldactone) to promote diuresis

b

58. A patient has late stage chronic glomerulonephritis. Which educational brochure would be the most appropriate to prepare for the patient? a. "How to Take Your antiinfective Medications" b. "Important Points to Know about Dialysis." c. "What Are the Side Effects of Radiation Therapy?" d. "Precautions to Take During Immunosuppressive Therapy"

b

58. The nurse is reviewing the medication list and appropriate dose a adjustments made for a patient with CKD. The nurse would question the use and/or dosage adjustment of which type of medication? a. antibiotics b. Magnesium antacids c. Oral antidiabetics d. Opioids

b

59. A patient is admitted for an elective orthopedic surgical procedure. The patient also has a personal and family history for urolithiasis. Which circumstance creates the greatest risk for recurrent urolithiasis? a. Giving the patient milk with every meal tray b. Keeping the patient NPO for extended periods c. Giving the patient an opioid narcotic for pain d. Inserting an indwelling catheter for the procedure

b

6. Which sign/symptom is one of the first indicators of hyperthyroidism that is often noticed by the patient? a. Eyelid or globe lag b. Vision changes or tiring of the eyes c. Protruding eyes d. Photophobia

b

60. The nurse is reviewing the patient's history, assessment findings, and laboratory results for a patient with suspected kidney problems. Which manifestation is the main feature of nephrotic syndrome? a. Flank asymmetry b. Proteinuria greater than 3.5 g of protein in 24 hours c. Serum sodium 148 mmol/L d. Serum cholesterol (total) 190 mg/dL

b

61. A patient had a renal scan. What is included in the post-procedural care for this patient? a. Administer laxatives to cleanse the bowel. b. Encourage oral fluids to assist excretion of isotope. c. Administer captopril (Capoten) to increase blood flow. d. Insert a urinary catheter to measure urine output.

b

62. A patient with CKD is restless, anxious, and short of breath. The nurse hears crackles that begin at the base of the lungs. The pulse rate is increased and the patient has frothy, blood-tinged sputum. What does the nurse do first? a. Facilitat transfer to the ICU for aggressive treatment. b. Place the patient in a high-Fowler's position. c. Continue to monitor vital signs and assess breath sounds. d. Administer a loop diuretic such as furosemide (Lasix).

b

64. A patient returns to the unit after a renal scan. Which instruction about the patient's urine does the nurse give to the UAP caring for the patient? a. It is radioactive, so it should be handled with special biohazard precautions. b. It does not place anyone at risk because of the small amount of radioactive material. c. Its radioactivity is dangerous only to those who are pregnant. d. It is potentially dangerous if allowed to sit for prolonged periods in the commode.

b

64. A patient with diabetic nephropathy reports having frequent hypoglycemic episodes "so my doctor reduced my insulin, which means my diabetes is improving." What is the nurse's best response? a. "Congratulations! You must be following the diet and lifestyle instructions very carefully." b. "When kidney function is reduced, the insulin is available for a longer time and thus less of it is needed." c. "You should probably talk to your doctor again. you have been diagnosed with nephropathy and that changes the situation." d. "Let me get you a brochure about the relationship of diabetes and kidney disease. It is a complex topic and hard to understand."

b

65. The student nurse is assisting in the postoperative care of a patient who had a recent nephrectomy. The student demonstrates a reluctance to move the patient to change the linens because "the patient seems so tired." The nurse reminds the student that a priority assessment for this patient is to assess for which factor? a. Skin breakdown on the patient's back b. Blood on the linens beneath the patient c. Urinary incontinence and moisture d. The patient's ability to move self in bed

b

65. What is an advantage of a renal scan compared to a CT scan for diagnosing the perfusion, function, and structure of the kidneys? a. Renal scan is more readily tolerated by elderly patients and small children. b. Renal scan is preferred if the patient is allergic to iodine or has impaired kidney function. c. Renal scans are more likely to detect pathologic changes that CT scans do not detect. d. Renal scan requires less pre- and post-procedural care than CT scan

b

66. After a nephrectomy, a patient has a large urine output because of adrenal insufficiency. What does the nurse anticipate the priority intervention for this patient will be? a. ACE inhibitor to control the hypertension and decrease protein loss in urine b. Straight catheterization or bedside bladder scan to measure residual urine c. IV fluid replacement because of subsequent hypotension and oliguria d. IV infusion of tesirolimus (Torisel), to inhibit cell division

b

66. The nurse is teaching a patient scheduled for an ultrasonography. What pre-procedural instruction does the nurse give the patient? a. Void just before the test begins. b. Drink water to fill the bladder. c. Stop routine medications. d. Have nothing to eat or drink after midnight.

b

66. Which clinical manifestation indicates to the nurse that interventions for the patient's renal colic are effective? a. Urine is pink-tinged. b. Patient reports that pain is relieved. c. Urine output is 50 mL/min. d. Bladder scan shows no residual urine.

b

67. As a result of kidney failure, excessive hydrogen ions cannot be excreted. With acid retention, the nurse is most likely to observe what type of respiratory compensation? a. Cheyne-Stokes respiratory pattern b. Increased depth of breathing c. Decreased respiratory rate and depth d. Increased arterial carbon dioxide levels

b

69. A patient has undergone a kidney biopsy. What does the nurse monitor for in the patient related to this procedure? a. Nephrotoxicity b. Hemorrhage c. Urinary retention d. Hypertension

b

7. The nurse is caring for a patient who sustained major injuries in an automobile accident. Which blood pressure will result in compromised kidney function, in particular the glomerular filtration rate (GFR)? a. 150/70 mm Hg b. 70/40 mm Hg c. 80/60 mm Hg d. 140/80 mm Hg

b

7. The nurse reads in the patient's chart that he has acute-on-chronic kidney disease. How does the nurse interpret this information? a. Kidney disease has progressed to the need for dialysis or transplant. b. Patient has chronic kidney disease and has sustained an acute kidney injury. c. Acute kidney injury requires aggressive management to prevent chronic disease. d. The condition could by acute or chronic; further diagnostic testing is needed.

b

70. The health care provider informs the nurse that there is a change in orders because the patient has a decrease in creatinine clearance rate. What change does the nurse anticipate? a. Fluid restriciton b. Reduction of drug dosage c. Limitations on activity level d. Modifications of diet

b

72. A patient and family are trying to plan a schedule that coordinates with the patient's dialysis regimen. The patient asks, "How often will I have to go and how long does it take?" What is the nurse's best response? a. "If you are compliant with the diet and fluid restrictions, you spend less time in dialysis; about 12 hours a week." b. "Most patients require about 12 hours per week; this is usually divided into three 4-hour treatments." c. "It varies from patient to patient. You will have to call your health care provider for specific instructions." d. "If you gain a large amount of fluid weight, a longer treatment time may be needed to prevent severe side effects."

b

74. The employee health nurse is conducting a presentation for employees who work in a paint manufacturing plant. In order to protect against bladder cancer, the nurse advises that everyone who works with chemicals should do what? a. Shower with mild soap and rinse well before they come to work. b. Use personal protective equipment such as gloves and masks. c. Limit their exposure to chemicals and fumes at all times. d. Avoid hobbies such as furniture refinishing that further expose to chemicals.

b

75. After a nephrectomy, one adrenal gland remains. Based on this knowledge, which type of medication replacement therapy does the nurse expect if the remaining adrenal gland function is insufficient? a. Potassium b. Steroid c. Calcium d. Estrogen

b

75. The nurse is assessing a patient's extremity with an areriovenous graft. The nurse notes a thrill and a bruit, and the patient reports numbness and a cool feeling in the fingers. How does the nurse interpret this information in regard to the graft? a. The graft is functional and these symptoms are expected. b. The patient has "steal syndrome" and may need surgical intervention. c. The graft is patent, but the blood is flowing in the wrong direction. d. The patient needs to increase active use of hands and fingers.

b

78. A patient is diagnosed with kidney cancer and the health care provider recommends the best therapy. Which treatment does the nurse anticipate teaching the patient about? a. Chemotherapy b. Surgical removal c. Hormonal therapy d. Radiation therapy

b

8. Damage to which renal structure or tissues can change the actual production of urine? a. Kidney parenchyma b. Convoluted tubules c. Calyces d. Ureters

b

8. For a patient who needs an indwelling catheter for at least 2 weeks, which intervention would help reduce the bacterial colonization along the catheter? a. Secure the catheter to the female patient's thigh. b. Consider the use of a coated catheter. c. Wash the urine bag and outflow tube every day. d. Apply antiseptic ointment to the catheter tubing.

b

8. The laboratory results for a 53-year-old patient indicate a low T3 level and elevated thyroid-stimulating hormone (TSH). What do these results indicate? a. Hyperthyroidism b. Hypothyroidism c. Malfunctioning pituitary gland d. Normal laboratory values for this age

b

80. The nurse is caring for a postoperative nephrectomy patient. The nurse notes during the first several hours of the shift a marked and steady downward trend in blood pressure. How does the nurse interpret this finding? a. Hypertension has been corrected. b. Internal hemorrhage is possible. c. The other kidney is failing. d. This is an expected response to medication.

b

86. The nurse is teaching a patient about performing PD at home. In order to identify the earliest manifestation of peritonitis, what does the nurse instruct the patient to do? a. Monitor temperature before starting PD. b. Check eh effluent for cloudiness. c. Be aware of feelings of malaise. d. Monitor for abdominal pain.

b

9. A patient with PKD reports sharp flank pain followed by blood in the urine. How does the nurse interpret these signs/symptoms? a. Infection b. Ruptured cyst c. Increased kidney size d. Ruptured renal artery aneurysm

b

9. The nurse hears in report that the patient is being treated for a fungal UTI. In addition to performing routine care and assessments, the nurse is extra-vigilant for signs/symptoms of which systemic disorder that may underlie the fungal UTI? a. Chronic cardiac disease b. Immune system compromise c. Respiratory system dysfunction d. Connective tissue disorder

b

9. a patient admitted for sickle cell crisis is being discharged home. Which statement by the patient indicates the need for further post-discharge instruction? a. "I will stop running 2 miles every morning." b. "I will visit my friends in Denver." c. "I will avoid the sauna at the gym." d. "I will not drink alcoholic beverages."

b

95. The nurse is caring for the kidney transplant patient who is 3 days postsurgery. The nurse notes a sudden and abrupt decrease in urine. The nurse alerts the health care provider because this is a sign of which anomaly? a. Rejection b. Thrombosis c. Stenosis d. Infection

b

A patient entering the cardiac rehabilitation unit seems optimistic and at time unexpectedly cheerful and upbeat. Which statement by the patient causes the nurse to suspect a maladaptive use of denial in the patient? a. "I am sick and tired of talking about these dietary restriction. Could we talk about it tomorrow?" b. "Oh, I don't really need that medication information. I'm sure that I'll soon be able to get by without it" c. "This whole episode of heart problem has been an eye-opener for me, but I really can't wait to get out of her" d. "That doctor is really driving me crazy with all his instructions. Could you put all that information away in my suitcase?"

b

A patient has been prescribed thyroid hormone for treatment of hypothyroidism. Within what time frame does the patient expect improvement in mental awareness with this treatment? a. A few days b. 2 weeks c. 1 month d. 3 months

b

A patient who has been diagnosed with Graves' disease is going to receive radioactive iodine (RAI) in the oral form of 131I. What does the nurse teach the patient about how this drug works? a. It destroys the hormones T3 and T4. b. It destroys the tissue that produces thyroid hormones. c. It blocks thyroid hormone production. d. It prevents T4 from being converted to T3.

b

After hospitalization for myxedema, a patient is prescribed thyroid replacement medication. Which statement by the patient demonstrates a correct understanding of this therapy? a. "I'll be taking this medication until my symptoms are completely resolved." b. "I'll be taking thyroid medication for the rest of my life." c. "Now that I'm feeling better, no changes in my medication will be necessary." d. "I'm taking this medication to prevent symptoms of an overactive thyroid gland."

b

Based on knowledge of albumin's role in maintaining osmotic pressure of the blood, which sign/symptom would the nurse observe for if the patient has low albumin levels? a. Fever b. Edema c. Dizziness d. Pain

b

For a patient who has a dysfunction of the bone marrow, which sign/symptom is the nurse most likely to observe? a. Long bone pain b. Fatigue c. Loss of appetite d. Weight gain

b

In a hypovolemic patient, stretch receptors in the blood vessels sense a reduced volume or pressure and send fewer impulses to the central nervous system. As a result, which signs/symptoms does the nurse expect to observe in the patient? a. Reddish mottling to skin and a blood pressure elevation b. Cool, pale skin and tachycardia c. Warm, flushed skin with low blood pressure d. Pale pink skin with bradycardia

b

In assessing a patient who has come to the clinic for a physical exam, the nurse sees that the patient has pallor. What is this finding most indicative of? a. Anemia b. Thrombophlebitis c. Heart disease d. Stroke

b

The health care provider orders orthostatic vital signs on a patient who experienced dizziness and feeling lightheaded. What is the nurse's first action a. Patient changes position to sitting or standing b. Measure the blood pressure when the patient is supine c. Place the patient in supine position for at least 3 minutes d. Wait for at least 1 minute before auscultating blood pressure and counting the radial pulse

b

The home health nurse is reviewing the patient's medication list and sees that new medications were added during a recent hospitalization. In addition, the patient reports he takes a daily low dose of aspirin, but aspirin is not on the final discharge list. Because of the aspirin usage, the nurse is most likely to call the prescribing health care provider for clarification of which type of medication? a. Vitamin supplement b. Platelet inhibitor c. Antihypertensive d. Erythrocyte stimulating agent

b

The laboratory results for a 53-year-old patient indicate a low T3 level and elevated thyroidstimulating hormone (TSH). What do these results indicate? a. Hyperthyroidism b. Hypothyroidism c. Malfunctioning pituitary gland d. Normal laboratory values for this age

b

The nurse assesses a patient in the emergency department (ED) and finds the following: constipation, fatigue with increased sleeping time, impaired memory, facial puffiness, and weight gain. Which deficiency does the nurse recognize? a. Hyperthyroidism b. Hypothyroidism c. Hyperparathyroidism d. Hypoparathyroidism

b

The nurse is assessing a patient diagnosed with hyperthyroidism and observes dry, waxy swelling of the front surfaces of the lower legs. How does the nurse interpret this finding? a. Globe lag b. Pretibial myxedema c. Exophthalmos d. Goiter

b

The nurse is performing an assessment on a patient brought in by emergency personnel. The nurse immediately observes that the patient has spontaneous respirations and the skin is cool, pale, and moist. What is the priority patient problem? a. Abnormal body temperature b. Impaired oxygenation c. Altered skin integrity d. Potential for peripheral neurovascular dysfunction

b

The nurse is providing instructions to a patient who is taking the antithyroid medication propylthiouracil (PTU). The nurse instructs the patient to notify the health care provider immediately if which sign/symptom occurs? a. Weight gain b. Dark-colored urine c. Cold intolerance d. Headache

b

The nurse working in a women's health clinic is reviewing the risk factors for several patients for stroke and MI. Which patient has the highest risk for MI? a. 49-year-old on estrogen replacement therapy b. 40-year-old taking oral contraceptives who smokes c. 23-year-old with diabetes that is currently not well-controlled d. 60-year-old with well-controlled hypertension

b

The nurse working in the public health department is reviewing data for populations at risk fro CVD. Which group has the greatest need for intervention to reduce CVD risk? a. Government employees who make approximately $40,000 a year b. Part-time fast-food workers who make approximately $9,000 a year c. Hospital-employed nurses who make approximately $52,000 a year d. Chain retail employees who make approximately $18,000 a year

b

The patient has smoked half a pack of cigarettes per day for 2 years. How many pack-years has this patient smoked? a. 1/2 pack year b. 1 back year c. 1 1/5 pack years d. 2 pack years

b

The patient is admitted for a chronic liver disorder and will be receiving vitamin K to address one of the problems associated with the disorder. Which clinical manifestation is the nurse most likely to observe before the vitamin K therapy is initiated? a. Sore throat and a smooth tongue b. Bruising and bleeding at venipuncture sites c. Fever and increase in WBC count d. Calf swelling due to deep vein thrombosis

b

The patient reports a history of splenectomy. Based on this information, what is the nurse most likely to assess for? a. Signs of bleeding b. Signs of infection c. Digestive problems d. Jaundice of the skin

b

The patient with a history of allergy to iodine-based contrast dyes is scheduled for a cardiac catheterization. What action does the nurse expect with regard to the scheduled test? a. Delay the test for a week or more b. Administer an antihistamine and.or steroid before the test c. The test will be performed without administration of contrast dye d. The patient will receive anticoagulation therapy before the test

b

What is the correct technique for assessing a patient with arterial insufficiency in the right lower leg? a. Use the Doppler to find the dorsalis pedis and posterior tibial pulses on the right leg b. Palpate the peripheral arteries in a head-to-toe approach with a side-to-side comparison c. Check all the pulse points in the right leg independent and supine positions d. Palpate the major arteries, such as the radial and femoral, and observe for pallor

b

What is the most common cause of hyperthyroidism? a. Radiation to thyroid b. Graves' disease c. Thyroid cancer d. Thyroiditis

b

What is the significance of a sodium level of 130 mEq/L for a patient with heart failure? a. Increased risk for ventricular dysrhythmias b. Dilutional hyponatremia and fluid retention c. Potential for electrical instability of the heart d. Slowed conduction of impulse through the heart

b

What term describes the difference between systolic and diastolic values, which is an indirect measure of cardiac output? a. Paradoxical blood pressure b. Pulse pressure c. Ankle-brachial index d. Normal blood pressure

b

Which assessment finding in a patient who has had a cardiac catheterization does the nurse report immediately to the provider? a. Pain at the catheter insertion site b. Catheterized extremity dusky with decreased peripheral pulses c. Small hematoma at the catheter insertion site d. Pulse pressure of 40 mm Hg with a slow, bounding pulse

b

Which condition is a life-threatening emergency and serious complication of untreated or poorly treated hypothyroidism? a. Endemic goiter b. Myxedema coma c. Toxic multinodular goiter d. Thyroiditis

b

Which description best defines the cardiovascular concept afterload? a. Degree of myocardial fiber stretch at end of diastole and just before heart contracts b. Amount of pressure or resistance that the ventricles must overcome to eject blood through the semilunar valves and into the peripheral blood vessels c. Pressure that ventricle must overcome to open aortic valve d. Force of contraction independent of preload

b

Which factor is a main assessment finding that signifies hypothyroidism? a. Irritability b. Cold intolerance c. Diarrhea d. Fatigue

b

Which sign/symptom is one of the first indicators of hyperthyroidism that is often noticed by the patient? a. Eyelid or globe lag b. Vision changes or tiring of the eyes c. Protruding eyes d. Photophobia

b

Question 4 of 41 A 32-year-old client is recovering from a sickle cell crisis. His discomfort is controlled with pain medications and he is to be discharged. What medication does the nurse expect to be prescribed for him before his discharge? a. Heparin (Heparin) b. Hydroxyurea (Droxia) c. Tissue plasminogen activator (t-PA) d. Warfarin (Coumadin)

b Hydroxyurea (Droxia) Hydroxyurea (Droxia) has been used successfully to reduce sickling of cells and pain episodes associated with sickle cell disease (SCD). Clients with SCD are not prescribed anticoagulants such as heparin or warfarin (Coumadin). t-PA is used as a "clot buster" in clients who have had ischemic strokes.

Question 40 of 41 The nurse is assessing the endurance level of a client in a long-term care facility. What question does the nurse ask to get this information? a. "Are your feet or hands cold, even when you are in bed?" b. "Do you feel more tired after you get up and go to the bathroom?" c. "How much exercise do you get?" d. "What is your endurance level?"

b "Do you feel more tired after you get up and go to the bathroom?" Asking about feeling tired after using the bathroom is pertinent to the client's activity and provides a comparison. The specific activity helps the client relate to the question and provide needed answers. Asking about cold feet or hands does not address the client's endurance. The hospitalized client typically does not get much exercise; this would be a difficult assessment for a client in long-term care facility to make. Asking the client about his or her endurance level is too vague; the client may not know how to answer this question.

Question 19 of 41 The nurse is educating a group of young women who have sickle cell disease (SCD). Which comment from a class member requires correction? a. "Frequent handwashing is an important habit for me to develop." b. "Getting an annual 'flu shot' would be dangerous for me." c. "I must take my penicillin pills as prescribed, all the time." d. "The pneumonia vaccine is protection that I need."

b "Getting an annual 'flu shot' would be dangerous for me." The client with SCD should receive annual influenza and pneumonia vaccinations; this helps prevent the development of these infections, which could cause a sickle cell crisis. Handwashing is a very important habit for the client with SCD to develop because it reduces the risk for infection. Prophylactic penicillin is given to clients with SCD orally twice a day to prevent the development of infection.

Ch.61 Question 1 of 16 The nurse is assessing a client for endocrine dysfunction. Which comment by the client indicates a need for further assessment? a. "I am worried about losing my job because of cutbacks." b. "I don't have any patience with my kids. I lose my temper faster." c. "I don't seem to have any stressors now." d. "My weight has been stable these past few years."

b "I don't have any patience with my kids. I lose my temper faster." Many endocrine problems can change a client's behavior, personality, and psychological responses; the client stating that he or she has become short-tempered warrants further assessment. Worrying about losing a job is a normal concern but does not give any indication of a need for further assessment. The nurse will need to assess the client's claim that he or she has no stressors at present because the client's response does not provide enough information to make this determination; however, the client's statement about losing patience is the priority. Weight gain or loss may or may not be an indication of an endocrine disorder.

Question 1 of 16 The nurse is assessing a client for endocrine dysfunction. Which comment by the client indicates a need for further assessment? a. "I am worried about losing my job because of cutbacks." b. "I don't have any patience with my kids. I lose my temper faster." c. "I don't seem to have any stressors now." d. "My weight has been stable these past few years."

b "I don't have any patience with my kids. I lose my temper faster." Many endocrine problems can change a client's behavior, personality, and psychological responses; the client stating that he or she has become short-tempered warrants further assessment. Worrying about losing a job is a normal concern but does not give any indication of a need for further assessment. The nurse will need to assess the client's claim that he or she has no stressors at present because the client's response does not provide enough information to make this determination; however, the client's statement about losing patience is the priority. Weight gain or loss may or may not be an indication of an endocrine disorder.

Ch.64 Question 9 of 25 The nurse is teaching a client with diabetes about proper foot care. Which statement by the client indicates that teaching was effective? a. "I should go barefoot in my house so that my feet are exposed to air." b. "I must inspect my shoes for foreign objects before putting them on." c. "I will soak my feet in warm water to soften calluses before trying to remove them." d. "I must wear canvas shoes as much as possible to decrease pressure on my feet."

b "I must inspect my shoes for foreign objects before putting them on." To avoid injury or trauma to the feet, shoes should be inspected for foreign objects before they are put on. Diabetic clients should not go barefoot because foot injuries can occur. To avoid injury or trauma, a callus should be removed by a podiatrist, not by the client. The diabetic client must wear firm support shoes to prevent injury.

Ch.64 Question 4 of 25 The nurse is teaching a client about the manifestations and emergency treatment of hypoglycemia. In assessing the client's knowledge, the nurse asks the client what he or she should do if feeling hungry and shaky. Which response by the client indicates a correct understanding of hypoglycemia management? a. "I should drink a glass of water." b. "I should eat three graham crackers." c. "I should give myself 1 mg of glucagon." d. "I should sit down and rest."

b "I should eat three graham crackers." Eating three graham crackers is a correct management strategy for mild hypoglycemia. Water or resting does not remedy hypoglycemia. Glucagon should be administered only in cases of severe hypoglycemia.

Ch.61 Question 11 of 16 The nurse is teaching a client about the correct procedure for a 24-hour urine test for creatinine clearance. Which statement by the client indicates a need for further teaching? a. "I should keep the urine container cool in a separate refrigerator or cooler." b. "I should not eat any protein when I am collecting urine for this test." c. "I won't save the first urine sample." d. "To end the collection, I must empty my bladder, adding it to the collection."

b "I should not eat any protein when I am collecting urine for this test." Eating protein does not interfere with collection or testing of the urine sample. Because the specimen must be kept cool, it can be placed in an inexpensive cooler with ice; the client should not keep the specimen container with food or beverages. The timing of the 24-hour collection begins after the initial void. To end a 24-hour urine specimen, emptying the bladder and adding it to the collection is the proper procedure.

Question 39 of 41 The nurse is transfusing 2 units of packed red blood cells to a postoperative client. What post-transfusion electrolyte imbalance does the nurse want to rule out? a. Hypercalcemia b. Hyperkalemia c. Hypomagnesemia d. Hyponatremia

b Hyperkalemia During transfusion, some cells are damaged. These cells release potassium, thus raising the client's serum potassium level (hyperkalemia). This complication is especially common with packed cells and whole-blood products. High serum calcium levels, low magnesium levels, or low sodium levels are not expected with blood transfusions.

Question 11 of 16 The nurse is teaching a client about the correct procedure for a 24-hour urine test for creatinine clearance. Which statement by the client indicates a need for further teaching? a. "I should keep the urine container cool in a separate refrigerator or cooler." b. "I should not eat any protein when I am collecting urine for this test." c. "I won't save the first urine sample." d. "To end the collection, I must empty my bladder, adding it to the collection."

b "I should not eat any protein when I am collecting urine for this test." Eating protein does not interfere with collection or testing of the urine sample. Because the specimen must be kept cool, it can be placed in an inexpensive cooler with ice; the client should not keep the specimen container with food or beverages. The timing of the 24-hour collection begins after the initial void. To end a 24-hour urine specimen, emptying the bladder and adding it to the collection is the proper procedure.

Question 1 of 41 Which client statement indicates that stem cell transplantation that is scheduled to take place in his home is not a viable option? a. "I don't feel strong enough, but my wife said she would help." b. "I was a nurse, so I can take care of myself." c. "I will have lots of medicine to take." d. "We live 5 miles from the hospital."

b "I was a nurse, so I can take care of myself." Stem cell transplantation in the home setting requires support, assistance, and coordination from others. The client cannot manage this type of care on his own. The client must be emotionally stable to be a candidate for this type of care. It is acceptable for the client's spouse to support the client undergoing this procedure. It is not unexpected for the client to be taking several prescriptions. Five miles is an acceptable distance from the hospital, in case of emergency.

Ch.64 Question 24 of 25 The nurse is providing discharge teaching to a client with newly diagnosed diabetes. Which statement by the client indicates a correct understanding about the need to wear a MedicAlert bracelet? a. "If I become hyperglycemic, it is a medical emergency." b. "If I become hypoglycemic, I could become unconscious." c. "Medical personnel may need confirmation of my insurance." d. "I may need to be admitted to the hospital suddenly."

b "If I become hypoglycemic, I could become unconscious." Hypoglycemia is the most common cause of medical emergency in clients with diabetes. A MedicAlert bracelet is helpful if the client becomes hypoglycemic and is unable to provide self-care. Hyperglycemia is not a medical emergency unless it is acidosis; people with diabetes tolerate mild hyperglycemia routinely. Insurance information and information needed for hospital admission do not appear on a MedicAlert bracelet.

Ch.63 Question 11 of 21 Family members of a client diagnosed with hyperthyroidism are alarmed at the client's frequent mood swings. What is the nurse's response? a. "How does that make you feel?" b. "The mood swings should diminish with treatment." c. "The medications will make the mood swings disappear completely." d. "Your family member is sick. You must be patient."

b "The mood swings should diminish with treatment." Telling the family that the client's mood swings should diminish over time with treatment will provide information to the family, as well as reassurance. Asking how the family feels is important; however, the response should focus on the client. Any medications or treatment may not completely remove the mood swings associated with hyperthyroidism. The family is aware that the client is sick; telling them to be patient introduces guilt and does not address the family's concerns.

Question 32 of 41 The nurse is reinforcing information about genetic counseling to a client with sickle cell disease who has a healthy spouse. What information does the nurse include? a. "Sickle cell disease will be inherited by your children." b. "The sickle cell trait will be inherited by your children." c. "Your children will have the disease, but your grandchildren will not." d. "Your children will not have the disease, but your grandchildren could."

b "The sickle cell trait will be inherited by your children." The children of the client with sickle cell disease will inherit the sickle cell trait, but may not inherit the disease. If both parents have the sickle cell trait, their children could get the disease.

Ch.64 Question 7 of 25 A client with type 1 diabetes mellitus received regular insulin at 7:00 a.m. The client should be monitored for hypoglycemia at which time? a. 7:30 a.m. b. 11:00 a.m. c. 2:00 p.m. d. 7:30 p.m.

b 11:00 a.m. Onset of regular insulin is ½ to 1 hour; peak is 2 to 4 hours. Therefore, 11:00 a.m. is the anticipated peak time for regular insulin received at 7:00 a.m. For regular insulin received at 7:00 a.m., 7:30 a.m., 2:00 p.m., and 7:30 p.m. are not the anticipated peak times.

Question 8 of 41 Which client does the nurse assign as a roommate for the client with aplastic anemia? a. A 23-year-old with sickle cell disease who has two draining leg ulcers b. A 28-year-old with glucose-6-phosphate dehydrogenase (G6PD) deficiency anemia who is receiving mannitol (Osmitrol) c. A 30-year-old with leukemia who is receiving induction chemotherapy d. A 34-year-old with idiopathic thrombocytopenia who is taking steroids

b A 28-year-old with glucose-6-phosphate dehydrogenase (G6PD) deficiency anemia who is receiving mannitol (Osmitrol) Because clients with aplastic anemia usually have low white blood cell counts that place them at high risk for infection, roommates such as the client with G6PD deficiency anemia should be free from infection or infection risk. The client with sickle cell disease has two draining leg ulcer infections that would threaten the diminished immune system of the client with aplastic anemia. The client with leukemia who is receiving induction chemotherapy and the client with idiopathic thrombocytopenia who is taking steroids are at risk for development of infection, which places the client with aplastic anemia at risk, too.

6. A patient has a low serum cortisol level. Which hormone would the nurse expect to be secreted to correct this? a. Thyroid-stimulating hormone (TSH) b. Adrenocorticotropic hormone c. Parathyroid hormone d. Antidiuretic hormone

b Adrenocorticotropic hormone

Ch.61 Question 6 of 16 A client is hospitalized for pituitary function testing. Which nursing action included in the client's plan of care will be most appropriate for the RN to delegate to the LPN/LVN? a. Assess the client for clinical manifestations of hypopituitarism. b. Inject regular insulin for the growth hormone stimulation test. c. Palpate the thyroid gland for size and firmness. d. Teach the client about the adrenocorticotropic hormone stimulation test.

b Inject regular insulin for the growth hormone stimulation test. Injection of insulin is within the LPN/LVN scope of practice. Client assessment for clinical manifestations of hypopituitarism, palpating the thyroid gland, and client education are complex skills requiring training and expertise, and are best performed by an RN.

Ch.63 Question 15 of 21 A client being treated for hyperthyroidism calls the home health nurse and mentions that his heart rate is slower than usual. What is the nurse's best response? a. Advise the client to go to a calming environment. b. Ask whether the client has increased cold sensitivity or weight gain. c. Instruct the client to see his health care provider immediately. d. Tell the client to check his pulse again and call back later.

b Ask whether the client has increased cold sensitivity or weight gain. Increased sensitivity to cold and weight gain are symptoms of hypothyroidism, indicating an overcorrection by the medication. The client must be assessed further because he may require a lower dose of medication. A calming environment will not have any effect on the client's heart rate. The client will want to notify the health care provider about the change in heart rate. If other symptoms such as chest pain, shortness of breath, or confusion accompany the slower heart rate, then the client should see the health care provider immediately. If the client was concerned enough to call because his heart rate was slower than usual, the nurse needs to stay on the phone with the client while he re-checks his pulse. This time could also be spent providing education about normal ranges for that client.

3. Which mechanism is used to transport the substance produced by the endocrine glands to their target tissue? a. Lymph system b. Bloodstream c. Direct seeding d. Gastrointestinal system

b Bloodstream

Ch.64 Question 14 of 25 A client newly diagnosed with diabetes is not ready or willing to learn diabetes control during the hospital stay. Which information is the priority for the nurse to teach the client and the client's family? a. Causes and treatment of hyperglycemia b. Causes and treatment of hypoglycemia c. Dietary control d. Insulin administration

b Causes and treatment of hypoglycemia The causes and treatment of hypoglycemia must be understood by the client and family to manage the client's diabetes effectively. The causes and treatment of hyperglycemia is a topic for secondary teaching and is not the priority for the client with diabetes. Dietary control and insulin administration are important, but are not the priority in this situation.

39. What is the correct nursing action before beginning a 24-hour urine collection for endocrine studies? a. Place each voided specimen in a separate collection container. b. Check whether any preservatives are needed in the collection container. c. Start the collection with the first voided urine. d. Weigh the patient before beginning the collection.

b Check whether any preservatives are needed in the collection container.

Ch.64 Question 16 of 25 The nurse has just taken change-of-shift report on a group of clients on the medical-surgical unit. Which client does the nurse assess first? a. Client taking repaglinide (Prandin) who has nausea and back pain b. Client taking glyburide (Diabeta) who is dizzy and sweaty c. Client taking metformin (Glucophage) who has abdominal cramps d. Client taking pioglitazone (Actos) who has bilateral ankle swelling

b Client taking glyburide (Diabeta) who is dizzy and sweaty The client taking glyburide (Diabeta) who is dizzy and sweaty has symptoms consistent with hypoglycemia and should be assessed first because this client displays the most serious adverse effect of antidiabetic medications. Although the client taking repaglinide who has nausea and back pain requires assessment, the client taking glyburide takes priority. Metformin may cause abdominal cramping and diarrhea, but the client taking it does not require immediate assessment. Ankle swelling is an expected side effect of pioglitazone.

18. Which is the major function of the hormones produced by the adrenal cortex? a. "Fight-or-flight" response b. Control of potassium, sodium, and water c. Regulation of cell growth d. Calcium and stress regulation

b Control of potassium, sodium, and water

31. Which disease involves a disorder of the islets of Langerhans? a. Diabetes insipidus b. Diabetes mellitus c. Addison's disease d. Cushing's disease

b Diabetes mellitus

Ch.64 Question 11 of 25 A client has just been diagnosed with diabetes. Which factor is most important for the nurse to assess in the client before providing instruction about the disease and its management? a. Current lifestyle b. Educational and literacy level c. Sexual orientation d. Current energy level

b Educational and literacy level A large amount of information must be synthesized; typically written instructions are given. The client's educational and literacy level is essential information. Although lifestyle should be taken into account, it is not the priority. Sexual orientation will have no bearing on the ability of the client to provide self-care. Although energy level will influence the ability to exercise, it is not essential.

Ch.63 Question 21 of 21 A client admitted with hyperthyroidism is fidgeting with the bedcovers and talking extremely fast. What does the nurse do next? a. Calls the provider b. Encourages the client to rest c. Immediately assesses cardiac status d. Tells the client to slow down

b Encourages the client to rest The client with hyperthyroidism often has wide mood swings, irritability, decreased attention span, and manic behavior. The nurse should accept the client's behavior and provide a calm, quiet, and comfortable environment. Because the client's behavior is expected, there is no need to call the provider. Monitoring the client's cardiac status is part of the nurse's routine assessment. Telling the client to slow down is unsupportive and unrealistic.

Question 29 of 41 Which intervention most effectively protects a client with thrombocytopenia? a. Avoiding the use of dentures b. Encouraging the use of an electric shaver c. Taking rectal temperatures d. Using warm compresses on trauma sites

b Encouraging the use of an electric shaver The client with thrombocytopenia should be advised to use an electric shaver instead of a razor. Any small cuts or nicks can cause problems because of the prolonged clotting time. Dentures may be used by clients with thrombocytopenia as long as they fit properly and do not rub. To prevent rectal trauma, rectal thermometers should not be used. Oral or tympanic temperatures should be taken. Ice (not heat) should be applied to areas of trauma.

15. Which statement about pituitary hormones is correct? a. ACTH acts on the adrenal medulla. b. Follicle-stimulating hormone (FSH) stimulates sperm production in men. c. Growth hormone promotes protein catabolism. d. Vasopressin decreases systolic blood pressure.

b Follicle-stimulating hormone (FSH) stimulates sperm production in men.

16. The nurse is providing instructions to a patient taking levothyroxine (Synthroid). When does the nurse tell the patient to take this medication? a. With breakfast in the morning. b. At lunchtime immediately after eating. c. In the morning on an empty stomach. d. At dinnertime within 15 minutes after eating.

c

Question 7 of 41 The nurse is caring for a client with sickle cell disease. Which action is most effective in reducing the potential for sepsis in this client? a. Administering prophylactic drug therapy b. Frequent and thorough handwashing c. Monitoring laboratory values to look for abnormalities d. Taking vital signs every 4 hours, day and night

b Frequent and thorough handwashing Prevention and early detection strategies are used to protect the client in sickle cell crisis from infection. Frequent and thorough handwashing is of the utmost importance. Drug therapy is a major defense against infections that develop in the client with sickle cell disease, but is not the most effective way that the nurse can reduce the potential for sepsis. Continually assessing the client for infection and monitoring the daily complete blood count with differential white blood cell count is early detection, not prevention. Taking vital signs every 4 hours will help with early detection of infection, but is not prevention.

Ch.61 Question 5 of 16 To best determine how well a client with diabetes mellitus is controlling blood glucose, which test does the nurse monitor? a. Fasting blood glucose b. Glycosylated hemoglobin (HbA1c) c. Oral glucose tolerance test d. Urine glucose level

b Glycosylated hemoglobin (HbA1c) Glycosylated hemoglobin indicates the average blood glucose over several months and is the best indicator of overall blood glucose control. Fasting blood glucose can be used to monitor glucose control, but it is not the best method (although this may be the method that clients are most familiar with). Oral glucose testing and urine glucose levels look at one period of time and are not the best methods to look at overall effectiveness of treatment.

Question 5 of 16 To best determine how well a client with diabetes mellitus is controlling blood glucose, which test does the nurse monitor? a. Fasting blood glucose b. Glycosylated hemoglobin (HbA1c) c. Oral glucose tolerance test d. Urine glucose level

b Glycosylated hemoglobin (HbA1c) Glycosylated hemoglobin indicates the average blood glucose over several months and is the best indicator of overall blood glucose control. Fasting blood glucose can be used to monitor glucose control, but it is not the best method (although this may be the method that clients are most familiar with). Oral glucose testing and urine glucose levels look at one period of time and are not the best methods to look at overall effectiveness of treatment.

Ch.62 Question 18 of 23 The client is taking fludrocortisone (Florinef) for adrenal hypofunction. The nurse instructs the client to report which symptom while taking this drug? a. Anxiety b. Headache c. Nausea d. Weight loss

b Headache A side effect of fludrocortisone is hypertension. New onset of headache should be reported, and the client's blood pressure should be monitored. Anxiety is not a side effect of fludrocortisone and is not associated with adrenal hypofunction. Nausea is associated with adrenal hypofunction; it is not a side effect of fludrocortisone. Sodium-related fluid retention and weight gain, not loss, are possible with fludrocortisone therapy.

Ch.63 Question 16 of 21 A client had a parathyroidectomy 18 hours ago. Which finding requires immediate attention? a. Edema at the surgical site b. Hoarseness c. Pain on moving the head d. Sore throat

b Hoarseness Hoarseness or stridor is an indication of respiratory distress and requires immediate attention. Edema at the surgical site of any surgery is an expected finding. Pain when the client moves the head or attempts to lift the head off the bed is an expected finding after a parathyroidectomy. Any time a client has been intubated for surgery, a sore throat is a common occurrence in the postoperative period. This is especially true for clients who have had surgery involving the neck.

Ch.61 Question 4 of 16 In type 1 diabetes, insulin injections are necessary to maintain which action between insulin and glucose? a. Glucose intolerance b. Homeostasis c. Insulin intolerance d. Negative feedback

b Homeostasis Insulin injections maintain homeostasis, or normal balance, between insulin and glucose in the client with type 1 diabetes. Type 1 diabetes is a lack of insulin production, not glucose intolerance, and requires frequent doses of insulin. Negative feedback does not occur in type 1 diabetes because of lack of insulin.

Question 4 of 16 In type 1 diabetes, insulin injections are necessary to maintain which action between insulin and glucose? a. Glucose intolerance b. Homeostasis c. Insulin intolerance d. Negative feedback

b Homeostasis Insulin injections maintain homeostasis, or normal balance, between insulin and glucose in the client with type 1 diabetes. Type 1 diabetes is a lack of insulin production, not glucose intolerance, and requires frequent doses of insulin. Negative feedback does not occur in type 1 diabetes because of lack of insulin.

Ch.61 Question 15 of 16 Which statement is true about hormones and their receptor sites? a. Hormone activity is dependent only on the function of the receptor site. b. Hormones need a specific receptor site to work. c. Hormones need to be plasma-bound to activate the receptor site. d. Hormone stores are available for activation until needed.

b Hormones need a specific receptor site to work. In general, each receptor site type is specific for only one hormone. Hormone receptor actions work in a "lock and key" manner, in that only the correct hormone (key) can bind to and activate the receptor site (lock). Hormones travel through the blood to all body areas, but exert their actions only on target tissues. Not all hormones are plasma-bound; for example, thyroid hormones are plasma protein-bound, whereas posterior pituitary hormones are transported by axons. Only certain cells manufacture specific hormones and store the hormones in vesicles.

Question 15 of 16 Which statement is true about hormones and their receptor sites? a. Hormone activity is dependent only on the function of the receptor site. b. Hormones need a specific receptor site to work. c. Hormones need to be plasma-bound to activate the receptor site. d. Hormone stores are available for activation until needed.

b Hormones need a specific receptor site to work. In general, each receptor site type is specific for only one hormone. Hormone receptor actions work in a "lock and key" manner, in that only the correct hormone (key) can bind to and activate the receptor site (lock). Hormones travel through the blood to all body areas, but exert their actions only on target tissues. Not all hormones are plasma-bound; for example, thyroid hormones are plasma protein-bound, whereas posterior pituitary hormones are transported by axons. Only certain cells manufacture specific hormones and store the hormones in vesicles.

18. A patient is diagnosed with a fungal UTI. Which drug does the nurse anticipate the patient will be treated with? a. Trimethoprim/sulfamethoxazole (Bactrim) b. Ciprofloxacin (Cipro) c. Fluconazole (Diflucan) d. Amoxicillin (Amoxil)

c

Question 6 of 16 A client is hospitalized for pituitary function testing. Which nursing action included in the client's plan of care will be most appropriate for the RN to delegate to the LPN/LVN? a. Assess the client for clinical manifestations of hypopituitarism. b. Inject regular insulin for the growth hormone stimulation test. c. Palpate the thyroid gland for size and firmness. d. Teach the client about the adrenocorticotropic hormone stimulation test.

b Inject regular insulin for the growth hormone stimulation test. Injection of insulin is within the LPN/LVN scope of practice. Client assessment for clinical manifestations of hypopituitarism, palpating the thyroid gland, and client education are complex skills requiring training and expertise, and are best performed by an RN.

Ch.62 Question 21 of 23 A client has undergone a transsphenoidal hypophysectomy. Which intervention does the nurse implement to avoid increasing intracranial pressure (ICP) in the client? a. Encourages the client to cough and deep-breathe b. Instructs the client not to strain during a bowel movement c. Instructs the client to blow the nose for postnasal drip d. Places the client in the Trendelenburg position

b Instructs the client not to strain during a bowel movement Straining during a bowel movement increases ICP and must be avoided. Laxatives may be given and fluid intake encouraged to help with this. Although deep breathing is encouraged, the client must avoid coughing early after surgery because this increases pressure in the incision area and may lead to a cerebrospinal fluid (CSF) leak. If the client has postnasal drip, he or she must inform the nurse and not blow the nose; postnasal drip may indicate leakage of CSF. The head of the bed must be elevated after surgery.

Question 30 of 41 A recently admitted client who is in sickle cell crisis requests "something for pain." What does the nurse administer? a. Intramuscular (IM) morphine sulfate b. Intravenous (IV) hydromorphone (Dilaudid) c. Oral ibuprofen (Motrin) d. Oral morphine sulfate (MS-Contin)

b Intravenous (IV) hydromorphone (Dilaudid) The client needs IV pain relief, and it should be administered on a routine schedule (i.e., before the client has to request it). Morphine is not administered intramuscularly (IM) to clients with sickle cell disease (SCD). In fact, all IM injections are avoided because absorption is impaired by poor perfusion and sclerosed skin. Nonsteroidal anti-inflammatory drugs may be used for clients with SCD for pain relief once their pain is under control; however, in a crisis, this choice of analgesic is not strong enough. Moderate pain may be treated with oral opioids, but this client is in a sickle cell crisis; IV analgesics should be used until his or her condition stabilizes.

Ch.63 Question 13 of 21 A client has been diagnosed with hypothyroidism. What medication is usually prescribed to treat this disorder? a. Atenolol (Tenormin) b. Levothyroxine sodium (Synthroid) c. Methimazole (Tapazole) d. Propylthiouracil

b Levothyroxine sodium (Synthroid) Levothyroxine is a synthetic form of thyroxine (T4) that is used to treat hypothyroidism. Atenolol is a beta blocker that is used to treat cardiovascular disease. Methimazole and propylthiouracil are used to treat hyperthyroidism.

42. A patient is suspected of having a pituitary tumor. Which radiographic test aids in determining this diagnosis? a. Skull x-rays b. MRI/CT c. Angiography d. Ultrasound

b MRI/CT

Ch.63 Question 3 of 21 A client recently admitted with hyperparathyroidism has a very high urine output. Of these actions, what does the nurse do next? a. Calls the health care provider b. Monitors intake and output c. Performs an immediate cardiac assessment d. Slows the rate of IV fluids

b Monitors intake and output Diuretic and hydration therapies are used most often for reducing serum calcium levels in clients with hyperparathyroidism. Usually, a diuretic that increases kidney excretion of calcium is used together with IV saline in large volumes to promote renal calcium excretion. The health care provider does not need to be notified in this situation, given the information available in the question. Cardiac assessment is part of the nurse's routine evaluation of the client. Slowing the rate of IV fluids is contraindicated because the client will become dehydrated due to the use of diuretics to increase kidney excretion of calcium.

12. The maintenance of internal body temperature at approximately 98.6F (37C) is an example of which endocrine process? a. "Lock and key" manner b. Neuroendocrine regulation c. Positive feedback mechanism d. Stimulus-response theory

b Nueroendocrine regulation

Ch.62 Question 22 of 23 A client presents to the emergency department with acute adrenal insufficiency and the following vital signs: P 118 beats/min, R 18 breaths/min, BP 84/44 mm Hg, pulse oximetry 98%, and T 98.8° F oral. Which nursing intervention is the highest priority for this client? a. Administering furosemide (Lasix) b. Providing isotonic fluids c. Replacing potassium losses d. Restricting sodium

b Providing isotonic fluids Providing isotonic fluid is the priority intervention because this client's vital signs indicate volume loss that may be caused by nausea and vomiting and may accompany acute adrenal insufficiency. Isotonic fluids will be needed to administer IV medications such as hydrocortisone. Furosemide is a loop diuretic, which this client does not need. Potassium is normally increased in acute adrenal insufficiency, but potassium may have been lost if the client has had diarrhea; laboratory work will have to be obtained. Any restrictions, including sodium, should not be started without obtaining laboratory values to establish the client's baseline.

Question 34 of 41 The nurse assesses multiple clients who are receiving transfusions of blood components. Which assessment indicates the need for the nurse's immediate action? a. A partial thromboplastin time (PTT) that is 1.2 times normal in a client who received a transfusion of fresh-frozen plasma (FFP) b. Respiratory rate of 36 breaths/min in a client receiving red blood cells c. Sleepiness in a client who received diphenhydramine (Benadryl) as a premedication d. Temperature of 99.1° F (37.3° C) for a client with a platelet transfusion

b Respiratory rate of 36 breaths/min in a client receiving red blood cells An increased respiratory rate indicates a possible hemolytic transfusion reaction; the nurse should quickly stop the transfusion and assess the client further. Because FFP is not usually given until the PTT is 1.5 times above normal, a PTT that is 1.2 times normal indicates that the FFP has had the desired response. Sleepiness is expected when Benadryl is administered. Temperature elevations are not an indication of an allergic reaction to a platelet transfusion, although the nurse may administer acetaminophen (Tylenol) to decrease the fever.

19. The nurse is caring for a patient with thrombocytopenia. Which order does the nurse question? a. Test all urine and stool for occult blood. b. Avoid IM injections. c. Administer enemas. d. Apply ice to areas of trauma.

c

37. Which statement about performing a physical assessment of the thyroid gland is correct? a. The thyroid gland is easily palpated in all patients. b. The patient is instructed to swallow sips of water to aid palpation. c. The anterior approach is preferred for thyroid palpation. d. The thumbs are used to palpate the thyroid lobes.

b The patient is instructed to swallow sips of water to aid palpation.

8. The nurse is performing an assessment of an adult patient with new-onset acromegaly. What does the nurse expect to find? a. Extremely long arms and legs b. Thickened lips c. Changes in menses with infertility d. Rough, extremely dry skin

b Thickened lips

29. In addition to the pancreas that secretes insulin, which gland secretes hormones that affect protein, carbohydrate, and fat metabolism? a. Posterior pituitary b. Thyroid c. Ovaries d. Parathyroid

b Thyroid

36. The nurse is performing a physical assessment of a patient's endocrine system. Which gland can be palpated? a. Pancreas b. Thyroid c. Adrenal glands d. Parathyroids

b Thyroid

33. Which statement about age-related changes in older adults and the endocrine system is true? a. All hormone levels are elevated. b. Thyroid hormone levels decrease. c. Adrenal glands enlarge. d. The thyroid gland enlarges.

b Thyroid hormone levels decrease.

44. A patient is at risk for falling related to the effect of pathologic fractures as a result of bone demineralization. Which endocrine problem is this pertinent to? a. Underproduction of PTH b. Overproductive of PTH c. Underproduction of thyroid hormone d. Overproduction of thyroid hormone

b Overproductive of PTH

91. Which patients are likely to be excluded from receiving a transplant? (Select all that apply.) a. Patient who had breast cancer 6 years ago b. Patient with advanced and uncorrectable heart disease c. Patient with a chemical dependency d. Patient who is 70 years old and has a living related donor e. Patient with diabetes mellitus

b, c

Question 20 of 41 What are serious side effects of antiviral agents prescribed for a client with acute myelogenous leukemia? Select all that apply. a. Cardiomyopathy b. Nephrotoxicity c. Ototoxicity d. Stroke e. Diarrhea

b, c Nephrotoxicity, Ototoxicity Antiviral agents, although helpful in combating severe infection, have serious side effects, especially nephrotoxicity and ototoxicity. Cardiomyopathy and stroke are not serious side effects of antiviral agents. Diarrhea is a mild side effect associated with antibiotic therapy.

21. A patient with PKD would exhibit which signs/symptoms? (Select all that apply.) a. Frequent urination b. Increased abdominal girth c. Hypertension d. Kidney stones e. Diarrhea

b, c, d

34. What are the characteristics of continuous venovenous hemofiltration (CVVH)? (Select all that apply.) a. Requires placement of arterial and venous access b. Uses a pump to drive blood from the patient catheter into the dialyzer c. Risk of air embolus d. More commonly used for patients who are critically ill e. Most convenient method for home care patients

b, c, d

59. The nurse is caring for a patient with nephrotic syndrome. What interventions are included in the plan of care for this patient? (Select all that apply.) a. Fluids should be restricted. b. Administer mild diuretics. c. Assess for edema. d. Administer antihypertensive medications. e. Frequently assess the patient's mental status.

b, c, d

25. Which are the target organs of PTH in the regulation of calcium and phosphorus? (Select all that apply.) a. Stomach b. Kidney c. Bone d. Gastrointestinal tract e. Thyroid gland

b, c, d Kidney, Bone, Gastrointestinal tract

21. Which statements about the thyroid gland and its hormones are correct? (Select all that apply.) a. The gland is located in the posterior neck below the cricoid cartilage. b. The gland has two lobes joined by a thin tissue called the isthmus. c. T4 and T3 are two thyroid hormones. d. Thyroid hormones increase red blood cell production. e. Thyroid hormone production depends on dietary intake of iodine and potassium.

b, c, d The gland has two lobes joined by a thin tissue called the isthmus. T4 and T3 are two thyroid hormones. Thyroid hormones increase red blood cell production.

33. What are the key features associated with chronic pyelonephritis? (Select all that apply.) a. Abscess formation b. Hypertension c. Inability to conserve sodium d. Decreased urine-concentrating ability, resulting in nocturia e. Tendency to develop hyperkalemia and acidosis

b, c, d, e

33. Which are preoperative instructions for a patient having thyroid surgery? (Select all that apply.) a. Teach postoperative restrictions such as no coughing and deep-breathing exercises to prevent strain on the suture line. b. Teach the moving and turning technique of manually supporting the head and avoiding neck extension to minimize strain on the suture line. c. Inform the patient that hoarseness for a few days after surgery is usually the result of a breathing tube (endotracheal tube). d. Humidification of air may be helpful to promote expectoration of secretions. Suctioning may also be used. e. Clarify any questions regarding placement of incision, complications, and postoperative care. f. A supine position and lying flat will be maintained postoperatively to avoid strain on suture line.

b, c, d, e

69. A patient has been informed by the health care provider that treatment will be needed for renal artery stenosis. The nurse prepares to teach about a variety of treatment options. What treatments will the nurse include in the teaching plan (Select all that apply.) a. Kidney transplant b. Hypertension control c. Balloon angioplasty d. Renal artery bypass surgery e. Synthetic blood vessel graft f. Percutaneous ultrasonic pyelolithotomy

b, c, d, e

86. What might the nurse notice if the patient is experiencing problems with urinary elimination as a result of acute pyelonephritis? (Select all that apply.) a. Patient urinates large amounts of dilute urine. b. Patient reports pain and burning on urination. c. Patient reports back or flank pain. d. Patient reports back or flank pain. e. Urine may be darker or smoky or have obvious blood in it.

b, c, d, e

87. During PD, the nurse notes slowed dialysate outflow. What does the nurse do to troubleshoot the system? (Select all that apply.) a. Ensure that the drainage bag is elevated. b. Inspect the tubing for kinking or twisting. c. Ensure that clamps are open. d. Turn the patient to the other side. e. Make sure the patient is in good body alignment. f. Instruct the patient to stand or cough.

b, c, d, e

Which are preoperative instructions for a patient having thyroid surgery? (Select all that apply.) a. Teach postoperative restrictions such as no coughing and deep-breathing exercises to prevent strain on the suture line. b. Teach the moving and turning technique of manually supporting the head and avoiding neck extension to minimize strain on the suture line. c. Inform the patient that hoarseness for a few days after surgery is usually the result of a breathing tube (endotracheal tube) used during surgery. d. Humidification of air may be helpful to promote expectoration of secretions. Suctioning may also be used. e. Clarify any questions regarding placement of incision, complications, and postoperative care. f. A supine position and lying flat will be maintained postoperatively to avoid strain on suture line.

b, c, d, e

32. Which endocrine tissues are most commonly found to have reduced function as a result of aging? (Select all that apply.) a. Hypothalamus b. Ovaries c. Testes d. Pancreas e. Thyroid gland

b, c, d, e Ovaries, Testes, Pancreas, Thyroid gland

18. After a hypophysectomy, home care monitoring by the nurse includes assessing which factors? (Select all that apply.) a. Hypoglycemia b. Bowel habits c. Possible leakage of cerebrospinal fluid (CSF) d. 24-hour intake of fluids and urine output e. 24-hour diet recall f. Activity level

b, c, d, e, f

92. A daughter is considering donating a kidney to her mother for organ transplant. What information does the nurse give to the daughter about the criteria for donation? (Select all that apply.) a. Age limit is at least 21 years old. b. Systemic disease and infection must be absent. c. There must be no history of cancer. d. Hypertension or kidney disease must be absent. e. There must be adequate kidney function as determined by diagnostic studies. f. The donor must understand the surgery and be willing to give up the organ.

b, c, d, e, f

76. The nurse is caring for a patient with kidney cell carcinoma who manifests paraneoplastic syndromes. What findings does the nurse expect to see in this patient? (Select all that apply.) a. Urinary tract infection b. Erythrocytosis c. Hypercalcemia d. Liver dysfunction e. Decreased sedimentation rate f. Hypertension

b, c, d, f

11. A patient with a low white blood cell count is being discharged home. In which situations will the patient be instructed by the nurse to contact his or her health care provider? (Select all that apply.) a. When temperature goes over 102F (38.9C) b. A persistent cough develops with or without sputum c. Pus or foul-smelling drainage develops d. Whenever exposed to fresh fruit or vegetables or live plants e. Urine is cloudy or foul-smelling, or if burning on urination is expeirenced

b, c, e

25. What does the nurse instruct patients with permanent DI to do? (Select all that apply.) a. Continue vasopressin therapy until symptoms disappear. b. Monitor for recurrent of polydipsia and polyuria. c. Monitor and record weight daily. d. Check urine specific gravity three times a week. e. Wear a medical alert bracelet.

b, c, e

47. Which laboratory findings does the nurse expect to find with Cushing's syndrome? (Select all that apply.) a. Decreased serum sodium b. Increased serum glucose c. Increased serum sodium d. Increased serum potassium e. Decreased serum calcium

b, c, e

78. The nurse is caring for a patient with an arteriovenous fistula. What is included in the nursing care for this patient? (Select all that apply.) a. Keep small clamps handy by the bedside. b. Encourage routine range-of-motion exercises. c. Avoid venipuncture or IV administration on the arm with the access device. d. Instruct the patient to carry heavy objects to build muscular strength. e. Assess for manifestations of infection of the fistula. f. Instruct the patient to sleep on the side with the affected arm in the dependent position.

b, c, e

19. Which statements about the hormone cortisol being secreted by the adrenal cortex are accurate? (Select all that apply.) a. Cortisol peaks occur late in the day, with lowest points 12 hours after each peak. b. Cortisol has an effect on the body's immune function. c. Stress causes an increase in the production of cortisol. d. Blood levels of cortisol have no effect on its secretion. e. Cortisol affects carbohydrate, protein, and fat metabolism.

b, c, e Cortisol has an effect on body's immune function. Stress causes an increase in the production of cortisol. Cortisol affects carbohydrate, protein, and fat metabolism.

5. Postrenal kidney injury can result from which conditions? (Select all that apply.) a. Septic shock b. Cervical cancer c. Nephrolithiasis or ureterolithiasis d. Heart failure e. Neurogenic bladder f. Prostate cancer

b, c, e, f

4. Which hormones are secreted by the posterior pituitary gland? (Select all that apply.) a. Testosterone b. Oxytocin c. Growth hormone (GH) d. Antidiuretic hormone (ADH) e. Cortisol

b, d Oxytocin, Antidiuretic hormone (ADH)

3. What are common causes of prerenal kidney injury? (Select all that apply.) a. Urethral cancer b. Hypovolemic shock c. Enlarged prostate gland d. Sepsis e. Severe burns

b, d, e

Question 12 of 41 What are the typical clinical manifestations of anemia? Select all that apply. a. Decreased breath sounds b. Dyspnea on exertion c. Elevated temperature d. Fatigue e. Pallor f. Tachycardia

b, d, e, f Dyspnea on exertion, Fatigue, Pallor, Tachycardia Difficulty breathing—especially with activity—is common with anemia. Lower levels of hemoglobin carry less O2 to the cells of the body. Fatigue is a classic symptom of anemia; lowered O2 levels contribute to a faster pulse (i.e., cardiac rate) and tend to "wear out" a client's energy. Lowered O2 levels deliver less oxygen to all cells, making clients with anemia pale—especially their ears, nail beds, palms, and conjunctivae and around the mouth. Respiratory problems with anemia do not include changes in breath sounds; dyspnea and decreased oxygen saturation levels are present. Skin is cool to the touch, and an intolerance to cold is noted; elevated temperature would signify something additional, such as infection

47. Which foods will the nurse instruct a patient with hypoparathyroidism to avoid? (Select all that apply.) a. Canned vegetables b. Yogurt c. Fresh fruit d. Red meat e. Milk f. Processed cheese

b, e, f

A nursing student learns that many drugs can impair the immune system. Which drugs does this include? (Select all that apply.) a. Acetaminophen (Tylenol) b. Amphotericin B (Fungizone) c. Ibuprofen (Motrin) d. Metformin (Glucophage) e. Nitrofurantoin (Macrobid)

b. Amphotericin B (Fungizone) c. Ibuprofen (Motrin) e. Nitrofurantoin (Macrobid) ANS: B, C, E Amphotericin B, ibuprofen, and nitrofurantoin all can disrupt the hematologic (immune) system. Acetaminophen and metformin do not.

A client is having a bone marrow biopsy today. What action by the nurse takes priority? a. Administer pain medication first. b. Ensure valid consent is on the chart. c. Have the client shower in the morning. d. Premedicate the client with sedatives.

b. Ensure valid consent is on the chart. ANS: B A bone marrow biopsy is an invasive procedure that requires informed consent. Pain medication and sedation are important components of care for this client but do not take priority. The client may or may not need or be able to shower.

A nurse works in a gerontology clinic. What age-related changes cause the nurse to alter standard assessment techniques from those used for younger adults? (Select all that apply.) a. Dentition deteriorates with more cavities. b. Nail beds may be thickened or discolored. c. Progressive loss of hair occurs with age. d. Sclerae begin to turn yellow or pale. e. Skin becomes dry as the client ages.

b. Nail beds may be thickened or discolored. c. Progressive loss of hair occurs with age. e. Skin becomes dry as the client ages. ANS: B, C, E Common findings in older adults include thickened or discolored nail beds, dry skin, and thinning hair. The nurse adapts to these changes by altering assessment techniques. Having more dental caries and changes in the sclerae are not normal age-related changes.

A client admitted for sickle cell crisis is distraught after learning her child also has the disease. What response by the nurse is best? a. "Both you and the father are equally responsible for passing it on." b. "I can see you are upset. I can stay here with you a while if you like." c. "It's not your fault; there is no way to know who will have this disease." d. "There are many good treatments for sickle cell disease these days."

b. "I can see you are upset. I can stay here with you a while if you like."

45. A patient with acute leukemia has been receiving an erythropoiesis-stimulating agent. When would the nurse call the health care provider to have this order discontinued? a. Hemoglobin level is 6 mg/dL. b. Hematocrit is 20%. c. Hemoglobin level is 10.5 mg/dL. d. Platelet count is 50,000/mm3

c

The nurse is caring for a group of clients who have sustained myocardial infarction (MI). The nurse observes the client with which type of MI most carefully for the development of left ventricular heart failure? a. Inferior wall b. Anterior wall c. Lateral wall d. Posterior wall

b. Anterior wall Due to the large size of the anterior wall, the amount of tissue infarction may be large enough to decrease the force of contraction, leading to heart failure. The client with an inferior wall MI is more likely to develop right ventricular heart failure. Clients with obstruction of the circumflex artery may experience a lateral wall MI and sinus dysrhythmias or a posterior wall MI and sinus dysrhythmias.

The family of a neutropenic client reports the client "is not acting right." What action by the nurse is the priority? a. Ask the client about pain. b. Assess the client for infection. c. Delegate taking a set of vital signs. d. Look at today's laboratory results.

b. Assess the client for infection.

A client has just returned from coronary artery bypass graft surgery. For which finding does the nurse contact the surgeon? a. Temperature 98.2° F b. Chest tube drainage 175 mL last hour c. Serum potassium 3.9 mEq/L d. Incisional pain 6 on a scale of 0 to 10

b. Chest tube drainage 175 mL last hour Some bleeding is expected after surgery; however, the nurse should report chest drainage over 150 mL/hr to the surgeon. Although hypothermia is a common problem after surgery, a temperature of 98.2° F is a normal finding. Serum potassium of 3.9 mEq/L is a normal finding. Incisional pain of 6 on a scale of 0 to 10 is expected immediately after major surgery; the nurse should administer prescribed analgesics.

A nurse in a hematology clinic is working with four clients who have polycythemia vera. Which client should the nurse see first? a. Client with a blood pressure of 180/98 mm Hg b. Client who reports shortness of breath c. Client who reports calf tenderness and swelling d. Client with a swollen and painful left great toe

b. Client who reports shortness of breath

The nurse in the coronary care unit is caring for a group of clients who have had myocardial infarction. Which client does the nurse see first? a. Client with dyspnea on exertion when ambulating to the bathroom b. Client with third-degree heart block on the monitor c. Client with normal sinus rhythm and PR interval of 0.28 second d. Client who refuses to take heparin or nitroglycerin

b. Client with third-degree heart block on the monitor Third-degree heart block is a serious complication that indicates that a large portion of the left ventricle and conduction system are involved, so the client with the third-degree heart block should be seen first. Third-degree heart block usually requires pacemaker insertion. A normal rhythm with prolonged PR interval indicates first-degree heart block, which usually does not require treatment. The client with dyspnea on exertion when ambulating to the bathroom is not at immediate risk. The client's uncooperative behavior when refusing to take heparin or nitroglycerin may indicate fear or denial; he should be seen after emergency situations have been handled.

9. A nurse assesses clients for potential endocrine disorders. Which client is at greatest risk for hyperparathyroidism? a. A 29-year-old female with pregnancy-induced hypertension b. A 41-year-old male receiving dialysis for end-stage kidney disease c. A 66-year-old female with moderate heart failure d. A 72-year-old male who is prescribed home oxygen therapy

b. Clients who have chronic kidney disease do not completely activate vitamin D and poorly absorb calcium from the GI tract. They are chronically hypocalcemic, and this triggers overstimulation of the parathyroid glands. Pregnancy-induced hypertension, moderate heart failure, and home oxygen therapy do not place a client at higher risk for hyperparathyroidism.

The nurse is concerned that a client who had myocardial infarction (MI) has developed cardiogenic shock. Which findings indicate shock? (Select all that apply.) a. Bradycardia b. Cool, diaphoretic skin c. Crackles in the lung fields d. Respiratory rate of 12 breaths/min e. Anxiety and restlessness f. Temperature of 100.4° F

b. Cool, diaphoretic skin c. Crackles in the lung fields e. Anxiety and restlessness The client with shock has cool, moist skin. Because of extensive tissue necrosis, the left ventricle cannot forward blood adequately, resulting in pulmonary congestion and crackles. Because of poor tissue perfusion, a change in mental status, anxiety, and restlessness are expected. All types of shock (except neurogenic) present with tachycardia, not bradycardia. Due to pulmonary congestion, a client with cardiogenic shock typically has tachypnea. Cardiogenic shock does not present with low-grade fever; this would be more likely to occur in pericarditis.

The client in the cardiac care unit has had a large myocardial infarction. How does the nurse recognize onset of left ventricular failure? a. Urine output of 1500 mL on the preceding day b. Crackles in the lung fields c. Pedal edema d. Expectoration of yellow sputum

b. Crackles in the lung fields Manifestations of left ventricular failure and pulmonary edema are noted by listening for crackles and identifying their locations in the lung fields. A urine output of 1500 mL is normal. Edema is a sign of right ventricular heart failure. Yellow sputum indicates the presence of white blood cells and possible infection.

A client with chronic anemia has had many blood transfusions. What medications does the nurse anticipate teaching the client about adding to the regimen? (Select all that apply.) a. Azacitidine (Vidaza) b. Darbepoetin alfa (Aranesp) c. Decitabine (Dacogen) d. Epoetin alfa (Epogen) e. Methylprednisolone (Solu-Medrol)

b. Darbepoetin alfa (Aranesp) d. Epoetin alfa (Epogen)

A client receiving a blood transfusion develops anxiety and low back pain. After stopping the transfusion, what action by the nurse is most important? a. Documenting the events in the client's medical record b. Double-checking the client and blood product identification c. Placing the client on strict bedrest until the pain subsides d. Reviewing the client's medical record for known allergies

b. Double-checking the client and blood product identification

Which atypical symptoms may be present in a female client experiencing myocardial infarction (MI)? (Select all that apply.) a. Sharp, inspiratory chest pain b. Dyspnea c. Dizziness d. Extreme fatigue e. Anorexia

b. Dyspnea c. Dizziness d. Extreme fatigue Many women who experience an MI present with dyspnea, light-headedness, and fatigue. Sharp, pleuritic pain is more consistent with pericarditis or pulmonary embolism. Anorexia is neither a typical nor an atypical sign of MI.

A nurse is preparing to administer a blood transfusion. What action is most important? a. Correctly identifying client using two identifiers b. Ensuring informed consent is obtained if required c. Hanging the blood product with Ringer's lactate d. Staying with the client for the entire transfusion

b. Ensuring informed consent is obtained if required

After thrombolytic therapy, the nurse working in the cardiac catheterization laboratory would be alarmed to notice which sign? a. A 1-inch backup of blood in the IV tubing b. Facial drooping c. Partial thromboplastin time (PTT) 68 seconds d. Report of chest pressure during dye injection

b. Facial drooping During and after thrombolytic administration, the nurse observes for any indications of bleeding, including changes in neurologic status, which may indicate intracranial bleeding. A 1-inch backup of blood in the IV tubing may be related to IV positioning. If heparin is used, PTT reflects a therapeutic value. Reports of chest pressure during dye injection or stent deployment are considered an expected result of the procedure.

10. A nurse plans care for a client with hyperparathyroidism. Which intervention should the nurse include in this client's plan of care? a. Ask the client to ambulate in the hallway twice a day. b. Use a lift sheet to assist the client with position changes. c. Provide the client with a soft-bristled toothbrush for oral care. d. Instruct the unlicensed assistive personnel to strain the client's urine for stones.

b. Hyperparathyroidism causes increased resorption of calcium from the bones, increasing the risk for pathologic fractures. Using a lift sheet when moving or positioning the client, instead of pulling on the client, reduces the risk of bone injury. Hyperparathyroidism can cause kidney stones, but not every client will need to have urine strained. The priority is preventing injury. Ambulating in the hall and using a soft toothbrush are not specific interventions for this client.

Which characteristics place women at high risk for myocardial infarction (MI)? (Select all that apply.) a. Premenopausal b. Increasing age c. Family history d. Abdominal obesity e. Breast cancer

b. Increasing age c. Family history d. Abdominal obesity Increasing age is a risk factor, especially after 70 years. Family history is a significant risk factor in both men and women. A large waist size and/or abdominal obesity are risk factors for both metabolic syndrome and MI. Premenopausal women are not at higher risk for MI, and breast cancer is not a risk factor for MI.

A client has Hodgkin's lymphoma, Ann Arbor stage Ib. For what manifestations should the nurse assess the client? (Select all that apply.) a. Headaches b. Night sweats c. Persistent fever d. Urinary frequency e. Weight loss

b. Night sweats c. Persistent fever e. Weight loss

A client comes to the emergency department with chest discomfort. Which action does the nurse perform first? a. Administers oxygen therapy b. Obtains the client's description of the chest discomfort c. Provides pain relief medication d. Remains calm and stays with the client

b. Obtains the client's description of the chest discomfort A description of the chest discomfort must be obtained first, before further action can be taken. Neither oxygen therapy nor pain medication is the first priority in this situation; an assessment is needed first. Remaining calm and staying with the client are important, but are not matters of highest priority.

Prompt pain management with myocardial infarction is essential for which reason? a. The discomfort will increase client anxiety and reduce coping. b. Pain relief improves oxygen supply and decreases oxygen demand. c. Relief of pain indicates that the MI is resolving. d. Pain medication should not be used until a definitive diagnosis has been established.

b. Pain relief improves oxygen supply and decreases oxygen demand. The focus of pain relief is on reducing myocardial oxygen demand. Chest discomfort will increase anxiety, but it may not affect coping. Relief of pain is secondary to the use of opiates or indicates that the tissue infarction is complete. Although it used to be true that pain medication was not to be used for undiagnosed abdominal pain, this does not relate to MI.

A client has a serum ferritin level of 8 ng/mL and microcytic red blood cells. What action by the nurse is best? a. Encourage high-protein foods. b. Perform a Hemoccult test on the client's stools. c. Offer frequent oral care. d. Prepare to administer cobalamin (vitamin B12).

b. Perform a Hemoccult test on the client's stools.

5. A nurse cares for a client who presents with bradycardia secondary to hypothyroidism. Which medication should the nurse anticipate being prescribed to the client? a. Atropine sulfate b. Levothyroxine sodium (Synthroid) c. Propranolol (Inderal) d. Epinephrine (Adrenalin)

b. The treatment for bradycardia from hypothyroidism is to treat the hypothyroidism using levothyroxine sodium. If the heart rate were so slow that it became an emergency, then atropine or epinephrine might be an option for short-term management. Propranolol is a beta blocker and would be contraindicated for a client with bradycardia.

Syncope in the aging person can likely occur with which actions by the patient? (SATA) a. Laughing b. Turning the head c. Performing a Valsalva maneuver d. Walking briskly for 20 to 30 minutes e. shrugging the shoulders

bce

The nurse is caring for a patient at risk for heart problems. What are normal findings for the cardiovascular assessment of this patient? (SATA) a. Presence of a thrill b. Splitting of S2; decreases with expiration c. Jugular venous distention to level of the mandible d. Point of maximal impulse (PMI) in fifth intercostal space at midclavicular line e. Paradoxical chest movement with inspiration and expiration

bd

What are the purposes of angiography? (SATA) a. Determine an abnormal structure of the heart b. Identify an arterial obstruction c. Assess the cardiovascular response to increased workload d. Identify an arterial narrowing e. Identify an aneurysm

bde

The nurse is assessing a 62-year-old native Hawaiian woman. She is postmenopausal, diabetic fro 10 years, smokes 1 pack a day of cigarettes for 20 years, walks twice a week for 30 minutes, is an administrator, and describes her lifestyle as sedentary. For her weight and height she has a body mass index of 32. Which risk factors for this patient are controllable for CVD? (SATA) a. Ethnic background b. Smoking c. Age d. Obesity e. Postmenopausal f. Sedentary lifestyle

bdf

What measures are taken to prepare a patient for a pharmacologic stress echocardiogram? (SATA) a. Patient can eat his/her diet as ordered b. IV access needs to be present c. Oxygen at 2 L per nasal cannula is placed on patient 3 hours prior to test d. An oral laxative is given the day before the test e. Patient is to be NPO for 3 to 6 hours before the test

be

1. A 22-year-old patient comes to the clinic for a wellness check-up. History reveals that the patient's parent has the autosomal-dominant form of polycystic kidney disease (PKD). Which vital sign suggests that the patient should be evaluated for PKD? a. Pulse of 90 beats/min b. Temperature of 99.6 F c. Blood pressure of 136/88 mm Hg d. Respiratory rate of 22/min

c

1. The home health nurse reads in the patient's chart that the patient has asymptomatic bacterial urinary tract infection (ABUTI). Which intervention will the nurse perform? a. Obtain an order for urinalysis and urine culture and sensitivity. b. Check the patient's medication list for appropriate antibiotic order. c. Closely monitor for conditions that cause progression to acute infection. d. Ask the patient when the ABUTI first started and when it was diagnosed.

c

10. A patient with PKD reports a severe headache and is at risk for a berry aneurysm. What is the nurse's priority action? a. Assess the pain and give a prn pain medication. b. Reassure the patient that this is an expected aspect of the disease. c. Assess for neurlologic changes and check vital signs. d. Monitor for hematuria and decreased urinary output.

c

10. The nurse is caring for a patient who has an indwelling catheter and subsequently developed a UTI. The patient has been receiving antibiotics for several days, but develops hypotension, a rapid pulse, and confusion. The nurse suspects urosepsis and alerts the health care provider. Which diagnostics test is the provider most likely to order to confirm urosepsis? a. Culture of the drainage bag b. Culture of the catheter tip c. Blood culture d. Repeat urinalysis

c

11. A patient with PKD reports nocturia. What is the nocturia caused by? a. Increased fluid intake in the evening. b. Increased hypertension. c. Decreased urine-concentrating ability d. Detrusor irritability

c

11. Which combination of drugs is the most nephrotoxic? a. Angiotensin-converting enzyme (ACE) inhibitors and aspirin b. Antiotensin II receptor blockers and antiacids c. Aminoglycoside antibiotics and nonsteroidal antiinflammatory drugs (NSAIDs). n d. Calcium channel blockers and antihistamines

c

12. The nurse is reviewing laboratory results for a patient with PKD. Which laboratory abnormality indicates glomeruli involvement? a. Low specific gravity of urine b. Bacteria in urine c. Proteinuria d. Hematuria

c

13. A patient is suspected of having PKD. Which diagnostic study has minimal risk and can reveal PKD? a. Kidneys-ureters-bladder (KUB) x-ray b. Urography c. Renal sonography d. Renal angiography

c

13. For a patient with AKI, the nurse would consider questioning the order for which diagnostic test? a. Kidney biopsy b. Ultrasonography c. Computed tomography with contrast dye d. Kidney, ureter, bladder (KUB) x-ray

c

13. Which statement best describes globe lag in a patient with hyperthyroidism? a. Abnormal protrusion of the eyes b. Upper eyelid fails to descend when the patient gazes downward c. Upper eyelid pulls back faster than the eyeball when the patient gazes upward d. Inability of both eyes to focus on an object simultaneously

c

15. A patient requires 100 g of oral glucose for suppression testing and GH levels are measured serially for 120 minutes. The results of the suppression testing are abnormal. The nurse assesses for the signs and symptoms of which endocrine disorder? a. Adrenal insufficiency b. DI c. Hyperpituitarism d. Hypothyroidism

c

16. A young female patient reports experiencing burning with urination. What question does the nurse ask to differentiate between a vaginal infection and a urinary infection? a. "Have you noticed any blood in the urine?" b. "Have you had recent sexual intercourse?" c. "Have you noticed any vaginal discharge?" d. "Have you had fever or chills?"

c

45. In addition to regulation of calcium levels, PTH and calcitonin regulate the circulating blood levels of which substance? a. Potassium b. Sodium c. Phsophate d. Chloride

c

2. Which description of the recessive form of PKD is correct? a. Prognosis is better for the recessive form compared to the dominant form. b. 100% of people with this form of PKD develop kidney failure around age 50. c. Most people with this form of PKD die in early childhood. d. The recessive from only manifests if other kidney problems occurs.

c

21. Which patient narrative describes the symptom of dysuria? a. "I have to pee all the time." b. "I have to wait before the pee starts." c. "It burns when I pee." d. "It feels like I am going to pee in my pants."

c

22. The action of antidiuretic hormone (ADH) influences normal kidney function by stimulating which mechanism? a. Glomerulus to control the filtration rate b. Proximal nephron tubules to reabsorb water c. Distal nephron tubules and collecting ducts to reabsorb water d. Constriction of glomerular capillaries to prevent loss of protein in urine

c

24. An older adult male patient calls the clinic because he has "not passed any urine all day long." What is the nurse's best response? a. "Try drinking several large glasses of water and waiting for a few more hours." b. "If you develop flank pain or fever, then you should probably come in." c. "You could have an obstruction, so you should come in to be checked." d. "I am sorry, but I really can't comment about your problem over the phone."

c

24. The patient has multiple thyroid nodules resulting in thyroid hyperfunction. What is the most likely cause of this hyperthyroidism? a. Thyroid carcinoma b. Graves' disease c. Toxic multinodular goiter d. Pituitary hyperthyroidism

c

25. The new registered nurse is identifying a patient for blood transfusion. Which action by the new nurse warrants intervention by the supervising nurse? a. Checks the health care provider's order before the blood transfusion. b. Compares the hospital identification band name and number to those on the blood component tag c. Uses the patient's room number as a form of identification d. Examines blood bag tag and attached tag to ensure that the ABO and Rh types are compatible

c

25. The nurse is determining whether a patient has a history of hypertension because of the potential for kidney problems. Which question is best to elicit this information? a. "Do you have high blood pressure?" b. "Do you take any blood pressure medications?" c. "Have you ever been told that your blood pressure was high?" d. "When was the last time you had your blood pressure checked?"

c

27. A patient with exophthalmos from hyperthyroidism reports dry eyes, especially in the morning. THe nurse teaches the patient to perform which intervention to help correct this problem? a. Wear sunglasses at all times when outside in the bright sun. b. Use coll compresses to the eye four times a day. c. Tape the eyes closed with nonallergenic tape. d. There is nothing that can be done to relieve this problem.

c

27. A patient with exophthalmos from hyperthyroidism reports dry eyes, especially in the morning. The nurse teaches the patient to perform which intervention to help correct this problem? a. Wear sunglasses at all times when outside in the bright sun. b. Use cool compresses to the eye four times a day. c. Tape the eyes closed with nonallergenic tape. d. There is nothing that can be done to relieve this problem.

c

27. The cystoscopy results for a patient include a small-capacity bladder, the presence of Hunner's ulcers, and small hemorrhages after bladder distention. How does the nurse interpret this report? a. Urosepsis b. Complicated cystitis c. Interstitial cystitis d. Urethritis

c

27. The nurse is taking a nutritional history on a patient. The patient states, "I really don't drink as much water as I should." What is the nurse's best response? a. "We should probably all drink more water than we do." b. "It's an easy thing to forget; just try to remember to drink more." c. "What would encourage you to drink the recommended 2 liters per day?" d. "I'd like you to read this brochure about kidney health and fluids."

c

29. When caring for a patient after bone marrow stem cell transplantation, when does the nurse expect engraftment (the settling in of stem cells and the start of producing new cells) to occur? a. 8 to 12 hours after infusion b. 7 days after infusion c. 21 days after infusion d. 6 weeks after infusion

c

29. Which patient's history puts him or her at risk for developing SIADH? a. 27-year-old patient on high-dose steroids b. 47-year-old hospitalized adult patient with acute renal failure c. 58-year-old with metastatic lung or breast cancer d. Older adult with history of a stroke within the last year

c

3. Which description of the autosomal-dominant form of PKD is correct? a. 25% of patients with this from of PKD develop acute kidney failure by age 30. b. The dominant from is responsive to newer gene therapy treatments. c. 50% of people with this form of PKD develop kidney disease by age 50. d. Most people with this form of PKD die in young adulthood.

c

31. The effect of increased ADH in the blood results in which effect on the kidney? a. Urine concentration tends to decrease. b. Glomerular filtration tends to decrease. c. Tubular reabsorption of water increases. d. Tubular reabsorption of sodium increases.

c

32. Which hematologic disorder is most likely to cause the patient to have joint problems? a. Thrombocytopenia b. Aplastic anemia c. Hemophilia d. Warm antibody anemia

c

33. A patient is diagnosed with urethral stricture. What findings does the nurse expect to see documented in the patient's chart for this condition? a. Pain on urination b. Pain on ejaculation c. Overflow incontinence d. Hematuria and pyuria

c

34. Which type of IV fluid does the nurse use to treat a patient with SIADH when the serum sodium level is very low? a. D5 1/2 normal saline b. D5W c. 3% normal saline d. Normal saline

c

36. The health care provider advises the patient that diagnostic testing is needed to identify the possible presence of a renal abscess. Which test does the nurse prepare the patient for? a. Renal arteriography b. Cystourethrogram c. Radionuclide renal scan d. Urodynamic flow studies

c

70. The nurse is talking to a 68-year-old male patient who has lifestyle choices and occupational exposure that put him at high risk for bladder cancer. The nurse is most concerned about which urinary characteristic? a. Frequency b. Nocturia c. Painless hematuria d. Incontinence

c

36. The nurse is caring for an obese older adult patient with dementia. The patient is alert and ambulatory, but has functional incontinence. Which nursing intervention is best for this patient? a. Help the patient to lose weight. b. Help the patient apply an estrogen cream. c. Offer assistance with toileting every 2 hours. d. INtermittently catheterize the patient.

c

37. A patient with chronic peylonephritis returns to the clinic for follow-up. Which behavior indicates the patient is meeting the expected outcomes to conserve existing kidney function? a. Drinks a liter of fluid every day b. Considers buying a home blood pressure cuff c. Reports taking antibiotics as prescribed d. Takes pain medication on a regular basis

c

37. The nurse is assessing a patient after thyroid surgery and discovers harsh, high-pitched respiratory sounds. What is the nurse's best first action? a. Administer oxygen at 5 L via nasal cannula. b. Administer IV calcium chlroide. c. Notify the Rapid Response Team. d. Suction the patient for oral secretions.

c

37. To avoid transfusion reaction, the nurse is carefully monitoring the patient during a blood transfusion. When are hemolytic reactions to blood transfusion most likely to occur? a. 1 mL is sufficient b. 5 mL is typical c. Within the first 50 mL d. Occurs after 100 mL

c

38. A patient's laboratory results show an elevated creatinine level. The patient's history reveals no risk factors for kidney disease. Which question does the nurse ask the patient to shed further light on the laboratory result? a. "How many hours of sleep did you get the night before the test?" b. "How much fluid did you drink before the test?" c. "Did you take any type of antibiotics before taking the test?" d. "When and how much did you last urinate before having the test?"

c

38. The nurse is caring for an older adult patient with urinary incontinence. The patient is alert and oriented, but refuses to use the call bell and has fallen several times while trying to get to the bathroom. What is the nurse's priority concern for this patient? a. Managing noncompliance b. Accurately measuring urinary output c. Providing fall prevention measures d. Managing urinary incontinence

c

38. Which patient has the greatest risk for developing chronic pyelonephritis? a. 80-year-old woman who takes diuretics for mild heart failure b. 80-year-old man who drinks four cans of beer per day c. 36-year-old woman with diabetes mellitus who is pregnant d. 36-year-old man with diabetes insipidus

c

38. Which type of medication is used for patients receiving a platelet transfusion as premedications to prevent a reaction? a. Vitamin K and a diuretic b. Aspirin and hydroxyurea c. Diphenhydramine and acetaminophen d. Hydroxortisone and an antihypertensive

c

39. A patient is diagnosed with acute pyelonephritis. What is the priority for nursing care for this patient? a. Providing information about the disease process b. Controlling hypertension c. Managing pain d. Preventing constipation

c

4. The nurse is working in a long-term care facility. Which circumstance is cause for greatest concern, because the facility has a large number of residents who are developing UTIs? a. Residents are not drinking enough fluids with meals. b. Unlicensed personnel are not assisting with toileting in a timely fashion. c. A large percentage of residents have indwelling urinary catheters. d. Many residents have severe dementia and functional incontinence.

c

4. Which factor is a hallmark assessment finding that signifies hyperthyroidism? a. Weight loss b. Increased libido c. Heat intolerance d. Diarrhea

c

40. The nurse is performing the immediate postprocedure care for a bone marrow donor. What is the priority assessment that the nurse will perform? a. Monitoring for activity intolerance b. Monitoring for infection c. Monitoring for fluid loss d. Monitoring CBC and platelet counts

c

41. Ketones in the urine may indicate which occurrence or process? a. Increased glomerular membrane permeability b. Chronic kidney infection c. Body's use of fat for cellular energy d. Urinary tract infection

c

41. The nurse is assessing a patient with kidney injury and notes a marked increase in the rate and depth of breathing. The nurse recognizes this as Kussmaul respiration, which is the body's attempt to compensate for which condition? a. Hypoxia b. Alkalosis c. Acidosis d. Hypoxemia

c

42. A patient is diagnosed with renal osteodystrophy. What does the nurse instruct the unlicensed assistive personnel (UAP) to do in relation to this patient's diagnosis? a. Assist the patient with toileting every 2 hours. b. Gently wash the patient's skin with a mild soap and rinse well. c. Handle the patient gently because of risk for fractures. d. Assist the patient with eating because of loss of coordination.

c

42. The nurse and nutritionist are evaluating the diet and nutritional therapies for a patient with kidney problems. Blood urea nitrogen (BUN) levels for this patient are tracked because of the direct relationship to the intake and metabolism of which substance? a. Lipids b. Carbohydrates c. Protein d. Fluids

c

43. The nurse is assessing a patient with glomerulonephritis and notes crackles in the lung fields and neck vein distention. The patient reports mild shortness of breath. Based on these findings, what does the nurse do next? a. Check for CVA tenderness or flank pain. b. Obtain a urine sample to check for proteinuria. c. Assess for additional signs of fluid overload. d. Alert the health care provider about the respiratory symptoms.

c

43. The nurse would measure abdominal girth to monitor of which complication of hematopoietic stem cell transplantation? a. Failure to engraft b. Graft-versus-host disease c. Venoocclusive disease d. Septic shock

c

44. The nurse is evaluating outcome criteria for a patient being treated for urge incontinence. Which statement indicates the treatment has been successful? a. "I'm following the prescribed therapy, but I think surgery is my best choice." b. "I still lose a little urine when I sneeze, but I have been wearing a thin pad." c. "I had trouble at first, but now I go to the toilet every 3 hours." d. "I have been using the bladder compression technique and it works."

c

47. An older adult patient with a cognitive impairment is living in an extended-care facility. The patient is incontinent, but as the family points out, "he will urinate in the toilet if somebody helps him." Which type of incontinence does the nurse suspect in this patient? a. Urge b. Overflow c. Functional d. Stress

c

47. What type of breath odor is most likely to be noted in a patient with CKD? a. Fruity smell b. Fecal smell c. Smells like urine d. Smells like blood

c

48. A patient with continuous spasms of the muscles is diagnosed with hypoparathyroidism. The muscle spasms are a clinical manifestation of which condition? a. Nerve damage b. Seizures c. Tetany e. Decreased ppotassium

c

49. In addition to kidney disease, which patient condition causes the BUN to rise above the normal range? a. Anemia b. Asthma c. Infection d. Malnutrtion

c

5. The nurse has taught the patient about dietary modifications for his vitamin B12 deficiency anemia. Which statement by the patient indicates that additional teaching is needed? a. "Dairy products are a good source of vitamin B12." b. "Dried beans taste okay if they are prepared correctly." c. "Leafy green vegetables interfere with my therapy." d. "I like nuts and I will gladly eat them."

c

50. The community health nurse is talking to a group of African-American adults about renal health. The nurse encourages the participants to have which type of yearly examination to screen for kidney problems? a. Kidney ultrasound b. Serum creatinine and blood urea nitrogen c. Urinalysis and microalbuminuria d. 24-hour urine collection

c

50. The nurse notes an abnormal laboratory test finding for a patient with CKD and alerts the health care provider. The nurse also consults with the registered dietitian because an excessive dietary protein intake is directly related to which factor? a. Elevated serum creatinine level b. Protein presence in the urine c. Elevated BUN level d. Elevated serum potassium level

c

53. What does an increase in the ratio of BUN to serum creatinine indicate? a. Highly suggestive of kidney dysfunction b. Definitive for kidney infection c. Suggests non-kidney factors causing and elevation in BUN d. Suggest non-kidney factors causing an elevation in serum creatinine

c

54. Which dietary changes does the nurse suggest to a patient with urge incontinence? a. Limit fluid intake to no more than 2L/day. b. Peel all fruit before consuming. c. Avoid alcohol and caffeine. d. Avoid smoked or salted foods.

c

55. A patient has a urinalysis ordered. When is the best time for the nurse to collect the specimen? a. In the evening b. After a meal c. In the morning d. After a fluid bolus

c

57. A 24-hour urine specimen is required from a patient. Which strategy is best to ensure that all the urine is collected for the full 24-hour period? a. Instruct the unlicensed assitive personnel (UAP) to collect all the urine. b. Put a bedpan or commode next to the bed as a reminder. c. Place a sign in the bathroom reminding everyone to save the urine. d. Verbally remind the patient about the test.

c

6. A patient with a history of PKD reports dull, aching flank pain and the urinalysis is negative for infection. The health care provider tells the nurse that the pain is chronic and related to enlarging kidneys compressing abdominal contents. What nursing intervention is best for this patient? a. Administer trimethoprim/sulfamethoxazole (Bactrim). b. Apply cool compresses to the abdomen or flank. c. Teach methods of relaxation such as deep-breathing. d. Administer around-the-clock nonsteroidal antiinflammtory drugs (NSAIDs).

c

60. A patient reports severe flank pain. The report indicates that urine is turbid, malodorous, and rust-colored; RBCs, WBCs, and bacteria are present; and microscopic analysis shows crystals. What does this data suggest? a. Pyuria and cystitis b. Staghorn calculus with infection c. Urolithiasis and infection d. Dysuria and urinary retention

c

61. A patient is newly admitted with nephrotic syndrome and has proteinuria, edema, hyperlipidemia, and hypertension. What is the priority for nursing care? a. Consult the dietitian to provide adequate nutritional intake. b. Prevent urinary tract infection. c. Monitor fluid volume and the patient's hydration status. d. Prepare the patient for a renal biopsy.

c

61. Which behavior is the strongest indicator that a patient with ESKD is not coping well with the illness and may need a referral for psychological counseling? a. Displays irritability when the meal tray arrives b. Refuses to take one of the drugs because it causes nausea c. Repeatedly misses dialysis appointments d. Seems distracted when the health care provider talks about the prognosis

c

62. A patient is diagnosed with interstitial neprhitis. Which nursing action is relevant and specific for this patient's medical condition? a. Avoid analgesic use. b. Use disposable gloves. c. Monitor for fever. d. Place the patient in isolation.

c

62. Which diagnostic test incorporates contrast dye, but does not place a patient at risk for nephrotoxicity? a. Renal scan b. Renal angiography c. Voiding cystourethrogram d. Computed tomography

c

63. Which patient is the most likely candidate for CVVH? a. Patient with fluid volume overload b. Patient who needs long-term management c. Patient who is critically ill d. Patient who is ready for discharge to home

c

65. The nurse is caring for a patient with CKD. The family asks about when renal replacement therapy will begin. What is the nurse's best response? a. "As early as possible to prevent further damage in stage I." b. "When there is reduced kidney function and metabolic wastes accumulate." c. "When the kidneys are unable to maintain a balance in body functions." d. "It will be started with diuretic therapy to enhance the remaining function."

c

69. Which patient has the highest risk for bladder cancer? a. 60-year-old male patient with malnutrition secondary to chronic alcoholism and self-neglect b. 25-year-old male patient with type 1 diabetes mellitus, who is noncompliant with therapeutic regimen c. 60-year-old female patient who smokes two packs of cigarettes per day and works in a chemical factory d. 25-year-old female patient who has had three episodes of bacterial (Escherichia coli) cystitis in the past year

c

71. The nurse is planning the care for several patients who are undergoing diagnostic testing. Which patient is likely to need the most time for post-procedural care? a. Will have a kidney, ureter, and bladder x-ray b. needs a kdiney ultrasound c. Will have a cystoscopy d. Needs urine for culture and sensitivity

c

72. Which statement by a patient indicates effective coping with Kock's pouch? a. "I don't have any discomfort, but the pouch frequently overflow." b. "My wife has been irrigating the pouch daily. She likes to do it." c. "I check the pouch every 2 to 3 hours depending on my fluid and diet." d. "I never undress in front of anyone anymore, but I guess that is okay."

c

79. A patient returning to the unit after a left radical nephrectomy for kidney cell carcinoma reports having some soreness on the right side. What does the nurse tell the patient? a. "The right kidney was re-positioned to take over the function of both kidneys." b. "I'll call your doctor for an order to increase your pain medication." c. "The soreness is likely to be from being positioned on your right side during surgery." d. "Would you like to talk with someone who had this surgery last year and now is fully recovered?"

c

8. The nurse is talking to a group of healthy young college students about maintaining good kidney health and preventing AKI. Which health promotion point is the nurse most likely to emphasize with this group? a. "Have your blood pressure checked regularly." b. "Find out if you have a family history of diabetes." c. "Avoid dehydration by drinking at least 2 to 3 L of water daily." d. "Have annual testing for microalbuminuria and urine protein."

c

8. The unlicensed assistive personnel (UAP) is providing care to a patient in sickle cell crisis. Which action by the UAP requires intervention by the supervising nurse? a. Elevating the head of the bed to 25 degrees b. Assisting to remove any restrictive clothing c. Obtaining the blood pressure with an external cuff d. Offering the patient her beverage of choice

c

81. The nurse is caring for a patient after a nephrectomy. The nurse notes that the urine flow was 50 mL/hr at the beginning of the shift, but several hours later has dropped 30 mL. What would the nurse do first? a. Notify the health care provider for an order for an IV fluid bolus. b. Document the finidng and continue to monitor for downward trend. c. Check the drainage system for kinks or obstructions to flow. d. Obtain the patient's weight and compare it to baseline.

c

83. The health care provider has ordered intraperitoneal heparin for a patient with a new PD catheter to prevent clotting of the catheter by blood and fibrin formation. How does the nurse advise the patient? a. Watch for bruising or bleeding from the gums. b. Make a follow-up appointment for coagulation studies. c. Intraperitoneal heparin does not affect clotting times. d. Heparin will be given with a small subcutaneous needle.

c

84. The ED nurse is preparing a patient with kidney trauma for emergency surgery. What is the best task to delegate to the unlicensed assistive personnel (UAP)? a. Set the automated blood pressure machine to cycle every 2 hours. b. Inform the family about surgery and assist them to the surgery waiting area. c. Go to the blood bank and pick up the units of packed red cells. d. Insert a urinary catheter if there is no gross bleeding at the urethra.

c

85. A patient has sustained a kidney injury. In order to assist the patient to undergo the best diagnostic test to determine the extent of injury, what does the nurse do? a. Obtain a clean-catch urine specimen for urinalysis. b. Give an IV fluid bolus before renal arteriography. c. Give an explanation of computed tomography. d. Obtain a blood sample for hemoglobin and hematocrit.

c

88. A patient has recently started PD therapy and reports some mild pain when the dialysate is flowing in. What does the nurse do next? a. Immediately report the pain to the health care provider. b. Try warming the dialysate in the microwave oven. c. Reassure that pain should subside after the first week or two. d. Assess the connection tubing for kinking or twisting.

c

90. The nurse is monitoring a patient's PD treatment. The total outflow is slightly less than the inflow. What does the nurse do next? a. Instruct the patient to ambulate. b. Notify the health care provider. c. Record the difference as intake. d. Put the patient on fluid restriction.

c

94. The intensive care nurse is caring for the kidney transplant patient who was just transferred from the recovery unit. Which finding is the most serious within the first 12 hours after surgery and warrants immediate notification of the transplant surgeon? a. Diuresis with increased output b. Pink and bloody urine c. Abrupt decrease in urine d. Small clots in bladder irrigation fluid

c

A patient comes to the clinic stating "my right foot turns a darkish red color when I sit too long, and when I put my foot up, it turns pale." Which conditions does the nurse suspect? a. Central cyanosis b. Peripheral cyanosis c. Arterial insufficiency d. Venous insufficiency

c

A patient is diagnosed with iron deficiency anemia. Which assessment finding is the nurse most likely to observe in this patient? a. Neck veins are distended and edema is present. b. Lower extremities show signs of phlebitis. c. Systolic blood pressure is lower than normal. d. Palpation of ribs or sternum elicits tenderness.

c

A patient is scheduled to have an exercise electrocardiography test. What instruction does the nurse provide to the patient before the procedure takes place? a. "Have nothing to eat or drink after midnight." b. "Avoid smoking or drinking alcohol for at least 2 weeks before the test" c "Wear comfortable, loose clothing and rubber-soled, supportive shoes" d "Someone must drive you home because of possible sedative effects of the medications"

c

A patient is undergoing diagnostic testing for reports of chest pain. Which test is done to determine the location and extent of CAD? a. ECG b. Echocardiogram c. Cardiac catheterization d. Chest x-ray

c

A patient with exophthalmos from hyperthyroidism reports dry eyes, especially in the morning. The nurse teaches the patient to perform which intervention to help correct this problem? a. Wear sunglasses at all times when outside in the bright sun. b. Use cool compresses to the eye four times a day. c. Tape the eyes closed with nonallergenic tape. d. There is nothing that can be done to relieve this problem.

c

A young patient reports having frequent episodes of palpitations but denies having chest pain. Which follow-up question does the nurse ask to assess the patient symptom of palpitations? a. "Have you noticed a worsening of shortness of breath when you are lying flat?" b "Do your shoes feel unusually tight, or are your rings tighter than usual?" c. "Do you feel dizzy or have you lost consciousness with the palpitations?" d. "Does anyone in your family have a history of palpitations?"

c

Based on knowledge of physiologic triggers for RBC production, the nurse would anticipate which chronic health condition to be associated with an increase in RBC production? a. Diabetes mellitus b. Osteoarthritis c. Chronic obstructive pulmonary disease d. Chronic kidney disease

c

Based on the physiologic force that propels blood forward in the veins, the which patient has the greatest risk for venous stasis? a. Older adult patient with hypertension who rides a bicycle daily b. Middle-aged construction worker taking Coumadin c. Bedridden patient in the end stage of Alzheimer's disease d. Teenage patient with a broken leg who sits and plays videogames

c

The nurse is assessing a patient after thyroid surgery and discovers harsh, high-pitched respiratory sounds. What is the nurse's best first action? a. Administer oxygen at 5 L via nasal cannula. b. Administer IV calcium chloride. c. Notify the Rapid Response Team. d. Suction the patient for oral secretions.

c

The nurse is assessing a patient with Graves' disease and observes an abnormal protrusion of both eyeballs. How does the nurse document this assessment finding? a. Globe lag b. Pretibial myxedema c. Exophthalmos d. Goiter

c

The nurse is caring for a patient with the priority problem of decreased cardiac output. Which situation may result in decreased myocardial contractility that will further lower cardiac output? a. Administration of a positive inotropic medication b. Hyperventilation to correct respiratory acidosis c. Frequent endotracheal suctioning that result in hypoxemia d. Administration of IV fluids to correct underlying hypovolemia

c

The nurse is providing instructions to a patient taking levothyroxine (Synthroid). When does the nurse tell the patient to take this medication? a. With breakfast in the morning b. At lunchtime immediately after eating c. In the morning on an empty stomach d. At dinnertime within 15 minutes after eating

c

The nurse is taking report on a client who will be transferred from the cardiac intensive care unit to the general medical-surgical unit. The reporting nurse states that S4 is heard on auscultation of the heart. This indicates that the patient has which condition? a. Heart murmur b. Pericardial friction rub c. Ventricular hypertrophy d. Normal heart sounds

c

The nurse practitioner reads in a patient's chart that a carotid bruit was heard during the last two annual checkups. Today on auscultation, the bruit is not present. How does the nurse practitioner evaluate this data? a. The problem has resolved spontaneously b. There may have been an anomaly in previous findings c. The occlusion of the vessel may have progressed past 90% d. The antiplatelet therapy is working

c

The nurse sees that a 45-year-old woman has a low hemoglobin level. The nurse would perform a dietary assessment to identify a possible deficiency in which nutrient? a. Calcium b. Vitamin K c. Iron d. Vitamin D

c

The patient has multiple thyroid nodules resulting in thyroid hyperfunction. What is the most likely cause of this hyperthyroidism? a. Thyroid carcinoma b. Graves' disease c. Toxic multinodular goiter d. Pituitary hyperthyroidism

c

To assist the health care provider in determining whether a patient is a candidate for fibrinolytic therapy, the nurse is interviewing the patient diagnosed with a myocardial infarction. Why is determining the time of symptom onset essential in decision-making? a. Fibrinolytic drugs will not dissolve clots that are older than 6 hours. b. Clots that are older than 6 hours are too large and tightly meshed. c. Tissue that is anoxic for more than 6 hours is unlikely to benefit. d. After 6 hours, the patient is more likely to have excessive bleeding.

c

Which category of cardiovascular drugs increases heart rate and contractility? a. Diuretics b. Beta blockers c. Catecholamines d. Benzodiazepines

c

Which factor is a hallmark assessment finding that signifies hyperthyroidism? a. Weight loss b. Increased libido c. Heat intolerance d. Diarrhea

c

Which laboratory result would indicate that the prescription for Epogen is having the desired therapeutic effect? a. Increase in platelet count b. Increase in white blood cell (WBC) count c. Increase in red blood cell (RBC) count d. Increase in iron level

c

Which statement best describes globe lag in a patient with hyperthyroidism? a. Abnormal protrusion of the eyes b. Upper eyelid fails to descend when the patient gazes downward c. Upper eyelid pulls back faster than the eyeball when the patient gazes upward d. Inability of both eyes to focus on an object simultaneously

c

a 65-year-old patient comes to the clinic reporting fatigue. The patient would like to start an exercise program, but thinks "anemia might be causing the fatigue." What is the nurse's first action? a. Advise the patient to start out slowly and gradually build strength and endurance b. Obtain an order for a complete blood count and nutritional profile c. Assess the onset, duration, and circumstances associated with the fatigue d. Perform a physical assessment to include testing of muscle strength and tone.

c

Question 38 of 41 A hematology unit is staffed by RNs, LPN/LVNs, and unlicensed assistive personnel (UAP). When the nurse manager is reviewing documentation of staff members, which entry indicates that the staff member needs education about his or her appropriate level of responsibility and client care? a. "Abdominal pain relieved by morphine 4 mg IV; client resting comfortably and denies problems. B.C., RN" b. "Ambulated in hallway for 40 feet and denies shortness of breath at rest or with ambulation. T.Y., LPN" c. "Client reporting increased shortness of breath; oxygen increased to 4 L by nasal cannula. M.N., UAP" d. "Vital signs 37.0° C, heart rate 60, respiratory rate 20, blood pressure 110/68, and oximetry 98% on room air. L.D., UAP"

c "Client reporting increased shortness of breath; oxygen increased to 4 L by nasal cannula. M.N., UAP" Determination of the need for oxygen and administration of oxygen should be done by licensed nurses who have the education and scope of practice required to administer it. All other documentation entries reflect appropriate delegation and assignment of care.

Ch.62 Question 20 of 23 The nurse is providing discharge instructions to a client on spironolactone (Aldactone) therapy. Which comment by the client indicates a need for further teaching? a. "I must call the provider if I am more tired than usual." b. "I need to increase my salt intake." c. "I should eat a banana every day." d. "This drug will not control my heart rate."

c "I should eat a banana every day." Spironolactone increases potassium levels, so potassium supplements and foods rich in potassium, such as bananas, should be avoided to prevent hyperkalemia. While taking spironolactone, symptoms of hyponatremia such as drowsiness and lethargy must be reported; the client may need increased dietary sodium. Spironolactone will not have an effect on the client's heart rate.

Ch.63 Question 18 of 21 The nurse is teaching a client about thyroid replacement therapy. Which statement by the client indicates a need for further teaching? a. "I should have more energy with this medication." b. "I should take it every morning." c. "If I continue to lose weight, I may need an increased dose." d. "If I gain weight and feel tired, I may need an increased dose."

c "If I continue to lose weight, I may need an increased dose." Weight loss indicates a need for a decreased dose, not an increased dose. One of the symptoms of hypothyroidism is lack of energy; thyroid replacement therapy should help the client have more energy. The correct time to take thyroid replacement therapy is in the morning. If the client is gaining weight and continues to feel tired, that is an indication that the dose may need to be increased.

Ch.64 Question 2 of 25 A client expresses fear and anxiety over the life changes associated with diabetes, stating, "I am scared I can't do it all and I will get sick and be a burden on my family." What is the nurse's best response? a. "It is overwhelming, isn't it?" b. "Let's see how much you can learn today, so you are less nervous." c. "Let's tackle it piece by piece. What is most scary to you?" d. "Other people do it just fine."

c "Let's tackle it piece by piece. What is most scary to you?" Suggesting the client tackle it piece by piece and asking what is most scary to him or her is the best response; this approach will allow the client to have a sense of mastery with acceptance. Referring to the illness as overwhelming is supportive, but is not therapeutic or helpful to the client. Trying to see how much the client can learn in one day may actually cause the client to become more nervous; an overload of information is overwhelming. Suggesting that other people handle the illness just fine is belittling and dismisses the client's concerns.

Ch.62 Question 10 of 23 A client with Cushing's disease begins to laugh loudly and inappropriately, causing the family in the room to be uncomfortable. What is the nurse's best response? a. "Don't mind this. The disease is causing this." b. "I need to check the client's cortisol level." c. "The disease can sometimes affect emotional responses." d. "Medication is available to help with this."

c "The disease can sometimes affect emotional responses." The client may have neurotic or psychotic behavior as a result of high blood cortisol levels. Being honest with the family helps them to understand what is happening. Telling the family not to mind the laughter and that the disease is causing it is vague and minimizes the family's concern. This is the perfect opportunity for the nurse to educate the family about the disease. Cushing's disease is the hypersecretion of cortisol, which is abnormally elevated in this disease and, because the diagnosis has already been made, blood levels do not need to be redrawn. Telling the family that medication is available to help with inappropriate laughing does not assist them in understanding the cause of or the reason for the client's behavior.

Question 36 of 41 A client with leukemia is being discharged from the hospital. After hearing the nurse's instructions to keep regularly scheduled follow-up provider appointments, the client says, "I don't have transportation." How does the nurse respond? a. "A pharmaceutical company might be able to help." b. "I might be able to take you." c. "The local American Cancer Society may be able to help." d. "You can take the bus."

c "The local American Cancer Society may be able to help." Many local units of the American Cancer Society offer free transportation to clients with cancer, including those with leukemia. Suggesting a pharmaceutical company is not the best answer; drug companies typically do not provide this type of service. Although the nurse offering to take the client is compassionate, it is not appropriate for the nurse to offer the client transportation. Telling the client to take the bus is dismissive and does not take into consideration the client's situation (e.g., the client may live nowhere near a bus route).

Ch.64 Question 1 of 25 A diabetic client has a glycosylated hemoglobin (HbA1C) level of 9.4%. What does the nurse say to the client regarding this finding? a. "Keep up the good work." b. "This is not good at all." c. "What are you doing differently?" d. "You need more insulin."

c "What are you doing differently?" Assessing the client's regimen or changes he or she may have made is the basis for formulating interventions to gain control of blood glucose. HbA1C levels for diabetic clients should be less than 7%; a value of 9.4% shows poor control over the past 3 months. Telling the client this is not good, although true, does not take into account problems that the client may be having with the regimen and sounds like scolding. Although it may be true that the client needs more insulin, an assessment of the client's regimen is needed before decisions are made about medications.

30. Which patient should not be advised to take cranberry juice? a. 26-year-old pregnant woman with a history of uncomplicated UTI b. 23-year-old man with history of recurrent kidney stones c. 65-year-old man with urinary retention secondary to enlarged prostate d. 33-year-old woman with dysuria associated with interstitial cystitis

d

Ch.64 Question 6 of 25 A client with type 2 diabetes has been admitted for surgery, and the health care provider has placed the client on insulin in addition to the current dose of metformin (Glucophage). The client wants to know the purpose of taking the insulin. What is the nurse's best response? a. "Your diabetes is worse, so you will need to take insulin." b. "You can't take your metformin while in the hospital." c. "Your body is under more stress, so you'll need insulin to support your medication." d. "You must take insulin from now on because the surgery will affect your diabetes."

c "Your body is under more stress, so you'll need insulin to support your medication." Because of the stress of surgery and NPO status, short-term insulin therapy may be needed perioperatively for the client who uses oral antidiabetic agents. For those receiving insulin, dosage adjustments may be required until the stress of surgery subsides. No evidence suggests that the client's diabetes has worsened; however, surgery is stressful and may increase insulin requirements. Metformin may be taken in the hospital; however, not on days when the client is NPO for surgery. When the client returns to his or her previous health state, oral agents will be resumed.

Ch.64 Question 10 of 25 Which of these clients with diabetes does the endocrine unit charge nurse assign to an RN who has floated from the labor/delivery unit? a. A 58-year-old with sensory neuropathy who needs teaching about foot care b. A 68-year-old with diabetic ketoacidosis who has an IV running at 250 mL/hr c. A 70-year-old who needs blood glucose monitoring and insulin before each meal d. A 76-year-old who was admitted with fatigue and shortness of breath

c A 70-year-old who needs blood glucose monitoring and insulin before each meal A nurse from the labor/delivery unit would be familiar with blood glucose monitoring and insulin administration because clients with type 1 and gestational diabetes are frequently cared for in the labor/delivery unit. The 58-year-old with sensory neuropathy, the 68-year-old with diabetic ketoacidosis, and the 76-year-old with fatigue and shortness of breath all have specific teaching or assessment needs that are better handled by nurses more familiar with caring for older adults with diabetes.

Ch.61 Question 13 of 16 A client is hospitalized with a possible disorder of the adrenal cortex. Which nursing activity is best for the charge nurse to delegate to an experienced nursing assistant? a. Ask about risk factors for adrenocortical problems. b. Assess the client's response to physiologic stressors. c. Check the client's blood glucose levels every 4 hours. d. Teach the client how to do a 24-hour urine collection.

c Check the client's blood glucose levels every 4 hours. Blood glucose monitoring is within the nursing assistant's scope of practice if the nursing assistant has received education and evaluation in the skill. Assessing risk factors for adrenocortical problems is not part of a nursing assistant's education. Assessing the client's response to physiologic stressors requires the more complex skill set of licensed nursing staff. Teaching the proper method for a 24-hour urine collection is a multi-step process; this task should not be delegated.

Question 13 of 16 A client is hospitalized with a possible disorder of the adrenal cortex. Which nursing activity is best for the charge nurse to delegate to an experienced nursing assistant? a. Ask about risk factors for adrenocortical problems. b. Assess the client's response to physiologic stressors. c. Check the client's blood glucose levels every 4 hours. d. Teach the client how to do a 24-hour urine collection.

c Check the client's blood glucose levels every 4 hours. Blood glucose monitoring is within the nursing assistant's scope of practice if the nursing assistant has received education and evaluation in the skill. Assessing risk factors for adrenocortical problems is not part of a nursing assistant's education. Assessing the client's response to physiologic stressors requires the more complex skill set of licensed nursing staff. Teaching the proper method for a 24-hour urine collection is a multi-step process; this task should not be delegated.

Question 11 of 41 A 56-year-old client admitted with a diagnosis of acute myelogenous leukemia (AML) is prescribed IV cytosine arabinoside for 7 days and an infusion of daunorubicin for the first 3 days. An infection develops. What knowledge does the nurse use to determine that the appropriate antibiotic has been prescribed for this client? a. Evaluating the client's liver function tests (LFTs) and serum creatinine levels b. Evaluating the client's white blood cell (WBC) count level c. Checking the culture and sensitivity test results to be certain that the requested antibiotic is effective against the organism causing the infection d. Recognizing that vancomycin (Vancocin) is the drug of choice used to treat all infections in clients with AML

c Checking the culture and sensitivity test results to be certain that the requested antibiotic is effective against the organism causing the infection Checking the culture and sensitivity test results to be certain that the requested antibiotic is effective against the organism causing the infection is the best action to take. Drug therapy is the main defense against infections that develop in clients undergoing therapy for AML. Agents used depend on the client's sensitivity to various antibiotics for the organism causing the infection. Although LFTs and kidney function tests may be influenced by antibiotics, these tests do not determine the effectiveness of the antibiotic. Although the WBC count is elevated in infection, this test does not influence which antibiotic will be effective in fighting the infection. Vancomycin may not be effective in all infections; culturing of the infection site and determining the organism's sensitivity to a cohort of drugs are needed, which will provide data on drugs that are capable of eradicating the infection in this client.

Ch.62 Question 12 of 23 These data are obtained by the RN who is assessing a client who had a transsphenoidal hypophysectomy yesterday. What information has the most immediate implications for the client's care? a. Dry lips and oral mucosa on examination b. Nasal drainage that tests negative for glucose c. Client report of a headache and stiff neck d. Urine specific gravity of 1.016

c Client report of a headache and stiff neck Headache and stiff neck (nuchal rigidity) are symptoms of meningitis that have immediate implications for the client's care. Dry lips and mouth are not unusual after surgery. Frequent oral rinses and the use of dental floss should be encouraged because the client cannot brush the teeth. Any nasal drainage should test negative for glucose; nasal drainage that tests positive for glucose indicates the presence of a cerebrospinal fluid leak. A urine specific gravity of 1.016 is within normal limits.

31. A patient with chronic kidney disease (CKD) develops anorexia, nausea and vomiting, muscle cramping, and pruritus. How does the nurs einterperte these findings? a. Oliguria b. Azotemia c. Anuria d. Uremia

d

Ch.63 Question 12 of 21 An RN and LPN/LVN are caring for a group of clients on the medical-surgical unit. Which client will be the best to assign to the LPN/LVN? a. Client with Graves' disease who needs discharge teaching after a total thyroidectomy b. Client with hyperparathyroidism who is just being admitted for a parathyroidectomy c. Client with infiltrative ophthalmopathy who needs administration of high-dose prednisone (Deltasone) d. Newly diagnosed client with hypothyroidism who needs education about the use of thyroid supplements

c Client with infiltrative ophthalmopathy who needs administration of high-dose prednisone (Deltasone) Medication administration for the client with infiltrative ophthalmopathy is within the scope of practice of the LPN/LVN. Discharge teaching is a complex task that cannot be delegated to the LPN/LVN. A client being admitted for a parathyroidectomy needs preoperative teaching, which must be provided by the RN. A client who has a new diagnosis will have questions about the disease and prescribed medications; teaching is a complex task that is appropriate for the RN.

Ch.62 Question 2 of 23 Which client does the nurse identify as being at highest risk for acute adrenal insufficiency resulting from corticosteroid use? a. Client with hematemesis, upper epigastric pain for the past 3 days not relieved with food, and melena b. Client with right upper quadrant pain unrelieved for the past 2 days, dark-brown urine, and clay-colored stools c. Client with shortness of breath and chest tightness, nasal flaring, audible wheezing, and oxygen saturation of 85% for the second time this week d. Client with three emergency department visits in the past month for edema, shortness of breath, weight gain, and jugular venous distention

c Client with shortness of breath and chest tightness, nasal flaring, audible wheezing, and oxygen saturation of 85% for the second time this week Corticosteroids may be used to treat signs and symptoms of asthma, such as shortness of breath and chest tightness, nasal flaring, audible wheezing, and oxygen saturation of 85%. This places the client at risk for adrenal insufficiency. Corticosteroids are not used to treat signs and symptoms of GI bleeding or peptic ulcer disease (hematemesis, upper epigastric pain for the past 3 days not relieved with food, and melena), gallbladder disease (right upper quadrant pain unrelieved for the past 2 days, dark brown urine, and clay-colored stools), or congestive heart failure (edema, shortness of breath, weight gain, and jugular venous distention).

Ch.61 Question 12 of 16 A client has suspected alterations in antidiuretic hormone (ADH) function. Which diagnostic test does the nurse anticipate will be requested for this client? a. Adrenocorticotropic hormone (ACTH) suppression test b. Chest x-ray c. Cranial computed tomography (CT) d. Renal sonography

c Cranial computed tomography (CT) ADH is a hormone of the posterior pituitary. Brain abscess, tumor, or subarachnoid hemorrhage could cause alterations in ADH levels. These can be seen on a CT scan of the brain. ACTH triggers the release of cortisol from the adrenal cortex and is not related to ADH. A chest x-ray would not show a pituitary tumor or brain abscess. Even though ADH acts on distal convoluted tubules in the kidneys, a renal sonogram would diagnose the cause of syndrome of inappropriate antidiuretic hormone.

Question 12 of 16 A client has suspected alterations in antidiuretic hormone (ADH) function. Which diagnostic test does the nurse anticipate will be requested for this client? a. Adrenocorticotropic hormone (ACTH) suppression test b. Chest x-ray c. Cranial computed tomography (CT) d. Renal sonography

c Cranial computed tomography (CT) ADH is a hormone of the posterior pituitary. Brain abscess, tumor, or subarachnoid hemorrhage could cause alterations in ADH levels. These can be seen on a CT scan of the brain. ACTH triggers the release of cortisol from the adrenal cortex and is not related to ADH. A chest x-ray would not show a pituitary tumor or brain abscess. Even though ADH acts on distal convoluted tubules in the kidneys, a renal sonogram would diagnose the cause of syndrome of inappropriate antidiuretic hormone.

Ch. 61 Question 2 of 16 Which negative feedback response is responsible for preventing hypoglycemia during sleep in nondiabetic clients? a. Alpha cells of the pancreas b. Beta cells of the pancreas c. Glucagon release d. Insulin release

c Glucagon release Glucagon is the hormone that binds to receptors on liver cells. This causes the liver cells to convert glycogen to glucose, which keeps blood sugar levels normal during sleep. Alpha cells are responsible for synthesizing and secreting the hormone glucagon. Beta cells are responsible for synthesizing and secreting the hormone insulin. Insulin is the hormone responsible for lowering blood glucose. Insulin improves glucose uptake by the cell.

Question 2 of 16 Which negative feedback response is responsible for preventing hypoglycemia during sleep in nondiabetic clients? a. Alpha cells of the pancreas b. Beta cells of the pancreas c. Glucagon release d. Insulin release

c Glucagon release Glucagon is the hormone that binds to receptors on liver cells. This causes the liver cells to convert glycogen to glucose, which keeps blood sugar levels normal during sleep. Alpha cells are responsible for synthesizing and secreting the hormone glucagon. Beta cells are responsible for synthesizing and secreting the hormone insulin. Insulin is the hormone responsible for lowering blood glucose. Insulin improves glucose uptake by the cell.

2. What is the name of the substance secreted by the endocrine glands? a. Vasoactive amines b. Chemotaxins c. Hormones d. Cytotoxins

c Hormones

10. What are tropic hormones? a. Hormones that rigger female and male sex characteristics. b. Hormones that have a direct effect on final target tissues. c. Hormones produced by the anterior pituitary gland that stimulate other endocrine glands. d. Hormones that are synthesized in the hypothalamus and stored in the posterior pituitary gland

c Hormones produced by the anterior pituitary gland that stimulate other endocrine glands.

Ch.62 Question 16 of 23 Which laboratory result indicates that fluid restrictions have been effective in treating syndrome of inappropriate antidiuretic hormone (SIADH)? a. Decreased hematocrit b. Decreased serum osmolality c. Increased serum sodium d. Increased urine specific gravity

c Increased serum sodium Increased serum sodium due to fluid restriction indicates effective therapy. Hemoconcentration is a result of hypovolemic hyponatremia caused by SIADH and diabetes insipidus. Plasma osmolality is decreased as a result of SIADH. Urine specific gravity is decreased with diabetes insipidus and is increased with SIADH.

31. Which urine characteristic suggests that the patient is drinking a sufficient amount of fluids? a. Urine pH is between 6 to 6.5. b. Urine has a high specific gravity. c. Urine has a faint ammonia odor. d. Urine is a pale yellow color.

d

Ch.62 Question 7 of 21 A client with hypothyroidism is being discharged. Which environmental change may the client experience in the home? a. Frequent home care b. Handrails in the bath c. Increased thermostat setting d. Strict infection-control measures

c Increased thermostat setting Manifestations of hypothyroidism include cold intolerance. Increased thermostat settings or additional clothing may be necessary. A client with a diagnosis of hypothyroidism can be safely managed at home with adequate discharge teaching regarding medications and instructions on when to notify the health care provider or home health nurse. In general, hypothyroidism does not cause mobility issues. Activity intolerance and fatigue may be an issue, however. A client with hypothyroidism is not immune-compromised or contagious, so no environmental changes need to be made to the home.

Question 3 of 41 The nurse is infusing platelets to a client who is scheduled for a hematopoietic stem cell transplant. What procedure does the nurse follow? a. Administer intravenous corticosteroids before starting the transfusion. b. Allow the platelets to stabilize at the client's bedside for 30 minutes. c. Infuse the transfusion over a 15- to 30-minute period. d. Set up the infusion with the standard transfusion Y tubing.

c Infuse the transfusion over a 15- to 30-minute period The volume of platelets—200 or 300 mL (standard amount)—needs to be infused rapidly over a 15- to 30-minute period. Administering steroids is not standard practice in administering platelets. Platelets must be administered immediately after they are received; they are considered to be quite fragile. A special transfusion set with a smaller filter and shorter tubing is used to get the platelets into the client quickly and efficiently.

Ch.61 Question 3 of 23 A client diagnosed with hyperpituitarism resulting from a prolactin-secreting tumor has been prescribed bromocriptine mesylate (Parlodel). As a dopamine agonist, what effect does this drug have by stimulating dopamine receptors in the brain? a. Decreases the risk for cerebrovascular disease b. Increases the risk for depression c. Inhibits the release of some pituitary hormones d. Stimulates the release of some pituitary hormones

c Inhibits the release of some pituitary hormones Bromocriptine mesylate inhibits the release of both prolactin and growth hormone. It does not decrease the risk for cerebrovascular disease leading to stroke. Increased risk for depression is not associated with the use of bromocriptine mesylate; however, hallucinations have been reported as a side effect. Bromocriptine mesylate does not stimulate the release of any hormones.

Ch.63 Question 2 of 21 An older client with an elevated serum calcium level is receiving IV furosemide (Lasix) and an infusion of normal saline at 150 mL/hr. Which nursing action can the RN delegate to unlicensed assistive personnel (UAP)? a. Ask the client about any numbness or tingling. b. Check for bone deformities in the client's back. c. Measure the client's intake and output hourly. d. Monitor the client for shortness of breath.

c Measure the client's intake and output hourly. Measuring intake and output is a commonly delegated nursing action that is within the UAP scope of practice. Numbness and tingling is part of the client assessment that needs to be completed by a licensed nurse. Bony deformities can be due to pathologic fractures; physical assessment is a complex task that cannot be delegated. An older client receiving an IV at 150 mL/hr is at risk for congestive heart failure; careful monitoring for shortness of breath is the responsibility of the RN.

Ch.63 Question 19 of 21 Which type of thyroid cancer often occurs as part of multiple endocrine neoplasia (MEN) type II? a. Anaplastic b. Follicular c. Medullary d. Papillary

c Medullary Medullary carcinoma commonly occurs as part of MEN type II, which is a familial endocrine disorder. Anaplastic carcinoma is an aggressive tumor that invades surrounding tissue. Follicular carcinoma occurs more frequently in older clients and may metastasize to bone and lung. Papillary carcinoma is the most common type of thyroid cancer. It is slow growing and, if the tumor is confined to the thyroid gland, the outlook for a cure is good with surgical management.

Ch.63 Question 9 of 21 Which action does the postanesthesia care unit (PACU) nurse perform first when caring for a client who has just arrived after a total thyroidectomy? a. Assess the wound dressing for bleeding. b. Give morphine sulfate 4 to 8 mg IV for pain. c. Monitor oxygen saturation using pulse oximetry. d. Support the head and neck with sandbags.

c Monitor oxygen saturation using pulse oximetry. Airway assessment and management is always the first priority with every client. This is especially important for a client who has had surgery that involves potential bleeding and edema near the trachea. Assessing the wound dressing for bleeding is a high priority, although this is not the first priority. Pain control and supporting the head and neck with sandbags are important priorities, but can be addressed after airway assessment.

Question 17 of 41 The nurse is caring for a client with neutropenia who has a suspected infection. Which intervention does the nurse implement first? a. Hydrate the client with 1000 mL of IV normal saline. b. Initiate the administration of prescribed antibiotics. c. Obtain requested cultures. d. Place the client on Bleeding Precautions.

c Obtain requested cultures. Obtaining cultures to identify the infectious agent correctly is the priority for this client. Hydrating the client is not the priority. Administering antibiotics is important, but antibiotics should always be started after cultures are obtained. Placing the client on Bleeding Precautions is unnecessary.

4. The assessment findings of a male patient with anterior pituitary tumor include reports of changes in secondary sex characteristics, such as episodes of impotence and decreased libido. The nurse explains to the patient that these findings are a result of overproduction of which hormone? a. Gonadotropins inhibiting prolactin (PRL) b. Thyroid hormone inhibiting PRL c. PRL inhibiting secretion of gonadotropins d. Steroids inhibiting production of sex hormones

c PRL inhibiting secretion of gonadotropins

40. Which instructions are included when teaching a patient about urine collection for endocrine studies? (Select all that apply.) a. Fast before starting the urine collection. b. Measure the urine in mL rather than ounces. c. Empty the bladder completely, and then start timing. d. Time the test for exactly the instructed number of hours. e. Avoid taking any unnecessary drugs during endocrine testing. f. Empty the bladder at the end of the time period and keep that specimen.

c, d, e, f Empty the bladder completely, and then start timing. Time the test for exactly the instructed number of hours. Avoid taking any unnecessary drugs during endocrine testing. Empty the bladder at the end of the time period and keep that specimen.

Question 2 of 41 A 32-year-old client recovering from a sickle cell crisis is to be discharged. The nurse says, "You and all clients with sickle cell disease are at risk for infection because of your decreased spleen function. For this reason, you will most likely be prescribed an antibiotic before discharge." Which drug does the nurse anticipate the health care provider will request? a. Cefaclor (Ceclor) b. Gentamicin (Garamycin) c. Penicillin V (Pen-V K) d. Vancomycin (Vancocin)

c Penicillin V (Pen-V K) Prophylactic therapy with twice-daily oral penicillin reduces the incidence of pneumonia and other streptococcal infections and is the correct drug to use. It is a standard protocol for long-term prophylactic use in clients with sickle cell disease. Cefaclor (Ceclor) and vancomycin (Vancocin) are antibiotics more specific for short-term use and would be inappropriate for this client. Gentamicin (Garamycin) is a drug that can cause liver and kidney damage with long-term use.

Ch.64 Question 13 of 25 An intensive care client with diabetic ketoacidosis (DKA) is receiving an insulin infusion. The cardiac monitor shows ventricular ectopy. Which assessment does the nurse make? a. Urine output b. 12-lead electrocardiogram (ECG) c. Potassium level d. Rate of IV fluids

c Potassium level With insulin therapy, serum potassium levels fall rapidly as potassium shifts into the cells. Detecting and treating the underlying cause is essential. Insulin treats symptoms of diabetes by putting glucose into the cell as well as potassium; ectopy, indicative of cardiac irritability, is not associated with changes in urine output. A 12-lead ECG can verify the ectopy, but the priority is to detect and fix the underlying cause. Increased fluids treat the symptoms of dehydration secondary to DKA, but do not treat the cause.

Ch.64 Question 15 of 25 Which action is correct when drawing up a single dose of insulin? a. Wash hands thoroughly and don sterile gloves. b. Shake the bottle of insulin vigorously to mix the insulin. c. Pull back plunger to draw air into the syringe equal to the insulin dose. d. Recap the needle and save the syringe for the next dose of insulin.

c Pull back plunger to draw air into the syringe equal to the insulin dose. The plunger is pulled back to draw an amount of air into the syringe that is equal to the insulin dose. The air is then injected into the insulin bottle before withdrawing the insulin dose. Although handwashing is important before any medication administration, sterile gloves are not required. The bottle of insulin should be rolled gently in the palms of the hands to mix the insulin, not shaken. Insulin syringes are never recapped or reused; the syringe and needle should be disposed of (without recapping) in a puncture-proof container.

Question 16 of 41 The nurse is to administer packed red blood cells to a client. How does the nurse ensure proper client identification? a. Asks the client's name b. Checks the client's armband c. Reviews all information with another registered nurse d. Verifies the client's room number

c Reviews all information with another registered nurse With another registered nurse, verify the client by name and number, check blood compatibility, and note expiration time. Human error is the most common cause of ABO incompatibility reactions, even for experienced nurses. Asking the client's name and checking the client's armband are not adequate for identifying the client before transfusion therapy. Using the room number to verify client identification is never appropriate.

13. Which statement about hormone replacement therapy for hypopituitarism is correct? a. Once manifestations of hypofunction are corrected, treatment is no longer needed. b. The most effective route of androgen replacement is the oral route. c. Testosterone replacement is contraindicated in men with prostate cancer. d. Clomiphene citrate (Clomid) is used to to suppress ovulation in women.

c Testosterone replacement is contraindicated in men with prostate cancer.

Question 23 of 25 Which explanation best assists a client in differentiating type 1 diabetes from type 2 diabetes? a. Most clients with type 1 diabetes are born with it. b. People with type 1 diabetes are often obese. c. Those with type 2 diabetes make insulin, but in inadequate amounts. d. People with type 2 diabetes do not develop typical diabetic complications.

c Those with type 2 diabetes make insulin, but in inadequate amounts. People with type 2 diabetes make some insulin but in inadequate amounts, or they have resistance to existing insulin. Although type 1 diabetes may occur early in life, it may be caused by immune responses. Obesity is typically associated with type 2 diabetes. People with type 2 diabetes are at risk for complications, especially cardiovascular complications.

Ch.63 Question 1 of 21 A client has hyperparathyroidism. Which incident witnessed by the nurse requires the nurse's intervention? a. The client eating a morning meal of cereal and fruit b. The physical therapist walking with the client in the hallway c. Unlicensed assistive personnel pulling the client up in bed by the shoulders d. Visitors talking with the client about going home

c Unlicensed assistive personnel pulling the client up in bed by the shoulders The client with hyperparathyroidism is at risk for pathologic fracture. All members of the health care team must move the client carefully. A lift sheet should be used to re-position the client. The client with hyperparathyroidism is not restricted from eating and should maintain a balanced diet. The client can benefit from moderate exercise and physical therapy, and is not restricted from having visitors.

36. The nurse assesses a patient postthyroidectomy for layrngeal nerve damage. Which findings indicate this complication? (Select all that apply.) a. Dyspnea b. Sore throat c. Hoarseness d. Weak voice e. Dry cough

c, d

63. Which ethnic groups are mostly likely to develop end-stage kidney disease related to hypertension? (Select all that apply.) a. Caucasian Americans b. Asian Americans c. American Indicans d. African Americans e. Hispanic Americans

c, d

The nurse assesses a patient postthyroidectomy for laryngeal nerve damage. Which findings indicate this complication? (Select all that apply.) a. Dyspnea b. Sore throat c. Hoarseness d. Weak voice e. Dry cough

c, d

23. Several patients at the clinic have just been diagnosed with UTIs. Which patients may need longer antibiotic treatment (7 to 21 days) or different agents than the typical first-line medications? (Select all that apply.) a. Postmenopausal patient b. Patient with urethritis c. Diabetic patient d. Immunosuppressed patient e. Pregnant patient

c, d, e

2. A malfunctioning posterior pituitary gland can result in which disorders? (Select all that apply.) a. Hypothyroidism b. Altered sexual function c. Diabetes insipidus (DI) d. Growth retardation e. Syndrome of inappropriate antidiuretic hormone (SIADH)

c, e Diabetes insipidus Syndrome of inappropriate antidiuretic hormone (SIADH)

2. A nurse cares for a client with elevated triiodothyronine and thyroxine, and normal thyroid-stimulating hormone levels. Which actions should the nurse take? (Select all that apply.) a. Administer levothyroxine (Synthroid). b. Administer propranolol (Inderal). c. Monitor the apical pulse. d. Assess for Trousseau's sign. e. Initiate telemetry monitoring.

c, e. The client's laboratory findings suggest that the client is experiencing hyperthyroidism. The increased metabolic rate can cause an increase in the client's heart rate, and the client should be monitored for the development of dysrhythmias. Placing the client on a telemetry monitor might also be a precaution. Levothyroxine is given for hypothyroidism. Propranolol is a beta blocker often used to lower sympathetic nervous system activity in hyperthyroidism. Trousseau's sign is a test for hypocalcemia.

A nurse is caring for four clients. After reviewing today's laboratory results, which client should the nurse see first? a. Client with an international normalized ratio of 2.8 b. Client with a platelet count of 128,000/mm3 c. Client with a prothrombin time (PT) of 28 seconds d. Client with a red blood cell count of 5.1 million/μL

c. Client with a prothrombin time (PT) of 28 seconds ANS: C A normal PT is 11 to 12.5 seconds. This client is at high risk of bleeding. The other values are within normal limits.

A client has thrombocytopenia. What client statement indicates the client understands self-management of this condition? a. "I brush and use dental floss every day." b. "I chew hard candy for my dry mouth." c. "I usually put ice on bumps or bruises." d. "Nonslip socks are best when I walk."

c. "I usually put ice on bumps or bruises."

A client undergoing coronary artery bypass grafting asks why the surgeon has chosen to use the internal mammary artery for the surgery. Which response by the nurse is correct? a. "This way you will not need to have a leg incision." b. "The surgeon prefers this approach because it is easier." c. "These arteries remain open longer." d. "The surgeon has chosen this approach because of your age."

c. "These arteries remain open longer." Mammary arteries remain patent much longer than other grafts. Although no leg incision will be made with this approach, veins from the legs do not remain patent as long as the mammary artery graft does. Long-term patency, not ease of the procedure, is the primary concern. Age is not a determining factor in selection of these grafts.

An LPN/LVN is scheduled to work on the inpatient "stepdown" cardiac unit. Which client does the charge nurse assign to the LPN/LVN? a. A 60-year-old who was admitted today for pacemaker insertion because of third-degree heart block and who is now reporting chest pain b. A 62-year-old who underwent open heart surgery 4 days ago for mitral valve replacement and who has a temperature of 38.2° C c. A 66-year-old who has a prescription for a nitroglycerin (Nitro-Dur) patch and is scheduled for discharge to a group home later today d. A 69-year-old who had a stent placed 2 hours ago in the left anterior descending artery and who has bursts of ventricular tachycardia

c. A 66-year-old who has a prescription for a nitroglycerin (Nitro-Dur) patch and is scheduled for discharge to a group home later today The LPN/LVN scope of practice includes administration of medications to stable clients. Third-degree heart block is characterized by a very low heart rate and usually by required pacemaker insertion; the skills of the RN are needed to care for this client. Fever after surgery requires collaboration with the health care provider, which is more consistent with the role of the RN. The client with a recent stent placement and having bursts of ventricular tachycardia is unstable and is showing ventricular irritability; he will need medications and monitoring beyond the scope of practice of the LPN/LVN.

15. A nurse plans care for a client who has hypothyroidism and is admitted for pneumonia. Which priority intervention should the nurse include in this client's plan of care? a. Monitor the client's intravenous site every shift. b. Administer acetaminophen (Tylenol) for fever. c. Ensure that working suction equipment is in the room. d. Assess the client's vital signs every 4 hours.

c. A client with hypothyroidism who develops another illness is at risk for myxedema coma. In this emergency situation, maintaining an airway is a priority. The nurse should ensure that suction equipment is available in the client's room because it may be needed if myxedema coma develops. The other interventions are necessary for any client with pneumonia, but having suction available is a safety feature for this client.

An older adult client, 4 hours after coronary artery bypass graft (CABG), has a blood pressure of 80/50 mm Hg. What action does the nurse take? a. No action is required; low blood pressure is normal for older adults. b. No action is required for postsurgical CABG clients. c. Assess pulmonary artery wedge pressure (PAWP). d. Give ordered loop diuretics.

c. Assess pulmonary artery wedge pressure (PAWP). Decreased preload as exhibited by decreased PAWP could indicate hypovolemia secondary to hemorrhage or vasodilation; hypotension could cause the graft to collapse. Low blood pressure is not normal in older adults or postoperative clients. The cause of hypotension must be found and treated; further action is needed to determine additional interventions. Hypotension could be caused by hypovolemia; giving loop diuretics increases hypovolemia.

The nurse is teaching a group of teens about prevention of heart disease. Which point should the nurse emphasize? a. Reduce abdominal fat. b. Avoid stress. c. Do not smoke or chew tobacco. d. Avoid alcoholic beverages.

c. Do not smoke or chew tobacco. Tobacco exposure, including secondhand smoke, reduces coronary blood flow; causes vasoconstriction, endothelial dysfunction, and thickening of the vessel walls; increases carbon monoxide; and decreases oxygen. Because it is highly addicting, beginning smoking in the teen years may lead to decades of exposure. Teens are not likely to experience metabolic syndrome from obesity, but are very likely to use tobacco. Avoiding stress is a less modifiable risk factor, which is less likely to cause heart disease in teens. The risk of smoking outweighs the risk of alcohol use. Tobacco exposure, including secondhand smoke, reduces coronary blood flow; causes vasoconstriction, endothelial dysfunction, and thickening of the vessel walls; increases carbon monoxide; and decreases oxygen. Because it is highly addicting, beginning smoking in the teen years may lead to decades of exposure. Teens are not likely to experience metabolic syndrome from obesity, but are very likely to use tobacco. Avoiding stress is a less modifiable risk factor, which is less likely to cause heart disease in teens. The risk of smoking outweighs the risk of alcohol use.

8. A nurse cares for a client who has hypothyroidism as a result of Hashimoto's thyroiditis. The client asks, "How long will I need to take this thyroid medication?" How should the nurse respond? a. "You will need to take the thyroid medication until the goiter is completely gone." b. "Thyroiditis is cured with antibiotics. Then you won't need thyroid medication." c. "You'll need thyroid pills for life because your thyroid won't start working again." d. "When blood tests indicate normal thyroid function, you can stop the medication."

c. Hashimoto's thyroiditis results in a permanent loss of thyroid function. The client will need lifelong thyroid replacement therapy. The client will not be able to stop taking the medication.

A client has a platelet count of 25,000/mm3. What actions does the nurse delegate to the unlicensed assistive personnel (UAP)? (Select all that apply.) a. Assist with oral hygiene using a firm toothbrush. b. Give the client an enema if he or she is constipated. c. Help the client choose soft foods from the menu. d. Shave the male client with an electric razor. e. Use a lift sheet when needed to re-position the client.

c. Help the client choose soft foods from the menu. d. Shave the male client with an electric razor. e. Use a lift sheet when needed to re-position the client.

A nurse is caring for a client who is about to receive a bone marrow transplant. To best help the client cope with the long recovery period, what action by the nurse is best? a. Arrange a visitation schedule among friends and family. b. Explain that this process is difficult but must be endured. c. Help the client find things to hope for each day of recovery. d. Provide plenty of diversionary activities for this time.

c. Help the client find things to hope for each day of recovery.

6. A nurse plans care for a client with hypothyroidism. Which priority problem should the nurse plan to address first for this client? a. Heat intolerance b. Body image problems c. Depression and withdrawal d. Obesity and water retention

c. Hypothyroidism causes many problems in psychosocial functioning. Depression is the most common reason for seeking medical attention. Memory and attention span may be impaired. The client's family may have great difficulty accepting and dealing with these changes. The client is often unmotivated to participate in self-care. Lapses in memory and attention require the nurse to ensure that the client's environment is safe. Heat intolerance is seen in hyperthyroidism. Body image problems and weight issues do not take priority over mental status and safety.

A client has a sickle cell crisis with extreme lower extremity pain. What comfort measure does the nurse delegate to the unlicensed assistive personnel (UAP)? a. Apply ice packs to the client's legs. b. Elevate the client's legs on pillows. c. Keep the lower extremities warm. d. Place elastic bandage wraps on the client's legs.

c. Keep the lower extremities warm.

The visiting nurse is seeing a client postoperative for coronary artery bypass graft. Which nursing action should be performed first? a. Assess coping skills. b. Assess for postoperative pain at the client's incision site. c. Monitor for dysrhythmias. d. Monitor mental status.

c. Monitor for dysrhythmias. Dysrhythmias are the leading cause of prehospital death; the nurse should monitor the client's heart rhythm. Assessing mental status, coping skills, or postoperative pain is not the priority for this client.

3. A nurse assesses a client who is recovering from a subtotal thyroidectomy. On the second postoperative day the client states, "I feel numbness and tingling around my mouth." What action should the nurse take? a. Offer mouth care. b. Loosen the dressing. c. Assess for Chvostek's sign. d. Ask the client orientation questions.

c. Numbness and tingling around the mouth or in the fingers and toes are manifestations of hypocalcemia, which could progress to cause tetany and seizure activity. The nurse should assess the client further by testing for Chvostek's sign and Trousseau's sign. Then the nurse should notify the provider. Mouth care, loosening the dressing, and orientation questions do not provide important information to prevent complications of low calcium levels.

The nurse is caring for a client in phase 1 cardiac rehabilitation. Which activity does the nurse suggest? a. The need to increase activities slowly at home b. Planning and participating in a walking program c. Placing a chair in the shower for independent hygiene d. Consultation with social worker for disability planning

c. Placing a chair in the shower for independent hygiene Phase 1 begins with the acute illness and ends with discharge from the hospital; it focuses on promoting rest and allowing clients to improve their activities of daily living based on their abilities. Phase 2 begins after discharge and continues through convalescence at home, including consultation with a social worker for long-term planning; it consists of achieving and maintaining a vital and productive life while remaining within the limits of the heart's ability to respond to increases in activity and stress. Phase 3 refers to long-term conditioning, such as a walking program.

A client has received a bone marrow transplant and is waiting for engraftment. What actions by the nurse are most appropriate? (Select all that apply.) a. Not allowing any visitors until engraftment b. Limiting the protein in the client's diet c. Placing the client in protective precautions d. Teaching visitors appropriate hand hygiene e. Telling visitors not to bring live flowers or plants

c. Placing the client in protective precautions d. Teaching visitors appropriate hand hygiene e. Telling visitors not to bring live flowers or plants

A nurse is caring for a young male client with lymphoma who is to begin treatment. What teaching topic is a priority? a. Genetic testing b. Infection prevention c. Sperm banking d. Treatment options

c. Sperm banking

A client has been treated for a deep vein thrombus and today presents to the clinic with petechiae. Laboratory results show a platelet count of 42,000/mm3. The nurse reviews the client's medication list to determine if the client is taking which drug? a. Enoxaparin (Lovenox) b. Salicylates (aspirin) c. Unfractionated heparin d. Warfarin (Coumadin)

c. Unfractionated heparin

42. The nurse and nutritionist are evaluating the diet and nutritional therapies for a patient with kidney problems. Blood urea nitrogen (BUN) levels for this patient are tracked because of the direct relationship to the intake and metabolism of which substance? a. Lipids b. Carbohydrates c. Protein d. Fluids

c?

What is included in postprocedural care of a patient after a cardiac catheterization? (SATA) a. Patient remains on bedrest for 12 to 24 hours b. Patient is placed in a high-Fowlers position c. Dressing is assessed for bloody drainage or hematoma d. Peripheral pulses in the affected extremity, as well as skin temperature and color, are monitored with every vital sign check e. Adequate oral and IV fluids are provided for hydration f. Vital signs are monitored every hour for 24 hours

cde

1. The nurse is performing a physical examination of a patient's thyroid gland. Precautions are taken in performing the correct technique because palpitation can result in which occurrence? a. Damage to the esophagus causing gastric reflux b. Obstruction of the carotid arteries causing a stroke c. Pressure on the trachea and laryngeal nerve causing hoarseness d. Exacerbation of symptoms by releasing additional thyroid hormone

d

11. The nurse assessing a patient palpates enlargement of the thyroid gland, along with noticeable swelling of the neck. How does the nurse interpret this finding? a. Globe lag b. Myxedema c. Exophthalmos d. Goiter

d

12. The nurse is teaching a man about how to prevent UTIs. What information does the nurse include? a. "Have a minimal fluid intake of 5 L daily, unless contraindicated. b. "Empty your bladder before and after sexual intercourse. c. "Make sure that spermicides are used with condoms." d. "Gently wash the genital area before intercourse."

d

12. Which food should a patient with a low white blood cell count be encouraged to eat? a. Fresh strawberries b. Raw carrots c. Green leaf lettuce d. Well-done poultry

d

14. A female patient has been prescribed hormone replacement therapy. What does the nurse instruct the patient to do regarding this therapy? a. Report any recurrence of symptoms, such as decreased libido, between injections. b. Monitor blood pressure at least weekly for potential hypotension. c. Treat leg pain, especially in the calves, with gentle muscle stretching. d. Take measures to reduce risk for hypertension and thrombosis.

d

14. Which pain management strategy does the nurse teach a patient who has pain from infected kidney cysts of PKD? a. Take nothing by mouth. b. Increase the dose of NSAIDs. c. Assume a high-Fowler's position. d. Apply dry heat to the abdomen or flank.

d

15. A patient has been started on oxybutynin (Ditropan) for urinary incontinence. What is the major action of this medication? a. Increases blood flow to the urethra b. Blocks acetylcholine receptors c. Causes slight numbing of the bladder d. Relaxes bladder muscles

d

17. A patient has been diagnosed with AKI, but the cause is uncertain. The nurse prepares patient educational material about which diagnostic test? a. Flat plate of the abdomen b. Renal ultrasonography c. Computed tomography d. Kidney biopsy

d

19. The health care provider tells the nurse that the patient with PKD has salt wasting. Which intervention is the nurse likely to use related to nutrition therapy? a. Talk to the patient about seasonings that are alternative for salt. b. Help the patient select a lunch tray with low-sodium items. c. Obtain an order for fluid restriction to prevent loss of sodium during urination. d. Advise that a low-sodium diet is not currently necessary.

d

2. Which event is most likely to trigger renin production? a. Patient participates in strenouous exercise. b. Patient becomes anxious and nervous. c. Patient has urge to urinate during the night. d. Patient sustains significant blood loss.

d

22. A patient has UTI symptoms but there are no bacteria int the urine. The health care provider suspects interstitial cystitis. The nurse prepares patient teaching material for which diagnostic test? a. Urography b. Abdominal sonography c. Computed tomography (CT) d. Cystoscopy

d

23. Laboratory findings of elevated T3 and T4, decreased TSH, and high thyrotropin receptor antibody titer indicate which condition? a. Multinodular goiter b. Hyperthyroidism related to overmedication c. Pituitary tumor suppressing TSH d. Graves' disease

d

23. What is the common problem of hydronephrosis, hydroureter, and urethral stricture in kidney;function? a. Dilute urine b. Tubular cell damage c. Dehydration d. Obstruction

d

25. A patient with AKI is ill and has a poor appetite. What would the health care team try first? a. IV normal saline to prevent dehydration b. Familiar foods brought by the family c. Nasogastric tube for enteral feedings d. Oral supplements designed for kidney patients

d

26. A patient appears very uncomfortable with the nurse's questions about urinary functions and patterns. What is the best technique for the nurse to use to elicit relevant information and decrease the patient's discomfort? a. Defer the questions until a later time. b. Direct the questions toward a family member. c. Use anatomic or medical terminology. d. Use the patient's own terminology.

d

28. A patient has AKI related to nephrotoxins. In order to maintain cell integrity, improve GFR, and improve blood flow to the kidneys, which type of medication does the nurse anticipate the health care provider will prescribe? a. Loop diuretics b. Alpha-adrenergic blockers c. Beta blockers d. Calcium channel blockers

d

29. The health care provider verbally informs the nurse that the patient needs a fluoroquinolone antibiotic to treat a UTI. The pharmacy delivers gabapentin (Neurontin). What hsould the nurse do first? a. Administer the medication as ordered. b. Call the pharmacist and ask for a read back of the order. c. Call the health care provider for clarification of the order. d. Look at the written order to clarify the name of the medication.

d

29. Which over-the-counter product used by a patient does the nurse further explore for potential impact on kidney function? a. Mouthwash with alcohol b. Fiber supplement c. Vitamin C d. Acetaminophen

d

35. The nurse is assisting an inexperienced health care provider to assess a patient who has an aneurysm. The nurse would intervene if the provider performed which action? a. Inspected the flank for bruising or redness b. Listened for a bruit over the renal artery c. Auscultated the abdomen for bowel sounds d. Palpated deeply to locate masses or tenderness

d

36. The nurse is taking a history on a patient with diabetes and hypertension. Because of the patient's high risk for developing kidney problems, which early sign of chronic kidney disease (CKD) does the nurse assess for? a. Decreased output with subjective thirst b. Urinary frequency of very small amounts c. Pink or blood-tinged urine d. Increased output of very dilute urine

d

37. Increased BUN and creatinine, hyperkalemia, and hypernatremia are all characteristics of which stage of kidney disease? a. Stage I CKD b. Mild CKD c. Moderate CKD d. ESKD

d

38. A patient reports flank pain and tenderness. What technique does the nurse use to assess for costovertebral angle (CVA) tenderness? a. Percuss the nontender flank and assess for rebound. b. Thump the CVA area with the flat surface of the hand. c. Thump the CVA area with a clenched fist. d. Place one palm over the CVA area, thump with other fist.

d

39. The nurse expects to perform which diagnostic test for pheochromocytoma? a. 24-hour urine collection for sodium, potassium, and glucose b. Catecholamine-stimulation test c. Administration of beta-adrenergic blocking agent and monitor results. d. 24- hour urine collection for fractionated metanephrine and catecholamine levels.

d

39. The nurse is performing an assessment on a patient with probable stress incontinence. Which assessment technique does the nurse use to validate stress incontinence? a. Assess the abdomen to estimate bladder fullness. b. Check for residual urine using a portable ultrasound. c. Catheterize the patient immediately after voiding. d. Ask the patient to cough while wearing a perineal pad.

d

40. A healthy female patient has no physical symptoms, but urinalysis results reveal a protein level of >0.8 mg/dL and a white blood cell count of 4 per high-powered filed. What question would the nurse ask the patient in order to assist the health care provider to correctly interpret the urinalysis results? a. "Have you ever been treated for a urinary tract infection?" b. "Do you have a family history of cardiac or biliary disease?" c. "Are you sexually active and if so, do you use condoms?" d. "Have you recently performed any strenuous exercise?"

d

42. Serum calcium levels are maintained by which hormone? a. Cortisol b. Luteinizing hormone c. Antidiuretic hormone (ADH) d. Parathyroid hormone (PTH)

d

42. Which diuretic is ordered by the health care provider to treat hyperaldosteronism? a. Furosemide (Lasix) b. Ethacrynic acid (Edecrin) c. Bumetanide (Bumex) d. Spironolactone (Aldactone)

d

43. Which statement about hyperaldosteronism is correct? a. Painful "charley horses" are common from hyperkalemia. b. It occurs more often in men than in women. c. It is a common cause of hypertension in the population. d. Hypokalemia and hypertension are the main issues.

d

44. All patient with hypertension or diabetes should have yearly screenings for which factor? a. Creatinine b. BUN c. Glycosuria d. Microalbuminuria

d

44. For a patient with acute glomerulonephritis, a 24-hour urine test was initiated and the glomerular filtration rate (GFR) results are pending. What are the clinical implications of the test results? a. GFR is normal; the therapy can be discontinued. b. GFR is high; the patient is at risk for dehydration. c. GFR is low; the patient is at risk for infection. d. GFR is low; the patient is at risk for fluid overload.

d

44. When diagnosed with Cushing's syndrome, the manifestations are most likely related to an excess production of which hormone? a. Insulin from the pancreas b. ADH from posterior pituitary gland c. PRL from anterior pituitary gland d. Cortisol from the adrenal cortex

d

45. A patient is very ill and is admitted to the intensive care unit with rapidly progressing glomerulonephritis. The nurse monitors the patient for manifestations of which organ system failure? a. Immune system b. Cardiovascular system c. Neurologic system d. Renal system

d

45. What is the most common cause of endogenous hypercortisolism, or Cushing's disease? a. Pituitary hypoplasia b. Insufficient ACTH production c. Adrenocortical hormone deficiency d. Hyperplasia of the adrenal cortex

d

49. The nurse is assessing the skin of a patient with ESKD. Which clinical manifestation is considered a sign of very late, premorbid, advanced uremic syndrome? a. Ecchymoses b. Sallowness c. Pallor d. Uremic frost

d

5. A patient has a family hisotryof the autosomal-dominant form of PKD and has therefore been advised to monitor for and report symptoms. What is an early symptom of PKD? a. Headache b. Pruritus c. Edema d. Notcuria

d

50. The nurse is teaching a patient a behavioral intervention for bladder compression. In order to correctly perform the Crede method, what does the nurse teach the patient to do? a. Insert the fingers into the vagina and gently push against the vaginal wall. b. Breathe in deeply and direct the pressure towards the bladder during exhalation. c. Empty the bladder, wait a few minutes, and attempt a second bladder emptying. d. Apply firm and steady pressure over the bladder area with the palm of the hand.

d

52. The nurse is taking a history on a patient with chronic glomerulonephritis. What is the patient most likely to report? a. History of antibiotic allergy b. Intense flank pain c. Poor appetite and weight loss d. Occasional edema and fatigue

d

52. The nurse is teaching a patient being discharged after bilateral adrenalcectomy. What medication information does the nurse emphasize in the teaching plan? a. The dosage of steroid replacement drugs will be consistent throughout the patient's lifetime. b. The steroid drugs should be taken in the evening so as not to interfere with sleep. c. The patient should take the drugs on an empty stomach. d. The patient should learn how to give himself an intramuscular injection of hydrocortisone.

d

53. The nurse is caring for a patient with functional incontinence. The UAP reports that "the linens have been changed four times within the past 6 hours, but the patient refuses to wear a diaper." What does the nurse do next? a. Thank the UAP for the hard work and advise to continue to change the linens. b. Call the health care provider to obtain an order for an indwelling catheter. c. Instruct the UAP to stop using the word "diaper" and instead use "incontinence pants." d. Assess the patient for any new urinary problems and ask about toileting preferences.

d

54. A patient with acute glomerulonephritis has edema of the face. The blood pressure is moderately elevated and the patient has gained 2 pounds within the past 24 hours. The patient reports fatigue and refuses to eat. What is the priority for nursing care? a. Cluster care to allow rest periods for the patient. b. Obtain a dietary consult to plan an adequate nutritional diet. c. Monitor urine output with accurate intake and output amounts. d. Assess for signs and symptoms of fluid volume overload.

d

55. A patient is considering vaginal cone therapy, but is a little hesitant because she does not understand how it works. What does the nurse tell her about how vaginal cone therapy improves incontinence? a. It mechanically obstructs urine loss from the urethra. b. It repositions the bladder to reduce compression. c. It increases the normal flora of the perineum. d. It strengthens pelvic floor muscles.

d

55. What is the most common cause of death from myxedema coma? a. Myocardial infarction b. Acute kidney failure c. High serum level of iodide d. Respiration failure

d

56. Which nursing intervention is applicable for a patient with acute glomerulonephritis? a. Restricting visitors who have infections b. Assessing the incision site c. Inspecting the vascular access d. Measuring weight daily

d

58. The nurse determines that the administration of hydrocortisone for the Addisonian crisis is effective when which assessment is made? a. Increased urine output b. No signs of pitting edema c. Weight gain d. Lethargy improving; patient alert and oriented

d

58. What role does drug therapy have as an intervention for reflex (overflow) urinary incontinence? a. Captopril (Capoten) is given to lower urine cystine levels. b. Levofloxacin (Levaquin) is given to prevent UTIs with this type of incontinence. c. Midorine (ProAmatine) is given to increase the contractile force of the bladder. d. Bethanechol chloride (Urecholine) may be used short-term after surgery.

d

59. Which nursing intervention is a preventive measure for adrenocortical insufficiency? a. Maintaining diuretic therapy b. Instructing the patient on salt restriction c. Reducing high-dose glucocorticoid therapy quickly d. Reducing high-dose glucocorticoid doses gradually

d

61. A Patient comes to the clinic and reports severe flank pain, bladder distention, and nausea and vomiting with increasingly smaller amounts of urine with frank blood. The patient states, "I have kidney stones and I just need a prescription for pain medication." What is the nurse's priority concern? a. Controlling the patient's pain b. Checking the quantity of blood in the urine c. Flushing the kidneys with oral fluids d. Determining if there is an obstruction

d

62. The nurse is caring for a patient with urolithiasis. Which medication is likely to be given in the acute phase to relieve the patient's severe pain? a. Ketorolac (Toradol) b. Oxybutynin chloride (Ditropan) c. Propantheline bromide (Pro-Bantine) d. Morphine sulfate (Astramorph)

d

65. A patient with a history of kidney stones presents with severe flank pain, nausea, vomiting, pallor, and diaphoresis. He reports freely passing urine, but it is bloody. The priority for nursing care is to monitor for which patient problem? a. Possible dehydration b. Impaired tissue perfusion c. Impaired urinary elimination d. Severe pain

d

68. A patient is scheduled for retrograde urethrography. Post-procedural care is similar to post-procedural care given for which test? a. Ultrasonography b. Computed tomography c. Renal angiogram d. Cystoscopy

d

7. In which patient circumstance would the nurse question the order for the insertion of an indwelling catheter? a. Patient is critically ill and at risk for hypovolemic shock. b. Patient has urinary retention with beginnings of hydronephrosis. c. Patient was in a card accident and has a possible spinal cord injury. d. Patient has functional incontinence related to Alzheimer's disease.

d

7. Which laboratory result is consistent with a diagnosis of hyperthyroidism? a. Decreased serum triiodothyronine (T3) and thyroxine (T4) levels b. Elevated serum thyrotropin-releasing hormone (TRH) level c. Decreased radioactive idoine uptake d. Increased serum T3 and T4

d

71. A patient has had surgery for bladder cancer. To prevent recurrence of superficial bladder cancer, the nurse anticipates that the health care provider is likely to recommend which treatment? a. No treatment is needed for this benign condition. b. Intravesical instillation of single-agent chemotherapy. c. Radiation therapy to the bladder, ureters, and urethra. d. Intravesical instillation of bacille Calmette Guerin.

d

71. The nursing student is explaining principles of hemodialysis to the nursing instructor. Which statement by the student indicates a need for additional study and research on the topic? a. "Dialysis works as molecules from an area of higher concentration move to an area of lower concentration." b. "Blood and dialyzing solution flow in opposite directions across an enclosed semipermeable membrane." c. "Excess water, waste products, and excess electrolytes are removed from the blood." d. "Bacteria and other organisms can also pass through the membrane, so the dialysate must be kept sterile."

d

73. A patient diagnosed with renal cell carcinoma that has metastasized to the lungs is considered to be in which stage of cancer? a. I b. II c. III d. IV

d

73. A patient has had a bladder suspension and a supra-pubic catheter is in place. The patient wants to know how long the catheter will remain in place. What is the nurse's best response? a. "Typically it remains for 24 hours postoperatively." b. "It will be removed at your first clinic visit." c. "When you can void on your own, it will be removed." d. "It will be removed when you can void and residual urine is less than 50 mL."

d

74. The nurse is caring for a patient with an arteriovenous fistula. What instructions are given to the UAP regarding the care of this patient? a. Palpate for thrills and auscultate for bruits every 4 hours. b. Check for bleeding at needle insertion site. c. Assess the patient's distal pulses and circulation. d. Do not take blood pressure readings in the arm with the fistula.

d

75. A patient is returning from the postanesthesia care unit after surgery for bladder cancer resulting in a cutaneous ureterostomy. Where does the nurse expect the stoma to be located? a. On the perineum b. At the belt-line c. On the posterior flank d. In the mid-abdominal area

d

77. The nurse is assessing a patient who has just returned from hemodialysis. Which assessment finding is cause for greatest concern? a. Feeling of malaise b. Headache c. Muscle cramps in the legs d. Bleeding at the access site

d

81. Which patient with kidney problems is the best candidate for peritoneal dialysis (PD)? a. Patient with peritoneal adhesion b. Patient with a history or extensive abdominal surgery c. Patient with peritoneal membrane fibrosis d. Patient with a history of difficulty with anticoagulants

d

82. The nurse is caring for a patient who had a nephrectomy yesterday. To manage the patient's pain, what is the best plan for analgesia therapy? a. Limit narcotics because of respiratory depression. b. Give an oral analgesic when the patient can eat. c. Alternate parenteral and oral medications. d. Give parenteral medications on a schedule.

d

9. Which hematologic disorder is most likely to occur if the hormonal function of the kidneys is not working properly? a. Leukemia b. Thrombocytopenia c. Neutropenia d. Anemia

d

93. The nurse is caring for the kidney transplant patient in the immediate postoperative period. During this initial period, the nurse will assess the urine output at least every hour for how many hours? a. First 8 hours b. First 12 hours c. First 24 hours d. First 48 hours

d

A nurse is monitoring the patient blood pressure and ECG during a stress test. Which parameter indicates the patient should stop exercising? a. Increase in heart rate b. Increase in blood pressure c. ECG showing the PQRS complex d. ECG showing St-segment depression

d

A patient has a suspected hematologic problem. Which instruction is the nurse most likely to give to the UAP? a. Record urine output for the shift. b. Take the vital signs every 2 hours. c. Assess the patient for fatigue after exertion. d. Handle the patient gently to avoid bruising.

d

A patient reports severe cramping in the legs while attempting to walk for exercise. The provider diagnoses the patient with intermittent claudication. What does the nurse advise the patient to do? a. Elevate the legs on a pillow b. Buy and wear supportive shoes c. Massage the legs before walking d. Rest the legs in a dependent position

d

After a thyroidectomy, a patient reports tingling around the mouth and muscle twitching. Which complication do these assessment findings indicate to the nurse? a. Hemorrhage b. Respiratory distress c. Thyroid storm d. Hypocalcemcia

d

An experienced nurse is supervising a new nurse who is assessing a patient with a suspected hematologic problem. The experienced nurse would intervene if the new nurse performed which action? a. Palpated the edge of the liver in the right upper quadrant b. Auscultated the heart for abnormal heart sounds or irregular rhythms c. Used the fingertips to firmly press over the ribs or sternum d. Palpated the left upper quadrant to locate an enlarged spleen

d

Laboratory findings of elevated T3 and T4, decreased TSH, and high thyrotropin receptor antibody titer indicate which condition? a. Multinodular goiter b. Hyperthyroidism related to overmedication c. Pituitary tumor suppressing TSH d. Graves' disease

d

Severe anemia could cause enlargement of which organ? a. Gallbladder b. Kidneys c. Colon d. Liver

d

The nurse assessing a patient palpates enlargement of the thyroid gland, along with noticeable swelling of the neck. How does the nurse interpret this finding? a. Globe lag b. Myxedema c. Exophthalmos d. Goiter

d

The nurse is assessing a patient with suspected CVD. When assessing the precordium, which assessment technique does the nurse begin? a. Percussion b. Palpation c. Auscultation d. Inspection

d

The nurse is performing a dietary assessment on a 45-year-old business executive at risk for CVD. Which assessment method used by the nurse is the most reliable and accurate? a. Ask the patient to identify foods he or she eats that contain sodium, sugar, cholesterol, fiber, and fat b. Ask the patient's spouse, who does the cooking and shopping, to identify the types of foods that are consumed c. Ask the patient how cultural beliefs and economic status influence the choice of food items d. Ask the patient to recall the intake of food, fluids, and alcohol during a typical 24-hour period

d

The nurse is performing a physical examination of a patient's thyroid gland. Precautions are taken in performing the correct technique because palpation can result in which occurrence? a. Damage to the esophagus causing gastric reflux b. Obstruction of the carotid arteries causing a stroke c. Pressure on the trachea and laryngeal nerve causing hoarseness d. Exacerbation of symptoms by releasing additional thyroid hormone

d

The nurse knows that erythropoietin is a growth factor that is required for stem cells specialization. Which sign/symptom would the nurse observe if erythropoietin is lacking or not performing its role? a. Elevated body temperature b. Bruising and ecchymosis c. Swelling of lymph nodes d. Easily fatigued

d

Venous stasis is considered an intrinsic factor that could result in activating which physiologic process? a. Increased RBC production b. Adjustment of osmotic fluid pressure c. Initiation of anticlotting forces d. Initiation of blood clotting cascade

d

When assessing the patient with darker skin for pallor and cyanosis, which area would the nurse examine? a. Chest and abdomen b. General appearance of face c. Fingertips and toes d. Oral mucous membranes

d

Which exercise regimen for an older adult meets the recommended guidelines of physical fitness to promote heart health? a. 6-hour bike ride every Saturday b. Golfing for 4 hours two times a week c. Running for 15 minutes three times a week d. Brisk walk 30 minutes every day

d

Which laboratory result is consistent with a diagnosis of hyperthyroidism? a. Decreased serum triiodothyronine (T3) and thyroxine (T4) levels b. Elevated serum thyrotropin-releasing hormone (TRH) level c. Decreased radioactive iodine uptake d. Increased serum T3 and T4

d

Which medications will the nurse be sure to hold until after a patient's cardiac catheterization? a. Daily vitamin and enteric coated aspirin b. Atenolol and IV antibiotic c. Potassium and folic acid d. Digoxin and furosemide

d

Which patient has an abnormal heart sound? a. S1 in a 45-year-old pateint b. S2 in a 30-year-old patient c. S3 in a 15-year-old patient d. S3 in a 54-year-old patient

d

Which statement about hematologic changes associated with aging is true? a. The older adult has increased blood volume. b. The older adult has increased levels of plasma proteins. c. Platelet counts decrease with age. d. Antibody levels and responses are lower and slower in older adults.

d

While listening to a patient's heart sounds, the nurse detects a murmur. What does the nurse understand about the cause of murmurs? a. A murmur is caused by the closing of the aortic and pulmonic valves b. A murmur is caused when blood flows from the atrium to a noncompliant ventricle c. A murmur is caused by anemia, hypertension, or ventricular hypertrophy d. A murmur is caused when there is turbulent blood flow through normal or abnormal valves

d

Ch.63 Question 14 of 21 A client with thyroid cancer has just received 131I ablative therapy. Which statement by the client indicates a need for further teaching? a. "I cannot share my toothpaste with anyone." b. "I must flush the toilet three times after I use it." c. "I need to wash my clothes separately from everyone else's clothes." d. "I'm ready to hold my newborn grandson now."

d "I'm ready to hold my newborn grandson now." Clients undergoing 131I therapy should avoid close contact with pregnant women, infants, and young children for 1 week after treatment. Clients should remain at least 1 meter (39 inches, or roughly 3 feet) away, and limit exposure to less than 1 hour per day. Some radioactivity will remain in the client's salivary glands for up to 1 week after treatment. Care should be taken to avoid exposing others to the saliva. Flushing the toilet three times after use will ensure that all urine has been diluted and removed. Clothing needs to be washed separately and the washing machine then needs to be run empty for a full cycle before it is used to wash the clothing of others.

Question 26 of 41 A client with thrombocytopenia is being discharged. What information does the nurse incorporate into the teaching plan for this client? a. "Avoid large crowds." b. "Drink at least 2 liters of fluid per day." c. "Elevate your lower extremities when sitting." d. "Use a soft-bristled toothbrush."

d "Use a soft-bristled toothbrush." Using a soft-bristled toothbrush reduces the risk for bleeding in the client with thrombocytopenia. Avoiding large crowds reduces the risk for infection, but is not specific to the client with thrombocytopenia. Increased fluid intake reduces the risk for dehydration, but is not specific to the client with thrombocytopenia. Elevating extremities reduces the risk for dependent edema, but is not specific to the client with thrombocytopenia.

Question 33 of 41 A client with multiple myeloma reports bone pain that is unrelieved by analgesics. How does the nurse respond to this client's problem? a. "Ask your doctor to prescribe more medication." b. "It is too soon for additional medication to be given." c. "I'll turn on some soothing classical music for you." d. "Would you like to try some relaxation techniques?"

d "Would you like to try some relaxation techniques?" Because most clients with multiple myeloma have local or generalized bone pain, analgesics and alternative approaches for pain management, such as relaxation techniques, are used for pain relief. This also offers the client a choice. Before prescribing additional medication, other avenues should be explored to relieve this client's pain. Even if it is too soon to give additional medication, telling that to the client is not helpful because it dismisses the client's pain concerns. Although music therapy can be helpful, this response does not give the client a choice.

Ch.63 Question 20 of 21 A client is taking methimazole (Tapazole) for hyperthyroidism and would like to know how soon this medication will begin working. What is the nurse's best response? a. "You should see effects of this medication immediately." b. "You should see effects of this medication within 1 week." c. "You should see full effects from this medication within 1 to 2 days." d. "You should see some effects of this medication within 2 weeks."

d "You should see some effects of this medication within 2 weeks." Methimazole is an iodine preparation that decreases blood flow through the thyroid gland. This action reduces the production and release of thyroid hormone. The client should see some effects within 2 weeks; however, it may take several more weeks before metabolism returns to normal. Although onset of action is 30 to 40 minutes after an oral dose, the client will not see effects immediately. Effects will take longer than 1 week to become apparent when methimazole is used. Methimazole needs to be taken every 8 hours for an extended period of time. Levels of triiodothyronine (T3) and thyroxine (T4) will be monitored and dosages adjusted as levels fall.

Ch.64 Question 12 of 25 In reviewing the health care provider admission requests for a client admitted in a hyperglycemic-hyperosmolar state, which request is inconsistent with this diagnosis? a. 20 mEq KCl for each liter of IV fluid b. IV regular insulin at 2 units/hr c. IV normal saline at 100 mL/hr d. 1 ampule NaHCO3 IV now

d 1 ampule NaHCO3 IV now NaHCO3 is given for the acid-base imbalance of diabetic ketoacidosis, not the hyperglycemic-hyperosmolar state, which presents with hyperglycemia and absence of ketosis/acidosis. KCl 20 mEq for each liter of IV fluid will correct hypokalemia from diuresis. IV regular insulin at 2 units/hr will correct hyperglycemia. IV normal saline at 100 mL/hr will correct dehydration.

Question 5 of 41 The nurse assesses the client with which hematologic problem first? a. A 32-year-old with pernicious anemia who needs a vitamin B12 injection b. A 40-year-old with iron deficiency anemia who needs a Z-track iron injection c. A 67-year-old with acute myelocytic leukemia with petechiae on both legs d. An 81-year-old with thrombocytopenia and an increase in abdominal girth

d An 81-year-old with thrombocytopenia and an increase in abdominal girth An increase in abdominal girth in a client with thrombocytopenia indicates possible hemorrhage; this warrants further assessment immediately. The 32-year-old with pernicious anemia, the 40-year-old with iron deficiency anemia, and the 67-year-old with acute myelocytic leukemia do not indicate any acute complications, so the nurse can assess them after assessing the client with thrombocytopenia.

Ch.64 Question 25 of 25 A client with type 2 diabetes who is taking metformin (Glucophage) is seen in the diabetic clinic. The fasting blood glucose is 108 mg/dL, and the glycosylated hemoglobin (HbA1C) is 8.2%. Which action does the nurse plan to take next? a. Instruct the client to continue with the current diet and metformin use. b. Discuss the need to check blood glucose several times every day. c. Talk about the possibility of adding rapid-acting insulin to the regimen. d. Ask the client about current dietary intake and medication use.

d Ask the client about current dietary intake and medication use. The nurse's first action should be to assess whether the client is adherent to the currently prescribed diet and medications. The client's current diet and medication use have not been successful in keeping glucose in the desired range. Checking blood glucose more frequently and/or using rapid-acting insulin may be appropriate, but this will depend on the assessment data. The HbA1C indicates that the client's average glucose level is not in the desired range, but discussing the need to check blood glucose several times every day assumes that the client is not compliant with the therapy and glucose monitoring. The nurse should not assume that adding insulin, which must be prescribed by the provider, is the answer without assessing the underlying reason for the treatment failure.

Ch.63 Question 10 of 21 The nurse reviews the vital signs of a client diagnosed with Graves' disease and sees that the client's temperature is up to 99.6° F. After notifying the health care provider, what does the nurse do next? a. Administers acetaminophen b. Alerts the Rapid Response Team c. Asks any visitors to leave d. Assesses the client's cardiac status completely

d Assesses the client's cardiac status completely If the client's temperature has increased by even 1°, the nurse's first action is to notify the provider. Continuous cardiac monitoring should be the next step. Administering a nonsalicylate antipyretic such as acetaminophen is appropriate, but is not a priority action for this client. Alerting the Rapid Response Team is not needed at this time. Asking visitors to leave would not be the next action, and if visitors are providing comfort to the client, this would be contraindicated.

Ch.64 Question 22 of 25 Which nursing action can the home health nurse delegate to a home health aide who is making daily visits to a client with newly diagnosed type 2 diabetes? a. Assist the client's spouse in choosing appropriate dietary items. b. Evaluate the client's use of a home blood glucose monitor. c. Inspect the extremities for evidence of poor circulation. d. Assist the client with washing the feet and applying moisturizing lotion.

d Assist the client with washing the feet and applying moisturizing lotion. Assisting with personal hygiene is included in the role of home health aides. Assisting with dietary choices, evaluating the effectiveness of teaching, and performing assessments are complex actions that should be implemented by licensed nurses.

23. Which hormone responds to elevated serum calcium blood level by decreasing bone resorption? a. PTH b. T4 c. T3 d. Calcitonin

d Calcitonin

Ch.63 Question 5 of 21 The RN has just received change-of-shift report on the medical-surgical unit. Which client will need to be assessed first? a. Client with Hashimoto's thyroiditis and a large goiter b. Client with hypothyroidism and an apical pulse of 51 beats/min c. Client with parathyroid adenoma and flank pain due to a kidney stone d. Client who had a parathyroidectomy yesterday and has muscle twitching

d Client who had a parathyroidectomy yesterday and has muscle twitching A client who is 1 day postoperative for parathyroidectomy and has muscle twitching is showing signs of hypocalcemia and is at risk for seizures. Rapid assessment and intervention are needed. Clients with Hashimoto's thyroiditis are usually stable; this client does not need to be assessed first. Although an apical pulse of 51 is considered bradycardia, a low heart rate is a symptom of hypothyroidism. A client with a kidney stone will be uncomfortable and should be asked about pain medication as soon as possible, but this client does not need to be assessed first.

Ch.62 Question 7 of 23 The charge nurse is making client assignments for the medical-surgical unit. Which client will be best to assign to an RN who has floated from the pediatric unit? a. Client in Addisonian crisis who is receiving IV hydrocortisone b. Client admitted with syndrome of inappropriate antidiuretic hormone (SIADH) secondary to lung cancer c. Client being discharged after a unilateral adrenalectomy to remove an adrenal tumor d. Client with Cushing's syndrome who has elevated blood glucose and requires frequent administration of insulin

d Client with Cushing's syndrome who has elevated blood glucose and requires frequent administration of insulin An RN who works with pediatric clients would be familiar with glucose monitoring and insulin administration. A client in Addisonian crisis would best be monitored by an RN from the medical-surgical floor. Although the float RN could complete the admission history, the client with SIADH secondary to lung cancer might require teaching and orientation to the unit that a nurse more familiar with that area would be better able to provide. Discharge teaching specific to adrenalectomy should be provided by the RN who is regularly assigned to the medical-surgical floor and is more familiar with taking care of postoperative adult clients with endocrine disorders.

Ch.64 Question 3 of 25 The nurse receives report on a 52-year-old client with type 2 diabetes: Physical Assessment - cold to touch - Right great toe mottled - Lungs- clear - Client states wears eyeglasses to read Diagnostic Findings - Glucose- 179 mg/dL - Hemoglobin A1c 6.9% mg/dL Provider Prescriptions - Regular insulin 8 units if blood glucose 250 to 275 mg/dL - Regular insulin 10 units if glucose 275 to 300 Which complication of diabetes does the nurse report to the provider? a. Poor glucose control b. Visual changes c. Respiratory distress d. Decreased peripheral perfusion

d Decreased peripheral perfusion A cold, mottled toe may indicate arterial occlusion secondary to arterial occlusive disease or embolization; this must be reported to avoid potential gangrene and amputation. Although one glucose reading is elevated, the hemoglobin A1c indicates successful glucose control over the past 3 months. After the age of 40, reading glasses may be needed due to difficulty in accommodating to close objects. Lungs are clear and no evidence of distress is noted.

16. Which statement about the gonads is correct? a. Gonads are reproductive glands found in males only. b. The function of the hormones begins at birth in low, undetectable levels. c. The placenta secretes testosterone for the development of male external genitalia. d. External genitalia maturation is stimulated by gonadotropins during puberty.

d External genitalia maturation is stimulated by gonadotropins during puberty.

7. Patients diagnosed with an anterior pituitary tumor can have symptoms of acremegaly or gigantism. These symptoms are a result of overproduction of which hormone? a. ACTH b. PRL c. Gonadotropins d. GH

d GH

Ch.63 Question 17 of 21 What effect can starting a dose of levothyroxine sodium (Synthroid) too high or increasing a dose too rapidly have on a client? a. Bradycardia and decreased level of consciousness b. Decreased respiratory rate c. Hypotension and shock d. Hypertension and heart failure

d Hypertension and heart failure Hypertension and heart failure are possible if the levothyroxine sodium dose is started too high or raised too rapidly, because levothyroxine would essentially put the client into a hyperthyroid state. The client would be tachycardic, not bradycardic. The client may have an increased respiratory rate. Shock may develop, but only as a late effect and as the result of "pump failure."

Question 27 of 41 A client who has been newly diagnosed with leukemia is admitted to the hospital. Avoiding which potential problem takes priority in the client's nursing care plan? a. Fluid overload (overhydration) b. Hemorrhage c. Hypoxia d. Infection

d Infection The main objective in caring for a newly diagnosed client with leukemia is protection from infection. Fluid overload, hemorrhage, and hypoxia are not priority problems for the client with leukemia.

30. The bloodstream delivers glucose to the cells for energy production. Which hormone controls the cells' use of glucose? a. T4 b. Growth hormone c. Adrenal steroids d. Insulin

d Insulin

27. Which statement about glucagon secretion is correct? a. It is stimulated by an increase in blood glucose levels. b. It is stimulated by a decrease in amino acid levels. c. It exerts its primary effect on the pancreas. d. It acts to increase blood glucose levels.

d It acts to increase blood glucose levels.

7. The target tissue for ADH is which organ? a. Hypothalamus b. Thyroid c. Ovary d. Kidney

d Kidney

43. After an ultrasound of the thyroid gland, which diagnostic test determines the need for surgical intervention for thyroid nodules? a. CT scan b. MRI c. Angiography d. Needle biopsy

d Needle biopsy

Question 10 of 41 Which would be an appropriate task to delegate to unlicensed assistive personnel (UAP) working on a medical-surgical unit? a. Administering erythropoietin to a client with myelodysplastic syndrome b. Assessing skin integrity on an anemic client who fell during ambulation c. Assisting a client with folic acid deficiency in making diet choices d. Obtaining vital signs on a client receiving a blood transfusion

d Obtaining vital signs on a client receiving a blood transfusion Obtaining vital signs on a client is within the scope of practice for UAP. Administering medication, assessing clients, and assisting with prescribed diet choices are complex actions that should be done by licensed nurses.

Question 41 of 41 The nurse is caring for a client who is in sickle cell crisis. What action does the nurse perform first? a. Apply cool compresses to the client's forehead. b. Encourage the client's use of two methods of birth control. c. Increase food sources of iron in the client's diet. d. Provide pain medications as needed.

d Provide pain medications as needed. Analgesics are needed to treat sickle cell pain. Warm soaks or compresses can help reduce pain perception. Cool compresses do not help the client in sickle cell crisis. Birth control is not the priority for this client. Increasing iron in the diet is not pertinent for the client in sickle cell crisis.

Ch.62 Question 11 of 23 A client with a possible adrenal gland tumor is admitted for testing and treatment. Which nursing action is most appropriate for the charge nurse to delegate to the nursing assistant? a. Assess skin turgor and mucous membranes for hydration status. b. Discuss the dietary restrictions needed for 24-hour urine testing. c. Plan ways to control the environment that will avoid stimulating the client. d. Remind the client to avoid drinking coffee and changing position suddenly.

d Remind the client to avoid drinking coffee and changing position suddenly. Drinking caffeinated beverages and changing position suddenly are not safe for a client with a potential adrenal gland tumor because of the effects of catecholamines. Reminding the client about previous instructions is an appropriate role for a nursing assistant who may observe the client doing potentially risky activities. Client assessment, client teaching, and environment planning are higher-level skills that require the experience and responsibility of the RN, and are not within the scope of practice of the nursing assistant.

1. Problems in the hypothalamus that change the function of the anterior pituitary gland result in which condition? a. Adenohypophysis b. Panhypopituitarism c. Primary pituitary dysfunction d. Secondary pituitary dysfunction

d Secondary pituitary dysfunction

Ch.61 Question 16 of 16 The nurse is reviewing the laboratory test results for a client admitted with a possible pituitary disorder. Which information has the most immediate implication for the client's care? a. Blood glucose 125 mg/dL b. Blood urea nitrogen (BUN) 40 mg/dL c. Serum potassium 5.2 mEq/L d. Serum sodium 110 mEq/L

d Serum sodium 110 mEq/L The normal range for serum sodium is 135 to 145 mEq/L; a result of 110 mEq/L is considered hyponatremia and is extremely dangerous. The client is at risk for increased intracranial pressure, seizures, and death. The RN must act rapidly because this situation requires immediate intervention. The normal range for fasting blood glucose is 60 to 110 mg/dL; 125 mg/dL is high, but is not considered dangerous. The normal range for BUN is 7 to 20 mg/dL; 40 mg/dL is high. An elevated BUN can be an indication of kidney failure, dehydration, fever, increased protein intake, and shock, so the client should have a creatinine drawn for a more complete picture of kidney function. The normal range for serum potassium is 3.5 to 5.2 mEq/L; 5.2 mEq/L is high normal.

Question 16 of 16 The nurse is reviewing the laboratory test results for a client admitted with a possible pituitary disorder. Which information has the most immediate implication for the client's care? a. Blood glucose 125 mg/dL b. Blood urea nitrogen (BUN) 40 mg/dL c. Serum potassium 5.2 mEq/L d. Serum sodium 110 mEq/L

d Serum sodium 110 mEq/L The normal range for serum sodium is 135 to 145 mEq/L; a result of 110 mEq/L is considered hyponatremia and is extremely dangerous. The client is at risk for increased intracranial pressure, seizures, and death. The RN must act rapidly because this situation requires immediate intervention. The normal range for fasting blood glucose is 60 to 110 mg/dL; 125 mg/dL is high, but is not considered dangerous. The normal range for BUN is 7 to 20 mg/dL; 40 mg/dL is high. An elevated BUN can be an indication of kidney failure, dehydration, fever, increased protein intake, and shock, so the client should have a creatinine drawn for a more complete picture of kidney function. The normal range for serum potassium is 3.5 to 5.2 mEq/L; 5.2 mEq/L is high normal.

Question 31 of 41 What is the most important environmental risk for developing leukemia? a. Direct contact with others with leukemia b. Family history c. Living near high-voltage power lines d. Smoking cigarettes

d Smoking cigarettes According to the American Cancer Society (ACS), the only proven lifestyle-related risk factor for leukemia is cigarette smoking. Leukemia is not contagious. Genetics is a strong indicator, but it is not an environmental risk factor. According to the ACS, living near high-voltage power lines is not a proven risk factor for leukemia.

26. Which statement about the pancreas is correct? a. Endocrine functions of the pancreas include secretion of digestive enzymes. b. Exocrine functions of the pancreas include secretion of glucagon and insulin. c. The islets of Langerhans are the only source of somatostatin secretion. d. Somatostatin inhibits pancreatic secretion of glucagon and insulin.

d Somatostatin inhibits pancreatic secretion of glucagon and insulin.

Question 6 of 41 A client who is receiving a blood transfusion suddenly exclaims to the nurse, "I don't feel right!" What does the nurse do next? a. Call the Rapid Response Team. b. Obtain vital signs and continue to monitor. c. Slow the infusion rate of the transfusion. d. Stop the transfusion.

d Stop the transfusion. The client may be experiencing a transfusion reaction; the nurse should stop the transfusion immediately. Calling the Rapid Response Team or obtaining vital signs is not the first thing that should be done. The nurse should not slow the infusion rate, but should stop it altogether.

Ch.62 Question 13 of 23 A client is referred to a home health agency after a transsphenoidal hypophysectomy. Which action does the RN case manager delegate to the home health aide who will see the client daily? a. Document symptoms of incisional infection or meningitis. b. Give over-the-counter laxatives if the client is constipated. c. Set up medications as prescribed for the day. d. Test any nasal drainage for the presence of glucose.

d Test any nasal drainage for the presence of glucose. Cerebrospinal fluid (CSF) will test positive using a glucose "dipstick." Nasal drainage that is positive for glucose after a transsphenoidal hypophysectomy would indicate a CSF leak that would require immediate notification of the health care provider. Home health aides can be taught the correct technique to perform this procedure. Assessing for symptoms of infection and documenting them in the record, medication administration, and setting up medication are not within the scope of practice of the home health aide.

Question 9 of 41 The nurse is mentoring a recent graduate RN about administering blood and blood products. What does the nurse include in the data? a. Obtain the client's initial set of vital signs (VS) within the first 10 minutes of the infusion. b. Remain with the client who is receiving the blood for the first 5 minutes of the infusion. c. Use a 22-gauge needle to obtain venous access when starting the infusion. d. Verify with another RN all of the data on blood products.

d Verify with another RN all of the data on blood products. All data are checked by two RNs. Human error is the most common cause of ABO incompatibilities in administering blood and blood products. Initial VS should be recorded before the start of infusion of blood, not after it has begun. The nurse remains with the client for the first 15 to 30 minutes (not 5) of the infusion. This is the period when any transfusion reactions are likely to happen. A 20-gauge needle (or a central line catheter) is used; the 22-gauge needle is too small.

Question 15 of 41 The nurse is transfusing a unit of whole blood to a client when the health care provider requests the following: "Furosemide (Lasix) 20 mg IV push." What does the nurse do? a. Add furosemide to the normal saline that is infusing with the blood. b. Administer furosemide to the client intramuscularly (IM). c. Piggyback furosemide into the infusing blood. d. Wait until the transfusion has been completed to administer furosemide.

d Wait until the transfusion has been completed to administer furosemide. Completing the transfusion before administering furosemide is the best course of action in this scenario. Drugs are not to be administered with infusing blood products; they can interact with the blood, causing risks for the client. Stopping the infusing blood to administer the drug and then restarting it is also not the best decision. Changing the admission route is not a nursing decision.

Ch.62 Question 4 of 23 The nurse is caring for a client with hypercortisolism. The nurse begins to feel the onset of a cold but still has 4 hours left in the shift. What does the nurse do? a. Asks another nurse to care for the client b. Monitors the client for cold-like symptoms c. Refuses to care for the client d. Wears a facemask when caring for the client

d Wears a facemask when caring for the client A client with hypercortisolism will be immune-suppressed. Anyone with a suspected upper respiratory infection who must enter the client's room must wear a mask to prevent the spread of infection. Although asking another nurse to care for the client might be an option in some facilities, it is not generally realistic or practical. The nurse, not the client, feels the onset of the cold, so monitoring the client for cold-like symptoms is part of good client care for a client with hypercortisolism. Refusing to care for the client after starting care would be considered abandonment.

Ch.62 Question 14 of 23 A client with Cushing's disease says that she has lost 1 pound. What does the nurse do next? a. Auscultates the lungs for crackles b. Checks urine for specific gravity c. Forces fluids d. Weighs the client

d Weighs the client Fluid retention with weight gain is more of a problem than weight loss in clients with Cushing's disease. Weighing the client with Cushing's disease is part of the nurse's assessment. Crackles in the lungs indicate possible fluid retention, which would cause weight gain, not weight loss. Urine specific gravity will help assess hydration status, but this would not be the next step in the client's assessment. Forcing fluids is not appropriate because usually excess water and sodium reabsorption cause fluid retention in the client with Cushing's disease.

Question 23 of 41 The nurse is caring for a client with neutropenia. Which clinical manifestation indicates that an infection is present or should be ruled out? a. Coughing and deep breathing b. Evidence of pus c. Fever of 102° F or higher d. Wheezes or crackles

d Wheezes or crackles Wheezes or crackles in the neutropenic client may be the first symptom of infection in the lungs. Coughing and deep breathing are not indications of infection, but can help prevent it. The client with leukopenia, not neutropenia, may have a severe infection without pus or with only a low-grade fever.

Ch.64 Question 5 of 25 A client recently admitted with new-onset type 2 diabetes will be discharged with a self-monitoring blood glucose machine. When is the best time for the nurse to explain to the client the proper use of the machine? a. Day of discharge b. On admission c. When the client states readiness d. While performing the test in the hospital

d While performing the test in the hospital Teaching the client about the operation of the machine while performing the test in the hospital is the best way for the client to learn. The teaching can be reinforced before discharge. Instructing the client on the day of admission or the day of discharge would be overwhelming to the client because of all of the other activities taking place on those days. The client may never feel ready to learn this daunting task; the nurse must be more proactive.

Ch.62 Question 5 of 23 How does the drug desmopressin (DDAVP) decrease urine output in a client with diabetes insipidus (DI)? a. Blocks reabsorption of sodium b. Increases blood pressure c. Increases cardiac output d. Works as an antidiuretic hormone (ADH) in the kidneys

d Works as an antidiuretic hormone (ADH) in the kidneys Desmopressin is a synthetic form of ADH that binds to kidney receptors and enhances reabsorption of water, thus reducing urine output. Desmopressin does not have any effect on sodium reabsorption. It may cause a slight increase or a transient decrease in blood pressure, but this does not affect urine output. Desmopressin does not increase cardiac output.

11. A deficiency of which anterior pituitary hormones is considered life-threatening? (Select all that apply.) a. GH b. Melanocyte-stimulating hormone (MSH) c. PRL d. Thyroid-stimulating hormone (TSH) e. ACTH

d, e Thyroid-stimulating hormone (TSH), ACTH

Question 21 of 41 An 82-year-old client with anemia is requested to receive 2 units of whole blood. Which assessment findings cause the nurse to discontinue the transfusion because it is unsafe for the client? Select all that apply. a. Capillary refill less than 3 seconds b. Decreased pallor c. Flattened superficial veins d. Hypertension e. Hypotension f. Rapid, bounding pulse

d, e, f Hypertension, Hypotension, Rapid, bounding pulse In an older adult receiving a transfusion, hypertension is a sign of overload, low blood pressure is a sign of a transfusion reaction, and a rapid and bounding pulse is a sign of fluid overload. In this scenario, 2 units, or about a liter of fluid, could be problematic. Capillary refill time that is less than 3 seconds is considered to be normal and would not pose a problem. Increased (not decreased) pallor and cyanosis are signs of a transfusion reaction, while swollen (not flattened) superficial veins are present in fluid overload in older adult clients receiving transfusions.

58. Place the steps of using a bedside bladder scanner in the correct order using the numbers 1 through 6. a. Select the male or female icon on the bladder scanner. b. Aim the scan head towards the expected location of the bladder. c. Place the probe midline about 1.5 inches (4cm) above the pubic bone. d. Explain the purpose and what sensations to expect. e. Place the ultrasound probe with gel right above the symphysis pubis. f. Press and release the scan button.

d-a-e-c-b-f Explain the purpose and what sensations to expect - Select the male or female icon on the bladder scanner - Place the ultrasound probe with gel right above the symphysis pubis - Place the probe midline about 1.5 inches (4cm) above the pubic bone - Aim the scan head towards the expected location of the bladder - Press and release the scan button

82. Place the sequence of steps of continous ambulatory peritoneal dialysis (CAPD) in the correct order using the numbers 1 through 4. a. Fluid stays in the cavity for a specified time prescribed by the health care provider. b. 1 to 2 L of dialysate is infused by gravity over a 10- to 20-minute period. c. Fluid flows out of the body by gravity into a drainage bag. d. Warm the dialysate bags before installation by using a heating pad to wrap the bag.

d-b-a- c d. Warm the dialysate bags before installation by using a heating pad to wrap the bag. b. 1 to 2 L of dialysate is infused by gravity over a 10- to 20-minute period. a. Fluid stays in the cavity for a specified time prescribed by the health care provider. c. Fluid flows out of the body by gravity into a drainage bag.

A nursing student wants to know why clients with chronic obstructive pulmonary disease tend to be polycythemic. What response by the nurse instructor is best? a. It is due to side effects of medications for bronchodilation. b. It is from overactive bone marrow in response to chronic disease. c. It combats the anemia caused by an increased metabolic rate. d. It compensates for tissue hypoxia caused by lung disease.

d. It compensates for tissue hypoxia caused by lung disease. ANS: D In response to hypoxia, more red blood cells are made so more oxygen can be carried and delivered to tissues. This is a physiologic process in response to the disease; it is not a medication side effect, the result of overactive bone marrow, or a response to anemia.

A hospitalized client has a platelet count of 58,000/mm3. What action by the nurse is best? a. Encourage high-protein foods. b. Institute neutropenic precautions. c. Limit visitors to healthy adults. d. Place the client on safety precautions.

d. Place the client on safety precautions. ANS: D With a platelet count between 40,000 and 80,000/mm3, clients are at risk of prolonged bleeding even after minor trauma. The nurse should place the client on safety precautions. High-protein foods, while healthy, are not the priority. Neutropenic precautions are not needed as the client's white blood cell count is not low. Limiting visitors would also be more likely related to a low white blood cell count.

A client is having a radioisotopic imaging scan. What action by the nurse is most important? a. Assess the client for shellfish allergies. b. Place the client on radiation precautions. c. Sedate the client before the scan. d. Teach the client about the procedure.

d. Teach the client about the procedure. ANS: D The nurse should ensure that teaching is done and the client understands the procedure. Contrast dye is not used, so shellfish/iodine allergies are not related. The client will not be radioactive and does not need radiation precautions. Sedation is not used in this procedure.

Which statement by a client scheduled for a percutaneous transluminal coronary angioplasty (PTCA) indicates a need for further preoperative teaching? a. "I will be awake during this procedure." b. "I will have a balloon in my artery to widen it." c. "I must lie still after the procedure." d. "My angina will be gone for good."

d. "My angina will be gone for good." Reocclusion is possible after PTCA. The client is typically awake, but drowsy, during this procedure. PTCA uses a balloon to widen the artery, and the client will have to lie still after the procedure because of the large-bore venous access. Time is necessary to allow the hole to heal and prevent hemorrhage.

A nursing student is struggling to understand the process of graft-versus-host disease. What explanation by the nurse instructor is best? a. "Because of immunosuppression, the donor cells take over." b. "It's like a transfusion reaction because no perfect matches exist." c. "The client's cells are fighting donor cells for dominance." d. "The donor's cells are actually attacking the client's cells."

d. "The donor's cells are actually attacking the client's cells."

A nursing student is caring for a client with leukemia. The student asks why the client is still at risk for infection when the client's white blood cell count (WBC) is high. What response by the registered nurse is best? a. "If the WBCs are high, there already is an infection present." b. "The client is in a blast crisis and has too many WBCs." c. "There must be a mistake; the WBCs should be very low." d. "Those WBCs are abnormal and don't provide protection."

d. "Those WBCs are abnormal and don't provide protection."

During discharge planning after admission for a myocardial infarction, the client says, "I won't be able to increase my activity level. I live in an apartment, and there is no place to walk." What is the nurse's best response? a. "You are right. Work on your diet then." b. "You must find someplace to walk." c. "Walk around the edge of your apartment complex." d. "Where might you be able to walk?"

d. "Where might you be able to walk?" Asking the client where he or she might be able to walk calls for cooperation and participation from the client; increased activity is imperative for this client. Telling the client to work on diet is an inappropriate response. Telling the client to find someplace to walk is too demanding to be therapeutic. Telling the client to walk around the apartment complex is domineering and will not likely achieve cooperation from the client.

To validate that a client has had a myocardial infarction (MI), the nurse assesses for positive findings on which tests? a. Creatine kinase-MB fraction (CK-MB) and alkaline phosphatase b. Homocysteine and C-reactive protein c. Total cholesterol, low-density lipoprotein cholesterol, and high-density lipoprotein cholesterol d. CK-MB and troponin

d. CK-MB and troponin CK-MB and troponin are the cardiac markers used to determine whether MI has occurred. Alkaline phosphatase is often elevated in liver disease. Homocysteine and C-reactive protein are markers of inflammation, which may represent risk for MI, but they are not diagnostic for MI. Elevated cholesterol levels are risks for MI, but they do not validate that an MI has occurred.

4. A nurse assesses a client on the medical-surgical unit. Which statement made by the client should alert the nurse to the possibility of hypothyroidism? a. "My sister has thyroid problems." b. "I seem to feel the heat more than other people." c. "Food just doesn't taste good without a lot of salt." d. "I am always tired, even with 12 hours of sleep."

d. Clients with hypothyroidism usually feel tired or weak despite getting many hours of sleep. Thyroid problems are not inherited. Heat intolerance is indicative of hyperthyroidism. Loss of taste is not a manifestation of hypothyroidism.

A nurse is preparing to hang a blood transfusion. Which action is most important? a. Documenting the transfusion b. Placing the client on NPO status c. Placing the client in isolation d. Putting on a pair of gloves

d. Putting on a pair of gloves

The nurse is caring for a client 36 hours after coronary artery bypass grafting, with a priority problem of intolerance for activity related to imbalance of myocardial oxygen supply and demand. Which finding causes the nurse to terminate an activity and return the client to bed? a. Pulse 60 beats/min and regular b. Urinary frequency c. Incisional discomfort d. Respiratory rate 28 breaths/min

d. Respiratory rate 28 breaths/min Tachypnea and tachycardia reflect activity intolerance; activity should be terminated. Pulse 60 beats/min and regular is a normal finding. Urinary frequency may indicate infection or diuretic use, but not activity intolerance. Pain with activity after surgery is anticipated; pain medication should be available.

The nurse is assessing a client with chest pain to evaluate whether the client is suffering from angina or myocardial infarction (MI). Which symptom is indicative of an MI? a. Chest pain brought on by exertion or stress b. Substernal chest discomfort occurring at rest c. Substernal chest discomfort relieved by nitroglycerin or rest d. Substernal chest pressure relieved only by opioids

d. Substernal chest pressure relieved only by opioids Substernal chest pressure relieved only by opioids is typically indicative of MI. Substernal chest discomfort that occurs at rest is not necessarily indicative of MI; it could be a sign of unstable angina. Both chest pain brought on by exertion or stress and substernal chest discomfort relieved by nitroglycerin or rest are indicative of angina.

A client with autoimmune idiopathic thrombocytopenic purpura (ITP) has had a splenectomy and returned to the surgical unit 2 hours ago. The nurse assesses the client and finds the abdominal dressing saturated with blood. What action is most important? a. Preparing to administer a blood transfusion b. Reinforcing the dressing and documenting findings c. Removing the dressing and assessing the surgical site d. Taking a set of vital signs and notifying the surgeon

d. Taking a set of vital signs and notifying the surgeon

The nurse assesses a client's oral cavity and makes the discovery shown in the photo below: What action by the nurse is most appropriate? a. Encourage the client to have genetic testing. b. Instruct the client on high-fiber foods. c. Place the client in protective precautions. d. Teach the client about cobalamin therapy.

d. Teach the client about cobalamin therapy.

A client with sickle cell disease (SCD) takes hydroxyurea (Droxia). The client presents to the clinic reporting an increase in fatigue. What laboratory result should the nurse report immediately? a. Hematocrit: 25% b. Hemoglobin: 9.2 mg/dL c. Potassium: 3.2 mEq/L d. White blood cell count: 38,000/mm3

d. White blood cell count: 38,000/mm3

A client with multiple myeloma demonstrates worsening bone density on diagnostic scans. About what drug does the nurse plan to teach this client? a. Bortezomib (Velcade) b. Dexamethasone (Decadron) c. Thalidomide (Thalomid) d. Zoledronic acid (Zometa)

d. Zoledronic acid (Zometa)

do online practice questions

do online practice questions

do study guide practice questions

do study guide practice questions


संबंधित स्टडी सेट्स

BJU Physical Science (6th ed.) - Chapter 4

View Set

Programming Fundamentals III Java FINAL

View Set

Chapter 5-8 study guide(includes quiz)

View Set

Homework 3 - Descriptive Statistics and Boxplots

View Set